339

Differential Diagnosis and Management Options inuploads.worldlibrary.org/uploads/pdf/20180107071525differential... · 83 Peripartum cardiomyopathy 165 84 Hypertension 167 85 Convulsions

  • Upload
    others

  • View
    9

  • Download
    0

Embed Size (px)

Citation preview

Page 1: Differential Diagnosis and Management Options inuploads.worldlibrary.org/uploads/pdf/20180107071525differential... · 83 Peripartum cardiomyopathy 165 84 Hypertension 167 85 Convulsions
Page 2: Differential Diagnosis and Management Options inuploads.worldlibrary.org/uploads/pdf/20180107071525differential... · 83 Peripartum cardiomyopathy 165 84 Hypertension 167 85 Convulsions

Differential Diagnosis andManagement Options in

Gynecology and Obstetrics

Page 3: Differential Diagnosis and Management Options inuploads.worldlibrary.org/uploads/pdf/20180107071525differential... · 83 Peripartum cardiomyopathy 165 84 Hypertension 167 85 Convulsions

Differential Diagnosis andManagement Options in

Gynecology and Obstetrics

Third Edition

Shashank V ParulekarMD, FCPS, DGO, DFPProfessor and Head

Department of Obstetrics and GynecologySeth GS Medical CollegeKEM Hospital, Mumbai

MD, FCPS, DGO, DFP

Page 4: Differential Diagnosis and Management Options inuploads.worldlibrary.org/uploads/pdf/20180107071525differential... · 83 Peripartum cardiomyopathy 165 84 Hypertension 167 85 Convulsions

© Shashank Parulekar, 2017First published in 1997 by Popular Prakashan Pvt LtdSecond Edition 2005 by Popular Prakashan Pvt LtdThird Edition 2017

All rights reserved. No part of this book may be reproduced or distributed in any form or by any means, or stored in a data base or retrieval system, without the prior permissionof the Author.

NoticeObstetrics and Gynecology are ever-changing sciences. The author has checked with sources believed to be reliable in order to provide information which is complete and inaccord with the standards acceptable at the time of publication of this book. However in view of the possible changes in these sciences and human error, neither the author northe publisher nor any other person involved in the publication of this book warrants that the information contained in it is in every respect accurate or complete, and they arenot responsible for any errors or omissions, or for the results obtained from the use of such information. Readers are encouraged to confirm the information contained in thisbook with other sources, including product information provided by manufacturers of different drugs, so as to be certain that changes have not been made in the recommendeddoses or in contraindications for the use of those drugs.Published by: Dr Shashank Parulekar

PRINTED IN INDIA

Amey
Typewriter
2018
Amey
Typewriter
Free to read
Page 5: Differential Diagnosis and Management Options inuploads.worldlibrary.org/uploads/pdf/20180107071525differential... · 83 Peripartum cardiomyopathy 165 84 Hypertension 167 85 Convulsions

PrefaceObstetrics and Gynecology, like all other branches of medicine, are changing constantly. What are considered advances today may become old before long. It is no

wonder that I have had to revise this book twice in a course of just twelve years. Some things just do not change, like gynecologic disorders, and conventional obstetrics. I havechanged those parts of the book a little. But investigative modalities and treatment keep changing, and therefore have been revised. Oncology keeps changing in staging andmanagement options. Those chapters have been thoroughly revised. Conditions which were not common before may gain importance due to either change in epidemiologicalfactors, or uncovering of new information due to modern investigations. As a result I have had to add a few chapters. I have combined a couple of chapters. Thus the overallnumber of chapters has increased by just one. The style of the book has been maintained, in the form of flow-charts and concise explanations. I hope my readers find thisnewer edition more useful than the previous one.

Preface to First EditionThe process of learning of a doctor is quite complex. The formal training involves a series of course in theory and practicals of different subject. The teaching is topic

oriented. Each session is complete, with exhaustive information on all aspects. However such teaching is text-book based, with little connection between different topics at thetime of teaching each of them. Some text-books include differential diagnosis in the discussion of individual topics. However, provision of salient features of each of theconditions in the differential diagnosis is often not enough to clear the student's mind and does not help him make up his mind about his patient's disease. The description ofvarious forms of treatments is also available in text-books. But a student may not be able to decide on the best course of action in a given clinical situation, if he relies solelyon his text-books.

The next phase of learning medical sciences is the phase of medical practice. A doctor sees patients and treats them After an extensive experience, his master computer,i.e. the brain gets programmed well. Once this stage is reached one can make an accurate diagnosis and decide the course of action without seeming to make a conscious effortat it. Of course the process is not automatic. It is a reflex which involves a series of steps, ruling out different conditions at each, and finally deciding the diagnosis and theform of treatment. A number of unimportant steps described in standard teaching are not followed because they do not seem to make any difference in that situation. This is analgorithm or a flow-chart. A successful clinician has all the flow charts in his mind, and he uses them as needed, based on his patient's symptoms or needs.

I have developed the flow-charts in this book with a view of shortening the personal-experience-phase of a doctor's learning. These flow-charts are based on years ofexperience of not only my colleagues and I, but all the great masters of Obstetrics and Gynecology, from whose teaching we have learned. These flowcharts are problem-basedI have refrained from making topic-based flow-charts as far as possible, because a patient presents to a doctor with a problem, not a diagnosis. Topic-based charts are usedwhen the diagnosis is already made using another flow-chart It must be understood that a certain degree of basic knowledge of the subject is necessary for being able to usethis book well. The book is meant for exactly what the title suggests, making a differential diagnosis and choosing the right option of management.

The flow-charts are printed the way the sequence of thoughts should be. The problem is outlined at the top. The clinical test or investigation to be done in assessment atany step is printed in italics The results of the tests are printed in ordinary type. The diagnoses are put in boxes The treatment is printed in ordinary type. Explanations aregiven in the text opposite capital letters printed in the flow-charts.

A chapter on therapeutics is added at the end, where commonly used drugs are presented in tabular form. I would have preferred to put these tables in their respectivechapters, but lack of space prevented me from doing so. I regret the inconvenience caused to my readers.

The chapter on "Request for Assisted Reproductive Techniques" is written by Dr Satish Sharma. I am extremely grateful to him for sharing his expertise with us all. Ithank my publishers for the remarkable job they have done in printing this book. If the readers can practice their art better by reading this book, I will feel that my efforts havebeen worthwhile.Shashank V. Parulekar

Page 6: Differential Diagnosis and Management Options inuploads.worldlibrary.org/uploads/pdf/20180107071525differential... · 83 Peripartum cardiomyopathy 165 84 Hypertension 167 85 Convulsions

CONTENTSI Menstrual disorders1 Dysmenorrhea 12 Primary amenorrhea 33 Secondary amenorrhea 54 Puberty menorrhagia 75 Menorrhagia in reproductive age 96 Postmenopausal vaginal bleeding 117 Metrorrhagia 138 Hypomenorrhea 159 Precocious vaginal bleeding 1710 Premenstrual syndrome 1911 Suspicion of endometrial hyperplasia 2112 Suspicion of luteal phase defect 23II General gynecology13 Acute lower abdominal and pelvic pain 2514 Tenderness on transverse cervical movements 2715 Low backache 2916 Endometriosis 3117 Pyometra 3318 Adnexal mass 3519 Abdominal enlargement 3720 Uterine leiomyoma 3921 Genital prolapse 4122 Leukorrhea 4323 Itching of the vulva 4524 Vulvar lesion 4725 Vulvodynia 4926 Female sexual dysfunction 5127 Painful dyspareunia 5328 Chronic Pelvic Pain 55III Gynecologic urology29 Urinary incontinence 5730 Frequency of micturition 5931 Retention of urine 61IV Gynecologic endocrinology32 Hirsutism 6333 Galactorrhea 65V Infertility34 Infertility 6735 Induction of ovulation 6936 Cervical factor and sperm mucus penetration test 7137 Male infertility 73VI Congenital malformations of genital tract38 Congenital absence of vagina 7539 Congenital malformations of the female genital tract 77

VII Gynecologic oncology40 Pap smear 7941 Cervical intraepithelial neoplasia 8142 Invasive cancer of cervix 8343 Undiagnosed cervical cancer in a total hysterectomy specimen 8544 Endometrial carcinoma 8745 Uterine sarcoma 8946 Ovarian carcinoma 9147 Suspicion of choriocarcinoma 9348 Suspicion of vulvar carcinoma 95VIII Operative gynecology49 Choice of laparotomy incision 9750 Difficult vaginal hysterectomy 9951 Myomectomy 10152 Repair of vesicovaginal fistula 10353 Request for reversal of female sterilization 10554 Postoperative fever 10755 Postoperative hemorrhage 10956 Suspicion of intraoperative bladder injury 11157 Suspicion of operative injury to ureter 11358 Operative injury to bowel 11559 Postoperative abdominal wound breakdown 11760 Posthysterectomy vaginal evisceration 11961 Needle breaking inside tissues 121IX General obstetrics62 Suspicion of pregnancy 12363 Last menstrual period (LMP) unknown 12564 Uterine size less than expected 12765 Uterine size more than expected 12966 Failure to gain weight 13167 Exposure to teratogen 13368 Request for prenatal diagnosis 135X Obstetric hemorrhage69 First trimester vaginal bleeding 13770 Suspicion of septic abortion 13971 Suspicion of tubal ectopic pregnancy 14172 Repeated abortions 14373 Antepartum hemorrhage 145XI Medical disorders74 Vomiting 14775 Edema in pregnancy 14976 Suspicion of anemia 15177 Subacute/chronic breathlessness 15378 Cough 15579 Pulmonary disease 157

Page 7: Differential Diagnosis and Management Options inuploads.worldlibrary.org/uploads/pdf/20180107071525differential... · 83 Peripartum cardiomyopathy 165 84 Hypertension 167 85 Convulsions

80 Cardiac disease 15981 Cyanotic heart disease 16182 Coarctation of aorta 16383 Peripartum cardiomyopathy 16584 Hypertension 16785 Convulsions 16986 Asymptomatic bacteriuria 17187 Chronic renal disease 17388 Jaundice in pregnancy 17589 Pruritus 17790 Out-of-wedlock pregnancy 17991 Venous thrombosis 18192 Pulmonary embolism 18393 Systemic lupus erythematosus 18594 Scleroderma 18795 Antiphospholipid antibody syndromeXII Obstetric disorders 18996 Rh-negative gravida 19197 Multiple pregnancy 19398 Umbilical cord prolapse 19599 Hydramnios 197100 Oligohydramnios 199101 Preterm labor 201102 Vaginal leakage of fluid 203103 Prolonged pregnancy 205104 Reduced fetal movements 207105 Induction of labor 209XIII Disorders of labor106 Occipitoposterior position 211107 Breech presentation 213108 Transverse lie 215109 Face presentation 217110 Brow presentation 219111 Compound presentation 221112 High floating presenting part at term 223113 Suspicion of pelvic contraction at term 225114 Nonprogress of labor 227115 Soft tissue dystocia 229116 Fetal macrosomia 231117 Fetal hydrocephalus 233118 Previous cesarean section 235XIV Surgical disorders119 Trauma 237120 Pelvic or abdominopelvic lump in pregnancy 239121 Burns during pregnancy 241122 Goitre in pregnancy 243123 Thyroid nodule in pregnancy 245

124 Nipple discharge during pregnancy 247125 Solitary breast mass in pregnancy 249126 Breast pain 251127 Hematemesis in pregnancy 253128 Epistaxis in pregnancy 255129 Hernias in pregnancy 257130 Varicose veins in pregnancy 259XV Maternal fetal infections131 Cytomegalovirus Infection 261132 Dengue In Pregnancy 263133 Ebola Virus Disease 265134 Group B Streptococcus 267135 Hepatitis B In Pregnancy 269136 Hepatitis C In Pregnancy 271137 Herpes Simplex 273138 HIV Infection 275139 Rash In Pregnancy 277140 Rubella. doc 281141 Toxoplasmosis 283142 Varicella Zoster 285143 Zika Virus 287

Fetal medicine144 Assessment of fetal well being 289145 Fetal arrhythmias 291146 Hydrops fetalis 293147 Craniospinal and facial anomalies 295148 Fetal alloimmune thrombocytopenia 297149 Intrapartum fetal monitoring 299150 Acute fetal distress 301151 Ambiguous genitalia 303XV1 Postpartum period152 Postpartum hemorrhage 305153 Postpartum hemorrhage : uterus not palpable 307154 Late postpartum hemorrhage 309155 Suspicion of coagulopathy 311156 Thrombocytopenia 313157 Sudden collapse in pregnancy 315158 Puerperal pyrexia 317159 Postpartum mastalgia 319XVII Operative obstetrics160 Cervical incompetence 321161 Difficult cesarean section 323XVIII Contraception162 Desire for contraception 325163 Missing threads of an IUCD 327164 Failure of contraception 329165 Desire for medical termination of pregnancy 331

Amey
Stamp
Amey
Stamp
Amey
Stamp
Amey
Stamp
Amey
Stamp
Page 8: Differential Diagnosis and Management Options inuploads.worldlibrary.org/uploads/pdf/20180107071525differential... · 83 Peripartum cardiomyopathy 165 84 Hypertension 167 85 Convulsions

1. DysmenorrheaDymenorrhea or painful menstruation, is one of the commonest

complaints in gynecology. The pain is severe enough to prevent normal activityand require medication. Dysmenorrhea is called primary when there is no organiccause, and secondary when there is an underlying organic cause.

A. The onset of primary dysmenorrhea is below the age of 25 years. The firstfew cycles of an adolescent girl are usually anovulatory and hence painless. Shestarts ovulating regularly about 2 years after menarche, when the dysmenorrheastarts. The onset of pain is 1 to 2 days before the onset of the menstrual flow.It lasts for 1 to 2 days after the onset of flow. The site of pain is lower abdomen,lower back, and/or medial aspects of the thighs, but never below the level ofthe knee joints. No abnormality is detected on a bimanual pelvic examination.Treatment is with any of the following regimes.

1. NSAlDs : from the onset of menstruation or pain, whichever is earlier, for theduration of pain.a. Ibuprofen : 400 mg PO q8h.b. Mefenamic acid : 500 mg PO loading dose, followed by 250-500 mg

PO q6h.c. Naproxen sodium : 550 mg PO loading dose, followed by 275 mg PO

q6-8h.d. Diclofenac sodium : 50 mg PO q8h.

2. Combination oral contraceptive pills : dose is 1 tablet a day PO for 21 days,starting from the 2nd day of the period. Subsequently the 21 day cycles arerepeated with a 7 day gap in between. This treatment prevents ovulation andhence dysmenorrhea. It is not recommended to adolescent girls because theestrogen in the pills hastens epiphysial fusion and limits increase in height. Itis recommended when contraception is desired too.

3. Dicyclomine 20 mg PO q8h.4. Calcium antagonists (e.g. nifedipine) show promise.5. Miscellaneous: complementary and alternate medicines (vitamin B1, vitamin E,

Fish oil-vitamin B12 combination, magnesium, ginger, Toki-shakuyaku-san, andNeptune krill oil), acupressure, acupuncture, aerobic exercises, local hotfomentation, massage, meditation, transcutaneous electrical nerve stimulationare techniques which may be useful to some women. Laparoscopic uterosacralligament resection is not useful. Cervical rapid dilatation (to No. 16 Hegar's),injection of paracervical plexus with alcohol, and presacral neurectomy havebecome obsolete.

B. Secondary dysmenorrhea develops above the age of 25 years. The relationof the pain to the menstrual period is important. In one group, the onset is 2 to 3days before the onset of the flow, and maximum pain is experienced near the endof the flow. A differentiation of the causes is made by a bimanual pelvicexamination.

1. The uterus is irregularly enlarged with multiple leiomyomas. With the former,

there is discordinate uterine activity and pain. See chapter 20 for further discussionon leiomyomas.

2. Uniform enlargement of the uterus may be due to adenomyosis or a single largeleiomyoma. The distinction is easily made by ultrasonography. With a singlesubmucous leiomyomatous polyp, the uterus is uniformly enlarged. With asubmucous leiomyoma there is intense colicky pain when the uterus contracts,trying to expel the leiomyoma. Adenomyosis is more common in multiparous,middle-aged women. It is often associated with menorrhagia.

3. If the uterus is of normal size but of abnormal shape, a malformation of theuterus may be present. A rudimentary horn, bicornuate uterus or septate uterus maycause dysmenorrhea due to asynchronous uterine contractions.

4. The uterus is retroverted and fixed, and one or both of the uterosacral ligamentsare nodular and tender in pelvic endometriosis. Pelvic ultrasonography andlaparoscopy help establish the diagnosis.

C. The woman may have constant pelvic discomfort. If it gets worse with menstrualflow, and is associated with tender and nodular uterosacral ligaments or retrovertedfixed uterus, it is likely to be pelvic endometriosis. If there is a tender masson one side of and close to the uterus, it is hemihematometra. It is due to distensionof a functioning but noncommunicating uterine horn in case of a bicornuateuterus. Revealed menstruation is due to a communicating horn. Thenoncommunicating horn has to be removed surgically. If the pain is worse for 2 to3 days before the flow and is relieved by the flow, it is congestive dysmenorrhea.The pelvic findings are normal in such cases. The treatment is pelvic exercises andchange from sedentary life style. In case of chronic pelvic inflammatory disease,there is fever and exacerbation of pain at the time of the menses. Pelvic findingsinclude a retroverted, fixed uterus, and uni- or bilateral forniceal tenderness with orwithout tender, fixed masses (tubo-ovarian masses). Treatment is antibiotics, NSAIDs,and surgical removal of the masses if resistant to conservative treatment. Anintrauterine contraceptive device causes local release of prostaglandins and hencepainful uterine contractions during menses. If it does not respond to NSAIDs, thedevice has to be removed.

D. If the woman has severe menorrhagia, she may pass blood clots during menses.She experiences severe pain when the clots pass through the cervix. The uterusmay be of normal size or bulky. Bulky uterus may the result of subinvolution of theuterus or a small leiomyoma. A normal-sized uterus with menorrhagia suggests thepresence of dysfunctional uterine bleeding. See chapter 5 further discussion onthese conditions.

E. Membranous dysmenorrhea is a rare condition in which there is an absenceof a proteolytic enzyme in the endometrium, as a result of which the endometriumdoes not fragment and is passed as a cast during menstruation. The woman experiencessevere pain as the cast passes through the cervix. This condition is resistantto all forms of treatment.

1

Page 9: Differential Diagnosis and Management Options inuploads.worldlibrary.org/uploads/pdf/20180107071525differential... · 83 Peripartum cardiomyopathy 165 84 Hypertension 167 85 Convulsions

DYSMENORRHEA

Check age at onset

AdolescenceBelow 25 (A)

Above 25

Check relation to menstrual cycle Check relation to menstrual cycle

Onset 1-2 d beforemenstrual flow, lastsfor 1-2 d after onsetof flow

Onset 2-3 d beforemenstrual flow, peaksnear the end of the flow

Constant throughout menstrual cycle (C) Onset withheavy flow (D)

Onset with passage ofendometrial cast (E)

Perform bimanual pelvic examination (B) Worse with flow

Perform bimanual pelvicexamination

Worse for 2-3 d beforeflow, relieved by it

Worse for2-3 dbeforeflow,fever

IUCD Performbimanual pelvicexamination

Membranousdysmenorrhea

Normal AbnormalTender nodule on side of uterus

Congestivedysmenorrhea

Characteristicfindings

Irregularenlargement ofuterus

Normal-sized orbulky uterus

Primarydysmenorrhea

Retroverted fixed uterus,nodular, tender uterosacralligament(s)

Endometriosis

Hemihematometra Chronic pelvicinflammatorydisease

Uterineleiomyomas

Menorrhagia

Irregularenlargement ofuterus

Uniform enlargement of uterus Abnormal shape ofnormal-sized uterus

Pelvic ultrasonography

Uterine leiomyomas Adenomyosis Uterine malformation, hemihematometra.

2

Page 10: Differential Diagnosis and Management Options inuploads.worldlibrary.org/uploads/pdf/20180107071525differential... · 83 Peripartum cardiomyopathy 165 84 Hypertension 167 85 Convulsions

2. Primary AmenorrheaA woman is evaluated for primary amenorrhea if she has not begun to menstruateby the age of 16 years in the presence of secondary sex characteristics, and 14years in their absence.

A. Secondary sex characters develop as a result of sex steroids secreted by thegonads. Hence they do not develop if the woman is hypogonadic. If these arefound to be developed normally, the woman is examined for an outflow tractobstruction (imperforate hymen, transverse vaginal septum, vaginal atresia, andcervical atresia) which produces cryptomenorrhea. The menstrual blood collectsabove the level of the obstruction and distends the genital tract (hematocolpos,hematocervix, hematometra) which is palpable on rectal-abdominal examination.See chapter 39 for further discussion on cryptomenorrhea.

B. If there is no outflow tract obstruction, the uterus is looked for clinicallyand with pelvic ultrasonography. If it is absent, the woman's karyotype is determined.46,XX karyotype in a woman with a blind pouch of vagina and absence of uterusis diagnostic of mullerian agenesis (Rokitansky Kuster Hauser syndrome). Ina similar presentation, if the karyotype is 46,XY, the diagnosis is total androgeninsensitivity. See chapter 39 for the management of these conditions. If the womanhas a uterus and no outflow tract obstruction, the evaluation is as for secondaryamenorrhea (see chapter 3).

C. If the woman has no secondary sex characteristics, she has in all probabilityhypogonadism. Her height is measured. If she is short, stigmas of Turner'ssyndrome are looked for, such as webbing of neck, congenital heart disease,cubitus valgus. If those are present, her karyotype is determined. It is 45,X inTurner syndrome, and 46,XX in Noonan syndrome. The latter has delay in pubertyin some cases. It is an autosomal dominant disorder, in which the gonads arenormal and not streaks as in Turner's syndrome. If she does not have stigmata ofTurner's syndrome, but has streak gonads, she is a variant with loss of a part of theshort arm of the X-chromosome, which can be diagnosed on karyotyping using abanding technique.

D. If a woman has eunuchoid habitus, streak gonads (on ultrasonography and/or laparoscopy) and 46,XX karyotype, she has pure gonadal dysgenesis.

E. If the woman has no physical stigmas of any genetic disorders, her serumFSH and LH levels are assayed, to differentiate between hypo- and hypergonadotropichypogonadism. If the levels are low, clinical disorders which can be diagnosedeasily are looked for.

1. Endocrine : Cushing's syndrome, hypothyroidism.

2. Gastrointestinal : regional enteritis, ulcerative colitis.

3. Neurologic: encephalocele, congenital hydrocephalus, pituitary tumor, birth

asphyxia or physical trauma.

4. Kallman syndrome : anosmia plus LHRH deficiency.

5. Laurence-Moon-Biedl syndrome.

6. Prader Willi syndrome.

F. High levels of FSH and LH (hypergonadotropic hypogonadism) indicategonadal failure. The damage may be genetic, which can be diagnosed bykaryotyping. It may be due to Swyer syndrome (XY pure gonadal dysgenesis), 17-alpha hydroxylase deficiency in gonad and adrenal (46,XY), or X-isochromosomeformation. In case of a normal karyotype, a history of exposure to agents damaginggonads may be obtained, e.g. radiation, antimalignancy chemotherapy, and rubella.The woman may be known to have a medical disorder like ataxia telangiectasia,myotonia dystrophica, or galactosemia. She may have familial gonadal failure. Ifshe does not have any of these conditions, laparoscopy-guided gonadal biopsy isdone to differentiate between infiltrative ovarian disease and Savage syndrome(gonadotropin resistance of ovary).

G. If the woman has ambiguous external genitalia and virilization at puberty, herkaryotype is obtained to differentiate between the following conditions.

1. Female pseudohermaphroditism (46.XX).2. Mixed gonadal dysgenesis (45,X/46,XY).3. Incomplete Swyer syndrome (46.XY).4. Pseudovaginal perineoscrotal hypospadias (PPSH) (46,XY).

H. If a woman with ambiguous external genitalia develops virilization andgynecomastia at puberty, her karyotype is confirmed to be 46.XY. Then androgenbinding capacity of her skin fibroblasts is checked. It is normal in 17-betahydroxysteroid dehydrogenase deficiency in the gonad, while it is low in incompleteandrogen insensitivity.

I. Ambiguous external genitalia and absence of any change at puberty is due tofemale pseudohermaphroditism, though some such women develop virilization atpuberty (see under G).

The treatment of primary amenorrhea consists of the following.

1. Treatment of the cause, if possible.

2. Estrogen replacement therapy for hypogonadism, to be combined with a progestinfor 10 days in each cycle.

3. Gonadotropin therapy for hypogonadotropism.

4. Removal of the gonads if the karyotype is 46,XY or a mosaic with 46,XY, toprevent development of tumors in the gonads.

5. Plastic surgery to correct ambiguity of the external genitalia and permit sexualactivity.

3

Page 11: Differential Diagnosis and Management Options inuploads.worldlibrary.org/uploads/pdf/20180107071525differential... · 83 Peripartum cardiomyopathy 165 84 Hypertension 167 85 Convulsions

PRIMARY AMENORRHEA

Assess secondary sex characteristics

Normal (A) Absent Ambiguous external genitalia

Look for outflow tract obstruction Measure height Assess changes at puberty

Present Absent Short (C) Tall (D) Normal Virilization (D) Virilization plusgynecomastia (H)

None

Determine nature Assess for presenceof uterus (B)

Look for features ofTurner’s syndrome

Image gonadsKaryotype

FSH, LH assayKaryotype Karyotypee

Femalepseudo-

Imperforatehymen

Transversevaginalseptum

Present Absent

Karyotype karyotype

Streak gonad46,XX

Low (E) High (F) XX XY XO/XY XY hermap-hroditism

Vaginalatresia

Cervicalatresia

Pure gonadaldysgenesis

Karyotype Femalepseudo

Mixedgonadal

Androgen bindingcapacity of skin

Absent

Karyotype

Present

Evaluate as forsecondary amenorr-hea (see chapter 3)

Loss of shortarm of X-

chromosomeLook for otherabnormalities

Anosmia

Hermaphroditism

IncompleteSwyersyndrome

Dysgenesis

PPSHNormal

fibroblasts

Low

46,XX 46,XY 45,X 46,XXKallmansyndrome

Normal17 beta hydroxy

Abnormal Steroid dehydro-genase deficiency

Incompleteandrogen

insensitivityMullerianagenesis

Totalandrogen

insensitivity

Turnersyndrome

Noonansyndrome

XY X-isoGonadal damage

SwyerRadiation Rubella syndrome

Chromosome

17 α Hydroxylasedeficiency

Cushing’ssyndrome

Regionalenteritis

Ulcerativecolitis

Neurologicdisease

Obesity

Prader Willi

Chemotherapy

syndromeHypothyroidism Determine type Laurence Moon

Biedl syndrome Yes NoCheck family history Yes Familial

Encephalocele PituitaryTumor

Birth traumaRadiation Rubella

No

Congenital hydrocephalus Birth asphyxiaChemotherapy

Known medical disorder No Ovarianbiopsy

Yes SavageSyndrome

Ataxia telangiectasia Myotonia dystrophica Galactosemia Infiltrativedisease

4

Page 12: Differential Diagnosis and Management Options inuploads.worldlibrary.org/uploads/pdf/20180107071525differential... · 83 Peripartum cardiomyopathy 165 84 Hypertension 167 85 Convulsions

3. Secondary AmenorrheaA woman is said to have secondary amenorrhea when she has been menstruating atfirst, and then does not do so for a period equal to the duration of 3 of her previousmenstrual periods. It differs from primary amenorrhea in that occurrence of normalmenstruation indicates that her hypothalamic-pituitary-ovarian axis has developednormally, and she has had a normal uterus, at least until she developed amenorrhea.The first step is to exclude a pregnancy. Amenorrhea associated with hirsutism isdiscussed in chapter 32 and that associated with galactorrhea is discussed inchapter 33.

A. If the woman with secondary amenorrhea does not have hirsutism orgalactorrhea, she is given a progesterone challenge test. The test consists ofadministration of progesterone in oil 100 mg IM qd X 2 days, ormedroxyprogesterone acetate 10 mg PO qd X 3 to 5 days. If the woman bleedsvaginally within 7 days of the last dose of the progestin, the test is positive. Itindicates that she has a uterus with endometrium which has been primed withendogenous estrogen coming from the ovaries i.e. she has anovulation. See chapter35 for treatment of anovulation. The test may be false negative if the woman is veryobese (inadequate dose of the progestin) of if the progestin causes an LH surge andovulation (in which case menstruation takes place 2 weeks later).

B. If the progesterone challenge test is negative, an estrogen challenge testis given by administration of ethinyl estradiol 0.05 mg PO ql2h for 20 days.It may be combined with medroxyprogesterone acetate 10 mg PO qd for last10 days of the test to prevent heavy bleeding. It is often unnecessary if the amenorrheais of long duration. A negative test is absence of vaginal bleeding within 7 daysof the last day of drug administration. It indicates that the woman does not haveany endometrium capable of responding to estrogen (end organ defect). The causeis found out by performing a hysteroscopy, dilatation and curettage, and ifnecessary a laparoscopy. The cause may be any of the following.

1. Asherman syndrome: the endometrium is replaced by bands of adhesion(synechiae) between opposite endometrial surfaces. It is the result of a verythorough curettage in a case of a septic abortion, uterine packing to control atonicpostpartum hemorrhage, endometrial tuberculosis or chronic endometritis. See

chapter 4 for the management of genital tuberculosis and chapter 8 for that ofAsherman syndrome.

2. Genital tuberculosis: in early cases, there is tuberculous granulation tissue inthe place of the endometrium. In late cases, there is just fibrous tissue.

3. Chronic endometritis: even if it is treated adequately with antibiotics, theamenorrhea may persist, because the fibrous tissue cannot be replaced byendometrium again.

C. If the estrogen challenge test is positive, it indicates that the woman hasendometrium capable of responding to estrogen, but she has no endogenousestrogen (hypogonadism). In such a case, serum FSH and LH levels are assayed todistinguish between hypergonadotropic (ovarian failure) and hypogonadotropic(hypothalamic - pituitary failure) hypogonadism. FSH and LH levels are high incase of the former because the negative feedback at the level of the hypothalamuscaused by the ovarian sex steroids is absent. Loss of ovarian function may havebeen due to premature ovarian failure (menopause below the age of 35 years),autoimmune oophoritis, pelvic irradiation, cytotoxic drugs, or mumps oophoritis. Ifthe woman has other autoimmune endocrine disorders associated with the ovariandisease, she is treated appropriately. All women with ovarian failure are treatedwith estrogen replacement therapy up to the mean age at menopause. Forinfertility, adoption is the best solution. An alternative is oocyte donation, in vitrofertilization and embryo transfer.

If FSH and LH levels are low, a GnRH test is not done because up to 40doses of GnRH may be required to prime the pituitary so that it can respond. If thewoman has lost a lot of weight due to dietetic fads or anorexia nervosa, she istreated by counseling for the former and psychotherapy plus feeding for the latter.A debilitating disorder causing loss of weight is treated appropriately. Any stresscan act on the hypothalamus. Menstruation returns on relieving the stress. Heavyexercise causes release of beta-endorphins and amenorrhea. It is cured by limitingthe exercise. The woman may have serious disorders like Simmond's disease,Sheehan's syndrome (postpartum hemorrhage and pituitary necrosis), empty sellasyndrome, space occupying lesion in the sella turcica (pituitary adenoma,histiocytosis X, sarcoidosis), pineal gland tumor, and hypothalamic tumor. Properendocrine and neurologic evaluation and treatment are given for these conditions.

5

Page 13: Differential Diagnosis and Management Options inuploads.worldlibrary.org/uploads/pdf/20180107071525differential... · 83 Peripartum cardiomyopathy 165 84 Hypertension 167 85 Convulsions

Exclude pregnancySECONDARY AMENORRHEA

Assess for hirsutism

Present Absent

See chapter 32 Assess for galactorrhea

Present Absent

See chapter 33 Progesterone challenge test (A)

Positive Negative

Anovulation Estrogen challenge test (B)

Negative Positive

Hysteroscopy Laparoscopy Dilatation and curettage High

Serum FSH, LH assay (C)

Low

Ashermansyndrome

Genital tuberculosis Ovarian failure Assess weight loss

Chronic endometritis Assess cause Present Absent

Prematureovarian failure

Autoimmuneoophoritis

Pelvicirradiation

Assess cause Assess for stress

Present Absent

Antimalignancychemotherapy

Mumps oophoritis Medicaldisorder

Anorexianervosa

Diet History of heavy exercise

Present Absent

Neurologic assessment

Panhypopituitarism Sheehan’s syndrome Empty sella syndrome Pineal gland tumor Hypothalamic tumor

6

Page 14: Differential Diagnosis and Management Options inuploads.worldlibrary.org/uploads/pdf/20180107071525differential... · 83 Peripartum cardiomyopathy 165 84 Hypertension 167 85 Convulsions

4. Puberty MenorrhagiaMenorrhagia is not uncommon in an adolescent girl. A dictum about abnormal orexcessive bleeding at this age is that it should be considered to be dysfunctionalunless proved otherwise. However, it is a sound policy to rule out organic conditionsfirst, rather than consider it to be dysfunctional primarily..

A. A girl can inherit coagulation disorders, such as idiopathic thrombocytopenicpurpura (ITP), von Willebrand's disease, and other coagulation factor deficiencies.She will then have a history of excessive bleeding from cuts and wounds fromearly childhood. Onset of menstruation provides just another route from whichshe bleeds excessively. This problem is discussed further in chapter 155.

B. A bimanual pelvic examination is done rectally. If the uterus is enlarged, a pelvicultrasonography done. Uterine leiomyoma and less commonly a sarcoma botryoidesare not unknown. A leiomyoma requires myomectomy or high frequency focusedultrasound induced myolysis. A sarcoma botryoides requires abdominalhysterectomy with bilateral salpingo-oophorectomy, and possibly additionalchemotherapy. If an adnexal mass is felt on per rectal bimanual pelvic examination,an ovarian tumor is suspected. It is diagnosed by pelvic ultrasonography. Afunctioning (sex steroid producing) ovarian tumor is a rare cause of menorrhagia inadolescent girls. It is treated by oophorectomy.

C. Genital tuberculosis is a not uncommon cause of puberty menorrhagia. It isusually secondary to pulmonary tuberculosis, with a latent period of about 10 to12 years. The girl gives a history of a contact with tuberculosis. She may or maynot have been treated for pulmonary tuberculosis in childhood. Constitutional symptomsof tuberculosis are not very commonly seen in genital tuberculosis. Though amenorrheais the commonest symptom of the disease in adults, it can cause menorrhagia dueto endometrial hyperemia and ulceration in adolescence. If pelvic examination isnormal and coagulation profile reveals no abnormality, investigations are done formaking a diagnosis of this condition including hemogram (lymphocytosis, elevationof ESR), chest radiograph - PA view, Mantoux test (>10 mm), laparoscopy,hysteroscopy and endometrial curettage. The endometrium is studied histologicallyfor features of tuberculosis and by polymerase chain reaction for tuberculosis.Endoscopy and endometrial curettage are invasive tests, and are resorted to onlywhen there is a very strong suspicion of genital tuberculosis. The following tableshows various regimes for treatment of genital tuberculosis.

D. When organic conditions have been ruled out, a diagnosis of dysfunctionaluterine bleeding (DUB) is made. Tests are done to detect ovulation. If the girlhas dysmenorrhea, she is ovulating. The diagnosis is confirmed by follicularstudy with ultrasonography, which shows development of a group of ovarian follicles,faster growth and eventually rupture of a (dominant) follicle. Diagnosis of ovulationis discussed in more details in chapter 34.

The acute bleeding can be controlled by any of the following regimes.1. Tranexamic acid 500 mg PO q8h X 2-3 d.2. Combination contraceptive pills, 1 tablet q8-6h X 24-48 h.

Category-1Newly diagnosed sputum positive pulmonarytuberculosisSevere form of extrapulmonary tuberculosis

2(H3 R3 Z3 E3) 4 ( H3 R3 )

Category-2Treatment failure casesRelapse casesReturn after interruption

2( S3 H3 R3 Z3 E3)+ 1( H3R3 Z3 E3 ) 5 ( H3 R3 E3)

Category-3Sputum negative pulmonary tuberculosis withminimal involvementLess severe form of extrapulmonary tuberculosis

2( H3 R3 Z3 ) 4 ( H3 R3 )

(Note: H: INH 600 mg, R: Rifampicin 450 mg, Z: Pyrazinamide 1500 mg, E:Ethambutol 1200 mg, S: Streptomycin 750 mgThe number before parentheses indicates duration in months, the number aftersymbol for each drug indicates frequency per week.)

3. Norethisterone enanthate 5 mg q 8-6 h X 48 h.4. Medroxyprogesterone acetate 10 mg q8h X 48 h.5. Ethinyl estradiol 0.05 mg q8h X 48 h.

After the bleeding stops, the dose is reduced to 1 table ql2h for 15 to 18days. In case of ethinyl estradiol, medroxyprogesterone has to be added in a dose of10 mg qd X 10 d during the last 10 days of the treatment in order to achieve amoderate flow. It can be repeated in every cycle.

Recurrence of anovulatory DUB is prevented by cyclical administration ofnorethisterone enanthate or medroxyprogesterone acetate in a dose of 5 or 10 mg qdfor 10 days, starting from 15th day of the cycle. It is given for 3 to 6 cycles.Ovulatory DUB can be controlled similarly too. Similar results can be. obtained withthe use of combination contraceptive pills too, given 1 qd for 21 days, from secondday of the first cycle and then cyclically with a gap of 21 days between two cycles..However the estrogen present in these pills can hasten the closure of epiphyses andarrest of increase in height of the girl. Hence progestins are preferred. Generaltreatment includes hematinics, and in some cases blood transfusions.

7

Page 15: Differential Diagnosis and Management Options inuploads.worldlibrary.org/uploads/pdf/20180107071525differential... · 83 Peripartum cardiomyopathy 165 84 Hypertension 167 85 Convulsions

PUBERTY MENORRHAGIA

Present

History suggestive of inherited disorder of coagulation (A)

Absent

Bimanual pelvic examination per rectum (B)

Normal Adnexal mass Uterine enlargement

Check coagulation profile Pelvic ultrasonography Pelvic ultrasonography

Abnormal Normal Ovarian tumor Leiomyoma Sarcoma botryoides

See chapter 155Appropriate treatment

Evaluation for genital tuberculosis (C)

Present Absent

Dysfunctional uterine bleeding (D)

Antituberculous therapy Assessment for ovulation

Present Absent

Ovulatory DUB Anovulatory DUB

Cyclical hormone therapy Cyclical progestin therapy

8

Page 16: Differential Diagnosis and Management Options inuploads.worldlibrary.org/uploads/pdf/20180107071525differential... · 83 Peripartum cardiomyopathy 165 84 Hypertension 167 85 Convulsions

5. Menorrhagia inReproductive Age

A. The woman may be a known case of coagulation disorder e.g. idiopathicthrombocytopenic purpura or von Willebrand's disease. She can suffer from anacquired coagulation disorder too e.g. thrombocytopenia due to various causes(see chapter 156). She may be receiving anticoagulants like warfarin or heparin forconditions like a prosthetic cardiac valve or deep vein thrombosis. The presence of acoagulation disorder is diagnosed by performing coagulation studies (see chapter155). Any such condition is treated appropriately.

B. A speculum examination is done to diagnose or rule out a lesion on the cervixor vagina as the cause of excessive bleeding. Such lesions include cervical polyp,cervical adenosis, cervical ulcers, cervical carcinoma, vaginal adenosis, vaginalcarcinoma, endometriosis of the rectovaginal septum spread to the posterior vaginalfornix, trauma, and metastasis of a choriocarcinoma to the vagina (posterior vaginalfornix or under the urethra).

C. If the uterus is found to be enlarged on a bimanual pelvic examination, itsshape is assessed. If it is multinodular and irregular, the diagnosis is uterine leiomyomas.If it is uniform, an ultrasonography is done. It may reveal a single large leiomyoma.If a woman with uterine leiomyoma(s) is above 40 and has completed her family,a total hysterectomy is done. If she is below 40 and has not completed her family,myomectomy is done (see chapter 51). Other options include drug therapy withGnRH analogues, selective uterine artery embolization, and MRI or ultrasoundguided high frequency focused ultrasonic myolysis. Uniform enlargement of theuterus may also be due to adenomyosis, endomyometrial hyperplasia, orsubinvolution of the uterus. The distinction between the three is possible bytransvaginal ultrasonography. If the patient presents with severe bleeding, she isgiven tranexamic acid 500 gm PO q8h with/without mefenamic acid 500 mg q8huntil bleeding stops. If it does not stop or reduce significantly in 48 hours, she mayneed a cervical dilatation and endometrial curettage to control it. Recurrence ofmenorrhagia can be prevented by cyclical progestin therapy (see chapter 4), but itmay not be as effective as in the management of dysfunctional uterine bleeding. Ifthe woman is above 40 and the hormone therapy fails, she is offered an option

between endometrial ablation (with a resectoscope, Nd-YAG laser, radiofrequencythermal ablation, or thermal balloon ablation) and total hysterectomy. Endometrialablation may not be curative in cases of adenomyosis, because there is extension of theendometrium into the myometrium which cannot be reached by ablation.

D. If the uterus is not enlarged and the coagulation studies are normal, herthyroid status is evaluated. If she is clinically euthyroid, further tests may notbe warranted. Serum T3, T , and TSH levels may be obtained. Hypothyroidismusually causes menorrhagia, while hyperthyroidism causes oligomenorrhea. Howeverit could be the other way too. Any disturbance of thyroid function is treatedappropriately.

E. If the thyroid function is normal, local endometrial conditions are sought.The investigation of choice is hysteroscopy and biopsy (if a lesion is found).If that is not possible, cervical dilatation and endometrial curettage are done.Various abnormalities which can be detected are as follows.

1. Proliferative endometrium: it is found in anovulation. The treatment is cyclicalprogestin therapy (see chapter 4) if the woman wants just control of menorrhagia,combination contraceptive pills if she wants contraception as well, and induction ofovulation with clomiphene citrate and other drugs (see chapter 35) if she desires apregnancy.

2. Uncontrolled endometrial stimulation with estrogen not countered by cyclicalprogestin therapy can lead to cystic glandular hyperplasia, adenomatous hyperplasia,and atypical hyperplasia. Untreated, these conditions progress to endometrialcarcinoma in 13, 26, and 78% cases respectively. If the woman is young, she istreated as for proliferative endometrium. If she is above the age of 40 and hascompleted her family, she treated by total hysterectomy.

3. A polyp (endometrial, leiomyomatous, or placental) is treated by polypectomy ifpossible, using polyp forceps at the time of a curettage. A more precise method is toremove it hysteroscopically, using a resectoscope or Nd-YAG laser.

4. Endometrial carcinoma : see chapter 44.

5. Choriocarcinoma: see chapter 47.

6. Uterine sarcoma: see chapter 45.

9

Page 17: Differential Diagnosis and Management Options inuploads.worldlibrary.org/uploads/pdf/20180107071525differential... · 83 Peripartum cardiomyopathy 165 84 Hypertension 167 85 Convulsions

MENORRHAGIA IN REPRODUCTIVE AGE

History of coagulation abnormality (A)

Absent Present

Speculum examination (A)

Local lesion Normal

Bimanual pelvic examination (B)

Uterus enlarged (C) Uterus not enlarged

Nonuniform, multiplenodules

Uterineleiomyomas

Uniform

Ultrasonography

Coagulation studies

Normal results Coagulopathypresent

(see chapter 155)

Singleleiomyoma

Adenomyosis Endomyometrialhyperplasia

SubinvolutionEvaluation of

thyroid function (D)

Normal Abnormal

Hysteroscopy and endometrial biopsy orDilatation and curettage (E)

Proliferativeendometrium

Cysticglandular

hyperplasia

Adnomatoushyperplasia

Atypicalhyperplasia

Polyp Endometrialcarcinoma

Choriocarcinoma Sarcoma

Anovulation See chapter 44 See chapter 47

10

Page 18: Differential Diagnosis and Management Options inuploads.worldlibrary.org/uploads/pdf/20180107071525differential... · 83 Peripartum cardiomyopathy 165 84 Hypertension 167 85 Convulsions

6. Postmenopausal VaginalBleeding

A woman is said to be menopausal when her menstruation ceases alongwith failure of ovarian function after the age of 35 years. However the average age atmenopause is 45 to 50 years, and not 35. Climacteric is the phase of the agingprocess in which the woman passes from the reproductive stage to the nonreproductivestage. Premenopause is that part of climacteric before menopause in which themenstrual cycle is likely to be irregular, and climacteric symptoms are likely to beexperienced. Postmenopause is that phase of life which follows the menopause. Awoman normally does not bleed vaginally after menopause. If she does, the causemust be considered to be organic until proved otherwise.

A. Hormone replacement therapy is given after the menopause for menopausalsymptoms or prevention or treatment of osteoporosis. Withdrawal bleeding occurswith such treatment. If the treatment has been combined with progestin therapyfor at least 10 days in every cycle, reassurance is all that is required. Endometrialaspiration should be done once every year to confirm that the endometrium doesnot develop hyperplasia or carcinoma.

B. A speculum examination of the lower genital tract reveals local causes ofpostmenopausal vaginal bleeding, such as the following.

1. Atrophic vagina: it is due to deficiency of estrogen. It is quite thin and sometimesbleeds readily.

2. Trauma: even if the woman has had coitus successfully all her adult life, she candevelop vaginal tears (usually in the posterior fornix) during vigorous coitus aftermenopause. It is due atrophy of the vagina. She often does not volunteer thisinformation out of embarrassment. The bleeding stops on placement of hemostaticsutures of No. 2-0 or 1-0 polyglactin. Colporrhexis is not very common in suchcases. If present, it needs to be repaired.

3. Foreign body: if the woman has had a ring pessary inserted for control of genitalprolapse and then forgotten about it, she may develop vaginal bleeding due toulceration of the vaginal mucosa. Removal of the pessary and antiseptic douchescure the condition.

4. Forgotten intrauterine contraceptive device (IUCD): a woman may forget to haveher IUCD removed after menopause. It may not cause any symptoms for years.Sometimes it causes endometrial ulceration and bleeding. Such an IUCD should beremoved. A fractional curettage should still be done to rule out an endometrial

carcinoma.

5. Cervical cancer : see chapter 42.

6. Vaginal cancer.

C. If the speculum examination reveals no abnormality other than blood escapingfrom the cervix, a bimanual pelvic examination is done. If the uterus is enlarged,pelvic ultrasonography is done to find out the cause of the same. A fractionalcurettage is done, obtaining endocervical and endometrial tissue separately. Thediagnosis of either endometrial carcinoma or uterine sarcoma is thus made. Presenceof an adnexal mass suggests the presence of an ovarian tumor, such as a granulosacell tumor which produces estrogen and causes postmenopausal bleeding. It cancause endometrial carcinoma too. It is treated by total abdominal hysterectomyand bilateral salpingo-oophorectomy.

D. If bimanual pelvic examination reveals no abnormality, a hysteroscopy isdone, and a biopsy is obtained from a suspicious area. If that is not possible,a fractional curettage is done. Various histological appearances of the materialremoved are as follows.

1. Secretory: it is due to ovulation of a stray follicle which had been left behind inthe ovary.

2. Atrophic.3. Proliferative.4. Simple without atypia5. Complex without atypia6. Simple with atypia7. Complex with atypia8. Endometrial carcinoma.

Women with secretory or atrophic endometrium can be left alone, beingasked to follow up in the event of a recurrence of the bleeding. Those withproliferative endometrium or simple hyperplasia without atypia are given progestintherapy cyclically for 6 months, stopping it earlier if there is no withdrawal bleeding(which signifies that the endogenous estrogen production has ceased). Those withcomplex hyperplasia without atypia, simple hyperplasia with atypia or complexhyperplasia with atypia are offered the option of hysterectomy in view of the risk ofdevelopment of endometrial carcinoma. If they decline, progestin therapy is givencontinuously for 6 months. A fractional curettage is then repeated. If the report isnormal, a close follow up is maintained, performing endometrial aspiration everyyear. If the hyperplasia is found to persist, total laparoscopic or abdominalhysterectomy with bilateral salpingo-oophorectomy is done. See chapter 44 forendometrial carcinoma.

11

Page 19: Differential Diagnosis and Management Options inuploads.worldlibrary.org/uploads/pdf/20180107071525differential... · 83 Peripartum cardiomyopathy 165 84 Hypertension 167 85 Convulsions

POSTMENOPAUSAL VAGINAL BLEEDING

History of estrogen therapy (A)

Yes No

Endometrial sampling Speculum examination (B)

Atrophic vagina Trauma Foreign body Normal Threads of IUCD Cervical cancer Vaginal cancer

See chapter 42.

Bimanual pelvic examination (C)

Uterus enlarged Ovarian tumor No abnormality

Endometrialcarcinoma

Uterine sarcoma Hysteroscopy plus biopsy or fractional curettage (D)

Secretoryendometrium

Atrophicendometrium

Proliferativeendometrium

Simplehyperplasia

with/withoutatypia

Complexhyperplasia

without atypia

Simple orcomplex

hyperplasia withatypia

Endometrialcarcinoma

See chapter 11 too. See chapter 44.

Page 20: Differential Diagnosis and Management Options inuploads.worldlibrary.org/uploads/pdf/20180107071525differential... · 83 Peripartum cardiomyopathy 165 84 Hypertension 167 85 Convulsions

7. MetrorrhagiaMetrorrhagia is acyclical or intermenstrual vaginal bleeding. Though not

excessive in amount, it can be quite disturbing to a woman, because it interferes withher social, religious, and sexual aspects of her life. It is of concern to a gynecologisttoo, because it can be due to serious gynecologic disorders.

A. A woman receiving hormone therapy in the form of combination contraceptivepills, minipill, injectables, implants or vaginal ring can have a breakthroughbleeding. If gynecologic examination reveals no abnormality, and the bleedingis occasional or mild, reassurance is sufficient. If she is bothered by her symptoms,her hormone therapy is changed. In case of combination contraceptive pills, apill with a lower progestational action has to be given. In case of progestationaltherapy, either an estrogen is added to control progesterone breakthrough bleeding,or another form of contraceptive is used.

B. If the woman is not receiving any hormone therapy, a gynecologic examinationis done to find out the source of the bleeding. Blood is seen escaping from theexternal os if the bleeding is of uterine origin. In that case, pelvic ultrasonographyis done. It may reveal any of the following conditions.

1. IUCD: it is treated with hormone therapy (see chapter 5). If the metrorrhagia stillpersists, the IUCD has to be removed.

2. Threatened abortion: the uterus is enlarged to a size appropriate for the period ofamenorrhea, and the cervix is closed. Fetal cardiac activity is seen on ultrasonographyafter 6 weeks of gestation. See chapter 69 for its management.

3. Incomplete abortion: the uterus is enlarged, soft, and the products of conceptionmay be felt through the open cervix. See chapter 69 for its management.

4. Ectopic pregnancy: the bleeding is due to hormone withdrawal, or passage ofpelvic blood to the uterine cavity through the fallopian tube. A pregnancy test onurine is usually positive. The uterine cavity is found to be empty on ultrasonography,while extrauterine pregnancy may or may not be seen. See chapter 71 for thediagnosis and treatment of ectopic pregnancy.

5. Leiomyomatous polyp: a submucous pedunculated leiomyomatous polyp mayproject into the vagina from the cervix. It is congested and its dependent portion maybe ulcerated. The diagnosis is evident on speculum examination. The treatment ispolypectomy. It is quite simple if the widest diameter of the polyp has passed outthrough the cervix. If the pedicle of the polyp can be reached, it is clamped, cut, andligated with a transfixion ligature of No. polyglactin. If it cannot be reached, thepolyp is held with a bulldog vulsellum, a tenaculum, or an Allis' forceps and it is

twisted until the pedicle breaks. If it bleeds, a sharp curettage is done, so that theresidual pedicle is removed and the blood vessels in it retract into the myometriumand stop bleeding. If the widest diameter of the polyp has not passed through thecervix, polypectomy cannot be done. In such cases, if the cervix is dilated to lessthan 1 cm, hysteroscopic resection of the polyp is possible. If it is dilated to morethan that, one has to wait for further dilatation of the cervix so that the polyp getsextruded. Alternatively one may perform a posterior cervicotomy, remove the polypand then suture the cervix with interrupted sutures of No. 1 polyglactin.

6. Endometrial polyp: it is a soft polyp projecting out of the cervix. Its dependentportion is ulcerated. It is treated by polypectomy followed by dilatation andcurettage.

C. A lesion may be found on the cervix on speculum examination. The followingconditions can cause metrorrhagia.

1. Erosion (see chapter 22).

2. Polyp.

3. Adenosis : it is seen with intrauterine exposure to diethyl stilbesterol. It isnecessary to rule out cervical adenocarcinoma by Pap smear, colposcopy, andbiopsy. The treatment of cervical adenosis is as that of a cervical erosion (seechapter 22).

4. Cancer (see chapter 42).

D. Any of the following lesions may be found on the vagina.

1. Trauma: it is usually due to forceful coitus, as in rape. Vagina of a lactatingwoman is often atrophic due to hypoestrogenism, and may tear during coitus.Another cause of vaginal trauma is insertion of a pointed foreign body into vagina.See chapter 6 for management of vaginal trauma.

2. Foreign body: a ring, Smith, or Hodge pessary may be forgotten in the vagina. Itcauses vaginal mucosal ulceration and bleeding. It is removed and the local infectionis controlled with antiseptic vaginal douches. The ulcers heal readily and thebleeding stops.

3. Adenosis: vaginal adenosis may develop with intrauterine exposure to diethylstilbesterol. It is managed like cervical adenosis.

4. Carcinoma of the vagina: it is an extremely rare cancer affecting a woman. Itmay be an excavating ulcer or an exophytic growth. It is managed by radical surgeryor radiotherapy, depending on its location and stage.

13

Page 21: Differential Diagnosis and Management Options inuploads.worldlibrary.org/uploads/pdf/20180107071525differential... · 83 Peripartum cardiomyopathy 165 84 Hypertension 167 85 Convulsions

METRORRHAGIA

Check history of receiving hormone therapy (A)

Yes No

Clinical examination Clinical examination

Normal Abnormal

Breakthroughbleeding

Check source of bleeding

Uterine (B)

Ultrasonography

Cervical (C) Vaginal (D)

Erosion Polyp Adenosis Cancer

IUCD Threatenedabortion

Incompleteabortion

Ectopicpregnancy

Trauma Foreignbody

Adenosis Cancer

Leiomyomatouspolyp

Endometrial polyp

14

Page 22: Differential Diagnosis and Management Options inuploads.worldlibrary.org/uploads/pdf/20180107071525differential... · 83 Peripartum cardiomyopathy 165 84 Hypertension 167 85 Convulsions

8. HypomenorrheaHypomenorrhea is scanty menstruation, in the form of only a drop or two

(staining of the pad). It can be quite distressing to a woman, because many womenbelieve that the menstrual blood contains impurities from the body, and if the flow isnot satisfactory, these impurities get retained. Reassurance and scientific explanationare enough to expel this myth. However hypomenorrhea needs investigation becausethe underlying condition may be quite serious.

A. If the woman is on combination contraceptive pills, she may get scanty withdrawalbleeding due to predominant progestational action. She should be given anotherpill with less progestational action. If she gets satisfactory flow after that, sheshould continue to use the same pill subsequently.

B. If she is not taking any combination pills, a history of progestin administrationin any form should be sought. Progestins tend to produce endometrial atrophyand scanty withdrawal bleeding. If the menstrual flow improves on stoppingprogestin administration, she is counseled and advised to continue progestin therapyif it is essential for her. If she was using progestin only contraception and desiresgood menstrual flow, she is offered alternative form of contraception so that themenstruation improves.

C. If there is no improvement after changing combination pills, or in case ofwomen who are not receiving any hormones, the patient is checked for galactorrheaand her serum thyroid stimulating hormone (TSH) and prolactin (PRL) levels arechecked. If there is thyroid dysfunction or hyperprolactinemia, it is managedappropriately. If these hormone levels are normal, the woman is given an estrogenchallenge test. She is given ethinyl estradiol 0.05 mg PO ql2h for 20 days. Ifshe gets satisfactory flow, she has endometrium capable of responding to estrogenbut is not menstruating well because of hypoestrogenism. If hypomenorrhea persists

after the estrogen challenge test, her endometrium is incapable of responding toestrogen.

D. Hypomenorrhea not responding to estrogen is evaluated by hysteroscopy.It may show any of the following conditions.

1. Asherman syndrome: there are bands of adhesion between opposite endometrialsurfaces. It is due to uterine packing after a childbirth or abortion to controlbleeding, thorough curettage in a case of septic abortion, genital tuberculosis, orchronic endometritis. These bands are lysed with either scissors, or Nd-YAG laserduring hysteroscopy. Then a Lippes' loop is inserted into the uterine cavity toprevent approximation of the opposite endometrial surfaces, and reformation of theadhesions. High dose estrogen therapy (ethinyl estradiol 0.05 mg ql2h) is given for 3months to stimulate endometrial growth.

2. Atrophic endometrium.

3. Fibrosis.

E. Dilatation and curettage is done if the endometrium is atrophic or if thereis fibrosis. Histopathological examination of the material removed may showany of the following.

1. Endometrial tuberculosis : see chapter 4.

2. Chronic endometritis: it is treated with broad spectrum antibiotics and cyclicalestrogen therapy to stimulate endometrial growth.

3. No material : no endometrial tissue is obtained if it has been replaced by fibroustissue as a result of chronic inflammation. The prognosis in such cases is quiteguarded. There is often no response to estrogen therapy, and finally there isocclusion of the endometrial cavity by adhesions (Asherman syndrome).

15

Page 23: Differential Diagnosis and Management Options inuploads.worldlibrary.org/uploads/pdf/20180107071525differential... · 83 Peripartum cardiomyopathy 165 84 Hypertension 167 85 Convulsions

HYPOMENORRHEA

Check for use of combination contraceptive pills (A)

Yes No

Change to lower progestin content pill Check for use of progestin therapy (B)

Satisfactory flow Hypomenorrhea No Yes

Check for galactorrhea, check serum TSH, PRL (C) Stop progestin therapy

Estrogen challenge test Absent Present Satisfactory flow

Satisfactory flow Hypomenorrhea See chapter 33

Hypoestrogenism Hysteroscopy (D)

Ashermansyndrome

Atrophicendometrium

Fibrosis

Dilatation and curettage

Genitaltuberculosis

Chronicendometritis

No material

16

Page 24: Differential Diagnosis and Management Options inuploads.worldlibrary.org/uploads/pdf/20180107071525differential... · 83 Peripartum cardiomyopathy 165 84 Hypertension 167 85 Convulsions

9. Precocious Vaginal BleedingA. The appearance of secondary sexual characteristics below the age of 8 yearsor onset of menstruation below the age of 9 years is considered precocious.If a girl presents with vaginal bleeding, it is not necessarily menstrual blood.The first step in her evaluation is assessment of development of secondary sexcharacteristics. If those are absent, a local cause of bleeding is a strong possibility,unless the girl has accidentally ingested estrogen tablets. If there is no suchhistory (and even if there is), local examination is done. If there is no causeexternally visible, a vaginoscopy is done. Various local causes responsible for suchbleeding are vulvovaginitis, foreign body in the vagina, local trauma (accident orassault), vulvar skin disorders, acute urethral prolapse, adenocarcinoma of the cervixor vagina, and botryoid sarcoma of the uterus (usually arising from the cervix at thisage). If any such cause is found, it is treated appropriately. If there is no cause, thediagnosis is precocious menarche. But it can be diagnosed only by exclusion of allother causes.

B. Prolonged administration of estrogen orally can produce development of breastsand menstrual bleeding. It should be stopped. General examination and abdominalplus pelvic examination often reveal causes of precocious puberty when the girlhas development of secondary sex characteristics, examples of which are as follows.

1. McCune Albright syndrome : the girl has cafe-au-lait spots on the skin,polyostotic fibrous dysplasia, and precocious puberty. Pathologic fractures mayoccur. The prognosis is unfavorable. Adult height is significantly reduced due to bothearly closure of epiphyses and pathologic bone fractures. There are menstrualabnormalities and frequently sterility.

2. Follicular cyst of the ovary: see chapter 18.

3. Ovarian tumor: a benign teratoma or cystadenoma may produce estrogen andcause precocious puberty. A granulosa cell tumor is less common in this age groupgirls.

4. Hepatoma: it causes hepatomegaly, and feminization. The diagnosis is confirmedby hepatic ultrasonography, liver scan, and a percutaneous biopsy. Such a girl isreferred to a surgeon for further management.

C. Serum levels of FSH, LH, and estradiol are assayed. They are in the adult range ifthe girl has GnRH dependent precocious puberty, and low in GnRH independentprecocious puberty. A GnRH stimulation test is done, in which 100 µg of GnRH isadministered intravenously. In case of GnRH dependent precocious puberty, LHrises to twice the baseline at 60 minutes after administration in the proliferativephase and 4 to 8 times in the luteal phase of the menstrual cycle. If the girl hasGnRH dependent precocity, she is examined neurologically. Even if no grossabnormalities are seen, a CT-scan of the head is obtained. Neurological disordersassociated with sexual precocity include hypothalamic hamartoma, optic glioma,other CNS tumors, and neurologic effects of head injury and radiation. If she has anysuch disorder, she is referred to a neurologist for further management.

D. If there is no neurologic disorder, thyroid function tests are done. Low levels ofT3 and T4, and high level of TSH suggest juvenile hypothyroidism. Such a girl isreferred to an endocrinologist for further treatment. If there is no abnormality in thethyroid function, a family history of sexual precocity is sought. If it is obtained, thediagnosis is familial sexual precocity. If it is absent, it is idiopathic sexual precocity.It is due to activating mutations in the KISS1R and KISS1 genes, inactivatingmutations in the MKRN3 gene or chromosomal abmormalities. Molecular geneticstudies help make this diagnosis. If positive, genetic counseling is done.

Besides treatment of the cause, GnRH dependent precocious puberty istreated also with a GnRH analog (GnRHa). GnRHa down-regulates receptors andinduces a temporary, reversible inhibition of hypothalamo-pituitary-ovarian axis.Thus the girl develops hypogonadotropic hypogonadism, amenorrhea, andregression of secondary sex characteristics. Increase in height continues. At thecorrect age of puberty, GnRHa administration is stopped, so that the girl achievespuberty. Treatment with drugs like danazol and medroxyprogesterone acetate is notas satisfactory as with a GnRHa.

17

Page 25: Differential Diagnosis and Management Options inuploads.worldlibrary.org/uploads/pdf/20180107071525differential... · 83 Peripartum cardiomyopathy 165 84 Hypertension 167 85 Convulsions

PRECOCIOUS VAGINAL BLEEDING

Assess secondary sex characteristics

Absent (A) Present

Check history of ingestion of sex steroids Clinical examination (B)

Present Absent Café-au-lait spots Adnexal mass Hepatomegaly Normal Prolongedestrogen

administration

Stop administration Assess local causes bylocal examinationand vaginoscopy

McCuneAlbright

syndrome

Pelvic ultrasonography Hepatoma Serum FSH, LH,estradiol assay (C)GnRH stimulation test Stop administration

Present Absent Ovarianfollicular cyst

Ovarian tumor Pubertal range Prepubertal range

Determine nature Premature menarche Neurological assessment: clinical, CT scan Abdominal CT scan

Trauma Vulvovaginitis Hypothalamichamartoma

Neurofibroma Head injury Adnexal adenoma

Normal

Hepatoma

Urethralprolapse

Botryoidsarcoma

Optic glioma Other CNS tumors Radiationeffect

Thyroid function tests (D)Foreign body Vulvar skin disorders

Juvenile hypothyroidismNormal Check family history

Adenocarcinoma of cervix or vagina Present

Familial

Absent

Idiopathic

18

Page 26: Differential Diagnosis and Management Options inuploads.worldlibrary.org/uploads/pdf/20180107071525differential... · 83 Peripartum cardiomyopathy 165 84 Hypertension 167 85 Convulsions

10. Premenstrual SyndromeA. Premenstrual syndrome (PMS) is a psychoneuroendocrine disorder with alarge number of symptoms. It occurs most often in late 20s to early 30s, andrarely in adolescents. Up to 90% women suffer from some recurrent PMS symptoms,20 to 40% are physically or mentally handicapped to some degree, and 5% havesevere distress. Commonest features are edema, weight gain, restlessness, irritability,and tension. Other symptoms include abdominal discomfort, bloating, headache,breast tenderness, mood swings, sleep changes, clumsiness, etc. Rather than relyingon the patient’s memory, the symptoms are recorded prospectively using a symptomdiary over two cycles. When that does not help establish a diagnosis of PMS, GnRHanalogues can be used to make the diagnosis of PMS.

B. The cyclicity of the symptoms is checked for. If there is no cyclicity and nosymptom-free interval in between, it is not PMS. Psychiatric referral is made then.For a diagnosis of PMS to be made, the following criteria should be satisfied.1. The symptoms should occur in the 5 days before her period for at least threemenstrual cycles in a row.2. The symptoms should end within 4 days after her period starts.3. The symptoms should interfere with some of her normal activities and be severeenough to require medical help.

C. When the diagnosis of PMS is confirmed, predominant symptom is checked. Ifthe condition is mild and quality of life is not affected, it is physiologicalpremenstrual symptom which requires just reassurance. If the woman has breasttenderness and/or engorgement, serum prolactin (PRL) level is estimated during thesymptomatic phase of PMS. If it is found to be elevated, she is treated withbromergocriptine (see chapter 33). If the level is normal, general measures are taken.If the woman suffers from weight gain and bloating, symptomatic treatmentcan be given with a diuretic like spironolactone. The dose is 25 mg PO qd (inthe morning). However it does not cure the condition, and is effective only aslong as it is administered. If the woman has dysmenorrhea, menorrhagia or headache,an NSAID is given. If there is no response, general measures are taken. If the womanhas mainly psychiatric symptoms like irritability, tension,restlessness, clumsiness, accident proneness, sleep changes, mood swings, andsocial withdrawal, psychiatric evaluation is done. It must be kept in mind thatpsychiatric illnesses have premenstrual exacerbations. If the woman is foundto have an underlying psychiatric illness, it is treated appropriately. Depression

is the commonest condition associated with PMS. It responds well to antidepressantdrugs. Psychiatric measures include, besides drug therapy, cognitive behavioraltherapy, individual and family therapy and self-help support group therapy. If thereis no psychiatric abnormality, general measures are adopted as discussed before.

D. General measures for control of PMS include the following.1. Dietary changes: low fat, low salt diet with complex carbohydrates is given, avoiding caffeineand alcohol. Eating schedule is changed to six small meals a day rather than three large meals, oreating smaller three meals and three light snacks.2. Exercise: aerobic.3. Adequate sleep at night.4. Cessation of smoking.5. Dietary supplements: vitamin B6 50-100 mg PO qd, calcium carbonate 1200 mg POqd, vitamin D, magnesium 360 mg PO qd are useful. Vitamin A, evening primroseoil, soy products and red clover are not found to be useful.6. Relaxation, meditation, anger management.

E. If the general measures fail to control PMS, a trial of antidepressants is given.Drugs commonly used are citalopram, escitalopram, fluoxetine , sertraline andparoxetine. Treatment is given continuously, though therapy only during the last 12days of cycle may be adequate and associated with less adverse effects. When usedcontinuously, discontinuation should be slow to avoid withdrawal symptoms. Thesedrugs have to be discontinued before planning and during a pregnancy.

F. If antidepressant therapy fails, a trial of anxiolytic therapy is given. Buspirone andbenzodiazepines are found to be useful.

G. If anxiolytic therapy fails, a trial of oral combination contraceptive pills isgiven. Pills containing 30 µg of ethinyl estradiol and drospirenone are used.Monophasic pills are preferred to bi- or triphasic pills. Continuous administration ispreferred to cyclic administration. Percutaneous estradiol combined with cyclicalprogestin is also effective.

H. If oral combination contraceptive pills fail, GnRH analogue is used to suppressovarian function. If it controls PMS, it can be continued for up to 3-6 months. Butlonger treatment is not recommended for fear of adverse effects. If it has to be usedfor more than 6 months, add-back therapy (combined estrogen-progesterone pills ortibolone) is required. In such cases, total hysterectomy with bilateral salpingo-oophorectomy may be advised. If GnRH therapy fails, the PMS is consideredresistant and total hysterectomy with bilateral salpingo-oophorectomy is advised.

19

19

Page 27: Differential Diagnosis and Management Options inuploads.worldlibrary.org/uploads/pdf/20180107071525differential... · 83 Peripartum cardiomyopathy 165 84 Hypertension 167 85 Convulsions

PREMENSTRUAL SYNDROME (A)

Check cyclicity of symptoms (B)

Cyclical, with symptom free interval in between Non-cyclical

Check predominant symptom (C) Psychological, not PMS

Mild condition, qualityof life not affected.

PMS, withprogestin therapy

Breast symptoms Weight gain, bloating

Spironolactone

Headache Irritability, tension, restlessness

NSAID Psychiatric evaluation

Physiological

Dysmenorrhea, menorrhagia

Stop progestin Serum prolactin assayNo response Good response Normal Abnormal

NSAIDHigh Normal Appropriate therapy

Bromergocriptine General measures (D)

Good response No response Good response No response

Antidepressant (E)

Good response No response

Anxiolytic therapy (F)

Good response No response

Oral contraceptive pills (G)

Good response No response

Good response GnRH (H) No response

Total hysterectomy with bilateral salpingo-oophorectomy Resistant PMS

20

Page 28: Differential Diagnosis and Management Options inuploads.worldlibrary.org/uploads/pdf/20180107071525differential... · 83 Peripartum cardiomyopathy 165 84 Hypertension 167 85 Convulsions

11. Suspicion of EndometrialHyperplasia

A. Endometrial hyperplasia (EH) is non-physiological, non-invasive proliferationof the endometrium which results in increase in volume of endometrial tissue,alterations of glandular shape and size and endometrial gland:stroma ratio of greaterthan 1. EH is a known precursor for endometrial adenocarcinoma. It occurs due tostimulation of the endometrium by estrogen which is not countered cyclically byprogestational action. Hence it should be suspected when a woman has chronicanovulation, polycystic ovarian disease, estrogen replacement therapy (not combinedcyclically with progestin therapy), tamoxifen therapy and menorrhagia. Pelvicfindings are not abnormal, except that in some cases the uterus may be bulky. Atransvaginal ultrasonography (USG) is done to detect EH. If endometrial thickness isless than 3 mm and 7 mm in post- and premenopausal women respectively, there isno EH. Besides endometrial thickening, other abnormalities suggesting presence ofEH on USG are increased vascularity, inhomogeneity of endometrium andparticulate fluid in the uterine cavity. EH may also be suspected when endometrialcells are seen in Pap smear. Endometrial aspiration cytology is useful to detect EHand screen for endometrial malignancy. It may be done with Isaac's cannula, Gravleejet washer or Mimark helix. Computerized tomography, diffusion-weighted magneticresonance imaging and biomarkers are not recommended as standard tests fordetection of EH.

B. The diagnosis is confirmed by an outpatient endometrial biopsy (EB). If it is notadequate, a hysteroscopy directed biopsy or dilatation and curettage is done.Hysteroscopy shows dark purple-red surface of the endometrium, with polypoidprojections. Gland openings are seen as indentations with a yellowtint. Even minor contact with the outer sleeve of the sheath causes endometrialbleeding or detachment of long strips of endometrium. On histopathology, two typesof hyperplasia are detected (WHO, 2014) - hyperplasia without atypia and atypicalhyperplasia.C. In case of EH without atypia, risk factors for development of EH are treated.The woman is counseled that the risk of EH without atypia progressing toendometrial cancer is less than 5% over 20 years and that it regressesspontaneously in the majority of cases. Use of progestin therapy hastens the

process of regression. Insertion of levonorgestrel IUCD is recommended. If that isnot possible due to contraindications for an IUCD, financial reasons or intolerance,then continuous oral progestin therapy is given (medroxyprogesterone 10–20 mg/dayor norethisterone 10–15 mg/day). Cyclical progestin therapy is not recommended asit is less effective than continuous therapy. In both cases transvaginal USG and (EB)are done every 6 months. If EH regresses, and there is no abnormal uterine bleeding,IUCD is continued for 5 years and oral progestins are continued for 6 months. EB isrepeated after 6 months. If two EBs done 6 months apart are negative and body massindex is below 35, further follow up is not necessary. But if it is 35 or more, EB isrepeated every year. In case of persistence of EH, relapse of EH after stoppingtreatment, development of atypia and abnormal uterine bleeding, total hysterectomywith bilateral salpingo-oophorectomy is done. Ovaries may be preserved inpremenopausal women on individual basis. Endometrial ablation is notrecommended because complete destruction of the endometrium cannot beensured and development of intrauterine adhesions may make follow up EBsdifficult or nonsatisfactory..

D. In case of atypical hyperplasia, if the woman is premenopausal and needs topreserve childbearing function, she is counseled about the risk of development ofendometrial carcinoma with conservative treatment. If she wishes to haveconservative treatment, insertion of levonorgestrel IUCD is recommended. If that isnot possible due to contraindications for an IUCD, financial reasons or intolerance,then oral progestin therapy is given. In both cases transvaginal USG and (EB) aredone every 3 months. If EH regresses, and there is no abnormal uterine bleeding,IUCD is continued for 5 years and oral progestins are continued for 6 months. EB isrepeated after 3 months. Twice. If both show absence of atypical hyperplasia, EB isdone every 6-12 months. In case of persistence of atypical hyperplasia, relapse ofatypical hyperplasia, abnormal uterine bleeding and in postmenopausal women totalhysterectomy with bilateral salpingo-oophorectomy is done. Ovaries may bepreserved in premenopausal women on individual basis. Lymph node dissection isnot recommended. Endometrial ablation is not recommended because completedestruction of the endometrium cannot be ensured and development ofintrauterine adhesions may make follow up EBs difficult or nonsatisfactory.

Regression of the disease should be achieved before the patient attempts to conceiveas it is associated with higher pregnancy rates. Assisted reproduction may bepreferred to natural conception as it achieves higher live birth rate is and it mayprevent relapse of the disease.

21

Page 29: Differential Diagnosis and Management Options inuploads.worldlibrary.org/uploads/pdf/20180107071525differential... · 83 Peripartum cardiomyopathy 165 84 Hypertension 167 85 Convulsions

Clinical features, basic investigations (A)

SUSPICION OF ENDOMETRIAL HYPERPLASIA

Hysteroscopy, Dilatation and curettage (B)

Absence of endometrial hyperplasia Endometrial hyperplasia

Check type

Hyperplasia without atypia (C) Atypical hyperplasia (D)

Consider levonorgestrel IUCD Check need to preserve childbearing function

Good result Does not tolerate Absent Present

Oral progestin therapy Consider levonorgestrel IUCD

Transvaginal USG and endometrial sampling 6 monthly Good result Does not tolerate

Regression of EH Persistence of EH Development of atypia Transvaginal USG and endometrial sampling 3 monthly Total hysterectomywith bilateral salpingo-

oophorectomyCheck if there is

abnormal bleedingTotal hysterectomy with bilateral salpingo-

oophorectomyRegression of EH Persistence of EH

Absent Present Continue levonorgestrel IUCD for 5 years, progestins for 6 months.

Continue levonorgestrel IUCD for5 years, progestins for 6 months.

Check EB every 3 months X 2

Check body mass index, do 6 monthly EBs Neative Positive

< 35 > 35 Check EB every 6-12 months

Stop follow up if 2 EBs negative Annual EBs if 2 EBs negative

22

Page 30: Differential Diagnosis and Management Options inuploads.worldlibrary.org/uploads/pdf/20180107071525differential... · 83 Peripartum cardiomyopathy 165 84 Hypertension 167 85 Convulsions

12. Suspicion of Luteal PhaseDefect

A. A woman with luteal phase defect (LPD) may be asymptomatic. On the otherhand, she may have any of the following symptoms.

1. Infertility.

2. Short luteal phase: the duration of the menstrual cycle is short. Normally theluteal phase is of 14 days, and any variation in the length of the cycle is due tovariation in the duration of the follicular phase. If the time of ovulation is fixed bybasal body temperature (BBT) chart (and later by investigations likeultrasonography), the luteal phase is found to be shorter than 14 days in a case ofLPD.

3. Irregular ripening: the corpus luteum starts degenerating early, causingdeclining serum progesterone levels earlier. The result is vaginal spotting for a fewdays prior to onset of regular menstrual flow.

4. BBT changes: the elevation in BBT may not be adequate (< 0.5° F) or maydecline early.

5. Repeated early first trimester abortions: these are due to inadequate decidualsupport by progesterone.

B. Serum progesterone level is estimated in the luteal phase of the menstrualcycle. If the level is 10 ng/ml or more, there is no LPD. If it is below 2 ng/ml, it is an anovulatory cycle. If it is less than 10 ng/ml, the test is repeatedagain after 3 days. If the sum of the two readings is 15 ng/ml or more, there is noLPD. If it is below 15 ng/ml, the diagnosis of LPD is confirmed. If the woman hasundergone an endometrial biopsy in the premenstrual phase and it shows the criteriadescribed by Georgeana Seager Jones as described below, corpus luteuminsufficiency can be diagnosed. However endometrial histopathology is not requiredto make a diagnosis of LPD and is rarely used in modern gynecology.Jones’ criteria for diagnosis of corpus luteum insufficiency are as follows.

1. Disparity in endometrial dating by calendar and histological features by morethan 2 days.

2. Disparity in endometrial dating by glandular and stromal appearance by morethan 2 days.

3. Disparity in endometrial dating in two different parts of the same slide by morethan 2 days.

C. The cause of LPD is checked. It may be hyperprolactinemia, hypothyroidism,clomiphene citrate therapy, hypercholesterolemia, polycystic ovary syndrome oridiopathic. Hyperprolactinemia is treated with bromocriptine or cabergoline (seechapter 33). Hypothyroidism is treated with levothyroxine. Hypercholesterolemia istreated with statins. Treatment of polycystic ovary syndrome is complex. Ovulationis induced with clomiphene citrate and if that fails, with hMG and hCG combination(see chapter 35). In case there is insulin resistance, metformin is administered. If thewoman is receiving clomiphene citrate for treatment of anovulation, the LPD may bedue to formation of a deficient corpus luteum after administration of the drug. Insuch a case, luteal phase support is given by one of the following regimes.

1. Progesterone 8% vaginal gel is used once or twice a day. Vaginal route ispreferred because it is close to the uterus where the drug is needed. Gel is betterthan a suppository because provides more receptor sites in the endometrium and istolerated better. Treatment is started 2 days after ovulation.

2. Progesterone suppository 25 mg ql2h from day 15 of the menstrual cycle.

3. 17-alpha hydroxyprogesterone acetate or caproate 250 mg deep IM on day 15and 21.

4. Allyl estranol 5 mg PO q8h from day 15 of the cycle.

5. hCG 5000 IU IM on day 15, and 2500 IU IM on day 21.

6. Micronized progesterone 100-200 mg qd PO from day 15 of the cycle.

Progesterone treatment is given for idiopathic corpus luteum insuffucuency too. Thedrug support is continued if the woman gets pregnant in that cycle (as confirmed byserum beta-hCG assay on day 27 of the cycle). If not, the drug is withdrawn andmenstruation is allowed to occur. If the woman is under treatment only for menstrualdisturbance and has not planned a pregnancy in that cycle, the drug is withdrawnafter day 28 of the cycle. The treatment is repeated in the next cycle.

23

Page 31: Differential Diagnosis and Management Options inuploads.worldlibrary.org/uploads/pdf/20180107071525differential... · 83 Peripartum cardiomyopathy 165 84 Hypertension 167 85 Convulsions

Clinical suspicion (A)SUSPICION OF LUTEAL PHASE DEFECT

Serum progesterone assay in luteal phase (B)

> 10 ng/ml 2-10 ng/ml < 2 ng/ml

No luteal phase defect Obtain another reading after 3 days and add the two values Anovulation

> 15 ng/ml < 15 ng/ml Clomiphene citrate

No luteal phase defect Check cause (C)

Hyperprolacinemia Hypothyroidism Clomiphene citrate Hypocholesterolemia Polycystic ovarysyndrome

Idiopathic

BromocriptineCabergoline

Levothyroxine Progesterone Statins Clomiphene citrate Progesterone

24

Page 32: Differential Diagnosis and Management Options inuploads.worldlibrary.org/uploads/pdf/20180107071525differential... · 83 Peripartum cardiomyopathy 165 84 Hypertension 167 85 Convulsions
Page 33: Differential Diagnosis and Management Options inuploads.worldlibrary.org/uploads/pdf/20180107071525differential... · 83 Peripartum cardiomyopathy 165 84 Hypertension 167 85 Convulsions

13. Acute Lower Abdominaland Pelvic Pain

A. There are a large number of non gynecologic and obstetric causes of acutelower abdominal and pelvic pain. These are excluded by a detailed history andappropriate examination. These include gastrointestinal (appendicitis, perirectalabscess, diverticulitis, irritable bowel syndrome, bowel obstruction, mesentericvenous thrombosis), urinary (cystitis, ureterolithiasis), and other (dissecting aorticaneurysm, porphyria, sickle cell crisis, malingering and somatization disorder).

B. If a woman with amenorrhea develops acute lower abdominal (iliac fossa)and pelvic pain, she is likely to have an ectopic pregnancy. The pain is due toleakage of some blood from the ectopic pregnancy. This amenorrhea is of 1.5to 2 months. Sometimes the ectopic pregnancy ruptures even before the womanmisses a period. See chapter 70 for diagnosis and further discussion on an ectopicpregnancy.

C. Presence of fever with or without chills is indicative of a pelvic infection.Such a woman has vaginal warmth, tenderness on transverse movements of thecervix, and a tender mass (tubo-ovarian) in one or both the lateral fornices. Seechapter 14 for further discussion on pelvic infections. It is possible that a milderform of the disease may not be associated with any fever or vaginal warmth.

D. If there is no vaginal warmth, transverse cervical movement tenderness maybe due to either an ectopic pregnancy or pelvic endometriosis. In case of theformer, the tenderness is on movement of the cervix only in one direction (seechapter 14), while in case of the latter, it is on movement of the cervix in bothdirections (see chapter 16).

E. If there is a tender mass in one of the lateral fornices, pelvic ultrasonographyis done. A transvaginal scan is more sensitive than a transabdominal scan in theassessment of small pelvic masses. When in doubt, the diagnosis can be confirmedby a laparoscopy. The tender adnexal mass may be any of the following.

1. Corpus luteum hematoma: if the hematoma is small and has not ruptured, the

woman may be given analgesics and observed. The hematoma gradually resolvesand no further treatment is then required. If the hematoma ruptures, the patientpresents with features of intraperitoneal hemorrhage (tachycardia, pallor, pain inabdomen, abdominal distension, and free fluid in the peritoneal cavity) andsometimes shock. Such a woman is resuscitated and then subjected to an exploratorylaparotomy. The ruptured hematoma is treated by a wedge resection of the ovary. Allblood in the peritoneal cavity is removed.

2. Ectopic pregnancy: the diagnosis is supported by a serum beta-hCG titerof 1800 mlU/ml or more, when the uterine cavity is empty. Serum estrogenand serum progesterone levels are also reported to be useful in diagnosis.An unruptured tubal ectopic pregnancy can be treated surgically as follows.

a. Partial salpingectomy and modified Coffey's repair by a laparotomy.

b. Conservative surgery by salpingotomy, salpingostomy, or completion oftubal abortion, either by laparotomy or by laparoscopy.

c. Injection of potassium chloride into the heart of the fetus, or methotrexateinto the gestational sac under ultrasonographic control. Resolution ofthe ectopic pregnancy by death and resorption is monitored by serial serumbeta-hCG titers and serial pelvic ultrasonography.

d. Conservative medical treatment by mere observation (if the serum beta-hCG level is falling) or observation after administration of methotrexate,watching for resorption of the pregnancy as above.

A ruptured tubal ectopic pregnancy presents like a ruptured corpus luteumhematoma. It is treated by resuscitation, laparotomy, and partial salpingectomy plusmodified Coffey's repair.

3. Torsion of a pedunculated structure like an ovarian cyst, ovarian tumor, subserouspedunculated leiomyoma, or normal tube and ovary is treated by laparotomyand surgical removal of the concerned structure. In case of a normal ovary or abenign ovarian cyst, the torsion is corrected and if the vascularity returns and thetissues appear viable, the ovary is conserved. In case of an ovarian cyst, ovariancystectomy is done. These procedures can be done laparoscopically too.

25

Page 34: Differential Diagnosis and Management Options inuploads.worldlibrary.org/uploads/pdf/20180107071525differential... · 83 Peripartum cardiomyopathy 165 84 Hypertension 167 85 Convulsions

Exclude gastrointestinal, urinary and othercauses of acute lower abdominal and pelvicpain (A)

ACUTE LOWER ABDOMINAL AND PELVIC PAIN

Check for amenorrhea (B)

Present Absent

Assess for an ectopic pregnancy(See chapter 71)

Check for fever (C)

Absent Present

Gynecological examination Gynecological examination

Acute pelvicinflammatory

disease

No vaginal warmthTransverse cervical movement

tenderness present (D)

Tender adnexal mass (E) Acute pelvic inflammatory disease

Antibiotics, NSAIDs Unilateral Bilateral Pelvic ultrasonography Antibiotics, NSAIDs

Assess for an ectopicpregnancy

(See chapter 71)

Assess endometriosis(See chapter 16)

Corpus luteumhematoma

Ectopicpregnancy

Torsion of anyof the following

Ovarian cyst ortumor

Subserouspedunculated

leiomyoma

Normal tubeand ovary

Exploratory laparotomy26

Page 35: Differential Diagnosis and Management Options inuploads.worldlibrary.org/uploads/pdf/20180107071525differential... · 83 Peripartum cardiomyopathy 165 84 Hypertension 167 85 Convulsions

14. Tenderness on TransverseCervical Movements

A. Tenderness on transverse cervical movements is a clinical sign which indicatesthe presence of an acutely painful pelvic condition. If the tenderness is elicited onmoving the cervix in only one direction, vaginal warmth is looked for. If it is present,the diagnosis is acute pelvic inflammatory disease. A cervical swab is obtained formicrobiologic studies. Then broad spectrum antibiotics and an NSAID areadministered. If the response is good, the course of the antibiotic is completed andobservation is continued for any recurrence of symptoms. If there is no response totreatment in 48 hours, the antibiotic is changed to one suggested by the antibioticsensitivity study on the cervical swab. If there is no response to that too in another 48hours, the woman is subjected to a laparoscopy. If the diagnosis is pelvicinflammatory disease, pelvic fluid (or saline washing in case there is no free fluid inthe pelvis) is obtained for microbiologic studies. Then she is treated with appropriateantibiotics based on the antibiotic sensitivity report. The other possibility is pelvic

endometriosis, which is discussed later.

B. If the tenderness is unidirectional and there is no vaginal warmth, it is likelyto be an ectopic pregnancy. A lateral forniceal tender mass may or may not bepresent in a case of an ectopic pregnancy (see chapter 71).

C. If there is tenderness on moving the cervix to either side, vaginal warmthis looked for. If it is present, the diagnosis is acute pelvic inflammatory disease.If it is absent, the likely diagnosis is pelvic endometriosis. Presence of tender massesin the lateral fornices supports the diagnosis of pelvic inflammatory disease(tuboovarian masses), and the presence of tender nodules in the uterosacralligaments supports the diagnosis of endometriosis, but their absence does not rule outthese conditions. Pelvic ultrasonography helps differentiate endometriosis and pelvicinflammatory disease. Final diagnosis is made by a laparoscopy.

D. If the woman has no abnormality found on extensive investigations includingpelvic ultrasonography and laparoscopy, and still has persistent tenderness on lateralcervical movements, she has somatoform pain. This psychosomatic finding is due tospasm of the pelvic diaphragm. This patient is referred for psychiatric evaluation andmanagement.

27

Page 36: Differential Diagnosis and Management Options inuploads.worldlibrary.org/uploads/pdf/20180107071525differential... · 83 Peripartum cardiomyopathy 165 84 Hypertension 167 85 Convulsions

TENDERNESS ON TRANSVERSE CERVICAL MOVEMENTS

Check direction (A)

Unidirectional Bidirectional (C)

Assess for vaginal warmth Assess for vaginal warmth

Absent (B) Present Absent Present

Check for lateral forniceal tender mass Acute pelvicinflammatory

disease

Assess for tender nodules in uterosacral ligaments Check for tender masses in lateral fornices

Present Absent Antibiotics, NSAIDs Absent Present Absent Present

Ectopicpregnancy

(See chapter 71)

? Ectopicpregnancy

(See chapter 71)

Noresponse

Goodresponse

Endometriosis ? Endometriosis Acute pelvicinflammatory

disease

Tuboovarianmasses

Ectopicpregnancy

Normal findings

Laparoscopy Observation Pelvic ultrasonography, Laparoscopy

Somatoform pain (D)Acute pelvic inflammatory disease

Endometriosis(see chapter 16)

Normal findings

Appropriate antibiotics Peritoneal fluid culture, antibiotic sensitivitySomatoform pain (D)

28

Page 37: Differential Diagnosis and Management Options inuploads.worldlibrary.org/uploads/pdf/20180107071525differential... · 83 Peripartum cardiomyopathy 165 84 Hypertension 167 85 Convulsions

15. Low BackacheA. Backache is an extremely common symptom in gynecologic patients. Gynecologicbackache is over the back of the sacrum, and sometimes over one or both thesacroiliac joints. In case of the latter, local tenderness is assessed. Women withpain over lumbar spine and tenderness over sacroiliac joint are referred to anorthopedic surgeon.

B. The woman may have backache only during menstruation (dysmenorrhea)which is discussed in chapter 1. Some women have the pain throughout the month,worse during menstruation. It may be due to pelvic endometriosis or pelvicinflammatory disease, discussed in chapters l3, 14 and 16. Some women haveno effect of the menstruation on the pain. The second and third groups are subjectedto gynecologic examination.

C. A speculum examination of the lower genital tract reveals the followingconditions which can cause low backache.

1. Chronic cervicitis: the cervix is diffusely hyperemic. Abnormal vessels andwhite discharge are also seen. See chapter 22 for the management of chroniccervicitis.

2. Cervical erosion: it does not cause backache by itself. However sometimes there iscervical infection along with it, which can cause backache.

3. Endometriosis: if there is endometriosis of the rectovaginal septum, it may be seenin the posterior vaginal fornix. It is the involvement of the uterosacral ligaments andposterior pelvic peritoneum that causes backache rather than the disease seen in theposterior fornix. However it serves as an indicator of the presence of the disease in thepelvis. See chapter 16 for the treatment of endometriosis.

4. Cervical cancer: stage IIIb cancer may have reached the sacrum along theuterosacral ligaments. Stage IVa cancer may involve the rectum. Such extension is felt onper rectal examination after making a diagnosis of the disease on speculumexamination. See chapter 42 for the management of cervical cancer.

D. A number of conditions which can cause backache can be diagnosed onbimanual pelvic examination. They are as follows.

1. Retroverted fixed uterus: adhesions between the back of the uterus and the rectum orposterior peritoneum may be due to chronic pelvic inflammatory disease (PID), pelvic

endometriosis, or a past pelvic operation. The uterine fundus is directed backwards and cannotbe moved forwards by digital pressure. The distinction between its causes can be made bypelvic ultrasonography, and if necessary a laparoscopy.

2. Tubo-ovarian mass: it is found in chronic PID. There is a firm mass closelyrelated to the lateral aspect of the uterus. It may or may not be tender. See chapter 18for its management.

3. Pelvic endometriosis: there are tender nodules in the uterosacral ligament(s) and otherfeatures (see chapter 16).

4. Past pelvic operation(s): the scar of an exploratory laparotomy is evident. Themobility of the uterus is restricted or absent (in retroverted position). Such adhesions arefound when a scar is produced on the posterior uterine wall, as with myomectomy,metroplasty, surgery for pelvic endometriosis etc.

5. Posterior cervical leiomyoma: it fills the sacral hollow and presses on the sacralperiosteum (see chapter 20).

6. Subserous pedunculated leiomyoma: if it gets impacted in the pouch of Douglas, itcompresses the rectum and irritates the sacral periosteum. See chapter 20 for itsmanagement.

7. Ovarian tumor : it may get impacted in the pouch of Douglas and cause backache(see chapter 18).

8. Chronic ectopic pregnancy: its effects are like those of a chronic tubo-ovarian mass(see chapter 18).

E. The muscles of the back are palpated in standing position. If there is a spasmof these muscles, the cause is orthopedic. Such a woman is referred to an orthopedicsurgeon for management. If the muscle tone is poor and the woman has stoopingposture, her backache is due to poor musculature of the back. She benefits fromback exercises.

F. If the woman has rectal symptoms like hard stools, mucus in stools, or tenesmus,it is a lower colonic disease which is probably responsible for her backache.She is advised gastroenterologic consultation.

G. If all possible causes of backache are ruled out, the woman is assessed forpsychological factors. Stress, anxiety, depression, marital disharmony and sexualdysfunction may be factors contributory to her backache.

29

Page 38: Differential Diagnosis and Management Options inuploads.worldlibrary.org/uploads/pdf/20180107071525differential... · 83 Peripartum cardiomyopathy 165 84 Hypertension 167 85 Convulsions

LOW BACKACHE

Check site (A)

Lumbar spine Sacroiliac joint Back of sacrum

Assess for local tenderness Check relation to menstruation (B)

Orthopedic referral Present Absent No effect Aggravation Only during menses

Speculum examination (C) Dysmenorrhea

Chroniccervicitis

Cervical erosion Endometriosis Cervical cancer Normal

Bimanual pelvic examination (D)

Retrovertedfixed uterus

Adnexal fixed mass Tender, nodularuterosacral ligaments

Past pelvic operation Normal Leiomyoma Ovarian tumor

Cervical cancerStage IIIb-IVa

Tubo-ovarianmass

Endometriosis Postoperativeadhesions

Check back muscles (E)

Chronic ectopic pregnancy

Pelvic ultrasonography Spasm Poor tone Normal

PID Endometriosis Orthopedic referral Back exercises Assess rectal symptoms (F)

Postoperative adhesions Present Absent

Surgical referral Assess psychological features (G)

Functional30

Page 39: Differential Diagnosis and Management Options inuploads.worldlibrary.org/uploads/pdf/20180107071525differential... · 83 Peripartum cardiomyopathy 165 84 Hypertension 167 85 Convulsions

16. EndometriosisA. Endometriosis is a condition characterized by growth of endometrium-liketissue at site(s) other than the uterine lining. It is found more often in highersocioeconomic class women who are career minded and postpone their first pregnanciesbeyond the age of 26 years. The woman has secondary dysmenorrhea (see chapter1), dyspareunia, infertility, and sometimes chronic pelvic pain. On bimanual pelvicexamination, the uterus is often retroverted and fixed. There may be tender nodulesin the uterosacral ligaments. Transverse cervical movements may be tender (seechapter 14). Uni- or bilateral adnexal masses may be present (endometriomas).B. Laparoscopy is diagnostic. Very early lesions may be white. Fresh lesions arelike raspberry spots. Old lesions have burnt match stick appearance. All lesions haveradiating scars around them. Adhesions are common. If a lesion gets opened duringmanipulation, chocolate-colored fluid escapes. Ovaries may be large(endometriomas). Laparoscopic staging of endometriosis is shown in table 16.1. Thescore is 1-5 in minimal disease, 6-15 in mild disease, 16-40 in moderate disease, andmore than 40 in severe disease. For management purpose, minimal and mild diseaseare grouped together in our practice.C. If a young woman who is not yet married, or is married but is using someform of contraception is found to have pelvic endometriosis, it is desirable thatshe gets pregnant as early as possible. An unmarried woman is counseled tomarry and plan a pregnancy. A woman with mild disease can plan a pregnancywithout any treatment. If she does not wish to get pregnant immediately, she is givenpseudopregnancy or pseudomenopause treatment for 6-9 and 3-6 monthsrespectively, after which she may plan a pregnancy. Pseudopregnancy regime isadministration of combination contraceptive pills 1 PO qd for 1 month, 1 PO ql2h forthe remaining duration of treatment, stepping up the dose by 1 tablet a day in theevent of breakthrough bleeding. Treatment is begun from the first day of a period,and amenorrhea is maintained for the duration of treatment. Pseudomenopauseregime involves administration of danazol from the first day of a period in adose of 200 mg PO q8h maintaining amenorrhea for the duration of treatment.It can also be achieved with the use of GnRH analogues, which are far moreexpensive than danazol.A woman with moderate disease is given pseudomenopause regime for 3 to 6months and then is advised to get pregnant. A woman with severe disease but patenttubes is treated similarly. If she has blocked tubes, conservative surgery is done afterthe pseudomenopause regime and then a pregnancy is planned. Conservativesurgery is done by laparotomy or laparoscopy, at which all disease is removed, alladhesions are lysed, and the uterus and at least one normal tube and ovary are leftbehind. If the woman does not conceive in one year, a second look laparoscopy isdone to look for residual disease and treat it.D. A young, infertile woman with mild disease, or moderate disease withoutpelvic adhesions is treated with pseudomenopause regime for 3 to 6 months andthen asked to plan a pregnancy. A woman with moderate disease with pelvicadhesions or severe disease is treated similarly, but is subjected to conservativesurgery before planning a pregnancy. A second look laparoscopy is done after12 months if there is no conception in that period, as described above.

E. Sometimes the disease is found in a woman who has already completed herfamily. If she has mild disease, pseudopregnancy or pseudomenopause treatment isgiven, depending on her tolerance. If she has moderate disease without any largepelvic masses, pseudomenopause treatment is given. If she has moderate diseasewith large pelvic masses or severe disease without any complications, conservativesurgery is done. If there are any complications like bowel or ureteric obstruction,radical surgery is done in which the uterus, tubes, ovaries, and all disease areremoved in addition to surgical treatment of the obstruction.

Table 16.1 American Fertility Society Classification of Endometriosis

Size (cm)

Parameters

<1 1-3 > 3

EndometriosisPeritoneum

Superficial 1 2 4

Deep 2 4 6

Right ovary

Superficial 1 2 4

Deep 4 16 20

Left ovary

Superficial 1 2 4

Deep 4 16 20

POD obliteration Partial: 4 Complete: 40

Adhesions < 1/3 enclosure 1/3-2/3 enclosure > 2/3 enclosure

Right ovary

Filmy 1 2 4

Dense 4 8 16

Left ovary

Filmy 1 2 4

Dense 4 8 16

Right tube

Filmy 1 2 4

Dense 4 8 16

Left tube

Filmy 1 2 4

Dense 4 8 16

Staging: stage I (minimal): 1-5; stage II (mild): 6-15; stage III (moderate): 16-40;stage IV (severe): >40.

31

Page 40: Differential Diagnosis and Management Options inuploads.worldlibrary.org/uploads/pdf/20180107071525differential... · 83 Peripartum cardiomyopathy 165 84 Hypertension 167 85 Convulsions

Clinical suspicion (A)

SUSPICION OF ENDOMETRIOSIS

Laparoscopy (B)

Endometriosis Endometriosis ruled out

Check age and parity

Young woman, fertility not tested (C) Young woman, infertile (D) Any age, family complete (E)

Check stage Check stage Check stage

Mild Moderate Severe Mild Moderate Severe

Pseudopregnancy Check tubal patency Assess for large pelvic masses Check for complications

PseudomenopauseFor 3-6 months

Patent Not patent Absent Present Absent Present

PseudomenopauseFor 3-6 months Conservative surgery

Advise conception Conservative surgery Pseudopregnancy Pseudomenopause Radicalsurgery

Mild Moderate Severe

Check for adhesions

Pseudomenopause For 3-6 monthsfollowed by advise to conceive

Absent Present Pseudomenopause for 3-6 months, followed byconservative surgery and advise to conceive

Second look laparoscopy if no conception in 12 months

32

Page 41: Differential Diagnosis and Management Options inuploads.worldlibrary.org/uploads/pdf/20180107071525differential... · 83 Peripartum cardiomyopathy 165 84 Hypertension 167 85 Convulsions

17. PyometraA. Pyometra is a condition characterized by uterine distension with pus. It maybe diagnosed when the woman presents for evaluation of vaginal bleeding andfoul smelling leukorrhea, and is found to have a cervical carcinoma with pyometra.Such cases do not need any further evaluation for the cause of pyometra. Thepyometra is drained by cervical dilatation, and the cervical carcinoma is treatedappropriately depending on its stage (see chapter 42).

B. In a number of cases, the pyometra is asymptomatic. The diagnosis is madewhen a pelvic examination reveals a large and soft uterus, and then a pelvicultrasonography is carried out. It shows uterine distension, thinning of the myometrium,and fluid in the uterine cavity. It is not a disease by itself, but rather the manifestationof a primary disease. The first step is to drain the pus under antibiotic cover,by performing rapid cervical dilatation. Hollow tubular dilators are used so thatpus in the uterine cavity does not get pushed through the fallopian tubes intothe pelvic peritoneal cavity by piston action of the dilator. Antibiotic cover preventsa flare up of the infection, and pelvic spread of the infection should a pelvicspill occur despite the use of hollow tubular dilators. The pus is sent for microbiologicstudies. Gravity drainage of the pus is achieved by giving the woman head highposition postoperatively, and ambulating her early. A course of an appropriateantibiotic is administered based on the antibiotic sensitivity report. Many times no

organisms are grown on culture and then no antibiotic therapy is required.

C. One week after drainage of the pus from the uterine cavity, rapid cervicaldilatation and fractional curettage are carried out. Great care must be exercised so asnot to perforate the uterus, because that would cause spread of pus and endometrialcancer cells (if present) to the peritoneal cavity. Various possibilities of thehistopathological diagnosis are as follows.

1. Cervical carcinoma: endocervical carcinoma may cause a pyometra by providing asource of infection and plugging the internal os (see chapter 42).

2. Genital tuberculosis: the endometrium gets replaced by tuberculous granulationtissue which exudes pus. Treatment of genital tuberculosis is described in chapter 4.

3. Endometrial carcinoma : see chapter 44.

4. Senile endometritis : the myometrium is atrophic and weak, so that it cannotexpel the pus exuded by the inflamed endometrium. The pyometra may not recurafter drainage and antibiotic therapy. If it does, a total hysterectomy with bilateralsalpingo-oophorectomy is carried out under cover of antibiotics.

5. Cervical stenosis : it may respond to cervical dilatation and antibiotic therapy. Ifit does not, cyclic estrogen therapy may help. Recurrent pyometra may occasionallyneed repeated cervical dilatation.

33

Page 42: Differential Diagnosis and Management Options inuploads.worldlibrary.org/uploads/pdf/20180107071525differential... · 83 Peripartum cardiomyopathy 165 84 Hypertension 167 85 Convulsions

PYOMETRA

Determine nature

Symptomatic (A)

Prophylactic antibiotics

Asymptomatic (B)

Rapid dilatation of cervix with hollow tubular dilatorsMicrobiologic study of pus removed

Gravity drainage of pusAppropriate antibiotics

Rapid cervical dilatation and fractional curettage after 1 week (C)

Cervical carcinoma Genital tuberculosis Endocervical carcinoma Senile endometritis Cervical stenosis

See chapter 42 See chapter 2 See chapter 44

34

Page 43: Differential Diagnosis and Management Options inuploads.worldlibrary.org/uploads/pdf/20180107071525differential... · 83 Peripartum cardiomyopathy 165 84 Hypertension 167 85 Convulsions

18. Adnexal MassA. A woman may present with pelvic pain or dyspareunia due to an adnexalmass. On the other hand, she may be asymptomatic and discovered to have anadnexal mass on routine pelvic examination. See chapter 13 for discussion ona tender adnexal mass. A nontender adnexal mass can be cystic or solid.

B. If a cystic mass is fixed, it is most likely to be a broad ligament cyst. Anotherpossibility is cystic degeneration of a broad ligament leiomyoma. The diagnosisis confirmed by a pelvic ultrasonography. Ureteric compression by the mass is apossible complication, which should be assessed by ultrasonography and excretoryurography. The mass is removed by an exploratory laparotomy. The ureter needs tobe safeguarded during this surgery. Ureteric catheterization prior to the operation isoptional. Some surgeons recommend it, so that the catheter can be felt during surgeryand the ureter can be identified. Others feel that it should not be done because acatheter within the ureter makes it less supple and it can be injured during dissectionmore easily than when there is no catheter in it, since the ureter slips away morereadily when it is supple.

If the mass is mobile, its size is determined. A small cyst is likely to be afunctional cyst of the ovary, e.g. a follicular cyst or a corpus luteum cyst. It couldalso be a nonovarian mass e.g. parovarian cyst, peritoneal pseudocyst, A mediumsized cyst may be a functional cyst, a cystic tumor of the ovary, or a pyosalpinx. Thediagnosis is made by pelvic ultrasonography. A simple unilocular cyst up to 5 cm indiameter can be just observed, since it often undergoes resolution in three months.Treating it with 3 cycles of combination contraceptive pills is not recommended. Asimple unilocular cyst measuring 5-7 cm is observed with annual follow up. Theresolution of the cyst is confirmed by ultrasonography. A pyosalpinx is treated bysalpingectomy. A nonovarian mass is excised. A large cyst is a cystic ovarian tumor.It needs to be differentiated from a mesenteric cyst (both being abdominal inlocation). An ovarian cyst can be moved freely in all directions. A mesenteric cystcan be moved only in a direction perpendicular to the line of attachment of themesentery to the posterior abdominal wall, which is from the tip of the left transverseprocess of the second lumbar vertebra to the right sacroiliac joint. The diagnosis isconfirmed by ultrasonography. A cystic ovarian tumor is treated by ovariancystectomy in a woman below the age of 35 years, oophorectomy between the agesof 35 and 50 years, and total hysterectomy and bilateral salpingo-oophorectomy atthe age of 50 years or more.

C. A nontender, solid adnexal mass may be mobile or fixed. A mobile mass maybe an ovarian tumor or a subserous pedunculated leiomyoma. The diagnosis is made

by ultrasonography. A benign ovarian tumor in a woman below the age of 50 yearsis treated by wedge resection of the ovary if the tumor is localized to only a part ofthe ovary, and by oophorectomy if it is diffuse. The treatment is total hysterectomyand bilateral salpingo-oophorectomy (TH + BSO) by laparotomy or laparoscopy atthe age of 50 years or more. The tumor is subjected to frozen section study at thelaparotomy. If it is malignant, it is treated appropriately (see chapter 45). Thetreatment of a subserous pedunculated leiomyoma, if symptomatic or very large, ismyomectomy below the age of 40 years. It can be done by laparotomy, orlaparoscopic division of its pedicle and hemostasis by electrocoagulation. Theleiomyoma is then removed by morcellation through a second port, or through aposterior colpotomy. The treatment at the age of 40 years or more is total abdominalhysterectomy. It is combined with bilateral salpingo-oophorectomy at the age of 50years or more.

D. An irregular and fixed adnexal mass is evaluated by pelvic ultrasonographyand if required, a laparoscopy. It may be any of the following.

1. Tubo-ovarian mass : if it is asymptomatic, mere observation is sufficient. Seechapter 14 for further discussion on a tubo-ovarian mass.

2. Endometriosis : see chapter 16.

3. Chronic ectopic pregnancy : it is removed by a laparoscopy or an exploratorylaparotomy for its removal.

E. A fixed adnexal mass which is regular and smooth is evaluated by pelvicultrasonography. It could be any of the following.

1. Ovarian carcinoma : see chapter 46.

2. Broad ligament leiomyoma : see chapter 20.

3. Pelvic kidney : the diagnosis in confirmed by excretory urography. In theabsence of any complications, observation is sufficient.

F. If an asymptomatic postmenopausal woman is found to have an ovarian cyst only onultrasonography, the size and the type of the cyst are checked. If it is 5 cm in diameter orlarger, or if it is multiloculated, TH + BSO are performed. If it is smaller than 5 cm indiameter and uniloculated, serum CA 125 levels are assayed. If the value is raised, TH +BSO are performed. If CA 125 level is normal, the woman is observed. If the cystresolves or remains of the same size, no further treatment is required. But if it enlarges, ordevelops solid areas or loculi, TH + BSO are performed. If frozen section report showsovarian cancer, additional treatment is given as described in chapter 46.

35

Page 44: Differential Diagnosis and Management Options inuploads.worldlibrary.org/uploads/pdf/20180107071525differential... · 83 Peripartum cardiomyopathy 165 84 Hypertension 167 85 Convulsions

ADNEXAL MASS

None, cystic mass only on ultrasonography inPostmenopausal woman (F)

Bimanual pelvic examination (A)

< 5 cm, unilocular 5-7 cm, unilocular > 5 cm/multilocular Tender Nontender

CA 125 assay Annual USG TH + BSO See chapter 14 Assess consistency

Normal Raised Cystic (B) Solid

Observation TH + BSO Assess mobility Assess mobility

Remains sameor regresses

Enlarges MobileDevelops solidareas or loculi

Fixed Mobile Fixed

Observation Assess size Broad ligament cyst

Excision

Pelvic ultrasonography Irregular (D) Regular, smooth (E)

Small Medium Large Ovariantumor

Subserouspedunculatedleiomyoma

Pelvic ultrasonographyLaparoscopy

Pelvic ultrasonography

Functionalcyst of ovary

Pelvic ultrasonography Pelvicultrasonography

Check age Check age Tubo-ovarianmass

ChronicEctopicpregnancy

Broadligamentleiomyoma

Observation orsuppression for 3months

Pyosalpinx Ovarian tumor > 50 < 50 < 40 > 40 Endometriosis Ovariancarcinoma

Suspicion ofpelvic kidney

Pelvicultrasonography

Excision Check age TH + BSO Check site inovary

Myomectomy TAH See chapter 16 See chapter 46 Seechapter 20

Paraovariancyst

< 35 35-50 > 50 Excretoryurography

Peritonealpseudocyst

Ovariancystectomy

Oophorectomy TH + BSO

Localized Diffuse Pelvic kidney

Wedge resection of ovary Oophorectomy

36

Page 45: Differential Diagnosis and Management Options inuploads.worldlibrary.org/uploads/pdf/20180107071525differential... · 83 Peripartum cardiomyopathy 165 84 Hypertension 167 85 Convulsions

19. Abdominal EnlargementA. Pregnancy is the commonest cause of abdominal enlargement in the reproductiveage. Such a woman has amenorrhea of more than 3 months. Various symptomsand signs which help make a diagnosis of pregnancy are discussed in chapter62. A pregnant woman is given antenatal care.

B. If the woman does not have any amenorrhea, abdominal and pelvic examinationis done. Various findings on such an examination are excess of abdominal wallfat, intraperitoneal lump, retroperitoneal mass, ascites, encysted ascites, and fullbladder. Obesity is treated by adjustment of diet and exercise if it is due to highintake of calories and lack of adequate exercise. A medical consultation is advisedfor other causes of obesity.

C. An intraabdominal lump may be cystic or solid. A fixed cystic lump is abroad ligament cyst or cystic degeneration of a broad ligament leiomyoma. Relationshipof the ureter to the lump and presence of back-pressure changes due to uretericobstruction by the mass are assessed by an excretory urography. The mass istreated by excision, safeguarding the ureter carefully.

A mobile cystic mass is either an ovarian or parovarian cyst, or a mesenteric cyst.The former two are mobile in all directions. The latter is mobile in only onedirection, which is perpendicular to the line of attachment of the mesentery to theposterior abdominal wall, from the tip of the left transverse process of the secondlumbar vertebra to the right sacroiliac joint. See chapter 18 for the treatment of anovarian cyst.

A solid mass of uterine origin is most commonly uterine leiomyoma(s). It can also beadenomyosis, endometrial carcinoma, or uterine sarcoma. See chapter 5 fordifferentiation of various causes of uterine enlargement. A solid, extrauterine mass isusually an ovarian tumor. See chapter 18 for the management of ovarian tumors.

D. A retroperitoneal mass is solid, fixed, and not very well defined. It is characterized

by a band of tympanic note (on percussion) passing transversely across its front(due to the transverse colon). Thus it differs from an ascites, which causes aninverted horse-shoe shaped area of dullness and shifting dullness; and also froma pelvi-abdominal lump, which causes dullness over itself, and tympanic noteon the sides and upper part of abdomen (due to displacement of the bowel). Asurgical consultation is advised for a retroperitoneal mass.

E. There are a large number of causes of ascites. In gynecology, it may be causedby an ovarian carcinoma or in Meigs’ syndrome. If an ovarian tumor is foundwith ascites, an abdominal paracentesis is done. If it shows malignant cells, thediagnosis is ovarian carcinoma (see chapter 46). However an absence of malignantcells does not rule out an ovarian carcinoma. If ovarian malignancy is ruledout by various tests, and the woman has right sided hydrothorax as well, thediagnosis is Meigs’ syndrome (ovarian fibroma, ascites, and right sided hydrothorax).The ascites and hydrothorax resolve when the ovarian fibroma is removed. Medicalor surgical referral is done for other causes of ascites.

F. Encysted ascites localized to the pelvis and lower abdomen may resemblea cystic lump. However it is less well defined. It is due to tuberculosis. Theremay be other features of tuberculosis. Ultrasonography is useful to make a diagnosis.A biopsy may be obtained by making a small abdominal incision.

G. Chronic retention with overflow of urine may not be recognized by the patientand from her account of her symptoms, by her physician as well. A high degreeof suspicion is necessary while evaluating a cystic pelvi-abdominal lump whichis relatively immobile. Simple catheterization results in emptying of the bladderand disappearance of the lump. A medical consultation is advised for the managementof this problem.

The clinical diagnosis may not be accurate in some cases. Imaging in the form ofultrasonography, computerized tomography and magnetic resonance imaging are ofa great help in differentiating different masses.

37

Page 46: Differential Diagnosis and Management Options inuploads.worldlibrary.org/uploads/pdf/20180107071525differential... · 83 Peripartum cardiomyopathy 165 84 Hypertension 167 85 Convulsions

ABDOMINAL ENLARGEMENT

Check for amenorrhea and symptoms of pregnancy (A)

Present Absent

Present Assess for pregnancy Absent Abdominal and bimanual pelvic examination (B)

Antenatal care Obesity Intraperitoneal lump Retroperitoneal mass(D)

Ascites (E) Encysted ascites (F) Full bladder (G)

Assess nature (C) Assess for an ovarian mass

Cystic Solid Present Absent

Assess mobility Assess origin Abdominal paracentesis for malignant cells Medical or surgicalcause

Mobile Fixed Uterine Extrauterine Present Absent

Assess direction ofmobility

Broad ligamentcyst

See chapter 5 Ovarian tumor Ovariancarcinoma

Assess for other features of malignancy

All One Present Absent

Ovariancyst

Mesentericcyst

Ovariancarcinoma

Ovarianfibroma

38

Page 47: Differential Diagnosis and Management Options inuploads.worldlibrary.org/uploads/pdf/20180107071525differential... · 83 Peripartum cardiomyopathy 165 84 Hypertension 167 85 Convulsions

20. Uterine LeiomyomaA leiomyoma is an estrogen dependant tumor. Hence it tends to undergo atrophyafter menopause. If a woman is known to have a leiomyoma develops menopause orit is diagnosed after menopause, the woman is observed, undergoing examinationand pelvic ultrasonography every year. If the leiomyoma reduces in size or remainsconstant in size, observation is continued. If it enlarges, a leiomyosarcoma issuspected and an extrafascial total hysterectomy and bilateral salpingo-oophorectomy is performed. Further treatment is given as for a leiomyosarcoma ifthe diagnosis is confirmed on histopathological examination.

A. Leiomyomas are not always symptomatic. If a woman without any symptoms isaccidentally found to have uterine leiomyomas, she need not be treated surgicallyimmediately. If she is single, she is advised to get married and have a pregnancy asearly as possible. That is so because leiomyomas tend to grow with time, as well asnew ones tend to appear. Thus if her fertility has not already been compromised by theexisting leiomyomas, it may get compromised after some time. Performing amyomectomy may result in pelvic adhesions which may cause infertility by causingtubal obstruction or alteration of tubo-ovarian relationship. Hence it is better to plana pregnancy prior to myomectomy, unless leiomyomas are responsible for theinfertility. A woman who is married, but is nulliparous and using a contraceptive isadvised to stop the use of the contraceptive and have a pregnancy for the samereason. If the woman is parous, she is assessed for the presence of complications ofleiomyomas, such as the following.

1. Uni- or bilateral ureteric obstruction and back-pressure changes like hydroureterand hydronephrosis. It is seen with large leiomyomas obstructing the ureters at thepelvic brim, and broad ligament leiomyomas obstructing the ureter of the same side.

2. Degeneration: acute, painful degeneration like cystic degeneration.

3. Torsion of a subserous pedunculated leiomyoma.

If there are any complications, the woman is treated by myomectomy below the ageof 40 years, total hysterectomy with bilateral salpingectomy between the ages of 40and 50 years, and total hysterectomy with bilateral salpingo-oophorectomy after the

age of 50 years. Conservative treatment may also be given by embolization in caseof intramural leiomyoma measuring up to 9 cm in diameter. It reduces uterine size.However it may be associated with complications of interventional radiology,neerosis of the leiomyoma, and intramyometrial abscess formation. This form oftreatment needs further evaluation. An alternative form of treatment is lysis ofleiomyomas (myolysis) using high frequency focused ultrasound (HIFU) under MRIor ultrasonographic guidance. This method is associated with less complications, butis more expensive, and is not suitable if leiomyomas are located near the uterinevessels.

If there are no complications, and the uterine size is up to that of 10 weeks ofgestation, observation is sufficient. She is examined every 6 months, any growth ofthe leiomyomas is looked for, and she is asked to report immediately if anysymptoms related to leiomyomas develop. If the uterine size is greater than that of10 weeks of gestation, she is treated like a parous woman with complications of theleiomyomas.

B. If an infertile woman is found to have uterine leiomyomas, she is assessedfor the presence of any other factors responsible for infertility (see chapters 34to 37). If she and her husband have any such factors, those are treated appropriately.If she still remains infertile, a myomectomy is advised. If all other factors arenormal, a myomectomy is performed (see chapter 51).

C. If she presents with symptoms like menorrhagia, lower abdominal pain,severe dysmenorrhea or repeated pregnancy wastage, any other cause for thesesymptoms is sought. If none is found, or if the symptoms persist despite adequatetreatment of the cause found, she is treated by myomectomy below the age of 40years, and total hysterectomy with bilateral salpingectomy at the age of 40 years ormore. If she is of the age of 50 years or more, bilateral salpingo-oophorectomy isperformed as well.

D. A woman who presents with uterine leiomyomas as a lump in the lowerabdomen, is treated surgically as outlined above.

E. A woman with a leiomyomatous polyp in the vagina usually presents withleukorrhea and metrorrhagia. The treatment of this condition is polypectomy.

39

Page 48: Differential Diagnosis and Management Options inuploads.worldlibrary.org/uploads/pdf/20180107071525differential... · 83 Peripartum cardiomyopathy 165 84 Hypertension 167 85 Convulsions

Observation

Size constant Enlargesor reduces

Menopausal

Treat as leiomyosarcoma

UTERINE LEIOMYOMA

Check menstrual statusAssess symptoms

Premenopausal

Asymptomatic (A) Infertility (B) Menorrhagia Pain Repeated pregnancywastage

Lump in abdomen(D)

Leucorrhea,metrorrhagia (E)

Check marital status Assess other causes of infertility Assess for presence of any other cause of symptoms (C) Vaginal polyp

Single Married Present Absent Present Absent Polypectomy

Advisemarriage

Check parity Treatappropriately

Myomectomy Treat cause

Nulliparous Parous Conceives

Antenatalcare

Infertile despite adequatetreatment for 1 year

Myomectomy

Symptom isrelieved

Symptom persists Check age

Adviseearlypregnancy

Assess forcomplications

6 monthly followup for assessingenlargement

< 40 40-50 > 50

Absent Present Enlargement Size constant orreduces

Observation Total hysterectomy plusbilateral salpimgo-

oophorectomyAssess uterine size Treat complication Total hysterectomy with bilateral

salpingectomy

< 10 weeks > 10 weeks

ObservationCheck age

< 40 40-50 > 50

Myomectomy Total hysterectomy withbilateral salpingectomy

Total hysterectomy plus bilateralsalpimgo-oophorectomy

Size constant Enlargement

40

Page 49: Differential Diagnosis and Management Options inuploads.worldlibrary.org/uploads/pdf/20180107071525differential... · 83 Peripartum cardiomyopathy 165 84 Hypertension 167 85 Convulsions

21. Genital ProlapseDescent of the genital tract to a level below normal is called genital prolapse. Theuterus is said to have prolapsed when the cervix lies below the level of the ischialspine. Various degrees of uterine prolapse are as follows (Baden Walker). The POP-Q classification is good for quantification and for pre- and postoperativecomparison, but has little advantage over the Baden Walker classification in makingdecisions on the management of the prolapse.

First degree: cervix between ischial spine and introitus.Second degree: cervix at introitus.Third degree: cervix below introitus, fundus above introitus.Fourth degree: fundus below introitus.

If the anterior vaginal wall and posterior vaginal wall are visible between the labiawithout having to separate them, the woman is said to have cystocele and/orurethrocele and rectocele respectively. If it is just visible, it is mild. If it is part of afourth degree prolapse (procidentia), it is severe. Moderate vaginal prolapse isbetween mild and severe forms. The treatment of genital prolapse depends, amongother things, on the age of the patient and the degree and type of the prolapse. Firstdegree uterine prolapse with mild cystocele and mild rectocele doesnot require any surgical treatment. The woman is prescribed Kegel's perinealexercises, in which she is asked to constrict her urinary and anal sphinctersfor 5 seconds followed by relaxation for 5 seconds for 5 minutes at a time 5times a day.

A. A congenital prolapse is seen in a newborn delivered as a breech. The uterusgets reduced by digital pressure and does not prolapse again. Usually there aremultiple anomalies, which require intensive treatment.

B. A premenarchal girl or a young nulliparous woman with more than first degreeuterine prolapse without any cystocele or rectocele may have lumbar spina bifida occultaor visceroptosis as the etiological factor. She is treated by one of the followingoperations. In case one of the operations fails, another operation from the group isperformed after three to six months.

1. Shirodkar's sling operation: the back of the cervix is suspended from the sacralpromontory with mersilene tape passed extraperitoneally, the left half of thesling being passed through a psoas loop to prevent constriction of the sigmoidcolon by the tape.

2. Khanna's sling operation: the back of the cervix is suspended from the anteriorsuperior iliac spines or lateral ends of the inguinal ligaments with mersilenetape passed extraperitoneally.

3. Purandare's cervicopexy: the front of the supravaginal cervix is suspended fromthe angles of the rectus sheath with mersilene tape passed extraperitoneally. It

cannot be used if the recti are weak.

C. A woman in the reproductive age with a desire to have more children or at leastcontinue menstrual function needs a conservative operation. If she has uterineprolapse alone, she is treated as in B. If she has cystocele, rectocele, or enterocele,she is treated with site specific anterior colporrhaphy, posterior colporrhaphy, andvaginal or abdominal repair of enterocele (Halban’s or Moschcowitz’s operation)respectively. Associated uterine prolapse is treated with advancement of theuterosacral ligaments to the front of the cervix in the absence of supravaginalelongation of the cervix. The cervix needs to be amputated and covered withSturmdorf sutures in addition if there is cervical elongation too. If the woman isabove 40 years and has completed her family and does not wish to retain themenstrual function, vaginal hysterectomy and vault suspension are done, combinedwith colporrhaphy and/or enterocele repair as appropriate.

D. If a postmenopausal woman is fit for anesthesia and surgery, she is treatedas a woman above 40 (see above). If the pelvic connective tissue is atrophic andunlikely to support the vagina, a polypropylene mesh is placed between the vaginaand the bladder and/or rectum (meshplasty). If she is unfit, the operation ispostponed until the medical problems are treated adequately. If she is very old, andhas multiple medical problems (ischemic heart disease, hypertension, diabetesmellitus, cerebrovascular accidents etc.) LeFort's operation is done under localanesthesia (bilateral pudendal block, and additional submucosal infiltration of 0.5%lignocaine). In this operation quadrangular strips of mucosa are removed from theanterior and posterior vaginal walls, and the raw areas are approximated, thusoccluding the vagina below the cervix. If the woman desires to retain sexualfunction, triangular mucosal flaps are removed, the base of each triangle being nearthe cervix. The raw areas are then approximated. This is called Goodall-Power'smodification of LeFort's operation.

E. Genital prolapse during pregnancy is managed by insertion of a ring pessaryuntil the uterus becomes an abdominal organ and is held up by the pelvic brim. Incase there is elongation of the supravaginal cervix, there is risk of cervical dystociaand a cesarean section may be required. Surgical treatment of the prolapse is givenafter the puerperium.

F. Vaginal prolapse after a hysterectomy in the past may be cystocele, rectocele, orvault prolapse. The former two conditions may have developed after thehysterectomy or may have been overlooked at the time of the previous operation. Inthose cases a colporrhaphy is done. If a colporrhaphy done in the past has failed, ameshplasty is done. A vault prolapse is treated with sacrospinous vault suspension orabdominal sacrocolpopexy.

All operations performed abdominally can also be performed laparoscopically.Though colporrhaphies can also be done laparoscopically, they are best done by thevaginal route, since the former offers no advantage over the latter, and is associatedwith abdominal scars and risk of complications of laparoscopy unnecessarily.

41

Page 50: Differential Diagnosis and Management Options inuploads.worldlibrary.org/uploads/pdf/20180107071525differential... · 83 Peripartum cardiomyopathy 165 84 Hypertension 167 85 Convulsions

GENITAL PROLAPSE

Check menstrual status

At birth (A) Premenarchal (B) Premenopausal (C) Postmenopausal (D) Pregnancy (E) Posthysterectomy (F)

Congenitalprolapse

Check nature and causeof prolapse

Check reproductive status Check type ofprolapse, health of

Ring pessary Check type of prolapse

Manual reduction Uterine prolapseDesires retention ofreproductivefunction

Does not desireretention ofreproductive function

pelvic connective tissue

Uterine Vaginal

Cystocele

Uterovaginal

Rectocele Vault

Spina bifida occulta Visceroptosis IdiopathicCheck type of prolapse Check age Vaginal hysterectomy,

vault suspensionAbdominal uterine suspension Uterine

prolapse< 40 > 40

Healthy tissue Weak tissueSucceeds Fails Check desire to retain menstruation

Sacrospinousvault suspensionor abdominalsacrocolpopexy

Observation AlternativeOperation

Yes No Vaginal hysterectomy,vault suspension,colporrhaphies.

Vaginal hysterectomy,vault suspension,meshplasty

Cystocele Rectocele Enterocele UterovaginalAny tissue, patient unfit for anesthesia

Anteriorcolporrhaphy

Posteriorcolporrhaphy

Repair

6.25 cm

Check utero-cervical length

> 6.25 cm

LeFort’s ormodified LeFort’s

operation

Check for prior colporrhaphy

Succeeds

Observation

Fails

MeshplastyCervical amputation,uterosacral advancementanterior and posteriorcolporrhaphy

Yes

Meshplasty

No

Colporrhaphy

Uterosacral advancement, anterior and posterior colporrhaphy

Succeeds Fails

Observation Abdominal uterine suspension plus meshplastyCheck type of prolapse

Uterovaginal Vaginal Uterine

Vaginal hysterectomy, vault suspension, colporrhaphies Vaginal hysterectomy, vault suspension

42

Page 51: Differential Diagnosis and Management Options inuploads.worldlibrary.org/uploads/pdf/20180107071525differential... · 83 Peripartum cardiomyopathy 165 84 Hypertension 167 85 Convulsions

22. LeukorrheaLeukorrhea means white discharge. However, the color may vary depending

on the cause. Normally some amount of vaginal discharge is present in everywoman. It is a mixture of cervical mucus, endometrial secretions, fluid fromperitoneal cavity and fallopian tubes, and vaginal cells. Its amount varies with thephase of the menstrual cycle, being more at the time of ovulation (cervical mucus)and premenstrually (pelvic congestion). It does not have odor, and it may leavebehind a brown stain on the underwear if hot water is used for washing it.

A. If the woman has leukorrhea and itching of the vulva together, the causeis trichomoniasis or candidiasis. A speculum examination helps differentiatethe two in the majority of cases. Trichomoniasis is associated with thin, yellowish,frothy, abundant, foul discharge of pH 5 to 7. The underlying vagina is inflamed,with red spots on a hyperemic background. Candidiasis causes thick, curdy, whitedischarge of pH 4 to 5.

B. A wet preparation is made in all cases, because the nature of the dischargeis misleading in some cases, and mixed infection is also possible. A drop ofthe discharge is placed on a glass slide and observed under a microscope.Trichomonads are seen to have flagellate motility, while the hyphae of Candidaare not seen without special preparation. For that, the discharge is first treated with10% KOH, which dissolves cells and tissue debris. Then India ink isadded to the mixture. It stains the background black, and leaves the hyphaeunstained.

C. If a diagnosis cannot be made by a wet preparation, the discharge is addedto Feinberg Whittington (liver peroxide) medium, which grows both the organisms.Candida can be grown on Saboraud’s agar too. A Pap smear is obtained too.The smear shows trichomonads as indistinct, faintly greyish-blue cells with avariety of shapes. The epithelial cells stain red and often have perinuclear halo.Monilial hyphae stain red, are straight or curved, unbranched, and may occurin great numbers. The spores are small, rounded, red refractile bodies whichare larger and thicker than cocci.

Trichomoniasis can be treated by any of the following regimes. Both sexual partnersare treated together. Sexual abstinence is maintained for 2 weeks.

1. Metronidazole 2 g PO once.2. Tinidazole 2 g PO once.3. Metronidazole 500 mg PO q12h for 7 days.4. Other agents: secnidazole, clotrimazole, nonoxynol-9, aminacrine, sodium

edetate, docusate sodium.

Treatment of candidiasis is by any of the following regimes. The male partner istreated only if infected.

1. Clotrimazole 1% cream 5 g PV qd for 7–14 days2. Clotrimazole 2% cream 5 g PV qd for 3 days

3. Miconazole 2% cream 5 g PV qd for 7 days4. Miconazole 4% cream 5 g PV qd for 3 days5. Miconazole 100 mg suppository, 1 PV HS for 7 days6. Miconazole 200 mg suppository, 1 PV HS for 3 days7. Tioconazole 6.5% ointment 5 g PV once8. Butoconazole 2% cream 5 g PV once9. Terconazole 0.4% cream 5 g PV qd for 7 days10. Fluconazole 150 mg PO once

Only topical azole therapies are recommended for use among pregnant women.

c. If the woman does not have vulvar itching along with leukorrhea, manyother causes are possible. A speculum examination is done to find a localcause.

1. Cervical erosion is a velvety pink area of columnar epithelium in continuitywith endocervical epithelium. If it is not due to pregnancy or use of combinationcontraceptive pills, and Pap smear is normal, it is destroyed withelectrocauterization, cryocauterization, or laser vaporization.

2. Cervicitis causes diffuse redness of the cervix. It is treated by doxycycline (100mg qd PO X 10 days) and vaginal povidone-iodine pessaries (200 mg qd PV X10 days). Chronic cervicitis is treated like a cervical erosion surgically.

3. A mucous or leiomyomatous polyp can cause leucorrhea. It is removedsurgically.

4. Bacterial vaginosis is caused by Gardenerella vaginalis. It causes thin, greyish,watery, odorless discharge at pH 5 to 5.5, which produces fishy odor on mixingwith 10% KOH. A wet preparation shows “clue cells”, which are stippled orgranulated epithelial cells (due to adhesion of the bacteria). A gram stain showssmall gram-negative bacilli and a relative absence of lactobacilli. It is treated byone of the following regimes.a. Metronidazole 500 mg q8h PO X 7 days.b. Metronidazole gel 0.75%, 5 g PV/day for 5 days.c. Clindamycin cream 2%, 5 g PV HS for 7 days.

4. Watery cervical mucus in second half of menstrual cycle suggests anovulation.5. Adenosis of the cervix and/or vagina can develop with intrauterine exposure to

diethyl stilbesterol (which was used in the past for treatment of threatenedabortion). A carcinoma has to be excluded by Pap smear and biopsy. Thetreatment is as of a cervical erosion.

6. Carcinoma of cervix: see chapter 42.7. Sexually transmitted diseases : see chapter 24.8. Foreign bodies commonly cause infection and vaginal discharge in preadolescent

girls. Cotton, paper, or other materials may be placed in the vagina. In adultsforgotten menstrual tampon, ring or Hodge pessary may cause malodorousdischarge. Treatment is to remove the foreign body.

9. Atrophic vaginitis: lactating and postmenopausal women lack estrogen, so thatthe vagina becomes atrophic, and susceptible to trauma and infection.Treatment is local application of estrogen cream (dinestrol 1%).

43

Page 52: Differential Diagnosis and Management Options inuploads.worldlibrary.org/uploads/pdf/20180107071525differential... · 83 Peripartum cardiomyopathy 165 84 Hypertension 167 85 Convulsions

LEUKORRHEA

Check for vulvar itching (A)

Present Absent (D)

Speculum examination: Nature of vaginal discharge Cervicalerosion

Bacterialvaginosis

Adenosis Sexuallytransmitteddisease

Atrophicvaginitis

Trichomoniasis (?) Candidiasis (?)Cervicitis Normal cervical

mucusCervical cancer Foreign body

Wet preparation (B) Polyp

Trichomoniasis Candidiasis No diagnosis possible

Culture (C)

Growth of organisms No growth

Trichomoniasis Candidiasis Pap smear

Trichomoniasis Candidiasis

44

Page 53: Differential Diagnosis and Management Options inuploads.worldlibrary.org/uploads/pdf/20180107071525differential... · 83 Peripartum cardiomyopathy 165 84 Hypertension 167 85 Convulsions

23. Itching of the VulvaA. Itching of the vulva is a not uncommon symptom of women presenting toa gynecologic clinic. It is often a distressing and embarrassing symptom. A combinationof itching of the vulva and leukorrhea is far more common than itching alone.It is due to either trichomoniasis or moniliasis (see chapter 22).

B. In absence of associated leukorrhea, a local skin lesion of the vulva is mostlikely. Local examination and appropriate investigations reveal one of the followingconditions.

1. Tenia cruris: there is a brick-red rash spreading from the inguinal foldsto the vulva and down inner thigh. There is an advancing red, scaly border,with a clearing in the center. A local scraping is treated with 10% KOHon a glass slide and observed. A useful modification is Swartz-Medrik stain,which shows the fungal hyphe faint blue against a pink background. Treatmentis to use loose, cool clothing, and topical antifungal cream like clotrimazole,miconazole etc., twice a day for 2 to 3 weeks.

2. Scabies: it is caused by the itch mite Sarcoptes scabiei, which is transmittedsexually and nonsexually both. There are fine, excoriated papules on vulva,fingerwebs, wrist folds, axillae, umbilicus, and groin. A few papules are surmountedby 1 to 4 mm, fine, etched burrows. Diagnosis is made by taking a superficial shave(not a scrape) and looking for mites or their eggs in it. The treatment is given to allfamily members at the same time. Permethrin cream 5%: two (or more) applications, each about a week apar Crotamiton lotion 10% and Crotamiton cream 10% Lindane lotion 1%: for patients who have failed treatment with or cannot tolerate

other medications. Benzyl benzoate (10%) is applied after a bath from chin to toes on 3 days.

3. Seborrheic dermatitis: there are mild to profuse, loose flaky scales, often onpoorly defined patches of mild pinkness. Itching is mild. Similar lesions are found onthe scalp and face. It is treated with hydrocortisone (1%) cream applied twice a day.Other forms of treatment include low-strength coal tar cream or gel, oriodochlorhydroxyquin (1%), either alone or in combination with a mildcorticosteroid.

4. Psoriasis : the lesions are over joints and extensor surfaces of limbs, trunk, palms,soles, scalp, and vulva. Itching is either absent, or mild to severe. There are

widespread bright pink plaques surmounted by loose, silvery scales. Various formsof topical treatment are salicylic acid, tar, ultraviolet light, anthralin, andcorticosteroids. Relapses can occur.

5. Pediculosis pubis: it is caused by Phthirus pubis transmitted sexually. It is a 2mm size, flat, crab-like creature which holds onto the base of a hair shaft andperiodically bites the host. Itching is mild to severe. Examination shows a grey,flake-like, slow moving organism, and grey, 1 mm nits on hairs. Both sexualpartners are treated simultaneously as follows.

a. Clothes, bed-linen, and pajamas are washed well before and after treatment.b. Permethrin (1% cream): applied to affected areas and washed off after 10

minutes.c. Pyrethrins with piperonyl butoxide: applied to the affected area and washed

off after 10 minutes.d. Malathion (0.5% lotion): applied to affected area and washed off after 8–12

hours.e. Ivermectin 250 μg/kg orally, repeated in 2 weeks.f. Lindane shampoo: for patients who have failed treatment with or cannot

tolerate other medications.

6. Vulvar carcinoma in situ : see chapter 48.

7. Allergic vulvitis: it needs sensitization and cannot develop with the first exposureto the antigen. The first time the rash starts 1 to 4 days after contact with theallergen, new lesions develop over 10 days, and the eruption subsides in 2 to 3weeks. Onset and development are faster with subsequent exposures. There isintense redness and itching. Patch testing helps diagnosis. The treatment is topicalcorticosteroids and avoiding contact with the allergen.

C. If there are no characteristic skin lesions, systemic conditions are sought,e.g. diabetes mellitus, hepatitis, drug allergy, vitamin A or B deficiency, anemia,and leukemia. The itching is widespread and not restricted to the vulva. Theseconditions, if present, are treated appropriately.

D. The diagnosis of a psychological factor responsible for vulvar itching is madeonly after exclusion of organic conditions discussed before. Such factors includenervous fatigue, marital disharmony, sexual frustration, guilt over masturbation,syphilophobia, and need for protection from sex. The treatment is education,counseling, and referral to a psychiatrist.

45

Page 54: Differential Diagnosis and Management Options inuploads.worldlibrary.org/uploads/pdf/20180107071525differential... · 83 Peripartum cardiomyopathy 165 84 Hypertension 167 85 Convulsions

ITCHING OF VULVA

Check for presence of leucorrhea (A)

Present Absent (B)

See chapter 22 Local examination for characteristic skin lesion

Present Absent

Tenia Scabies Seborrheicdermatitis

Psoriasis Pediculosispubis

Vulvarcarcinomain situ

Allergicvulvitis

Assess for presence of systemic disease (C)

Present Absent

Diabetesmellitus

Hepatitis Drugallergy

Vitamin Adeficiency

Vitamin Bdeficiency

Anemia Leukemia Assess for psychological factors (D)

Nervousfatigue

Maritaldisharmony

Sexualfrustration

Guilt overmasturbation

Syphilophobia Protectionfrom sex

46

Page 55: Differential Diagnosis and Management Options inuploads.worldlibrary.org/uploads/pdf/20180107071525differential... · 83 Peripartum cardiomyopathy 165 84 Hypertension 167 85 Convulsions

24. Vulvar LesionA. When a woman develops an ulcer on the vulva, prompt evaluation and treatmentare required, because it may be a serious disorder like a sexually transmitted diseaseor a cancer. Presence of lesions on other skin areas suggests the presence of adermatological condition rather than a gynecologic condition of the vulva. If thelesion is only on the vulva, local pain is assessed.

B. If the lesion is painful, the amount and nature of the discharge on the ulcer,and the appearance of the ulcer are assessed. The pain is severe and the dischargeis minimal in Behcet's disease. There are recurrent, single or multiple, small,round or oval superficial ulcers with reddened borders. Destruction and fenestration oflabia may be seen in chronic cases. Other features of the syndrome include oral ulceration,iritis, and polyarthritis. It is treated with oral steroids.Genital herpes causes severe local pain, and mild discharge. Initially the lesions are in theform of intradermal or subepithelial vesicles which break down to form superficial ulcers.They are recurrent. Primary infection may be associated with fever and inguinallymphadenopathy. It is treated with acyclovir 200 mg PO 5 times a day for 7 to 10 days.Severe cases are given the drug 5 mg/kg IV q8h. Analgesics and topical anesthetics aregiven for local pain. Sitting in a warm bath during micturition relieves severe dysuriacaused by the disease.Factitious ulcers are due to self-inflicted trauma.. They are moderately painful, recurrent,bizarre, irregular, tender, and covered by mild amount of purulent exudate. The woman isgiven an antiseptic cream for local application. She is referred to a psychiatrist for help.The lesions of chancroid are extremely painful. There are multiple tender noninduratedulcers on the clitoris or in the vestibule, with irregular ragged edges and granulatingsurface covered by moderate amount of pus. Larger ulcers have irregular outline. Thevulva is often edematous. Acute inguinal lymphadenitis is present. When nodes suppurate,they are aspirated. The disease is treated by any of the following regimes: Azithromycin 1g PO in a single dose, Ceftriaxone 250 mg IM in a single dose, Ciprofloxacin 500 mg POq12h for 3 days or Erythromycin base 500 mg PO q8h for 7 days..Hidradenitis suppurativa causes mild pain. There are multiple draining pustules whichintercommunicate. The amount of discharge is moderate. Inguinal lymphadenitis ispresent. It is treated by extensive local debridement. The raw area is left open and treatedby local cleaning and application of antiseptics to promote granulations. Antibiotics areused before and after surgery, based on the microbiologic studies. Skin grafting is usuallynot required. See chapter 48 for discussion on vulvar carcinoma.C. If the lesion is painless, the nature and amount of discharge, and the appearance of thelesion are assessed. The discharge is minimal in syphilis. The lesion of primary syphilis isa chancre on the labia or at the introitus. It is a firm, indurated lesion with a punched outcentre, clean floor, and marked edema. In secondary syphilis the lesion is calledcondyloma lata. It is a plateau-like excrescence raised slightly above the surface of theskin. Its surface is greyish, necrotic, moist, and somewhat depressed at the center. Thereare often multiple, confluent lesions at the mucocutaneous junction. Gumma is found intertiary syphilis. There is a tumor or an ulcer which is punched out and shallow. It leavesbehind tissue paper scar. The vulva and the surrounding structures may be destroyed by

the ulcer. The diagnosis is confirmed by tests like VDRL, Kahn, Wasserman, fluorescenttreponema antibody test etc. Syphilis is treated by Benzathine penicillin 2.4 million unitsdeep IM after a test done for early disease (<1 year), and once a week for 3 weeks for latedisease ( > 1 year), or Doxycycline 100 mg PO ql2h for 15 days for early disease, and 30days for late disease.Lymphogranuloma venereum (LGV) causes superficial vulvar ulceration in the beginning.Extensive ulceration combined with marked lymphedema leads to elephantiasis withdraining sinuses. Labia may fenestrate. Occasionally large ulcers destroy vulva and extendto the perineum and rectum. Inguinal nodes form bubos. The treatment is Doxycycline100 mg PO q12h, Erythromycin 500 mg PO q6h, Sulfisoxazole 1000 mg PO q12h, orTetracycline 500 mg PO q6h, each for 21 days..Granuloma inguinale (GI) is characterized by a small papule on vulva, which ulcerates.The ulcer spreads to inguinal and pubic regions. Its floor is of beefy-red, velvetygranulation tissue. Late cases have deep ulceration and extensive cicatrization. Treatmentis Azithromycin 1 g PO once per week or 500 mg qd, Doxycycline 100 mg PO q12h,Ciprofloxacin 750 mg PO q12h, Erythromycin base 500 mg PO q6h or Co-trimoxazole(160 mg/800 mg) PO q12h each for at least 3 weeks and until all lesions have completelyhealed.Tuberculous ulcer is found on labia or vestibule. It is irregular, ragged, and hasundermined edges. Its floor has granulations or caseous material. See chapter 4 fortreatment of tuberculosis.Actinomycosis of the vulva has multiple small abscesses leading to sinuses whichdischarge viscid pus containing yellow granules. It is treated with penicillin G 1.5-3million units IV q4h after a test dose, for at least 6 weeks. It is then followed by penicillinVK, 2-4 g PO qd for 6 to 12 months to prevent a relapse. Localized lesions may needsurgical drainage.D. If the lesion is intensely erythematous and itching, it is likely to be allergic dermatitis.There is history of contact with an allergen, which may be in the form of a deodorant,laundry detergent, topical cream, soap, powder, cosmetic (carried by fingers) or syntheticunderclothes. Reaction is often acute vesicular dermatitis. Less dramatic form is low-grade, chronic dermatitis. See chapter 23 for treatment of allergic vulvitis.E. A brick-red rash spreading from the inguinal folds to the vulva and down innerthigh. There is an advancing red, scaly border, with a clearing in the center is due totenia cruris. See chapter 23 for its management.F. Acanthosis nigricans is found in skin folds including axillae, groins, neck, thighs, andsubmammary areas. The lesion is wart-like, velvety, and hyperpigmented. An abdominalcancer may be associated with it, which needs to be looked for and treated appropriately.There is no satisfactory form of treatment for acanthosis nigricans.G. Lichen planus involves the vulva, wrists, and ankles. There are small (2 to 15 mm),polygonal, flat-topped, violaceous papules. The surface is usually shiny due to slightscale. The lesions last for 6 to 24 months. Deep pigmentation is left behind on healing.Itching may be absent or mild to severe only in the early part of the course of the disease.Treatment does not shorten the course of the disease, but relieves itching or improves thecosmetic appearance. It consists of topical corticosteroids, oral antihistamines, andintralesional corticosteroids (for hypertrophic lesions).H. Psoriasis : see chapter 23.I. Seborrheic dermatitis : see chapter 23.

47

Page 56: Differential Diagnosis and Management Options inuploads.worldlibrary.org/uploads/pdf/20180107071525differential... · 83 Peripartum cardiomyopathy 165 84 Hypertension 167 85 Convulsions

VULVAR LESION

Assess for similar lesions elsewhere (A)

Absent Present

Check for pain Note principal sites and characteristics of the lesion

Present (B) Absent (C) Vulva Intenselyerythematous

Assess discharge Assess discharge

Allergic vulvitis (D)Minimal Mild Moderate Minimal Mild

Assess appearanceof lesion

Assess appearanceof lesion

Assess appearanceof lesion

Assess appearanceof lesion

Assess appearanceof lesion

Vulva and adjacentskin: raised edge, lossof hair from the skin

Behcetsyndrome

Genitalherpes Syphilis LGV

Tenia cruris (E)

Factitiousulcer

Granulomainguinale

Actinomycosis

Vulva and bodyfolds:hyperpigmentation

Chancroid Hydradenitissuppuraitiva

Carcinoma Acanthosisnigricans (F)

Tuberculosis

Vulva and flexor surfaces of extremities:violaceous papules

Vulva and scalp or extensor surfaces ofextremities

Vulva and scalp: Eczematoid lesion

Lichen planus (G) Psoriasis (H) Seborrheic dermatitis (I)

48

Page 57: Differential Diagnosis and Management Options inuploads.worldlibrary.org/uploads/pdf/20180107071525differential... · 83 Peripartum cardiomyopathy 165 84 Hypertension 167 85 Convulsions

25. VulvodyniaVulvar pain syndromes are characterized by unexplained burning or anycombination of stinging, irritation, pain, or rawness located anywhere between themons and the anus and causing physical, sexual, and psychological distress(frustration, chronic stress, anxiety and depression). Vulvar pain may be due tosome disorder, or there may not be any local or neurological cause (vulvodynia).This pain may be generalized or localized, and each may be provoked (by directtouch, sexual touch or insertion of a tampon), unprovoked or mixed in nature.Trauma of the vestibular mucosa causes inflammation, which leads to varyingdegrees of vestibular mucosal hypersensitivity, pelvic floor muscle dysfunction,and general perception of pain by peripheral and central sensitization. Predisposingconditions include candidiasis, HPV infection, lichen planus, lichen sclerosus andmature neuroma.

A. Detailed history and local examination are carried out. If there is local vulvardisease, it is treated appropriately (see chapter 24).

B. In the absence of local disease, the vulva I palpated. If the patient permits thatThe site of pain is identified with the patient’s help. Q-tip test of vulva andvestibule is done in a clockwise manner (mons, labia majora, and perineal andperianal skin areas) ending in vagina. Pain mapping of vulva and vestibule is doneusing a visual pain analog scale (rated 0–10) quantify the pain.

C. If there is no pain on touching the vulva, a pelvic floor examination is donewith a lubricated finger. If it is tense, it is due to pelvic floor dysfunction. It may bedue to faulty childhood toilet training, holding patterns in adult life, imbalance ofthe pelvic girdle, disturbance of gait, postural abnormalities from musculo-skeletalabnormality such as short leg syndrome, and from vocations, anxiety and sexualabuse. Traumatic vaginal delivery can result in neuromuscular injury to the pelvicfloor. Levator avulsion injury, endometriosis, irritable bowel syndrome, interstitialcystitis or bladder pain syndrome are the most common sources of pelvic floorhypertonicity. The patient has pain at 4 o’clock of introitus if there is hypertonus ofpubococcygeus muscle, at 6 o’clock if puborectalis muscle is affected, urinaryfrequency, hesitancy, sensation of incomplete emptying, constipation, lowbackache, hip pain, rectal fissures, hemorrhoids, scoliosis and anxiety. The cause ismanaged appropriately. If the pelvic floor is relaxed, a speculum and vaginalexamination are done to identify conditions like chronic cervicitis, vaginitis, vulvarand vaginal atrophy and lichen planus. Any condition found is treatedappropriately. If the findings are normal, the diagnosis is vulvodynia. Generalmeasures include wearing 100% cotton underwear, no underwear at night, usingmild soaps for bathing with none applied to the vulva, avoiding vulvar contact withperfumes, shampoos and detergents, patting vulva dry after cleaning the vulva withwater only at bathing and after micturition, topical application of vegetable oil orplain petrolatum based emollient, using adequate lubrication during sexualintercourse and using 100% cotton sanitary pads during menses.

D. If there is pain on touching the vulva, a speculum and vaginal examination isdone after topical application of 1% lignocaine gel. See under ‘C’ for furtherevaluation and management.

E. If fungal culture is positive for Candida, it is treated (see chapter 22).

F. If the patient does not permit palpation, the cause is checked for. If it is fear, apsychiatric referral is done, reassurance and cognitive behavioral therapy are given.If it is vaginismus, pelvic floor dysfunction or vulvar malignancy, appropriatetreatment is give. After the condition is controlled, steps are taken as describedunder 'B'.

G. Congenital neuroproliferative vestibulodynia is diagnosed when the patient hasvulvar pain from time of first penetrative contact, erythema and umbilicalhypersensitivity (60% cases). It is treated with vestibulectomy. Acquiredneuroproliferative type is diagnosed when the patient has allergic reactions andchronic candidiasis along with vulvar pain. It is treated with tricyclicantidepressants, SNRI antidepressants, anticonvulsants (gabapentin, pregabalin),topical (lignocaine, capsain, gabapentin) cream, and if this treatment fails, avestibulectomy is done. Inflammatory type shows chronic infection, allergicreaction, abundant yellowish discharge, local erythema and induration. It is treatedwith interferon 1.5 MU SC thrice a week for 12 doses, triamcenolone, gabapentin,topical cromolyn, and if this treatment fails, a vestibulectomy is done.Desquamative vaginitis shows abundant yellowish leucorrhea, vulvar pruritus,vaginal erythema and cervicitis. It is treated with estradiol, clindamycin orhydrocortisone cream. Hormone induced type is due to estrogen deficiency. Itshows erythema of gland ostia, mucosal pallor, reduction in size of labia minoraand clitoris. It is managed by stopping therapy responsible (e.g. contraceptive pills,tamoxifen, spironolactone, aromatase inhibitors) and topical estradiol (0.01%) andtestosterone (0.1%) cream.

H. If the pain extends beyond the vestibule, its cause is checked for. Pudendalneuralgia shows unilateral tenderness at ischial spine (if bilateral, significantlygreater on one side), coccygeal trauma in the past, hip pain (worse with sitting,better with standing or in prone position). It is treated with pudendal nerve serialblocks, gabapentin, or pudendal nerve neuromodulation. Lichen planus showsWickham’s stria (ulceration in the vestibule), and adhesions and/or scarring of thevagina.. It is treated with clobetasol, elidel, if required systemic steroids andimmunosuppressants. Lichen sclerosus shows anogenital involvement in a “figure8” distribution without going inside the vagina. It is characterized by vulvarpruritus and pain, blotchy, wrinkled, white patches on vulva, easy bruising, tearing,blistering or ulceration. A biopsy is done to confirm the diagnosis and rule outmalignancy. It is treated with clobetasol cream, triamcenolone SC, and surgicaltreatment for phimosis or recurrent tearing of the vulva. Hypertonic pelvic floordysfunction is managed by diazepam suppositories, pelvic floor physiotherapy,vaginal dilatation and injections of botox as required.

Page 58: Differential Diagnosis and Management Options inuploads.worldlibrary.org/uploads/pdf/20180107071525differential... · 83 Peripartum cardiomyopathy 165 84 Hypertension 167 85 Convulsions

SUSPECTED VULVODYNIA

History, inspection of vulva (A)

Vulvar disease No vulvar disease

Evaluation and management (see chapter 24) Local palpation (B)

Identify site of pain with the patient’s helpPain mapping of vulva and vestibule

Q-tip test of vulva and vestibule, ending in vagina

Permits Does not permit (F)

No vulvar pain on touch (C) Vulvar pain on touch (D) Fungal culture +ve (E)Check cause

Pelvic floor examination withlubricated finger

Speculum and vaginal examination with topical applicationof 1% lignocaine gel

Fear Vaginismus Suspected malignancyPelvic floor dysfunction

Relaxed

Speculum and vaginalexamination

Tense

Pelvic floordysfunction

ReassuranceCognitive behavioral

therapyPsychiatric referral

Appropriate treatment Examination underanesthesia, biopsy.

Chronic cervicitis Vaginitis Lichen planus AtrophyNormal Pain in vestibule and beyond (H)

Vestibulodynia (G)Congenital

neuroproliferativeAcquired

neuroproliferativeInflammatory Desquamative

vaginitis

Check cause

Vestibulectomy Medical treatment

Fails

Medical treatment

Fails

Medical treatment

Hormone induced

Stop causative treatment, topical estradiol plus testosterone

Pudendal neuralgia Lichen planus Lichen sclerosus Hypertonic pelvic floor dysfunction

Pudendal nerve block,neuromodulation, gabapentin.

Clobetasol, elidel, corticosteroids,immunosuppressants

Clobetasol, triamcenolone SC, and surgicaltreatment for phimosis

50

Page 59: Differential Diagnosis and Management Options inuploads.worldlibrary.org/uploads/pdf/20180107071525differential... · 83 Peripartum cardiomyopathy 165 84 Hypertension 167 85 Convulsions

26. Female Sexual DysfunctionWomen have been so inhibited by their culture that some women live

through their marriage, childbearing, and menopause without ever experiencingsexual arousal or orgasm. However times are changing with women's education andliberalization. Now a number of women want to participate actively in their sexuallives. Some of them present with sexual dysfunction.

A. The most common female sexual dysfunction is inhibited sexual desire, whichis described by the woman as a decrease in or lack of interest in sex. It maybe from the beginning (primary) or acquired after a period of normal sexualdesire (secondary). Primary type may be due to sexual repression by parentalteaching and culture, sexual trauma in childhood or adolescence, or a poor partnerrelationship. Sympathetic and tactful conversation with the woman brings aboutthese facts. Some women may need hypnosis to get this repressed informationout. Secondary inhibition of sexual desire may be due to organic conditions likedebilitating diseases, diabetic neuropathy, drug abuse, major surgery, or psychologicalfactors like anxiety, stress, depression, fatigue, concern about an unwanted pregnancyand a change in body image. The treatment of the organic conditions resultsin restoration of sexual desire. Help of a psychiatrist should be sought for controlof the other factors with methods like education, support, psychotherapy, behavioraldesensitization, and psychoactive drugs.

B. Excitement phase dysfunction is less common than the other types of sexualdysfunction. The woman has normal sexual desire and the initiation of sexualactivity is normal. However there is decreased or no lubrication which is expected inthe excitement phase. It may be due to organic conditions like bilateral excision ofBartholin's cysts, atrophic vagina etc. When such conditions are ruled out, one has toconsider factors like poor partner relationship (differences in sexual appetites and

methods of the two partners), lack of adequate foreplay, and sexual phobias. Thefirst two causes can be corrected by explanations and counseling. A woman withsexual phobia develops patterns of avoidance which may be mistaken for low desire.The thought of sex arouses anxiety and panic. Such a woman benefits frompsychotherapy and behavioral desensitization.

C. Orgasm is a reflex with sensory input from the periphery (clitoris, lower vagina,nipples, and other parts of the body) and brain. The center is in the sacral segment ofthe cord. The efferents pass out from T11 to L2 segments causing rhythmiccontractions of pelvic muscles around the vagina at a rate of about 8/min. The centeris close to that of bladder and bowel control. Thus all 3 can be lost with injury to thelower portion of the spinal cord.

A woman with primary orgasmic dysfunction may have fear about sexualityand relationships. She may be afraid of losing control, getting pregnant, urinating, oreven giving herself pleasure. Performance anxiety is another problem faced by somewomen. Such a woman needs to be explained that being sexual is not wrong. Hersensory stimulation has to be enhanced and her involuntary control has to beextinguished. She is taught clitoral stimulation. Then she is taught to disregard herobsessive thoughts and concentrate on erotic thoughts and the premonitorysensations which precede orgasm. Use of fantasy is promoted. If she achievesorgasm by such autostimulation, normal sexual activity is allowed. Adoption of afemale-superior position helps her achieve orgasm because the coital movements arecontrolled by her in that position. If she cannot achieve an orgasm byautomanipulation, she is given sexual therapy. She may use a vibrator to achieve anorgasm, initially alone and later in presence of her partner. She may join a women'sgroup where educational issues are discussed, specific sexual tasks are given ashomework, fantasies are shared, and support is given to one another.

D. Dyspareunia is discussed in chapter 27.

.

51

Page 60: Differential Diagnosis and Management Options inuploads.worldlibrary.org/uploads/pdf/20180107071525differential... · 83 Peripartum cardiomyopathy 165 84 Hypertension 167 85 Convulsions

FEMALE SEXUAL DYSFUNCTION

Determine nature

Inhibited sexual desire (A) Excitement phase dysfunction (B) Orgasmic dysfunction (C) Dyspareunia(D)

Check typeAssess cause Assess cause See chapter 27

Primary Secondary Poorpartnerrelationship

Lack offoreplay

Sexualphobia

Fear Performanceanxiety

None Injury tolower endof thespinal cord

Assess cause Assess cause Autostimulation

Sexuallyrepressive

background

Poor partnerrelationship

Anorgasmic Orgasmic

Sexual trauma in childhoodor adolescence

Sexual therapy Normal sexFemale superior position

Debilitatingdisease

Drug abuse Anxiety Depression Concernaboutpregnancy

Surgery Stress Fatigue Change in body image

52

Page 61: Differential Diagnosis and Management Options inuploads.worldlibrary.org/uploads/pdf/20180107071525differential... · 83 Peripartum cardiomyopathy 165 84 Hypertension 167 85 Convulsions

27. Painful DyspareuniaDyspareunia is difficulty in sexual intercourse. It may be due to pain or

obstruction. If forced intercourse is attempted in presence of an obstructive lesion, itmay cause pain.

A. The pain on an attempt at sexual intercourse may be superficial or deep.If penetration has not been possible at any time in the past, then local examinationreveals the diagnosis. The vagina may be absent, as in mullerian agenesis, vaginalatresia, and total androgen insensitivity. Such women are treated by Mclndoe'svaginoplasty (see chapter 39). If the local examination reveals no abnormality,a bimanual pelvic examination is done. If the woman experiences severe painlocally, lifts her lower back and buttocks off the examination table, approximatesher thighs, and pushes away the examining hand on an attempt to pass a fingerin the vagina, the diagnosis is vaginismus. She needs the help of a psychiatristto get well. If vaginal examination is normal, the dyspareunia is due to lackof lubrication. Such a woman is advised to have adequate foreplay, and if thatfails to achieve adequate lubrication, is prescribed a lubricant jelly for local applicationbefore penetration.

B. If the woman has superficial dyspareunia, but penetration has been possiblebefore, local examination is done to find the cause. It is a painful condition aroundthe introitus, as follows.

1. Vulvitis: the vulva is red, tender and swollen. The cause is candidiasis, diabetesmellitus, or allergic vulvitis. The etiological condition is treated appropriately.

2. Bartholinitis: the Bartholin's gland is painful and tender. It is palpable in thedeep perineal pouch posterolateral to the introitus. It responds to antibiotics and anNSAID.

3. Bartholin's abscess: it is of shorter duration, and extremely painful. The womanhas fever with chills. There is a tender, fluctuant, cystic swelling deep to theposterior one-third of a labium majus.

4. Vaginitis: see chapter 22.5. Ulcerative lesion of the vulva: see chapter 24.6. Infected wound: episiotomy, perineal tear, or perineorrhaphy wound may get

infected and cause painful dyspareunia.7. Atrophic vulvar dystrophy: it causes a stricture of the introitus and hence

dyspareunia.

8. Scarring: it may be due to local application of caustic agents, such as rock saltinserted by certain tribes in Africa for restoring the original dimensions of thevagina after childbirth. It needs plastic repair.

C. Deep dyspareunia is pain felt deep inside the pelvis and vagina. It is related to apainful condition within the pelvis or upper vagina, such as the following.

1. Endometriosis: endometriotic lesions in the uterosacral ligaments and therectovaginal septum cause pain on contact with the penis. If the uterus isretroverted and fixed due to endometriosis, there is collision dyspareunia. Seechapter 16 for further discussion on endometriosis.

2. Pelvic inflammatory disease: there is acute inflammation of the pelvic structuresin acute pelvic infections. There is severe pain whenever any of these structures

Obstructive Lesions Which Cause Dyspareunia

Vaginal agenesisVaginal septum : longitudinal or transverseVaginal strictureVaginal condyloma acuminata (extensive)Elephantiasis of the vulvaLarge polyp in the vaginaOvercorrection in a perineorrhaphy

is touched vaginally, either during intercourse or vaginal examination. Inchronic pelvic infection, there are two sources of pelvic pain during coitus. Oneis tubo-ovarian masses, and the other is a retroverted fixed uterus. The formerare tender on palpation unless quiescent. The latter causes collisiondyspareunia. See chapters 13 and 14 for further discussion on pelvic infections.

3. Ovary in pouch of Douglas: the ovary may be prolapsed in the pouch ofDouglas and fixed there by adhesions. It causes a sickening type of pain onforceful contact with the penis (collision dyspareunia). It may be an iatrogeniccondition, when the woman undergoes a vaginal hysterectomy, and the cornualstumps are used to suspend the vault of vagina. In such a case, the ovary getsfixed to the vault of vagina, and is touched during coitus. Adoption of a lateralor female superior position may relieve this type of dyspareunia. If it does not,an ovarian suspension operation is required after separating the ovaries fromthe top of the vagina.

53

Page 62: Differential Diagnosis and Management Options inuploads.worldlibrary.org/uploads/pdf/20180107071525differential... · 83 Peripartum cardiomyopathy 165 84 Hypertension 167 85 Convulsions

PAINFUL DYSPAREUNIA

Check the site of pain

Superficial (A) Deep (C)

Check possibility of digital penetration Bimanual pelvic examination

Not possible Possible Retroverted, fixed uterusTender nodules inuterosacral ligaments

Vaginal warmth,retroverted fixeduterus, tender adnexalmasses

Tender nodule inposterior fornix

Local examination Local examination

NormalAbsence ofvagina

Infection Atrophy Scarring Endometriosis Pelvicinflammatorydisease

Ovarianprolapse inpouch ofDouglas

See chapter 38 Atrophic vulvardystrophy

Bimanual pelvic examination Inflamed, redvulva

Ulcerativelesions

Infectedwound

Withdrawal response

Vaginismus

Poor lubrication Vulvitis See chapter 24

Painful, tender swellingat 5 or 7 o’clock position

Assess consistency

Solid Cystic

Bartholinitis Bartholin’s abscess

54

Page 63: Differential Diagnosis and Management Options inuploads.worldlibrary.org/uploads/pdf/20180107071525differential... · 83 Peripartum cardiomyopathy 165 84 Hypertension 167 85 Convulsions

29. Urinary IncontinenceA woman in continent for urine as long as her urinary tract is intact and the

pressure closing the urethra remains greater than the intravesical pressure. If thenormal route is bypassed (e.g. a fistula) or if the pressure gradient is reversed, shebecomes incontinent.

A. If the woman loses urine continuously, she is most likely to have a urinaryfistula. She cannot void on her own in case of a vesico-vaginal fistula (VVF)unless it is very small. When it is very small, the rate of filling the bladder is greaterthan the rate of the leak, and she can pass some urine on her own too. She can voiddespite the incontinence in case of a ureterovaginal fistula (UVF) because the otherureter continues to drain urine into an intact urinary bladder. A speculumexamination may show a fistulous opening draining urine into the vagina. Even if itis not seen, sterile methylene blue solution is instilled into the bladder through atransurethral catheter. If the dye is found to leak into the vagina, it is VVF. If it is notseen to leak, it may be a small VVF or UVF. The differentiation is made byperforming a three-swab test. The urinary catheter is removed keeping 200 ml of thedye solution in the bladder. Three cotton swabs are placed in the vagina one belowanother. Then she is asked to walk about for half an hour. The swabs are thenremoved and checked for staining with the dye. If the upper and/or middle swab isstained, it is VVF. If none is stained, but the upper one is soaked with urine, it isUVF. If only the lower swab is stained, it is a type of urge incontinence. Urodynamicstudies are done to determine the type (see later).

B. If the woman in incontinent with stress, the nature and volume of urinaryloss are evaluated. In case of urinary stress incontinence (USI), there is a lossof a few drops of urine coincident with the stress. It is totally preventable byvoluntary contraction of external urinary sphincter. The diagnosis is confirmedby Bonney's test, in which she becomes continent on stress when the urethro-vesical junction is elevated above the urogenital diaphragm with a finger oneither side of it. Marchetti's test is similar, except that two Allis' forceps areused under local anesthesia in place of the fingers to avoid continence by directpressure of the fingers over the urethra. If the woman loses a stream of urine insteadof a few drops, and the incontinence begins a few seconds after the stress, she has

motor urge incontinence (MUI).

C. MUI is the result of uninhibited, involuntary detrusor contractions. It maybe with stress, or with stimuli which may be cold, running water (sight or sound),or psychogenic.

D. MUI may be manifest as spontaneous and intermittent loss of large volumesof urine, with symptoms like frequency and urgency. Urodynamic studies aredone to confirm the diagnosis. The findings are involuntary detrusor contractionsat rest, during bladder filling, or after provocative maneuvers like coughing,hand washing, and bouncing on heels. Cystourethroscopy often shows bladdertrabeculation due to hypertrophy of the detrusor.If the spontaneous and intermittent loss of a large volume of urine is associated withdysuria, it is likely to be sensory urge incontinence (SUI). If it is associated withlarge residual volume of urine, it is overflow incontinence. Urodynamic studies inthe former show a small cystometric capacity, urethral relaxation or a markedvariation in the urethral pressure profile, and an absence of detrusor activity.Cystourethroscopy may show the etiological condition like cystitis, diverticulum,foreign body, or neoplasia. In case of overflow incontinence, the bladder capacity islarge (often > 1200 ml). The bladder sensation is decreased. Detrusor contractility ispoor or absent.

Causes of Overflow Incontinence

Diabetic neuropathy

Lower motor neuropathyPostoperative obstruction

Pelvic massMassive prolapse with urethral kinking

Drugs: ganglionic blocking agents, alpha-adrenergic agonists,anticholinergic drugs.

E. Though a diagnosis of psychogenic incontinence is most often easy in womenwith psychiatric disorders, psychogenic factors of more subtle nature are responsiblein a number of cases. Behavior therapy is useful in such cases.

57

Page 64: Differential Diagnosis and Management Options inuploads.worldlibrary.org/uploads/pdf/20180107071525differential... · 83 Peripartum cardiomyopathy 165 84 Hypertension 167 85 Convulsions

URINARY INCONTINENCE

Check nature of incontinence

Constant (A)

Speculum examination

With stress (B) With stimuli (C) Spontaneous, intermittent,large volume (D)

Psychogenic

Fistula Fistula is notdetected

Few drops coincidentwith stress

Stream follows stress Check symptoms

Transurethral instillation of methylene bluesolution into bladder. Look for leakage into

vagina.

Clinical tests Frequency,urgency

Dysuria None

Yes No Urinary stressincontinence

Motor urge incontinence Sensory urgeincontinence

Large residualvolume

VVF Three swab test. Check swabstained with methylene blue.

UrodynamicstudiesCystoscopyUrethroscopy

Overflowincontinence

Upper and/or middle None, but uppersoaked with urine

Lower Urodynamicstudies

VVF UVF Urge incontinence

58

Page 65: Differential Diagnosis and Management Options inuploads.worldlibrary.org/uploads/pdf/20180107071525differential... · 83 Peripartum cardiomyopathy 165 84 Hypertension 167 85 Convulsions

30. Frequency of MicturitionA woman normally urinates 2 to 7 times during the waking hours, and may or maynot get up once during the sleeping hours for the same. Any increase in this numberis called frequency of micturition.

A. Presence of dysuria along with frequency is strongly suggestive of cystourethritisas the cause of the frequency. In addition, the woman may have the followingfeatures.

1. Fever with chills.2. Urgency, i.e. a sensation that immediate voiding cannot be delayed.3. Incontinence.4. Costovertebral angle pain (with pyelonephritis).

If more than 6 to 8 leukocytes are seen per high power field on microscopicexamination of a carefully collected centrifuged urine sample, urinary tract infectionis diagnosed. White blood cell casts may be found with pyelonephritis. Hemogram isnormal with cystourethritis, while leukocytosis is seen with pyelonephritis.Microbiologic studies are done on a mid-stream clean-catch urine sample. A colonycount of > 105/ml is diagnostic of urinary tract infection. If there is pyuria but noorganisms are grown, the diagnosis is tuberculous infection or chlamydial urethritis.The distinction can be made by cystourethroscopy.

B. If there is no infection of the urinary tract, urine output in 24 hours is measured.

If it is increased (polyuria), medical consultation is advised for conditions likediabetes mellitus, diabetes insipidus etc.

C. If the 24 hours urine output is normal, a local gynecologic or obstetric conditionis the most likely cause of the frequency of micturition. A speculum and bimanualpelvic examination is done, which may reveal any of the following conditions.

1. Cystocele: there is some residual urine left in the cystocele, so that addition ofsome more urine to it results in a bladder sensation and a desire to urinate. Residualurine also promotes urinary tract infection due to stasis. If infection is ruled out, thesymptoms are corrected by an anterior colporrhaphy.

2. Pregnancy: a gravid uterus may press on the urinary bladder, and cause frequencyof micturition. It is a not uncommon symptom in the first trimester. Even near term,when the head gets engaged, bladder compression and frequency of micturition maytake place.

3. A leiomyomatous uterus may press on the urinary bladder and produce frequency ofmicturition. See chapters 20 and 52 for further discussion on the management ofuterine leiomyomas.

4. Large residual volume of urine causes a frequency of micturition because thebladder is filled quickly to the threshold at which a desire to urinate is experienced.It is due to chronic retention of urine with overflow. Medical consultation is requiredto treat this condition.

59

Page 66: Differential Diagnosis and Management Options inuploads.worldlibrary.org/uploads/pdf/20180107071525differential... · 83 Peripartum cardiomyopathy 165 84 Hypertension 167 85 Convulsions

FREQUENCY OF MICTURITION

Assess for dysuria (A)

Present Absent

UrinalysisMicrobiologic studies

Check 24 hours urine output (B)

Colony count> 105/ml

Colony count< 105/ml

Pyuria, no growth ofbacteria

Increased Normal

Repeat test Cystourethroscopy Polyuria Gynecologic examination

> 105/ml < 105/ml Medical consultation Cystocele Pregnancy Leiomyoma

Urinary tract infection Contamination Large residual volume

Tuberculouscystitis

Chlamydiaurethritis

Retention with overflow

Medical consultation

60

Page 67: Differential Diagnosis and Management Options inuploads.worldlibrary.org/uploads/pdf/20180107071525differential... · 83 Peripartum cardiomyopathy 165 84 Hypertension 167 85 Convulsions

31. Retention of UrineA. If a woman suddenly develops retention of urine, the first step is to adoptconservative measures to help her pass urine. She is given a hot water bag forapplication to the suprapubic area. If that fails, she is taken to a bathroom andexposed to the sight and sound of running water. If she still fails to pass urine,she is catheterized aseptically. The volume of urine drained is measured. If shehas passed urine on her own after conservative measures or the volume of urinedrained is more than 350 ml, the cause of urinary retention is assessed.

B. Neurologic examination is done. Multiple sclerosis, prolapsed intervertebraldisc, spinal cord lesions, and cerebrovascular diseases can cause retention ofurine. Such a woman is advised neurologic consultation.

C. A history of ingestion of drugs is sought. Various drugs are known to causeretention of urine e.g. anticholinergics, narcotics, ganglion blocking agents,phenothiazines, alpha-adrenergics, and benzodiazepines. If a drug is found tobe responsible, an alternative drug for the same therapeutic action should beused.

D. A woman may develop retention of urine postoperatively due to various reasons,as follows.

1. A woman may not be able to pass urine in a lying down position. A propped upposition on a bed pan is useful.

2. Pain in perineal sutures of episiotomy, perineal tear repair, or perineorrhaphymay reflexly cause spasm of the external urinary sphincter and retention of urine.Adequate analgesia relieves the spasm.

3. Dennervation of the bladder due to dissection in operations like anteriorcolporrhaphy, repair of urinary stress incontinence by Kelly’s operation or urethralsling operation, and Wertheim's radical hysterectomy often causes retention of urine.Continuous decompression of the bladder with a self-retaining catheter for 3, 5, and

15 to 21 days respectively postoperatively prevents this complication. If it doesdevelop after removal of the catheter, the bladder is catheterized and drainedcontinuously again for at least 48 hours. Then the catheter is clamped. The clamp isreleased every 2 hours for 1 day and every 3 hours for the next day. Then thecatheter is removed, and the woman is advised to pass urine every 2 hours. Theresidual urine in the bladder is checked by catheterization after spontaneous voiding.If it is less than 50 ml, no further treatment is required. If it is more than 50 ml, eitherthe bladder is recatheterized for further decompression followed by training, or acholinergic drug is used, such as carbachol. Microbiologic studies are done on theurine, since urinary infection is likely after prolonged catheterization. If it is found, itis treated with appropriate antibiotic as suggested by the antibiotic sensitivity report.

E. Urinary retention may be due to obstruction. If it is due to a retrovertedgravid uterus, a Foley's catheter is passed into the bladder and it is emptied slowly.Rapid emptying is avoided because it may cause engorgement of vesical mucosalveins, their rupture, and hematuria. The woman is kept in a prone position. Manualcorrection is usually unnecessary. The uterus falls forwards and grows into theabdomen. Then the problem does not recur.

Other obstructive conditions responsible for retention of urine are cervical leiomyoma,hematocolpos, pelvic abscess, and uterine prolapse. In such cases the bladder isdrained through a Foley's catheter until definitive treatment of the etiologicalcondition is given.

F. Acute cystourethritis can cause severe pain and retention of urine. See chapter30 for further discussion on the management of this condition.

G. If the volume of urine drained is small, the diagnosis is oliguria (< 400ml/24 hrs) or anuria (< 100 ml/24 hours). The specific gravity of the urine ismeasured. If it is less than 1020, the diagnosis of renal decompensation is made, andmedical consultation is advised. If it is 1020 or greater, ureteral obstruction isthe likely diagnosis. A surgical consultation is required for such a case.

61

Page 68: Differential Diagnosis and Management Options inuploads.worldlibrary.org/uploads/pdf/20180107071525differential... · 83 Peripartum cardiomyopathy 165 84 Hypertension 167 85 Convulsions

RETENTION OF URINE

Conservative measures (A)

Patient passes urine Patient cannot pass urine

Catheterize bladderCheck volume of urine drained

Assess cause

> 350 ml Oliguria (G)

Neurologic condition(B)

Drug effect (C) Postoperative (D) Obstruction (E) Cystourethritis (F) Medical/surgicalconsultation

Neurologic management Use alternative form ofmedication

Gravity drainage byindwelling catheterBladder training

Retrovertedgravid uterus

Other Antibiotic therapy

Prone positionGravity drainageby indwellingcatheterBladder training

Gravity drainage by indwellingcatheter until definitive treatment of

the cause

62

Page 69: Differential Diagnosis and Management Options inuploads.worldlibrary.org/uploads/pdf/20180107071525differential... · 83 Peripartum cardiomyopathy 165 84 Hypertension 167 85 Convulsions

31. HirsutismA. Hirsutism is development of coarse, pigmented hair on the face, upper back,chest, and/or abdomen. Virilization differs from it in that the woman has temporalbaldness, deepening of voice, male type of musculature, and clitoromegaly inaddition to hirsutism. Hirsutism associated with elevation of serum androgenlevels is more often associated with menstrual disturbances (e.g. oligomenorrhea,amenorrhea) than when it is associated with normal androgen levels.

Classification of hirsutismDensity (D) Mild: 1 Moderate: 2 Profuse: 3Quality (Q) Fine: 1 Moderately coarse: 2 Very coarse: 3

Stage 1 D1Q1 No predilection for virilizing sites.Stage 2 D1Q2 Normal distribution of sites (Upper lip, sideburns,

intermammary, periareolar, midabdomen).Stage 3 D2Q2 to

D3Q3Sites as in stage 4, but without virilization.

Stage 4 D3Q3 Normal distribution sites, also over upper back, shoulder,inner thighs, supragluteal and gluteal areas, with temporalhair recession and other signs of virilization.

B. Hirsutism is known to occur in families. The cause may be increased sensitivityto androgens, so that normal levels of the same cause hirsutism. Even if thefamily history is positive, the woman needs to undergo full evaluation beforebeing diagnosed to have familial hirsutism. Late onset adrenal hyperplasia isknown to be familial.C. Drugs known to cause hirsutism are methyl testosterone, dilantin, danazol,corticosteroids, and anabolic steroids like oxandrolone. If alternative mode oftreatment is possible, it should be given instead of the drug which has causedhirsutism.D. Cushing's syndrome may occur due to any of the following conditions.

1. Adrenal adenoma, carcinoma, or nodular hyperplasia.2. Excessive ACTH production by the pituitary (Cushing's disease).3. Ectopic ACTH production by a nonpituitary tumor, e.g. small cell carcinoma of

lung or bronchial carcinoid.The clinical picture is the same with all of these conditions. These include trunkalobesity, muscle wasting, plethora, hypertension, diabetes mellitus, menstrualdisturbances, abdominal striae, and atrophy of subcutaneous tissue. Such a woman isreferred to an endocrinologist for further evaluation and treatment.If the woman does not have Cushing's syndrome, serum dehydroepiandrosteronesulfate (DHEAS) and testosterone (T) levels are assayed.E. If serum DHEAS and T levels are normal, the woman has idiopathic hirsutism.Increased peripheral androgen metabolism may be responsible for the excessivehair growth. Many women with idiopathic hirsutism have an elevated level ofserum androstanediol glucuronide, which reflects increased peripheral androgenmetabolism in the skin and hair follicles. The treatment is cosmetic measures.F. Serum DHEAS level above 7 μg/ml and normal serum T level indicate

the presence of a neoplasm, either in the adrenal cortex (producing DHEAS), or inthe pituitary (producing ACTH). CT scan of abdomen and head respectively confirmthe diagnosis. Such tumors are treated surgically. It must be remembered that a fewadrenal tumors produce testosterone too. If both the scans are normal, serum 17alpha-hydroxy progesterone [ 17 (OH) P ] level is estimated. A level above 9 nmol/L(> 3 μg/L) is indicative of late onset adrenal hyperplasia. Such a woman is referredto an endocrinologist for suppressive glucocorticoid therapy.G. Serum DHEAS level below 7 μg/ml and elevation of serum T above normal ( >80 ng/dl) suggests an ovarian cause of the hirsutism. If serum T level is below 200ng/ml, serum 17-OHP < 200 ng/dl assay is done. If it is < 200 ng/dl, the diagnosis ispolycystic ovarian disease (PCOD), ovarian stromal hyperplasia, ovarian stromalhyperthecosis, or theca lutein cysts. Pelvic ultrasonography is done to differentiatebetween these conditions. A biopsy of an ovary may be required to differentiatebetween ovarian stromal hyperplasia and hyperthecosis. Ovarian androgen productionis suppressed by the use of oral combination contraceptive pills. Use of a combinationpill containing drospirenone is useful, owing to its antiandrogenic action. PCOD istreated with cyclical therapy with medroxyprogesterone acetate to suppress LHlevels, break the vicious cycle of excessive androgen production and elevation of LHlevels, change endometrial proliferative or hyperplastic pattern to secretory pattern,and achieve regular menstruation. If the woman with PCOD desires a pregnancy,ovulation is induced with clomiphene citrate, and if that fails, with FSH and hCG.H. If serum T level is above 200 ng/ml and 17-OHP < 200 ng/dl, an androgenproducing tumor of the ovary (arrhenoblastoma, gynandroblastoma, hilus cell tumor,and adrenal rest tumor) is the most likely diagnosis. The diagnosis is confirmed bypelvic ultrasonography, and if necessary, a CT scan or MRI. Such a tumor isremoved by oophorectomy if the woman is aged up to 50 years, and by totalhysterectomy and bilateral salpingo-oophorectomy if she is aged above 50 years. Ifthe woman does not have an ovarian tumor, an adrenal tumor producing testosteroneis looked for with a CT scan or MRI.I. If T is < 200 ng/dl and 17-OHP > 200 ng/dl, ACTH stimulation test is done. If 17 -OHP is < 1000 ng/dl, i is heterozygote carrier of 21-hydroxylase deficiency. if 17-OHP is > 1000 ng/dl, it is late-onset adrenal hyperplasia/21-hydroxylase deficiency.thesepaiens are referred o an endocrinologist.Treatment of the causative condition is combined with cospetic measures and drugtherapy. Cosmetic measures are used while awaiting results of drug therapy. Theyinclude bleaching, plucking, shaving, waxing, chemical treatment and galvanicelectrolysis (alone or with thermolysis, laser therapy and intense pulsed light. Drugtherapy includes the following.1. Eflornithine (13.9%) cream: reversibly slows facial hair growth in 70% cases.2. Low-dose OCPs containing a neutral progestin (desogestrel or gestodene) or

an antiandrogen (cyproterone acetate, chlormadinone acetate or thespironolactone-derivative drospirenone).

3. Antiandrogen alone if estrogen is contraindicated: flutamide (250 mg/day),finasteride (5 mg/day) or spironolactone (100 mg/day).

4. Besides drug treatment to prevent further progress of the disease, the existinghair has to be removed for cosmetic reasons. This is achieved by bleaching,waxing, shaving, and/or electrolysis.

63

Page 70: Differential Diagnosis and Management Options inuploads.worldlibrary.org/uploads/pdf/20180107071525differential... · 83 Peripartum cardiomyopathy 165 84 Hypertension 167 85 Convulsions

HIRSUTISM

Check family history

Yes (B) No

Familial Drug administration (C)

Yes No

Drug induced Check for features of Cushing’s syndrome(D)

Yes No

Perform confirmatory tests Negative Serum DHEAS and T assay (G)

Positive DHEAS < 7 μg/mlT < 80 ng/ml (E)

DHEAS > 7 μg/mlT < 80 ng/dl (F)

Ovarian cause DHEAS < 7 μg/mlT > 80 ng/dl

Cushing’ssyndrome

Idiopathic CT scan/MRI Serum 17 OHP assay(H)

Normal T < 200 ng/dl17-OHP < 200 ng/dl

T > 200 ng/dl17 -OHP < 200 ng/dl

T < 200 ng/dl17-OHP > 200 ng/dl (I)

Adrenal tumor Pituitary tumorNo adrenal disease Pelvic imaging ACTH stimulation test

Late onset adrenal hyperplasia Pelvic USG Ovarian tumor17 -OHP < 1000 ng/dl 17-OHP > 1000 ng/dl

Anovulation PCOD Ovarianstromal

hyperplasia

Ovarian stromalhyperthecosis

Theca luteincysts

Heterozygotecarrier of 21-hydroxylasedeficiency

Late-onsetadrenal

hyperplasia/21-hydroxylasedeficiency

64

Page 71: Differential Diagnosis and Management Options inuploads.worldlibrary.org/uploads/pdf/20180107071525differential... · 83 Peripartum cardiomyopathy 165 84 Hypertension 167 85 Convulsions

33. GalactorrheaGalactorrhea is expression of milky fluid from the nipple(s). It can be

unilateral or bilateral. When in doubt a drop of the fluid is observed under amicroscope. Presence of fat droplets in the fluid confirms the diagnosis. Galactorrheaneeds evaluation not only because it is associated with menstrual disturbances andinfertility, but also because it may be due to a serious disorder. The first step is torule out a pregnancy or a local breast disease by clinical examination and amammography.

A. A number of drugs are known to cause galactorrhea, e.g. methyldopa, reserpine,amphetamine, metoclopramide, opiates, estrogen, medroxyprogesterone acetate,phenothiazines, tricyclic antidepressants, butyrophenones, isonex, and cimetidine.If the woman is receiving any of these drugs, she should be reassured that hergalactorrhea is a drug effect. An alternative drug may be used instead, if possible.

B. An irritant chest wall lesion may stimulate the afferents for milk let downreflex, which acts at the level of pituitary to cause release of oxytocin (for milklet down), and synthesis and release of prolactin. Examples of such lesions arethoracotomy scar, burns scar, and herpes zoster.

C. Breast stimulation may act in a similar manner to increase prolactin output.It may be due to excessive stimulation during sexual activity, breast lump, orbreast abscess. Sexual arousal by breast stimulation should be appropriately reduced.Breast lump and breast abscess need surgical management.

D. Serious neurologic disorders may be associated with galactorrhea.1. Hypothalamic tumors: they destroy areas responsible for secretion of prolactin

inhibiting factor (PIF).2. Hypothalamic infarction, infiltration, or irradiation: action is as above.3. Pineal tumors: the action is as above.4. Syringomyelia: there is stimulation of the afferent of the milk let down reflex at

the level of the spinal cord.5. Tabes dorsalis: action is as above.6. Pituitary stalk transection or compression: there is interruption of PIF transport

to the pituitary.7. Empty sella syndrome, aneurysm or cyst in sella turcica, pseudotumor cerebri:

action is as above.Neurologic referral is made for the management of such disorders.

E. Endocrine disorders like adrenocortical hypofunction and primary hypothyroidism,are known to cause galactorrhea. They are managed medically. Estrogen secretingovarian tumors and polycystic ovarian disease can cause galactorrhea. The formeris removed surgically. See chapter 35 for the management of the latter.

F. Some medical disorders are known to cause galactorrhea, e.g. cirrhosis ofliver, chronic renal failure, and acute intermittent porphyria. If any such conditionis diagnosed, the woman is referred to a physician for management.

G. If the conditions discussed so far are absent, serum prolactin (PRL) levelis estimated. Normal level is 5 to 20 ng/ml. If the level is normal and the woman'smenstrual pattern is also normal, no further evaluation is necessary. But if itis above 20 ng/ml or there is oligomenorrhea or amenorrhea, serum thyroidstimulating hormone (TSH) level is estimated. If it is elevated, the woman isreferred to an endocrinologist for management. If it is normal, the woman isinvestigated for the presence of a pituitary adenoma. Pituitary tumors more than1 cm in diameter (macroadenoma) can be detected by obtaining a skull radiograph(lateral view), or preferably a coned down view of the sella turcica. Radiologicfeatures of a pituitary tumor are enlargement of the sella turcica, destructionof the wall(s) of the sella turcica, double floor of the sella turcica, and separationof the posterior clinoid process from the posterior wall of the sella turcica. Ifthe woman has a pituitary macroadenoma, she is referred to a neurosurgeon forfurther management.

Pituitary adenomas measuring up to 1 cm in diameter (microadenomas)cannot be detected by plain radiography. A CT scan of the head is obtained fordetection of a microadenoma. If it is present, the woman is given bromergocriptine(see chapter 35), which causes lowering of serum PRL level, ovulation, and apregnancy if so desired. Cabergoline is more specific for type 2 (D2) dopaminereceptor. It is more potent than bromergocriptine and has a long half life. Its initialdose is 0.25-0.5 mg PO twice weekly. It is adjusted every 2-3 months untilstabilization. A median dose of 1 mg/wk is enough in most cases, which can bereduced to 0.5 mg/wk once prolactin level is controlled. It is effective in 85% ofbromergocriptine intolerant cases, and 70% of bromergocriptine resistant cases. Aclose watch is maintained clinically and with periodic CT scans for enlargement ofthe microadenoma, especially during a pregnancy. Joint consultation with aneurologist is essential in the management of such women.

65

Page 72: Differential Diagnosis and Management Options inuploads.worldlibrary.org/uploads/pdf/20180107071525differential... · 83 Peripartum cardiomyopathy 165 84 Hypertension 167 85 Convulsions

Rule out a pregnancy and breast disease.

GALACTORRHEA

Check for drug administration (A)

Present Absent

Check for chest wall lesion (B)

Present Absent

Check for breast stimulation (C)

Present Absent

Check for neurologic disease (D)

Present Absent

Check for endocrine disorder (E)

Present Absent

Check for medical disorder (E)

Present Absent

Serum prolactin assay

Normal Elevated

Assess menstrual pattern Oligomenorrhea Serum TSH assay

Normal Normal Elevated

Absent Assess for pituitary adenoma Present Thyroid function tests

Bromergocriptine/cabergoline Bromergocriptine/cabergoline, definitive therapy

66

Page 73: Differential Diagnosis and Management Options inuploads.worldlibrary.org/uploads/pdf/20180107071525differential... · 83 Peripartum cardiomyopathy 165 84 Hypertension 167 85 Convulsions

34. InfertilityA. If a couple has had at least one year of sexual intercourse without the useof any contraceptives and the woman has failed to conceive, investigations aredone for infertility. The menstrual history and coital history are important. Ifthe woman's menstrual cycles are every month, regular, and painful, she is likelyto be ovulating regularly. If the cycles are very short, very long, irregular, orpainless, anovulation is likely. Regular coitus around the time of ovulation isimportant for conception. Ovulation takes place 14 days before the onset ofmenstruation. Some women get vaginal spotting with ovulation. Abstinence fromsex at this time owing to the social and cultural custom of abstaining duringmenstruation would result in infertility.

B. Male factor is responsible for infertility in 30% cases. Evaluation of themale is easy and noninvasive. Semen is collected by masturbation and analyzed(see chapter 37). If the male factor is normal, the woman is investigated forinfertility.

C. If the male factor is found to be normal, serial cervical mucus studies anda postcoital test are done (see chapter 36). If sperm are not seen in the mucusthough the Insler score is good, it is confirmed that the couple did have sexualactivity the previous night. Sexual activity is a function which is highly dependenton the psychological state of the male, and though normally possible for him,a command performance may prove to be impossible. If coitus has not takenplace, the test is repeated in the next cycle. If it has taken place, all sperm might havepassed up the cervical canal (which cannot be diagnosed by conventional means)or they have not been able to penetrate the mucus. Such cases and those withimmotile sperm in the mucus are subjected to assessment of the cervical factor(see chapter 36). It is believed that the role of the postcoital test is limited in moderngynecology. If the test is normal, ovulation studies are done.

D. A large number of tests have been described to detect ovulation, such as basalbody temperature charting, changes in the cervical mucus and vaginal smear,endometrial biopsy, and serum progesterone assay. These tests are dependent on theeffects of progesterone, which is produced by the corpus luteum formed as a result ofovulation. Such effects can be produced whenever there is progesterone in circulation

irrespective of its source. Luteinized unruptured follicle produces progesteronewhich causes a false positive result in each of these tests. Follicular study byultrasonography is the only test which is diagnostic of ovulation, which helpspinpoint the day of ovulation, and which can be repeated cycle after cyclenoninvasively. Follicular growth, growth of one dominant follicle and its rupture aremonitored starting from eighth or ninth day of the cycle. Failure of follicular growthand failure of the dominant follicle to rupture can be differentiated onultrasonography. Corpus luteum insufficiency cannot be diagnosed by ultrasonography,and needs other tests (see chapter 12).

E. Hysteroscopy is done first. It detects abnormalities on the inside of the uterus,such as endometrial tuberculosis (see chapter 3), endometrial synechiae or Ashermansyndrome (see chapter 3), and submucous leiomyomas (see chapter 20). Theseconditions are treated appropriately. Laparoscopy and chromopertubation detecttubal factors and pelvic peritoneal factors like endometriosis (see chapter 16), andgenital tuberculosis (see chapter 3) responsible for infertility. Tubal factors includethe following.

1. Tubal block: the block may be due to occlusion of the tube in cornu, isthmus,ampulla, infundibulum, or fimbrial end (hydrosalpinx). It may be due to kinking ofthe tube due to pull of a band of adhesion between one surface of the tube and someadjacent structure. It may also be due to a band of adhesion passing across the tubeand compressing it.

2. Peritubal adhesions: these impair the mobility of the fallopian tube and thefunction of the fimbriae, thereby causing infertility.

If repair of the fallopian tube is possible, a tuboplasty is done. If that is notpossible or is contraindicated, the woman is advised to undergo in vitro fertilizationand embryo transfer (IVF-ET), or to adopt a child. In developed countries, tubalreconstructive surgery is not done much, and in vitro fertilization is offered as theprimary line of treatment for tubal factor in infertility. However owing to high costand lack of adequate number of centers offering that form of treatment, it is offeredonly if tubal reconstructive surgery cannot be done or has failed. If the tubal andpelvic peritoneal factors are found to be normal, the woman has unexplainedinfertility. It is managed by intrauterine insemination, and if that fails, in vitrofertilization and embryo transfer.

67

Page 74: Differential Diagnosis and Management Options inuploads.worldlibrary.org/uploads/pdf/20180107071525differential... · 83 Peripartum cardiomyopathy 165 84 Hypertension 167 85 Convulsions

INFERTILITYClinical assessment (A)

Assessment of male (B) Assessment of female

See chapter 37 Postcoital test (C)

Normal No sperm seen Immotile sperm seen

Ovulation studies (D) Confirm sexual activity Assess for spermantibodies

Anovulation Ovulation Corpus luteum insufficiency Present Absent See chapter 36

See chapter 35 See chapter 35 Assess cervical factor Repeat test in the next cycle

Hysteroscopy, laparoscopy with chromopertubation

Normal Uterine factor Endometriosis Tubal factor

Tuberculosis

See chapter 3

Ashermansyndrome

Leiomyoma Peritubaladhesions

Hydrosalpinx Midsegmentblock

Cornual block Irreparablydamagedtube

Unexplainedinfertility

See chapter 3 See chapter 20 Adhesiolysis Salpingostomy Confirm byhysterosalpingography

Confirm byhysterosalpingography

IVF-ET

Midsegmentreconstruction

Tubocornualanastomosis

68

Page 75: Differential Diagnosis and Management Options inuploads.worldlibrary.org/uploads/pdf/20180107071525differential... · 83 Peripartum cardiomyopathy 165 84 Hypertension 167 85 Convulsions

35. Induction of OvulationA. Anovulation is absence of ovulation. It may be of any of the following types.

1. Hypogonadotropic: such a woman has amenorrhea, negative result of aprogesterone challenge test, positive result of an estrogen challenge test, andlow levels of serum FSH and LH.

2. Eugonadotropic: such a woman has anovulatory cycles (see chapter 5), oramenorrhea which responds to a progesterone challenge test. It is not necessaryto estimate serum FSH and LH levels for diagnosis. These levels are anywaynormal.

3. Hypergonadotropic: such a woman has amenorrhea which does not respond to aprogesterone challenge test, but does respond to an estrogen challenge test.Serum FSH and LH levels are very high due to loss of negative feedback actionof estrogen and progesterone (due to ovarian failure).

4. Hyperprolactinemic: the woman usually has galactorrhea and amenorrhea,associated with elevation of serum prolactin (PRL) level.

B. Ovulation induction with human menopausal gonadotropin (hMG) and humanchorionic gonadotropin (hCG) is begun on day 1 of the menstrual cycle. Theprotocol is as follows.

1. hMG 2 ampoules IM qd X 3 days.2. Serum estradiol is estimated on day 4 morning. If it is twice the baseline level

on day 1, same dose of hMG is continued. If not, it is increased by a factor of0.5. This is continued until response is seen. Ultrasonographic follicular studyis done on alternate days till follicular diameter reaches 14 mm, and then everyday till it reaches 18 mm.

3. hCG 10000 IU IM is administered 24 to 36 hours after hMG, when plasmaestradiol is 500 to 1000 pg/ml. It is withheld if the level is above 2000 pg/ml orif follicular number exceeds 4, so as to avoid hyperstimulation syndrome.

4. If conception does not occur despite ovulation, treatment is begun in the nextcycle with estradiol doubling dose of hMG.

C. Eugonadotropic anovulation is treated with clomiphene citrate. The doseis 50 mg PO qd for 5 days, starting from second day of the cycle. Follicular growthand rupture are monitored ultrasonographically. If ovulation takes place, the samedose is repeated in subsequent cycles, for up to a period of 1 year or until pregnancytakes place, whichever is earlier. If follicular growth does not take place at all, thedose is stepped up as 100 mg PO qd, 150 mg PO qd, 200 mg PO qd and 250 mg POqd in successive cycles until response is seen. If no response is seen, serum

dehydroepiandrosterone sulfate (DHEAS) level is assayed. If it is elevated,dexamethasone is administered 0.5 mg PO qd for 2 weeks before clomiphene citrateto suppress DHEAS and permit ovulation. If DHEAS level is normal, clomiphenecitrate therapy is considered to have failed, and ovulation is induced with hMG plushCG therapy. Patients with polycystic ovarian disease may be treated with folliclestimulating hormone (FSH) plus hCG, because they already have high levels of LHand do not need LH present in hMG.

If follicular growth is noted with any dose of clomiphene citrate but rupture does nottake place, hCG is administered at follicular diameter of 20 mm in the next cycle.The dose is 5000 IU IM and 10000 IU IM in normoestrogenic and hypoestrogenicwomen respectively.

Letrozole, an aromatase inhibitor, has been used in cases not responding to clomiphenecitrate. Its dose is 2.5 mg PO qd from day 3 to day 7 of the menstrual cycle. Used incombination with hMG, it reduces the dosage requirement of hMG. Its use for thisindication is banned in this country.

D. Ovulation cannot be induced if the woman has hypergonadotropichypogonadism. Estrogen replacement therapy is given to prevent complicationsof hypoestrogenism.

E. Hyperprolactinemia is associated with hypogonadotropism and anovulation.Ovulation can take place spontaneously when elevated level of PRL is loweredto normal. That is achieved with bromergocriptine. Treatment is initiated witha dose of 1.25 PO ql2h for 1 week. If there is intolerance, the dose is reducedto 1.25 mg PO qd and then gradually increased. If it is tolerated well, the doseis increased to 2.5 mg PO ql2h for 4 weeks. Serum PRL is then estimated. Ifit is still raised, bromergocriptine administration is continued. Cabergoline is morespecific for type 2 (D2) dopamine receptor. It is more potent than bromergocriptineand has a long half life. Its initial dose is 0.25-0.5 mg PO twice weekly. It is adjustedevery 2-3 months until stabilization. A median dose of 1 mg/wk is enough in mostcases, which can be reduced to 0.5 mg/wk once prolactin level is controlled. It iseffective in 85% of bromergocriptine intolerant cases, and 70% of bromergocriptineresistant cases. If serum PRL level is normal, ovulation studies are done (see chapter33). Administration is continued until a pregnancy occurs if the woman is ovulating.If she is not ovulating, clomiphene citrate is added to her treatment. When a womanwith hyperprolactinemia ovulates with therapy, investigations must be done to detectcorpus luteum insufficiency, which needs to be treated adequately (see chapter 12).

69

Page 76: Differential Diagnosis and Management Options inuploads.worldlibrary.org/uploads/pdf/20180107071525differential... · 83 Peripartum cardiomyopathy 165 84 Hypertension 167 85 Convulsions

INDUCTION OF OVULATION

Determine type of anovulation (A)

Hypogonadotropic (B) Eugonadotropic (C) Hypergonadotropic (D) Hyperprolactinemic (E)

hMG + hCG Clomiphene citrate No treatmentis effective

Bromergocriptineor cabergoline

Ovulation Absence of follicular growth Anovulation despite follicular growth Serum PRL assay after5 weeks of therapy

Step up dose ofclomiphene citrate

Clomiphene citrate + hCG Normal High

Ovulation Absence of folliculargrowth with a dose of250 mg qd

Ovulation studies Increase dose

Serum DHEAS assay Ovulation Anovulation

Normal Elevated Assess for lutealphase defect

Add clomiphene citrate

hMG + hCG Dexamethasone + clomiphene Present Absent

See chapter 12

70

Page 77: Differential Diagnosis and Management Options inuploads.worldlibrary.org/uploads/pdf/20180107071525differential... · 83 Peripartum cardiomyopathy 165 84 Hypertension 167 85 Convulsions

36. Cervical Factor andSperm Mucus PenetrationTests

Serial cervical mucus studies are done from day 8 or 9 of a 28 dayspontaneous cycle, and after completion of drug administration in a clomiphene-induced cycle. Moghissi's cervical score is determined at each visit (see table 35.1).The score is favorable when 10-15, unfavorable when below 10, and hostile whenbelow 5. If it is favorable, a postcoital test (PCT) is done. If it is turbid and cellularnear the time of ovulation, cervical culture is done for microorganisms. Appropriatelocal and systemic antibiotic therapy is given as required.

A. PCT is performed in the next cycle, after the cervicitis is controlled. If themucus is scanty or turbid without growth of microorganisms, low-dose estrogentherapy is given in the next cycle (ethinyl estradiol 0.01 mg PO ql2h from day8 to day 14 of a spontaneous cycle). If the mucus improves, a PCT is done. Ifit does not improve, there is a loss of cervical glands due to chronic infection,surgical removal (amputation of cervix, endocervical cauterization etc.) or withoutany cause (idiopathic). Cervical stenosis due to chronic infection is treated bycervical dilatation. If all treatment fails to improve cervical score, intrauterineartificial insemination is done with husband's semen at the time of ovulation timedby ultrasonography.

B. The couple is asked to abstain from coitus while serial cervical scoring isbeing done. When the score is above 9, coitus is advised (day 12 to 14) at night.The woman is examined the next morning. Cervical mucus and posterior fornixaspirate are collected. If sperm are present in the mucus, their motility is assessed.Presence of even one sperm with normal progressive motility is considered apositive PCT, and then both the sperm and cervical factor are normal. Someworkers grade PCT depending on the number of sperm seen per high power field( 1-5: + 1; 6-10: +2; 11-15: +3; 16-20: +4, 21-25: +5; > 25:+6). Suchgrading is probably unnecessary. If the sperm motility is poor or absent, PCTis repeated in the next cycle, and if it is still negative, an in vitro cross mucuspenetration test (IVCMPT) is done.

If there are no sperm in the cervical mucus, the posterior fornix aspirate isexamined for sperm. If sperm are found in it, it is possible that all sperm in themucus had traveled up the uterus by the time a PCT was done. An early PCT is thendone in the next cycle (within 2.5 hours of coitus). If it is positive, no furtherevaluation is necessary. If it is still negative, IVCMPT is done. If there are no spermin the posterior fornix aspirate, the cause could be absence of coitus, retrogradeejaculation (see chapter 36), impotence, or azoospermia (see chapter 37).

Table 36.1 Moghissi's Cervical Score

Parameter Score

0 1 2 3

Amount (ml) 0 0.1 0.2 > 0.3

Viscosity Thick, highlyviscous

Intermediate Mild Normalmidcycle

Spinnbarkeit (cm) < 1 1 - 4 5 - 8 > 9

Ferning Nil Atypical Primary andsecondary stems

Tertiary andquaternary stems

Cellularity(cells/hpf)

> 11 6 - 10 1 - 5 0

C. Penetration of cervical mucus by sperm can be studied in vitro too. A drop offavorable cervical mucus and a drop of liquefied semen are placed on a slide side byside. A coverslip is placed over them such that the two drops touch each other. It isthen observed under a microscope. If sperm cross the interface between the dropsand travel into the mucus, the test is positive, and the sperm as well as the mucus aresaid to be normal. In IVCMPT, the test is carried out between donor's mucus andhusband's semen, wife's mucus and husband's semen, wife's mucus and donor'ssemen, and donor's mucus and donor's semen. The donor's mucus should be from afertile woman with a favorable cervical score. The semen donor should be of provedfertility and normal result of semen analysis. The donor-donor IVCMPT shouldalways be positive. Interpretation of results is as shown in table 36.2.

Table 36.2 In Vitro Cross Mucus Penetration Test

Combination Result Interpretation

Donor's sperm-Donor's mucus Positive As expected.

Husband's sperm-Wife's mucus PositiveNegative

Both normal, faulty timing of PCTOne or both faulty

Husband's sperm-Donor's mucus PositiveNegative

Sperm are normalSperm are abnormal

Donor's sperm-Wife's mucus PositiveNegative

Mucus is normalMucus is abnormal

In case normal sperm of the husband cannot penetrate wife's mucus, themucus should be tested for antisperm antibodies.

In modern gynecology, cervical mucus studies and postcoital test are notdone much. The entire procedure is bypassed by performing intrauterineinsemination.

71

Page 78: Differential Diagnosis and Management Options inuploads.worldlibrary.org/uploads/pdf/20180107071525differential... · 83 Peripartum cardiomyopathy 165 84 Hypertension 167 85 Convulsions

CERVICAL FACTOR AND POSTCOITAL TEST

Serial cervical mucus study (A)

Normal Scanty Turbid, cellular

Cervical culture

No growth Growth of bacteria

Estrogen Antibiotics

Postcoital test Cervical mucusimprovement

Absence ofimprovement

Repeat cervical culture

Check for presence of sperm in cervical mucus Cervical stenosis Cervicitis Idiopathic Growth of bacteria No growth

Present Absent Postcoital test in the next cycle

Assess sperm motility Look for sperm in vaginal fluid

Normal Poor Absent Present Absent

Positive test Repeat test in next cycle

Normal Abnormal

Coital problem No coitus Retrograde ejaculation Impotence Azoospermia

In vitro cross mucus penetration test (C)

72

Page 79: Differential Diagnosis and Management Options inuploads.worldlibrary.org/uploads/pdf/20180107071525differential... · 83 Peripartum cardiomyopathy 165 84 Hypertension 167 85 Convulsions

37. Male InfertilityA. Normal semen parameters are liquefaction in 30 minutes, volume 2-5 ml,pH 7.2 to 7.8, fructose present, sperm count 20-250 million/ml (average 80-120million/ml), sperm motility > 50% grade III, and > 50% sperm of normal morphology.If any of the parameters are abnormal, further evaluation is necessary.

B. Usually a general examination does not reveal any abnormality. If the featuresare dysmorphic, a karyotype is obtained. It may be 47,XXY (Klinefelter syndrome).If findings of general examination are normal, and karyotype is 46,XY in cases ofdysmorphic features, urologic examination is done. It may reveal the followingabnormalities.

1. Varicocele: there is varicosity of the pampiniform plexus. The diagnosis isconfirmed by Doppler studies. It is treated by high ligation of spermatic vein(Palomo's operation) or excision of the plexus.

2. Atrophic testes: it may be developmental or acquired, as in bilateral mumpsorchitis. The testes feel small. Exact volume of each testis is measured by anorchidometer. If a biopsy shows absence of seminiferous tubules andspermatogenesis, the treatment is artificial insemination with donor semen (AID) oradoption.

3. Chronic epididymitis: broad spectrum antibiotics and an NSAID are given. If it isfound to be tuberculous, it is treated appropriately.

C. If the semen volume is low (< 1.5 ml) and the man has abstained from coitusfor the prescribed 5 days before the test, his urine is examined for sperm. Presenceof sperm in urine suggests retrograde ejaculation. It may be due to extrophyof bladder, prostatectomy, resection of the bladder neck, diabetes mellitus, spinalcord trauma, sympathectomy, multiple sclerosis, urethral valves or strictures,or use of adrenergic blocking agents. If the condition persists despite adequatetreatment of the cause, urine is alkalinized (NaHC03 1 tsf PO q6h X 48 h), theman is asked to ejaculate while his bladder is empty, his urine is then collectedand centrifuged, and the sperm pellet is used for artificial inseminationafter resuspension in Ham's F-10, by intrauterine insemination (IUI).In the absence of retrograde ejaculation, the man is treated with testosterone (T) ifhis serum T level is low. If the volume remains low, a sperm concentrationtechnique is used, and that sample is used for IUI.

D. If other semen parameters are normal in presence of high volume (> 5 ml),no further treatment is required. If any parameters are abnormal, a sample isobtained for IUI by split ejaculate or swim-up technique.

E. High viscosity is managed by sperm washing and IUI.

F. Agglutination of spermatozoa may be due to antisperm antibodies, whichare detected by immunological tests. Treatment with glucocorticoids is not verysatisfactory. Sperm washing and IUI appears to be promising.

G. Presence of pus cells in semen (pyospermia) is due to infection of accessoryglands. Appropriate antibiotic is given after microbiologic studies. Howeverpyospermia does not impair fertility.

H. If fructose is absent, the vasa deferentia are palpated. If they are found to beabsent, a sample for IUI or in vitro fertilization (IVF) is obtained by epididymalaspiration.

I. If there is azoospermia, or all parameters are abnormal, including oligozoospermia(sperm count < 20 million/ml), asthenozoospermia (sperm motility < 50%, < gradeIII), necrozoospermia (dead sperm > 50%), teratozoospermia (abnormal sperm >50%), serum FSH, LH, and T levels are estimated.

J. If hormone levels are normal, testicular biopsy is done. If spermatogenesis isarrested or absent, adoption or AID is advised. If it is normal, vasography is done.Obstruction in a vas is treated by microsurgical reversal of the obstruction. Absenceof vasa is managed by aspiration of a spermatocele and the sample is used for AIH orIVF.

K. High levels of FSH, LH and low level of T indicate testicular failure. Thetreatment is adoption or artificial insemination with donor semen (AID).L. High level of FSH and normal levels of LH and T are due to isolated germinalcell failure. If there is oligozoospermia, the treatment is IUI or IVF, depending onthe count. If there is azoospermia, the treatment is adoption or AID.

M. Low levels of FSH, LH, and T are evaluated by serum prolactin (PRL) assay andCT scan of head. If both are normal, the condition is idiopathic. It is treated byhuman menopausal gonadotropin. High PRL level and normal CT scan are treatedwith bromergocriptine, repeating the CT scan after 6 months for a pituitarymicroadenoma. If a pituitary tumor is found (high PRL level, tumor in sella turcica),the patient is referred to a neurosurgeon for treatment.

N. Normal FSH, and high LH and T levels are due to androgen insensitivity. Thetreatment is adoption or AID.

O. Empiric forms of treatment are as follows.1. Stimulation: clomiphene, tamoxifen, hMG/hCG, testosterone, mesterolone,

testolactone.2. Antioxidative therapy: vitamin E.3. Mast cell blockage: ketotifen.4. Immunomodulatory therapy: zinc.5. Improving microcirculation: pentoxiphylline.6. Rebound therapy: testosterone.

73

Page 80: Differential Diagnosis and Management Options inuploads.worldlibrary.org/uploads/pdf/20180107071525differential... · 83 Peripartum cardiomyopathy 165 84 Hypertension 167 85 Convulsions

SEMEN ANALYSIS

Normal (A) Abnormal

General examination(B)

Normal Dysmorphic features

Urologic examination Normal Obtain karyotype Abnormal Genetic factor

Varicocele Atrophictestes

Chronicepididymitis

Normal

Check for abnormalities in semen analysis

Low volume(C)

High volume(D)

High viscosity(E)

Spermagglutination (F)

Pus cells(G)

Absence offructose (H)

Azoospermia All parametersabnormal

Examine urinefor sperm

Assess otherparameters

Sperm washing IUI Immunologicaltests

Semen culture Palpate for vasadeferentia

Confirm by repeatingtest

Present Absent Normal Abnormal Sperm antibodies Accessory glandinfection

Absent Serum FSH, LH, T assay

Retrogradeejaculation

Assessserum T

IUI IUI Epididymal aspiration for IUI

Normal Low Normal (J) High FSH, high LH,low T (K)

High FSH, normalLH, normal T (L)

Low FSH, low LH, low T (M) Normal FSH, highLH, high T (N)

Reassessvolumeafter T

Testicular biopsyAssess spermatogenesis Testicular

failureIsolated germinal

cell failureSerum PRL, CT skull Androgen

insensitivity

Low Normal Normal Arrested/absent AID/adoption Check sperm count

Sperm concentrationIUI

Vasography AID/adoption

Oligozoospermia Azoospermia

Absence of vasa Obstruction in vasa IUI/IVF dependingon the count

AID/adoption

Aspiration ofspermatocele for

IUI/IVF

Microsurgical reversalof obstruction

Normal PRL, normal CT High PRL, normal CT Normal or high PRL,abnormal CT

Idiopathic BromergocriptineRepeat CT after 6 months

Pituitary tumor

74

Page 81: Differential Diagnosis and Management Options inuploads.worldlibrary.org/uploads/pdf/20180107071525differential... · 83 Peripartum cardiomyopathy 165 84 Hypertension 167 85 Convulsions

38. Congenital Absence ofVagina

A. A girl may be brought with primary amenorrhea and a history of cyclicallower abdominal pain once every month. If she is found to have a blind pouchof vagina, and a tender cystic lump in the pelvis and lower abdomen, the diagnosisis a transverse vaginal septum and hematocolpos. Ultrasonography confirms thediagnosis. The treatment is excision of the septum.

B. If the vagina fails to develop totally, but the uterus is normal and functional,the menstrual blood collects in it and forms a hematometra. It needs to bedrained by creating a new vagina. If the cervix is developed, MRI shows thepresence of the dilated external os and a short length of a distended vagina below it.In such cases the results of vaginoplasty are satisfactory, when the short vagina nearthe cervix is anastomosed with the upper end of the neovagina. If the cervix has notdeveloped, attempts at creation of a new cervix usually fail. Most of the patientseventually need an abdominal hysterectomy after multiple attempts at creation of andmaintaining the patency of the cervix. It may be preferable to perform ahysterectomy in these cases primarily.

C. If there is no functioning uterus in the pelvis, one has to differentiate betweentotal androgen insensitivity (testicular feminization syndrome) and mullerianagenesis (Rokitansky Kuster Hauser syndrome). Though some persons with theformer condition have inguinal hernia and/or inguinal gonad (testis), most personsdo not have any clinical features which can help differentiate between the twoconditions. Two investigations are of importance for this purpose. One is Barrbodies ( > 15% in a woman, < 6% in a man), and the other is karyotyping (46,XX in a woman, 46, XY in a man).

Condition Barr bodies Karyotyping

Androgen insensitivity < 6% 46, XY

Mullerian agenesis > 15% 46, XX

D. A person with total androgen insensitivity needs to have the gonads removedafter the age of puberty because there is increased risk of development of testiculartumors. It is important not to reveal to the woman that she is really a geneticmale in so many words. The management of such a woman, and a woman withmullerian agenesis is otherwise the same. If the woman is married, or is not marriedbut is likely to be married soon, she needs a vagina of adequate depth for sexualfunction. She must be told that she can never menstruate, and can never havea child in the usual way. The depth of her blind pouch of vagina is assessed. Ifit is 2 cm or more, Frank's technique of vaginal dilatation is used. She is askedto make pressure with a vaginal dilator in the right direction in the vaginal pouchfor 15 minutes at a time at least twice a day. The vagina gradually deepens. Ifshe cannot achieve this, or if the pouch measures less than 2 cm, Mclndoe's vaginoplastyis done. In this operation new vagina is dissected between the bladder and urethrain front and the rectum behind. After achieving hemostasis, a mould of spongeover a rubber catheter, with a sterile condom sheath is used to softly press a partialthickness skin graft over the raw walls of the new vagina. After the graft takesand there are no raw areas left anywhere, the woman is asked to wear a similarmould every night for at least 3 months in case she is married, and until she marriesin the remaining cases. Then she is advised to have regular sexual activity sothat the new vagina does not close down.

E. If the woman has already undergone one or more attempts at Mclndoe'svaginoplasty in the past without any success, and there is no pouch of vagina, butonly dense fibrous tissue between the bladder and the rectum, she is advisedWilliams' vulvovaginoplasty. If she had an accidental bladder or rectal injuryduring a previous operation, which was either successfully repaired or whichended in a fistula which was repaired subsequently, Williams' operation is indicatedeven more strongly. In this operation, a new vagina is created as a tube, usingskin from the medial aspects of the labia majora. The lateral cut edges are underminedand approximated over the tube. This vagina is directed downwards and backwards,which is not very satisfactory for sexual function. However the couple learnsto adjust to this abnormal direction of vagina and have reasonably satisfactorysexual relations.

75

Page 82: Differential Diagnosis and Management Options inuploads.worldlibrary.org/uploads/pdf/20180107071525differential... · 83 Peripartum cardiomyopathy 165 84 Hypertension 167 85 Convulsions

CONGENITAL ABSENCE OF VAGINA

Look for hematocolpos (A)

Present Absent

Transverse vaginal septum Ultrasonography for hematometra (B)

Excision of septum Present Uterus absent

Vaginal agenesis Barr bodies, karyotyping (C)

MRI for cervical development Total androgen insensitivity

Gonadectomy

Mullerian agenesis

Cervical atresia Cervix present Check marital status (D)

Married Single

Matrimony likely Matrimony not likely

Assess depth of vagina

< 2 cm > 2 cm Nil, with scarring from previous failed attempt(s) at vaginoplasty

Frank’s technique Williams’ vulvovaginoplasty (E)

Not successful Successful

McIndoe’s vaginoplasty

76

Page 83: Differential Diagnosis and Management Options inuploads.worldlibrary.org/uploads/pdf/20180107071525differential... · 83 Peripartum cardiomyopathy 165 84 Hypertension 167 85 Convulsions

39. Congenital Malformationsof the Female GenitalTract

A. The malformations of the female genital tract can be classified as follows. I preferthis working classification to the American Fertility Society classification because itis more logical and easier for management of patients.

1. Agenesis: if both the mullerian ducts do not develop, there is total absence ofthe fallopian tubes and the uterus, while there is a blind pouch of vagina ofvariable depth. If only one mullerian tube develops, the result is a unicornuateuterus, in which there is a single uterine horn with one fallopian tube attached toit.

2. Hypoplasia: it varies from a small but functioning uterus to a small nodule ofsmooth muscle without any functioning endometrium within it.

3. Fusion defects: there is a variable degree of failure of fusion of the twomullerian ducts, resulting in the development of the following malformations.a. Arcuate uterus: there is a fundal depression, the least degree of fusion

defect.b. Uterus bicornis unicollis: there are two uterine horns attached to a single

cervix, with one fallopian tube attached to each horn.c. Uterus bicornis bicollis: there are two uteri with one cervix and one

fallopian tube each. The two cervixes open into a single or septate vagina(uterus pseudodidelphys).

d. Subseptate uterus: there is a septum hanging down from the uterinefundus, not reaching the cervix.

e. Septate uterus: there is a longitudinal septum from the fundus to thecervix.

4. Duplication: there are two sets of uterus, tubes, cervix, vagina, vulva, bladder,and urethra (uterus didelphys).

5. Atresia of outflow tract: it leads to obstruction and retention of menstrual blood.It may be of the following types.a. Imperforate hymen.b. Transverse vaginal septum.c. Vaginal atresia.d. Cervical atresia.e. A fusion defect with unilateral atresia.The diagnosis is made by a thorough clinical examination, ultrasonography,

and as required, laparoscopy plus hysteroscopy. Laparoscopy helps evaluateexternal surface of the uterus and tubes, while hysteroscopy reveals the interior ofthe uterine cavity. Excretory urography is done in all cases because urinary tractmalformations are often associated with malformations of the genital tract.Hysterosalpingography is done if it is not possible to do a hysteroscopy.

B. Mullerian agenesis: see chapter 38.

C. Hypoplastic uterus: if it is functioning, cyclical estrogen therapy is givento stimulate its growth. There is no treatment for a nonfunctioning uterine nodule.D. Arcuate uterus is an anatomical variation which usually causes no problemsand requires no treatment.

E. If one horn of a uterus bicornis unicollis is rudimentary while the otherhorn is well developed, the treatment depends on the condition of the rudimentaryhorn.

1. Nonfunctioning horn: it is left alone.2. Functioning noncommunicating horn: menstrual blood distends it to form a

hematometra. It is excised.3. Functioning communicating horn: its fallopian tube is ligated, so that a

pregnancy does not get implanted in it and rupture it in the second trimester.4. Rupture of a rudimentary horn due to a pregnancy: the woman is resuscitated

for intraperitoneal hemorrhage and then the horn is excised.

F. If both horns of a uterus bicornis unicollis are well developed, the womanmay not have any problems, gynecologic or obstetric. If she has had repeatedpregnancy wastage, Strassman's metroplasty is done. In this operation the medialaspect of each horn is incised to expose its cavity. The two horns are then unitedby suturing their walls, posterior to posterior and anterior to anterior. Such awoman may still need cervical cerclage in a future pregnancy for cervical incompetence,and such a pregnancy is always terminated by a cesarean section, owing to therisk of rupture of the scar during labor.

G. Uterus bicornis bicollis does not have any treatment.

H. A septate uterus may not cause any gynecologic or obstetric problems. Howeverit is more likely to cause repeated pregnancy wastage than a bicornuate uterus. Itmay cause first trimester abortions due to implantation on the septum, or secondtrimester abortions due to cervical incompetence. In such cases it is treated byhysteroscopic resection of the septum using Nd-YAG laser or resectoscope. If suchfacilities are not available, it is excised by a laparotomy, using Tompkin's or Jones'technique.

I. Uterus didelphys is extremely rare. There is no way of treating it.

J. A longitudinal vaginal septum may cause dyspareunia. It is then excised. If it isfirst diagnosed during labor causing obstruction, it is divided between clamps topermit childbirth. Then it is excised after the puerperium.

K. Transverse vaginal septum: it is excised (see chapter 38).

L. Imperforate hymen: it is excised (see chapter 2).

M. Cervical atresia: see chapter 2. All attempts at creation of a new cervix usuallyfail. Most of the patients eventually need a hysterectomy after multiple attempts atcreation of and maintaining the patency of the cervix. It may be preferable toperform a hysterectomy in these cases primarily.

77

Page 84: Differential Diagnosis and Management Options inuploads.worldlibrary.org/uploads/pdf/20180107071525differential... · 83 Peripartum cardiomyopathy 165 84 Hypertension 167 85 Convulsions

CONGENITAL MALFORMATIONS OF FEMALE GENITAL TRACTDetermine type (A)

Mullerianagenesis (B)

Hypoplasticuterus (C)

Arcuate uterus(D)

Uterus bicornisunicollis

Uterus bicornisbicollis (G)

Septate uterus(H)

Uterusdidelphys (I)

CounselingMcIndoe’s vaginoplasty

Estrogencyclically

Observation Check developmentof both horns (E)

Observation Check for thepresence of repeatedpregnancy wastage

Observation

Rudimentaryhorn

Both horns welldeveloped

Yes No

Nonfunctioning Hematometra Functioning,communicating

Rupture inpregnancy

Check for thepresence of repeatedpregnancy wastage

Resection of septum Observation

Observation Excision Resuscitaton,Excision of horn

Ligation of fallopian tube of the same side

Yes No

Strassman’s metroplasty Observation

Longitudinal vaginal septum (J) Imperforate hymen (L) Cervical atresia (M)

Dyspareunia Obstructed labor Excision Cervicoplasty

Excision Division, excisionafter puerperium

Transverse vaginal septum (K) Cervix remains patent Cervix closes

Excision Total hysterectomy

78

Page 85: Differential Diagnosis and Management Options inuploads.worldlibrary.org/uploads/pdf/20180107071525differential... · 83 Peripartum cardiomyopathy 165 84 Hypertension 167 85 Convulsions

40. Abnormal Pap SmearA. Cervical cancer screening should begin at age 21 years. Between 21 and 29 years,do cervical cytology (conventional or liquid-based) every 3 years. Between 30 and65 years, do cytology and HPV testing every 5 years, or cytology alone every 3years. These guideline do not apply to women who have a history of cervical cancer,have HIV infection/immunocompromise, or were exposed to diethylstilbestrol inutero. An unsatisfactory cytology preparation is not reliable for detecting epithelialabnormalities. But smears which are unsatisfactory due to the presence of blood orinflammation are associated with a higher risk of disease, especially when they areHPV +ve. HPV detection depends on cellularity of the smear, and hence a HPV –veresult may be due to poor cellularity. So HPV unknown and –ve smears are repeatedafter 2-4 months. If they are still unsatisfactory, colposcopy is done. If they are –ve,screening is done after 12 months. If they are abnormal, they are managedappropriately. HPV +ve cases can be subjected to colposcopy or screened after 2-4months.

B. Cytology negative for intraepithelial lesion or malignancy, but with limitedendocervical/transformation zone (EC/TZ), suggests that disease might have beenmissed. A negative cytology has a good negative predictive value and specificitydespite absent or insufficient EC/TZ. HPV testing is independent of sampling ofthe transformation zone. Up to the age of 30 years, routine screening is adequate forHPV negative cases. If it is positive, do cytology and HPV co-testing or genotypingafter 1 year. If on genotyping HPV type 16 or type 18 is present, colposcopy isdone. If the woman is HPV positive but cytology-negative, repeat co-testing at 1year. The preferred mode of screening in the age group 30-64 is cytology-HPV co-testing.

C. For women with HPV negative ASC-US, repeat co-testing at 3 years. For womenwith HPV positive ASC-US, do colposcopy. If colposcopy shows CIN, manageappropriately. Alternatively, for ASC-US and no HPV repeat cytology at 1 year. If itshows ASC-US or worse, do colposcopy; if the result is negative, do cytology testingevery 3 years. If the woman is 21-24 year old and ASC-US and is HPV positive ornegative, do repeat cytology in 12 months. Management of ASC-US is similar inpregnancy, except that colposcopy may be deferred until after the puerperium.

D. For LSIL and HPV negative repeat co-testing at 1 year, though colposcopy isacceptable too. If at this time cytology shows ASC-US or worse or the HPV test is

positive, do colposcopy. If both the tests are negative, do co-testing every 3 years.For LSIL and no HPV test or a positive HPV test, do colposcopy. If the report isCIN, take appropriate measures. For LSIL in the age group 21-24, do cytology every12 months. For LSIL during pregnancy, do colposcopy. Do not perform endocervicalcurettage. Colposcopy may be deferred until 6 weeks postpartum. For LSIL andHPV negative report in a postmenopausal woman, repeat cytologic testing at 6 and12 months, and perform colposcopy. If the HPV test is negative or if there is no CIN,repeat cytology after 12 months. If the HPV test is positive or cytology is ASC-USor greater, do colposcopy. If two consecutive repeat cytology tests are negative,return to routine screening.

E. For Atypical Squamous Cells, Cannot Exclude High-Grade SquamousIntraepithelial Lesion (ASC-H), do colposcopy. ASC-H confers higher risk for CIN3+ over time than ASC-US or LSIL, even for women in the age group 21-24 years.HPV testing is not recommended because rate of positive HPV report is high, and 5-year risk of cancer is also too high (2%) to permit observation.

F. With High-Grade Squamous Intraepithelial Lesion (HSIL), about 60% have CIN2 or greater. So do an immediate excision of the transformation zone, especially ifthe woman has completed childbearing or is likely to be lost to follow-up. If that isnot accepted, do a colposcopy. HPV testing and observation are not recommended.Do colposcopy for women aged 21-24 years. Observe for up to 24 months usingboth colposcopy and cytology every 6 months, provided colposcopy is adequate andendocervix is normal or CIN 1. If it is CIN 2 or higher, manage appropriately. Do adiagnostic excisional procedure if HSIL persists for 24 months without identificationof CIN 2 or higher, or primarily if colposcopy is inadequate.

G. For Atypical Glandular Cells, Cytologic Adenocarcinoma In Situ, and BenignGlandular Changes (AGC), although the cancer risk is lower in women below theage of 35 years, the risk of CIN 2 or greater is higher. Glandular and squamouslesions often coexist. CIN found in approximately 50% of women withadenocarcinoma in situ (AIS). Hence intensive assessment is done at all ages. IfHPV test is negative, the woman may be at greater risk for endometrial rather thancervical disease. For atypical endometrial cells, do colposcopy, endocervical andendometrial sampling. Do colposcopy with endocervical sampling for women withAGC but without atypical glandular cells. For women with AGC in whom CIN 2+is not identified, do co-testing at 12 and 24 months. If both are negative, 12months and 24 months is recommended. If both co-tests are negative, repeat co-testing after 3 years. Manage CIN and AIS appropriately.

79

Page 86: Differential Diagnosis and Management Options inuploads.worldlibrary.org/uploads/pdf/20180107071525differential... · 83 Peripartum cardiomyopathy 165 84 Hypertension 167 85 Convulsions

PAP Smear

Unsatisfactory cytology (A) NILM, EC/TZ Absent/Insufficient (B) ASC-US (C) LSIL (D)

Check HPV status Age 21 to < 30 Age > 30 Check HPV status Check HPV status

Unknown Negative Positive Routine screening Check HPV status NegativePositive

Negative Positive

Repeat cytology after 2-4 months Colposcopy Positive Co-testingafter 3 years

Co-testingafter 1 year

Negative Abnormal

Routine screening

Unsatis-factory

Negative Cytology + HPV orgenotyping after 1

yearColposcopy

Colposcopy

CIN 1 CIN 2or 3

Repeat co-testing after 1 year Cytology negative, HPV positive Cytology and HPVnegative

> ASC/HPV positive

Appropriatemanagement

Cytology and HPVnegative

ASC or HPVpositive

DNA typing after 1 yearASC-H (E) HSIL (F) AGC (G)

Repeat after 3 y Colposcopy HPV 16-18 positive HPV 16-18 negative Colposcopy Excision of thetransformation zone orcolposcopy

Atypical endometrial cells Other

Colposcopy, endocervical and endometrialsampling

Endocervical and endometrial sampling

80

Page 87: Differential Diagnosis and Management Options inuploads.worldlibrary.org/uploads/pdf/20180107071525differential... · 83 Peripartum cardiomyopathy 165 84 Hypertension 167 85 Convulsions

41. Cervical IntraepithelialNeoplasia

A. HPV infection results in CIN 1 histoloically. The natural history of CIN 1 andthat of HPV-positive ASC-US and LSIL is similar. Hence their management shouldbe similar. Though most cases of CIN 1 are associated with oncogenic HPV, HPV-16 is far less common in CIN 1 than in CIN 3. It tends to regress in a largepercentage of cases, especially in younger women. Its progression to CIN 2 or higheris uncommon. Lesser abnormalities include HPV-16, HPV-18 or persistentoncogenic HPV infection of any type when cytology is negative, ASC-US, andLSIL. These are associated with risk of CIN 3 or higher as much as with HSIL.Thesepatients are managed by testing after 1 year with Pap smear if below the age of 30years and co-testing if above 30 years. If prior Pap smear shows ASC-US or LSIL, a5-year risk of CIN 3 or higher is low (less than 4%). If it shows HSIL, ASC-H, orAGC, the risk is much higher (15%). In prior Pap smear shows ASC or if HPV is+ve, a colposcopy is done. If it is inadequate, a biopsy is obtained. If the diagnosis isCIN 1 which persists for at least 2 years, treatment is given or follow up iscontinued, based on the patient’s desire. When CIN 1 is seen in endocervicalsample but no CIN 2+ is seen in colposcopic biopsies, management is as for CIN1, plus repeat endocervical sampling in 12 months. If the colposcopic examinationis inadequate, the endocervical sample shows CIN 2,CIN 3, or ungraded CIN, or ifthe patient has been treated previously, a diagnostic excision is done. Application ofpodophyllin or podophyllin-related products, or a hysterectomy for histologicallydiagnosed CIN 1 is not recommended. For a woman aged 21-23 years having CIN 1after ASC-US or LSIL cytology, Pap is repeated at 12-month intervals.

B. For a woman with CIN 1 preceded by ASC-H or HSIL, if CIN 2 or higher is notseen on biopsy, an excisional biopsy is done. Alternatively observation with co-testing may be done at 12 and 24 months provided the colposcopy is adequate andthe endocervical sampling is negative. If the tests are negative, retesting is done in 3years. If any test is positive, repeat colposcopy is done. If it shows HSIL, anexcision biopsy is done. For a woman aged 21-24 years, with ASC-H or HSIL+ at

the 12-month follow up, colposcopy done. If ASC-US or worse is seen at the 24month follow up, colposcopy is done. After two consecutive tests are negative,routine screening is done.

C. Regression rates are lower and progression to cancer more common for womenwith CIN 2 and 3. This is more so for the latter. It is difficult to distinguish betweenCIN 2 and 3 in a given case. Treatment must be initiated for CIN 2 and higherlesions. Women with CIN 3 should not be observed as they can progress to aninvasive cancer. If colposcopy is found to be adequate, and the woman is notpregnant or young, excision and ablation are satisfactory. If the colposcopy is notadequate or if endocervical sample shows CIN 2, CIN 3, or CIN not graded, ablationshould not be done. An excision biopsy should be done in such cases. An excisionbiopsy is done for recurrent CIN 2 or CIN 3. Hysterectomy is not recommended asprimary therapy for CIN 2 or CIN 3. Observation of CIN 2 and CIN 3 withsequential cytology and colposcopy is recommended for pregnant women and youngwomen. Co-testing at 12 months and 24 months is done for treated women. If bothco-tests are negative, retesting is done after 3 years. If any test is abnormal,colposcopy and endocervical sampling is done. If all the tests are negative, routinescreening is done for at least 20 years irrespective of the age of the patient.Hysterectomy is not recommended if HPV test is positive. If CIN 2, or CIN 3 is seenat the margins of a diagnostic excisional procedure or in an endocervical sampleobtained immediately after the procedure, reassessment is done with Pap smear andendocervical sampling after 4-6 months. A repeat excision biopsy is acceptable too.If that is not possible, a hysterectomy may be done. If CIN 2 or 3 is found to persistor recur, a repeat excision biopsy o hysterectomy is done.

D. Adenocarcinoma in situ (AIS) can be associated with minimal changes oncolposcopy. It often extends into the endocervical canal. It can be multifocal. Sonegative margins on an excision biopsy do not guarantee complete excision of thedisease. Invasive cancer cannot be excluded without a hysterectomy. It isrecommended if the woman has completed her family. Observation is done if shedesires to retain her fertility. But it is associated with a 10% risk of persistence ofAIS. If a HPV test is negative after treatment, the risk of persistent or recurrent AISis low. Long-term follow-up is required for women who do not undergohysterectomy.

81

Page 88: Differential Diagnosis and Management Options inuploads.worldlibrary.org/uploads/pdf/20180107071525differential... · 83 Peripartum cardiomyopathy 165 84 Hypertension 167 85 Convulsions

CERVICAL INTRAEPITHELIAL NEOPLASIA

CIN 1 (A) AIS (D) CIN 2, CIN 3 (C)

Check preceding Pap report Check adequacy of colposcopy Recurrent CIN 2, 3Endocervical CIN 2,3

Lesser abnormalities(A)

ASC-H or HSIL (B) Adequate Inadequate

Co-testing at 12 months Co-testing at12 and 24months

Excisionbiopsy

Excision or ablationof transformationzone

Excision biopsy

Cytology and HPV –ve ASC/HPV +ve HPV –ve,Pap –ve

HPV/Pap+ve

HSIL Co-testing after 12 and 24 months

Follow up testingafter 1 year and thenafter 3 years.

Colposcopy Retestingafter 3years

Colposcopy Excisionbiopsy

Negative Any test positive

CIN 1 CIN 2,3,No CIN

Higher lesion Co-testing after 3 years Colposcopy plusendocervical sampling

Diagnosticexcision

Appropriatemanagement

Check if the family is complete

Follow uptesting

Treat if persists for atleast 2 years

Yes

Hysterectomy

No

Excision biopsy

Check if the margins are free of disease andendocervical curettage is negative

Yes No

Follow up Re-excision

82

Page 89: Differential Diagnosis and Management Options inuploads.worldlibrary.org/uploads/pdf/20180107071525differential... · 83 Peripartum cardiomyopathy 165 84 Hypertension 167 85 Convulsions

42. Invasive Cancer of CervixFIGO staging of cancer of the cervix is determined by clinical examination

and appropriate investigations like cystoscopy, proctoscopy, excretory urography,and CT scan or MRI.

FIGO staging of Carcinoma of Cervix (2009)Stage I The carcinoma is strictly confined to the cervix (extension to the

corpus would be disregarded)

lA Invasive carcinoma which can be diagnosed only bymicroscopy, with deepest invasion < 5.0 mm and largestextension < 7.0 mm

IAl Measured stromal invasion of < 3.0 mm in depth and horizontalextension of < 7.0 mm

IA2 Measured stromal invasion of > 3.0 mm and not > 5.0 mm withan extension of < 7.0 mm

lB Clinically visible lesions limited to the cervix uteri or preclinicalcancers greater than stage IA

IB1 Clinically visible lesion < 4.0 cm in greatest dimensionIB2 Clinically visible lesion > 4.0 cm in greatest dimension

Stage II Cervical carcinoma invades beyond the uterus, but not to thepelvic wall or to the lower third of the vagina

IIA Without parametrial invasionIIA1 Clinically visible lesion < 4.0 cm in greatest dimensionIIA2 Clinically visible lesion > 4.0 cm in greatest dimensionIIB With obvious parametrial invasion

Stage III The tumour extends to the pelvic wall and/or involves lowerthird of the vagina and/or causes hydronephrosis or non-functioning kidney

IIIA Tumour involves lower third of the vagina, with no extension tothe pelvic wall

IIIB Extension to the pelvic wall and/or hydronephrosis or non-functioning kidney

Stage IV The carcinoma has extended beyond the true pelvis or hasinvolved (biopsy proven) the mucosa of the bladder orrectum. A bullous edema does not permit a case to be allottedto stage IV

IVa Spread to adjacent organs.IVb Spread to distant organs.

A. For stage 1A1 without lymphovascular space involvement (LVSI) in a womandesirous of sparing fertility, perform a cone biopsy with 3 mm free margin. If themargin comes positive, repeat cone biopsy or perform a trachelectomy. Treat stage1A1 with LVSI and stage 1A2 as stage 1A1, with pelvic lymph node dissection

with/without para-aortic lymph node sampling. Sentinel lymph node (SLN) mappingmay be done. For stage 1B1 and possibly for stage 1A2, radical trachelectomy,pelvic lymph node dissection with/without paraaortic lymph node sampling is done.Sentinel lymph node (SLN) mapping may be done for all except stage 1a1 withoutLVSI. If preservation of fertility is not desired, do cone biopsy for stage 1A1 withoutLVSI. If margins are negative for cancer, do extrafascial hysterectomy if operableand observe if not operable. Do Extrafascial or modified radical hysterectomy, pelviclymph node dissection if margins are positive for cancer. For stage IA1 with LVSIand stage IA2, do modified radical hysterectomy, pelvic lymph node dissectionwith/without paraaortic lymph node sampling, or give pelvic radiotherapy plusbrachytherapy (70–80 Gy).B. For stages IB1 and IIA1, do radical hysterectomy, pelvic lymph node dissection,with/without para-aortic lymph node sampling, or pelvic radiotherapy, brachytherapy(total 80–85 Gy at point A), with/without concurrent cisplatin chemotherapy. Forstages 1B2 and 2A2, give Pelvic radiotherapy, concurrent cisplatin, brachytherapy(total ≥85 Gy at point A), or do radical hysterectomy, pelvic lymph node dissection,with/without paraaortic lymph node sampling, or pelvic radiotherapy, concurrentcisplatin, brachytherapy (total 75–80 Gy at point A), adjuvant hysterectomy. If themargins, nodes and parametrium are negative, observation may be done, or if Sedliscriteria are met, pelvic radiotherapy with/without cisplatin therapy may be given. Ifany of the three is positive, give pelvic radiotherapy, concurrent cisplatin,with/without brachytherapy. If paraaortic nodes are positive, give paraaortic lymphnode radiotherapy, concurrent cisplatin, pelvic radiotherapy, with/withoutbrachytherapy. If distant metastases are present too, give systemic therapywith/without individualized radiotherapy.C. For stages IB2, IIA2, IIB2, IIIa, IIIB, and IVA, check lymph node involvementby imaging and extraperitoneal or laparoscopic lymph node dissection. If pelvicnodes are positive and paraaortic nodes are negative, give pelvic radiotherapy,concurrent cisplatin, and brachytherapy with/without paraaortic lymph noderadiotherapy. If both nodes are positive, give extended field radiotherapy, concurrentcisplatin, and brachytherapy. If there are distant metastases, give systemic therapyand individualized radiotherapy.D. Educate the patient about features of recurrence. Give advice on use of vaginallubricants and dilators. Do cytology and biopsy as required every 3–6 mo for 2 y,every 6–12 mo for 3–5 y, then annually based on patient’s risk of disease recurrence.Do cervical/vaginal cytology annually to detect lower genital tract neoplasia. DoCBC, BUN, creatinine chest radiography, CT, PET, PET/CT, MRI as required.E. For local or regional recurrence without prior radiotherapy to that area, dosurgical resection if feasible. If it is not, give tumor-directed radiotherapy, cisplatinwith/without brachytherapy. For central recurrence without prior radiotherapy, dopelvic exenteration with/without intraoperative radiotherapy, or if the lesionmeasures less than 2 cm, do radical hysterectomy or give brachytherapy. For non-central disease without prior radiotherapy, give tumor-directed radiotherapywith/without chemotherapy. Treat distant metastases amenable to local treatmentwith local resection/ablation and radiotherapy, or radiotherapy and concurrentchemotherapy. Treat distant metastases not amenable to local treatment withchemotherapy or palliative care.

83

Page 90: Differential Diagnosis and Management Options inuploads.worldlibrary.org/uploads/pdf/20180107071525differential... · 83 Peripartum cardiomyopathy 165 84 Hypertension 167 85 Convulsions

INVASIVE CANCER OF CERVIX

Check type

Primary Stump (F) Recurrent (G)

Check stage Check length ofstump

Check previoustreatment

IA1 (A) IB1, IA2, non-bulky IIA (B) IB2, bulky IIA (C) IIB to IVA (D) IVB (E)

Simple hysterectomyor cervical conization

Full external and intracavitarypelvic irradiation with

concurrent chemotherapy

Palliativechemotherapy

Radical hysterectomy or trachelectomy,bilateral pelvic lymphadenectomy withpostoperative irradiation, + concurrent

chemotherapy in selected high-riskpatients

Full external and intracavitary pelvic irradiationwith concurrent chemotherapy, then extrafascialhysterectomy) or radical abdominal hysterectomy

and pelvic ± periaortic lymphadenectomy

> 2 cm < 2 cm

External plus intracervical irradiation External plus vaginal irradiation

Radical hysterectomy + pelvic lymph node dissection Radiotherapy

Check site Check site

Central pelvic Pelvic sidewall Distant Distant Central pelvic Pelvic sidewall

External radiation + brachytherapy Check site Exenteration Palliation

Bone Lung Brain Supraclavicular lymph nodes

External radiation Resection of solitary lesion, chemotherapy formultiple lesions

Whole brain radiation + steroids Localized radiation

84

Page 91: Differential Diagnosis and Management Options inuploads.worldlibrary.org/uploads/pdf/20180107071525differential... · 83 Peripartum cardiomyopathy 165 84 Hypertension 167 85 Convulsions

43. Undiagnosed CervicalCancer in a TotalHysterectomy Specimen

It is essential that every woman who undergoes a hysterectomy has a Pap smear priorto the operation. If there is any suspicion of cervical cancer, adequate evaluationmust be done (see chapter 42) before undertaking any form of surgical treatment.Despite such precautions, it is possible that cervical cancer is missed due toinadequate smear, or inadequate cytological technique. The treatment of a womandiagnosed to have invasive cervical cancer in a total hysterectomy specimen dependson the stage and pathological findings. Usually the stage is IA, as grossly visibledisease I or should not be missed clinically.

A. If the stage is 1A1, the histopathology should be reassessed for lympovascularspace involvement (LVSI). If these spaces are not involved, a follow up is adequate(see chapter 42). If these spaces are involved by the cancer, the case is treated asstage 1A2.

B. Stage 1A1 with LVSI and stage 1A2 are assessed for residual disease and fitnessfor further treatment, with histopathology, hemogram, liver and renal function tests,chest radiography, CT/PET and MRI as indicated. Assessment is done to see if themargins are free of the disease, lymph nodes are involved, and if there is residualdisease.

C. If the margins are free and there is no nodal involvement, give pelvic

radiotherapy, brachytherapy, with/without concurrent cisplatin or do completeparametrectomy, upper vaginectomy, pelvic lymph node dissection with/withoutparaaortic lymph node sampling. If the lymph nodes are not involved, follow up thepatient (see chapter 42). If the lymph nodes are involved, give pelvic radiotherapy,paraaortic radiotherapy (for nodal involvement), concurrent cisplatin, with/withoutbrachytherapy (for disease in vaginal margins).

D. If the margins are involved or there is large residual disease, assess forinvolvement of lymph nodes by imaging. If there is no nodal involvement, givepelvic radiotherapy, paraaortic radiotherapy (for nodal involvement), concurrentcisplatin, with/without brachytherapy (for disease in vaginal margins). If the nodesare involved, do debulking of the large nodes, and then give pelvic radiotherapy,paraaortic radiotherapy (for nodal involvement), concurrent cisplatin, with/withoutbrachytherapy (for disease in vaginal margins).

If the stage of the disease is higher than 1A2, treat it as invasive carcinoma asdescribed in chapter 42. However it may not prove to be adequate because tumor-bearing tissue has already been cut across, increasing the risk of metastases. Risk offailure of treatment increases further directly proportional to the interval between theprimary operation and the time of referral.

Five year survivals of different types of cases are as follows.1. Microscopic evidence of invasive cancer: 96%.2. Gross tumor in surgical specimen with negative surgical margins: 84%.3. No gross residual disease, but positive surgical margins: 87%.4. Residual disease present, referral less than 6 months after hysterectomy or evenlater: 47%.5. Referral 6 months after hysterectomy or even later: 37%.

85

Page 92: Differential Diagnosis and Management Options inuploads.worldlibrary.org/uploads/pdf/20180107071525differential... · 83 Peripartum cardiomyopathy 165 84 Hypertension 167 85 Convulsions

UNDIAGNOSED CERVICAL INVASIVE CANCER IN A TOTAL HYSTERECTOMY SPECIMEN

Check stage

1A1 (A) 1A2 (B)

Check for lymph vascular space involvement

Absent Present

Investigations for fitness and residual disease or lymph node involvement

Follow up Margins free, no nodal involvement (C) Involvement of margins or large residual disease (D)

Pelvic radiotherapy, brachytherapy, ±concurrent chemotherapyor

Imaging for nodal involvement

Upper vaginectomy, parametrectomy, pelviclymph node dissection ± paraaortic lymph nodebiopsy

Negative Positive

Check for nodal involvement or disease inmargins/parametrium

Resection of largenodes

Absent Present

Observation Pelvic and paraaortic radiotherapy, concurrent chemotherapy, ± brachytherapy (for disease in vaginal margins)

86

Page 93: Differential Diagnosis and Management Options inuploads.worldlibrary.org/uploads/pdf/20180107071525differential... · 83 Peripartum cardiomyopathy 165 84 Hypertension 167 85 Convulsions

44. Endometrial CarcinomaEndometrial carcinoma is the third commonest gynecologic malignancy, cancer ofthe cervix and ovary being the first two. However it is the least common cause ofcancer death in women. Relatively low malignant potential of the tumor and earlypresentation in postmenopausal women account for the lower mortality. There is astrong correlation between positive cervical cytology and high-risk disease (high-grade tumor, deep myometrial invasion and nodal disease). Screening of endometrialcarcinoma in high-risk cases is done by pelvic ultrasonography (endometrialthickness > 3 mm), MRI (cervical and nodal involvement), endometrial aspirationcytology with Pipelle or histopathology with Vabra, hysteroscopy-guided biopsy (seechapter 6) and if these tests are not available a fractional curettage.

FIGO staging of Endometrial Cancer

I Carcinoma confined to the corpuslA (G123) Tumor limited to the endometrium or invades < 1/2 myometriumlB (G 123) Invasion to > 1/2 myometriumII Involvement of the stromal connective tissue of the cervixIII Spread outside the uterus, confined to the pelvisIIIA (G 123) Tumor invades serosa and/or adnexa (directly or by metastasis)IllB (G 123) Vaginal involvement (directly or by metastasis) or of parametriumIIIC (G 123) Metastases to pelvic (C1) and/or paraaortic lymph nodes (C2)IV Spread to the bladder mucosa, and/or bowel mucosa, and/or distant sitesIVA (G 123) Tumor invasion of bladder and/or bowel mucosaIVB (G 123) Distant metastases including inguinal lymph nodes and/or

intraabdominal disease

FIGO staging of endometrial cancer is a surgical staging, that requires a laparotomy.Degree of differentiation is expressed histopathologically as follows.

Gl < 5% of a nonsquamous or nonmorular solid growth pattern.G2 6-50% of a nonsquamous or nonmorular solid growth pattern.G3 > 50% of a nonsquamous or nonmorular solid growth pattern.

A. If a pure endometrioid carcinoma is confined to the uterus, check if the patientdesires fertility sparing. If she does, check if she meets the criteria - IAG1 confirmedby histopathology after D&C and MRI), no contraindications to medical therapy orpregnancy, willing to accept the risk involved. If she does, treat her with continuousprogestin based therapy with medroxyprogesterone, megestrol or levonorgestrelIUD. Check endometrial histopathology every 3 months. If the response is completeat 6 months, encourage her to get pregnant. Perform total abdominal hysterectomy(TAH) and bilateral salpingo-oophorectomy (BSO) and surgical staging after herchildbearing is over. If there is no response, or if she is unwilling or ineligible for

fertility sparing treatment, perform TAH, BSO and surgical staging. Surgical taggingincludes biopsy of any intraperitoneal lesions, omental biopsy, excision of suspiciouspelvic and paraaortic lymph nodes, but not peritoneal cytology (to be recordedseparately). Systematic pelvic lymphadenectomy is not recommended. If the patientis not fit for surgery, give tumor directed radiotherapy (TDRT), or in selected caseshormone therapy. If there is cervical involvement and the tumor is operable, performradical hysterectomy RH, BSO and surgical staging. Alternatively, for cases withcervical stromal involvement, 75-80 Gy irradiation may be given to point A followedby TH, BSO, and surgical staging. For inoperable cases, TDRT is given, followed bysurgery if operable. Treat unresectable extrrauterine pelvic disease with radiotherapy(RT) + chemotherapy (ChT) + surgery. Treat intraabdominal disease with TAH,BSO, and staging. Treat extraabdominal or hepatic disease with TAH, BSO, staging,+ ChT + RT with/without hormone therapy. After staging, treat stage IAG1 withobservation or brachytherapy (BrT), IAG2 and IBG1,2 with BrT + pelvicradiotherapy (PRT), IAG3 with BrT + PRT or ChT, and IBG3 with PRT + BrT +chemotherapy (ChT). Treat stage IIG1 with brachytherapy (BrT) + pelvicradiotherapy (PRT), IIG2 with BrT and PRT, IIG3 with PRT,BRT + ChT. Treatstage IIIA with ChT + RT or PRT + BrT or TDRT + ChT, IIIB with ChT + TDRT,IIIC with ChT + TDRT. Treat stage IVA with anterior and posterior exenteration asappropriate. Treat IVB with systemic palliative hormone or ChT and surgery.B. A serous or clear cell carcinoma and a carcinosarcoma are treated similarly. AfterTAH, BSO and surgical staging, treat a stage IA G1,2; IBG1,2 disease with BrT oronly observation (if there is no residual disease in hysterectomy specimen). TreatIBG3 with PRT, II with PRT + BrT, III and IV with ChT. Give sequential RT forlymph node disease and ChT + RT for palliation in metastatic disease.Vaginal brachytherapy after hysterectomy is limited to the upper vagina, at vaginalsurface or at a depth of 0.5 cm from the vaginal surface. The dosage is 2 or 3fractions of 4-6 Gy when used as a boost to external beam RT. Dosage of vaginalbrachytherapy alone is 5 fractions of 6 Gy to the vaginal surface or 7 Gy thrice at adepth of 0.5 cm from the vaginal surface.Single agent chemotherapy is with bevacizumab carboplatin, cisplatin, doxorubicin,docetaxel, ifosfamide, paclitaxel, ridaforolimus, temsirolimus, topotecan.Combination chemotherapy regimens include carboplatin-paclitaxel, cisplatin-doxorubicin, carboplatin-docetaxel, ifosfamide-paclitaxel, cisplatin-ifosfamide andcisplatin-doxorubicin-paclitaxel.Hormone therapy is with medroxyprogesterone acetate, megestrol acetate, GnRHanalogs or aromatase inhibitors.C. Follow up schedule is clinical examination every 3 months for 2 years, every 6months for 3 years and then every year. Ultrasonography, CT, PET and MRI scansare obtained as required. Genetic testing is done for patients below the age of 50years and if there is a family history of endometrial or colorectal cancer.Microsatellite instability is found in patients with hereditary non-polyposis coloncancer.

D. Treat vaginal recurrence with external beam RT (EBRT) + BrT, central pelvicwith surgery or RT, and regional pelvic RT + ChT.

87

Page 94: Differential Diagnosis and Management Options inuploads.worldlibrary.org/uploads/pdf/20180107071525differential... · 83 Peripartum cardiomyopathy 165 84 Hypertension 167 85 Convulsions

ENDOMETRIAL CARCINOMA

Check histopathological type

Pure endometrioid (A) Serous/clear cell Carcinosarcoma(B)

Confined to uterus Cervical involvement ExtrauterineTAH + BSO + Resection of all disease + Surgical staging

Check operability Check operability Unresectable ex-trrauterine pelvic

IA G1,2; IBG1,2 IBG3 II III, IVYes No

TDRT

Yes No

TDRT

RT +Chemotherapy+ surgery

Intraab-dominal BrT or observation PRT PRT + BrT ChT

Check desire andeligibility forfertility sparing

RH + BSO +surgical stagingor radiotherapy +TAH+BSO+ staging

Extraabdom-inal or hepatic

TAH+BSO

+ stagingFollow up

Yes NoII

TAH + BSO + ChT/ + RT/+ hormone therapy

Fertility sparingTherapy Radiotherapy +

Chemotherapy III IV

Cure at6 months

Noresponse

Permitpregnancy

Chemotherapy +Radiotherapy

TAH+BSO+surgical staging

Childbearingcomplete

Vaginal EBRT+ BrT

IA IB Centralpelvic

SurgeryOr RT

ObserveRadiotherapy

Radiotherapy +Chemotherapy Regional

pelvicRT +ChT

Follow up Recurrence (D)

88

Page 95: Differential Diagnosis and Management Options inuploads.worldlibrary.org/uploads/pdf/20180107071525differential... · 83 Peripartum cardiomyopathy 165 84 Hypertension 167 85 Convulsions

45. Uterine SarcomaStaging of Uterine Sarcomas

I Tumor limited to the uterusIA Tumor 5 cm or less in greatest dimensionlB Tumor more than 5 cmIl Tumor extends beyond the uterus, within the pelvislIA Tumor involves adnexaIIB Tumor involves other pelvic issuesIII Tumor infiltrates abdominal tissues, not just protrudes into the abdomenIIIA One siteII1B More than one siteIIIC Regional lymph node metastasesIVA Tumor invades bladder or rectumIVB Distant metastasis (excluding adnexa, pelvic, and abdominal tissues)

A. For a sarcoma confined to the uterus, perform total abdominal hysterectomy(TAH) plus bilateral salpingo-oophorectomy (BSO). TAH may be substituted for bylaparoscopic surgery too. If the woman is young, the ovaries may be spared. If thedisease is diagnosed after TAH and BSO has been done, do imaging (CT, MRI, orPET-CT) to see if there is any spread and resect those lesions. If the patient is not fitto undergo an operation, give pelvic radiotherapy with/without brachytherapy and/orchemotherapy or hormone therapy (only for endometrial stromal sarcoma). Furthertreatment depends on the histopathological type of the tumor, as discussed under ‘B’.

B. In case of spread beyond the uterus, check if the tumor is resectable. If it is,perform TAH, BSO, and resection of the metastases. Check the histopathologicaltype of the tumor if it is not resectable, and also of cases of all stages that have beentreated with surgical resection. An endometrial stromal sarcoma (ESS) showsmorphologic features of proliferative phase endometrial stroma and any mitoticindex. Treat stage I ESS with hormone therapy (medroxyprogesterone acetate,megestrol acetate, GnRH analogs or aromatase inhibitors). Treat stage II, III, andIVA ESS with hormone therapy and tumor directed radiotherapy (TDRT). TDRT isdirected at sites of tumor. It includes external beam radiotherapy (EBRT) and/orbrachytherapy (BT). EBRT for microscopic disease is 45-50 Gy. Vaginalbrachytherapy after hysterectomy is limited to the upper vagina, at vaginal surface orat a depth of 0.5 cm from the vaginal surface. The dosage is 2 or 3 fractions of 4-6

Gy when used as a boost to EBRT. Dosage of vaginal brachytherapy alone is 5fractions of 6 Gy to the vaginal surface or 7 Gy thrice at a depth of 0.5 cm from thevaginal surface.

C. High grade endometrial sarcoma shows pleomorphism more than that seen inproliferative phase endometrial stroma or it completely lacks any stromaldifferentiation. Its mitotic index is above 10/10 hpf. High grade endometrialsarcoma and leiomyosarcoma are treated similarly. Observe stage I cases or givethem chemotherapy. Combination chemotherapy regimens include docetaxel-gemcitabine (for leiomyosarcoma), doxorubicin-dacarbazine, doxorubicin-ifosfamide, gemcitabine-dacarbazine, and gemcitabine-vinorelbine. Single agentchemotherapy is with dacarbazine, docetaxel, doxorubicin, epirubicin, gemcitabine,ifosfamide, pazopanib, temozolomide, and vinorelbine. Treat stage II and III caseswith hormone therapy and TDRT. Treat stage IVA cases with chemotherapy and/orradio therapy. Treat stage IVB cases with chemotherapy with/without palliativeradiotherapy.

D. If the patient has a recurrence, check the site of the recurrence. In case it is a localrecurrence, check if the patient has received radiotherapy before. If she has, performresection with/without intraoperative radiotherapy or give chemotherapy. In case shehas not, perform resection with/without intraoperative radiotherapy or give tumordirected radiotherapy with/without chemotherapy. If the patient has distant isolatedmetastases, check if the lesion is resectable. If it is, perform resection with/withoutpostoperative radiotherapy, hormone therapy or chemotherapy. If it is not resectable,give chemotherapy with/without palliative radiotherapy or give radiotherapy alone.In case of disseminated metastases, check the histopathological type. If it isendometrial stromal sarcoma, give hormone therapy with/without palliativeradiotherapy. For all the other types, give chemotherapy with/without palliativeradiotherapy.

E. Follow up schedule is as follows.

1. Clinical examination every 3 months for 2 years, then every 6 months for 2 yearsand then every 12 months.2. Obtain CT scan of the chest, abdomen, and pelvis every 6 months for 4 years andthen every 12 months. Obtain MRI or PET scan as required.

89

Page 96: Differential Diagnosis and Management Options inuploads.worldlibrary.org/uploads/pdf/20180107071525differential... · 83 Peripartum cardiomyopathy 165 84 Hypertension 167 85 Convulsions

UTERINE SARCOMA

Check stage and nature of the tumor

Confined to uterus(A) Spread beyond uterus (B) Recurrence (D)

Check fitness for surgery Check resectability of the tumor Local Distant isolated Disseminated

No Yes Resectable Not resectable Check if radiotherapywas given in past

Check if resectable Check type

Pelvic radiotherapy +brachytherapy and/or

chemotherapy orhormone therapy

TAH+BSO TAH+BSO +resection ofmetastases Yes No Yes No

Endometrialstromalsarcoma

Other

Follow up Check histopathological type of the tumor Chemotherapy +resection, hormonetherapy, radiotherapy

Resection +postoperativeradio, hormone orchemotherapy

Hormone therapy+ radiotherapy

Endometrial stromalsarcoma

High gradeendometrial sarcoma

Leiomyosarcoma Resection +intraoperativeradiotherapy ortumor directedradiotherapy +chemotherapy

Chemotherapy +radiotherapy or

radiotherapy

Chemotherapy+ radiotherapy

Check stage Check stage (C)

I II, III, IVA IVB I II III IVA IVB

Hormonetherapy

Hormonetherapy +

Tumordirected

radiotherapy

Hormonetherapy +

radiotherapy

ObserveChemoth-rapy

Chemoth-erapy andradiothe-rapy

Chemoth-erapy andradiothe-rapy

Chemoth-erapyand/ orradio-therapy

Chemotherapy +radiotherapy

Follow up (E)

90

Page 97: Differential Diagnosis and Management Options inuploads.worldlibrary.org/uploads/pdf/20180107071525differential... · 83 Peripartum cardiomyopathy 165 84 Hypertension 167 85 Convulsions

46. Ovarian CarcinomaThe diagnosis of ovarian carcinoma is based on clinical findings (see

chapter 18), ultrasonography, CT scan or MRI, peritoneal fluid cytology, tumormarker study, and in some cases, fine needle aspiration cytology or biopsy.Investigations are also done for involvement of urinary tract (renal chemistry andexcretory urography) and distant metastases. When in doubt about the diagnosis, afrozen section is obtained during the laparotomy, so that definitive treatment can begiven.

FIGO Staging of Ovarian Cancer (2014)I Growth limited to ovarieslA Tumor limited to 1 ovary, capsule intact, no tumor on surface, negative

washings.IB Tumor involves both ovaries otherwise as in IA.IC Tumor limited to 1 or both ovaries, with:IC1 Surgical spillIC2 Rupture of capsule before surgery or tumor on ovarian surface.IC3 Malignant cells in the ascites or peritoneal washings.II Growth involving one or both ovaries with pelvic extension.IIA Extension and/or implant on uterus and/or fallopian tubes.IIB Extension to other pelvic intraperitoneal tissuesIII Tumor involves 1 or both ovaries with cytologically or histologically

confirmed spread to the peritoneum outside the pelvis and/or metastasis tothe retroperitoneal lymph nodes.

IIIA Positive retroperitoneal lymph nodes and /or microscopic metastasisbeyond the pelvis

IIIA1 Positive retroperitoneal lymph nodes onlyIIIA1(i): Metastasis ≤ 10 mmIIIA1(ii): Metastasis > 10 mm

IlIA2 Microscopic, extrapelvic (above the brim) peritoneal involvement ±positive retroperitoneal lymph nodes

IIIB Macroscopic, extrapelvic, peritoneal metastasis ≤ 2 cm ± positiveretroperitoneal lymph nodes. Includes extension to capsule of liver/spleen.

IIIC Macroscopic, extrapelvic, peritoneal metastasis > 2 cm ± positiveretroperitoneal lymph nodes. Includes extension to capsule of liver/spleen.

IVA Pleural effusion with positive cytologyIVB Hepatic and/or splenic parenchymal metastasis, metastasis to

extraabdominal organs, including inguinal lymph nodes and lymph nodesoutside of the abdominal cavity

Other major recommendations are as follows:1. Histologic type including grading should be designated at staging.2. Primary site (ovary, fallopian tube or peritoneum) should be designated where

possible.3. Tumors in stage I but involved with dense adhesions justify upgrading to stage

II if tumor cells are present in the adhesions.

A. For stage I perform peritoneal washings for cytology, total abdominalhysterectomy (TAH), bilateral salipngooophorectomy (BSO) and omentectomy. Forstages II, III, IV, perform debulking surgery in addition. Any residual disease shouldbe < 1 cm in diameter. IA grade 1 and 2 may be observed. Or all IA cases exceptgrade 1 may be given 3-6 cycles of taxane or carboplatin chemotherapy (ChT). StageII-IV require 6-8 cycles of ChT. Additional surgery may be done after that to removeresidual tumor in these cases. Paclitaxel therapy is optional. Genetic risk evaluationis done. Follow up is done every months for 3 years, every 4-5 months for 3 years,and then every year. Clinical examination and CA125 levels are checked.Radiological imaging is used as required. In case of a recurrence or failure torespond to primary ChT (over 2 cycles), alternative ChT is give. Platinum sensitivepatients may be treated with carboplatin alone or with docetaxel,gemcitabine+bevacizumab, doxorubicin, paclitaxel, or cisplatin+gemcitabine.Platinum resistant patients may be treated with docetaxel, etoposide, gemcitabine,doxorubicin, bevacizumabd, paclitaxel, or topotecan.B. Borderline epithelial tumors are of low malignant potential. Observation issufficient if the entire tumor mass was removed initially. If it was not and the patientdesires further childbearing, unilateral salpingooophorectomy (USO) is done and thepatient is observed. If the patient has completed her family, the tumor is resected. Ifthere is recurrence under observation, it is treated as epithelial ovarian cancer.C. Germ cell tumors include dysgerminomas, immature teratomas, embryonaltumors, and endodermal sinus (yolk sac) tumors. They occur usually in children andyoung women and are often in stage I. If the woman desires childbearing function,USO is done for stage I. For other stages and for all who do not desire childbearingfunction, perform TAH, BSO, omentectomy, debulking and staging. Tumor markersfor these tumors include βhCG (choriocarcinoma), alpha-fetoprotein (endodermalsinus tumor), LDH (dysgerminoma, immature teratoma and endodermal sinustumor). Treat > II disease with 3-4 cycles of BEP (bleomycin 30 units/week,etoposide 100 mg/m2/d on days 1-5, cisplatin 20 mg/m2/d on days 1-5). If follow upwith tumor markers shows no elevation of the markers but residual disease onimaging, it is surgically resected. If the tumor markers are elevated, give ChT withpaclitaxel-ifosfamide-cisplatin.D. Of sex cord stromal tumors, granulosa cell tumors (adult, juvenile), fibrosarcoma,Sertoll-Leydig cell tumors (intermediate or poor differentiation, with heterologouselements), sex cord tumors with annular tubules, gynandroblastoma, steroid celltumors are malignant. Thecoma (luteinized, with increased mitotic figures), fibroma(cellular), Sertoll-Leydig cell tumors (well differentiated), gynandroblastoma havemalignant potential. Patients with stage I disease and desire to preserve childbearingfunction are treated with USO and observation. All others are managed with TAH,BSO, omentectomy and staging. Tumor markers are used, such as inhibin forgranulosa cell tumor. They are then given platinum-based ChT. ChT regimens forgerm cell tumors can also be used.

E. Carcinosarcomas are also known as malignant mixed mullerian tumors. Theirmanagement is as of epithelial ovarian cancers.

91

Page 98: Differential Diagnosis and Management Options inuploads.worldlibrary.org/uploads/pdf/20180107071525differential... · 83 Peripartum cardiomyopathy 165 84 Hypertension 167 85 Convulsions

OVARIAN CANCER

Epithelial (A) Borderline epithelial (B) Germ cell (C) Sex cord stromal ((D) Carcinosarcoma

I II, III, IV Check if initial resectionwas complete

Check histopathological type Check stage and desire forchildbearing

Manage as epithelialovarian cancer

TAH, BSO,omentectomy

TAH, BSO,omentectomy,

debulking Yes No

Dysgerminoma Immatureteratomas

Embryonaltumor Stage I, desires

childbearingAll

others

Observe/ChT ChT Observation Check desirefor childbearing

Endodermal sinus tumor USO TAH, BSO,omentectomy,

stagingRemission Relapse Yes No Check desire for childbearing Observation

ChT

Progression USO Resect residualdisease

Yes No Norecur-rence

Recur-rence

ChT Observation USOfor I

TAH, BSO, cytology,omentectomy

No relapse Relapse ChT

Surgery, ChT

Observation Treat as epithelialcancer Observation

No residual disease or recurrence Residual disease/recurrence

Check tumor markers

Normal Elevated

Resection ChT

Follow up

92

Page 99: Differential Diagnosis and Management Options inuploads.worldlibrary.org/uploads/pdf/20180107071525differential... · 83 Peripartum cardiomyopathy 165 84 Hypertension 167 85 Convulsions

47. Suspicion ofChoriocarcinoma

Gestational trophoblastic tumor is diagnosed easily if it follows a molarpregnancy. A high degree of suspicion is required in case of other pregnancy events.Usually hCG titers and histopathology are adequate for making a diagnosis. Findingtrophoblasts in uterine curettings may be confusing.

A. Intermediate trophoblast (IT) is extravillous trophoblast. It lines the intervillousspace and the placental bed. It is a transitional form between cytotrophoblast(CT) and syncytiotrophoblast (ST). IT is usually mononuclear, sometimes multinuclearwith a dense eosinophilic cytoplasm. Study of tissue architecture differentiatesbetween syncytial endometritis and placental site trophoblastic tumor, when ITis found in uterine curettings.

B. Villous trophoblast consists of CT and ST. They cover the chorionic villi.CT has mononuclear cells with clear cytoplasm and high mitotic rate. ST hasmultinucleated cells with abundant, dense cytoplasm. If histology shows CT andST on chorionic villi, the diagnosis is normal products of conception or a vesicularmole. If the trophoblast is nonvillous, the tissue architecture is studied to determineif it is a choriocarcinoma. It shows abnormal trophoblastic hyperplasia and anaplasiawith absence of villi and presence of extensive hemorrhage and necrosis. Thecells are found to infiltrate between myometrial cells. If only benign simple trophoblastis found, close follow-up is sufficient. If there is a suspicion of choriocarcinoma,history of antecedent pregnancy is checked. If the woman has had a term pregnancyor an abortion and now shows tissue like a choriocarcinoma, it is most probablya choriocarcinoma. If she has had a vesicular mole up to 4 months ago, it isa retained mole. Such a woman needs a close follow-up to detect developmentof a choriocarcinoma early. If the mole was evacuated more than 4 months ago,the diagnosis is most probably a choriocarcinoma. In case a diagnosis of achoriocarcinoma is made, the WHO score is determined, as shown in table 47.1.

C. A score of up to 4 indicates low risk. If the woman is young and wishes topreserve her childbearing function, she is given single agent chemotherapy.Methotrexate with folinic acid "rescue" and actinomycin D are the most commonlyused agents. If she responds, 2 or 3 additional courses of chemotherapy are givenafter normalization of hCG levels. If hCG levels plateau for 3 consecutive weeks ormore, new lesions develop, or metastases occur, she is given multiagentchemotherapy with methotrexate, folinic acid, actinomycin D, andcyclophosphamide (MAC). If the woman has completed her family, hysterectomy isperformed.

D. A score of 5 to 7 indicates medium risk. Such a woman is treated with combinationchemotherapy with MAC. If she responds, the treatment is completed as for low riskgroup. If she does not respond, or if resistance develops to the chemotherapy later, she istreated with combination of using etoposide, actinomycin D, methotrexate, folinicacid, vincristine, and cyclophosphamide (EMACO).

Table 47.1 WHO Scoring System Based on Prognostic Factors

Prognostic factor Score

0 1 2 4

Age (years) < 39 > 39

Antecedent pregnancy Mole Abortion Term

Pregnancy event to treatmentinterval (months)

< 4 4 - 6 7-12 > 12

hCG (IU/L) < 103 103 - 104 104-105 > 105

ABO groups (Female X male) 0 X AA X 0

BAB

Largest tumor size (cm) 3-5 > 5

Site of metastases Spleen,kidney

GI tract,liver

Brain

No. of metastases 1-4 5-7 8

Prior chemotherapy Single drug > 2 drugs

E. A score of 8 or more indicates high risk. Such a woman is treated withEMACO regime, and in case of resistance to it, with POMB regime, which includesvincristine, methotrexate, folinic acid, bleomycin, and cisplatin. In case of cerebralmetastases; EMACO regime is combined with intrathecal methotrexate (10-12.5mg). Surgical procedures like hysterectomy, thoracotomy and laparotomy are usefulto remove drug resistant tumor, control hemorrhage, relieve bowel obstruction, andcontrol sepsis.

Follow-up of a fully treated woman is done as follows.1. Quantitative serum beta-hCG assays weekly for 3 months, monthly for 3

months, every 2 months for 6 months, every 3 months for 1 year, and thenevery 6 months.

2. Pelvic examination and chest radiography every 6 months.3. Contraception by barrier methods or oral combination contraceptive pills for 2

years.

93

Page 100: Differential Diagnosis and Management Options inuploads.worldlibrary.org/uploads/pdf/20180107071525differential... · 83 Peripartum cardiomyopathy 165 84 Hypertension 167 85 Convulsions

TROPHOBLAST IN UTERINE CURETTINGS

Check histological type

Intermediate (A) Cyto- and syncytiotrophoblast (B)

Check tissue architecture Check tissue architecture

Syncytial endometritis Placental site trophoblastic tumor Nonvillous Villous

Assess for malignancy

Benign Suspicious of choriocarcinoma Choriocarcinoma Products ofconception

Mole

Simpletrophoblast

Check history of antecedent pregnancy

Term pregnancy Abortion Vesicular mole

Check duration since evacuation of mole

Choriocarcinoma > 4 months < 4 months

Retained moleCheck WHO score

< 4 (C) 5-7 (D) > 8

Check childbearing function MAC chemotherapy EMACO chemotherapy

Incomplete Complete Response Resistance Response Resistance

Single agent chemotherapy Hysterectomy EMACO chemotherapy POMB chemotherapy

Response Resistance

MAC chemotherapy

94

Page 101: Differential Diagnosis and Management Options inuploads.worldlibrary.org/uploads/pdf/20180107071525differential... · 83 Peripartum cardiomyopathy 165 84 Hypertension 167 85 Convulsions

48. Suspicion of VulvarCarcinoma

A. Vulvar intraepithelial neoplasia (VIN) presents most commonly as a whitelesion. Sometimes it is red or pigmented. The lesion is usually raised, papular,or localized, most often on the inferior aspect of the labia minora. The lesionmay be multicentric, or covering the entire vulva. Leukoderma is a depigmentationdisorder in which the skin is otherwise normal, and similar lesions are foundelsewhere on the body too. Vulvar colposcopy is not as satisfactory as colposcopyof the cervix and vagina. It is useful only if the lesion is devoid of hyperkeratosisand is located on the mucosal part of the introitus, as vulvar skin does not revealabnormal vascular pattern on colposcopy. Vulvar biopsy is essential for the diagnosisof VIN. If there is no atypia of the epithelium, presence of local irritants is lookedfor. Irritants like detergents on underwear, deodorants, soaps, powders cause vitiligo.The treatment is avoiding contact with such irritants, controlling local infection(if any), and keeping the area clean and dry. If there is no local irritant, thediagnosis is chronic vulvar dystrophy. It is treated depending on its type. Lichensclerosus is treated by local application of 2% testosterone propionate. If thewoman does not respond to it, progesterone in oil is applied locally twice a dayfor 6 weeks. Perineoplasty is done if there is dyspareunia due to constrictionof the introitus. Hyperplastic vulvar dystrophy is treated with local applicationof a corticosteroid cream. Wide local excision may be done in cases with markedatypia, especially when the lesions are resistant to topical steroid therapy.

VIN is treated by wide local excision, with a 1 cm disease free surgical margin. Ifthe woman is young and has multicentric lesion, skinning vulvectomy is done withor without partial clitorectomy, followed by application of a partial thickness skingraft. Total vulvectomy is done when an elderly woman has extensive VIN. CO2

laser vaporization of VIN produces excellent cosmetic results and low recurrencerates. Topical application of 5-FU (5%) has lower success rate. It is not effective ifthe lesion is hyperkeratotic. Photodynamic therapy appears promising, and is stillunder evaluation.

Intraepithelial Paget's disease is treated by wide local excision of the lesion with atleast 2.5 cm of tumor free margin. The depth of excision is kept at the level ofunderlying fat to rule out missing adjacent glands which are involved. Large surgicaldefects are covered with full thickness rotational flap grafts. Total vulvectomy is doneif the disease is multicentric.

B. An early squamous cell cancer of the vulva is a small, shallow, indurated ulcerwith elevated, nodular margins. An advanced cancer is exophytic, papillomatous, andfungoid. Hyperkeratosis with or without superficial erosions and ulcerations is seenin the surrounding skin. A basal cell carcinoma is an ulcerated nodule with bloody,foul-smelling discharge. The ulcer is superficial, with slightly elevated, rolled edges(rodent ulcer). An adenocarcinoma often presents like an inflammatory mass whichfails to heal after marsupialization or abscess drainage. A verrucous carcinoma is a

florid, large, exophytic, white, cauliflower-like growth with superficial ulceration. Amalignant melanoma is a black colored ulcerative lesion. Occasionally it is seen as anevus or an exophytic mass. A biopsy is done to confirm the diagnosis of vulvarcancer and determine its type. The treatment depends on the type of cancer and itsstage.

FIGO Staging of Vulvar Cancer

Stage I Tumor confined to vulva and/or perineum, no nodes.

Stage IA ≤ 2cm in size, < 1 mm stromal invasion.

Stage IB > 2cm in size, > 1 mm stromal invasion.

Stage II Tumor of any size with adjacent spread (1/3 lower urethra, 1/3 lowervagina, anus), no nodes.

Stage IIIA Tumor of any size with positive inguino-femoral lymph nodes(i) 1 lymph node metastasis greater than or equal to 5 mm(ii) 1-2 lymph node metastasis(es) of less than 5 mm

Stage IIIB (i) 2 or more lymph nodes metastases > 5 mm(ii) 3 or more lymph nodes metastases less than 5 mm

Stage IIIC Positive node(s) with extracapsular spread

Stage IV Tumor invasion of upper urethra, bladder mucosa, rectal mucosa,pelvic bone, regional or pelvic lymph nodes.

Stage IVA (i) Tumor invades other regional structures (upper 2/3 urethra, upper2/3 vagina), bladder mucosa, rectal mucosa, or fixed to pelvic bone(ii) Fixed or ulcerated inguino-femoral lymph nodes

Stage IVB Any distant metastasis including pelvic lymph nodes.

Basal cell carcinoma is treated by wide local excision. Treatment ofsquamous cell carcinoma and adenocarcinoma is similar. Stage I tumor with lessthan 1 mm stromal invasion is treated by radical local excision (RLE), while thatwith 1 mm or greater invasion is treated depending on its location, as follows.

1. Lateral: radical local excision, plus unilateral groin node dissection (UGND).2. Midline: radical vulvectomy (RV) plus bilateral groin node dissection (BGND).3. Perineum: radical local excision plus BGND.4. If edges are positive for cancer, adjuvant external beam radiotherapy (EBRT).5. If lymph nodes are positive, EBRT + concurrent chemotherapy (CT).

Stage II disease is treated by RV plus BGND, EBRT and CT. Thatmeasuring more than 4 cm in diameter and all stage III tumors are treated by RV,BGND, and bilateral pelvic node dissection, EBRT and CT. Groin EBRT is given toselected cases. Selected cases of stage IV disease benefit from an anterior exenteration.Others are given EBRT + systemic CT or palliative treatment.

Verrucous carcinoma is treated by surgical excision and lymphadenectomy.Malignant melanoma may be treated by wide local excision with a 3 cm tumor freesurgical margin if it is unifocal and has less than 0.5 mm depth of invasion. In allother cases, RV and BGND is the treatment of choice.

95

Page 102: Differential Diagnosis and Management Options inuploads.worldlibrary.org/uploads/pdf/20180107071525differential... · 83 Peripartum cardiomyopathy 165 84 Hypertension 167 85 Convulsions

SUSPICION OF VULVAR CANCER

Assess nature of lesion

White lesion (A) Cancerous growth (B)

Assess for similar lesion elsewhere on skin Vulvar biopsy

Present Absent Invasive cancer

Assess local skin lesion

Vulvar colposcopyand biopsy

Determine type

Normaldepigmentedskin

Absence ofneoplasia

Intraepithelialcancer

Basal cellcarcinoma

Squamous cellcarcinoma

Adenocarcinoma Verrucouscarcinoma

Malignantmelanoma

Leukoderma

Assess for chronic irritants

Wide local excision Check stage Surgical excision +lymphadenectomy

RV + BGND

Present Absent

Stage I Stage II Stage III Stage IV

VitiligoChronicdystrophy

Check stromal invasion Assess tumor size Anterior exenteration(selected cases)

Determine type

VIN Intraepithelial Paget’s disease

< 1 mm > 1 mm < 4 cm > 4 cm

Lateral

Radical local excision Check location

Midline Perineum

RV + BGND

RLE + UGND RV + BGND RLE + BGND RV + BGND + bilateral pelvic node dissection

96

Page 103: Differential Diagnosis and Management Options inuploads.worldlibrary.org/uploads/pdf/20180107071525differential... · 83 Peripartum cardiomyopathy 165 84 Hypertension 167 85 Convulsions

49. Choice of LaparotomyIncision

A. Most of the gynecologic abdominal operations can be done through aninfraumbilical incision. Examples of infraumbilical incision are vertical midline,vertical paramedian (right or left), Pfannenstiel, Maylard, and Czerney. Sometimesaccess to upper abdomen is desired too, in addition to the access to the pelvis.Examples are omentectomy for ovarian carcinoma, paraaortic lymph node dissection,or separation of bowel adhesions which are unexpectedly encountered duringsurgery. A very large cystic ovarian tumor also needs a very large incision becauseit has to be removed intact. In such cases a vertical incision is made, becauseit can be extended upwards. A paramedian incision is extended along the sameline. A midline incision is continued around the umbilicus and then in midlineabove the umbilicus.

If the woman has a malignant disease, a vertical incision is preferred for variousreasons, as follows.

1. Wertheim's radical hysterectomy has best access if the incision extends abovethe umbilicus.

2. Ovarian carcinoma needs the following procedures, which necessitate access tothe upper abdomen.a. Omentectomy.b. Paraaortic lymph node biopsy.c. Removal of metastases from the upper peritoneal cavity, if any.

3. Krukenberg tumor has a primary in the stomach or colon, which has to belooked for, if not found on preoperative evaluation.

B. If access to the peritoneal cavity is urgent, a vertical midline incision isthe best, because it involves cutting the skin, subcutaneous fat, linea alba andperitoneum, which is quite rapid. An example of such a situation is a womanwith a ruptured ectopic pregnancy and massive hemoperitoneum.

C. Even a very good clinician can sometimes make a wrong diagnosis. If oneencounters a condition which cannot be dealt with adequately well through theincision made, the only options left are to struggle for a long time, make anotherincision, or extend the same incision in a surgically incorrect or disfiguring way.None of these options is really satisfactory. An example of this situation is tomake a Pfannenstiel incision for management of a misdiagnosed ruptured ectopicpregnancy, while the true cause of the hemoperitoneum is a splenic tear. Anotherexample is to open the abdomen through McBurney's grid-iron incision for removingappendix in a case of misdiagnosed acute appendicitis while the real diagnosis ispelvic endometriosis. Whenever there is any doubt about the diagnosis, a verticalincision is preferred, because it can be extended as required for optimum access.

D. If the woman already has a scar on the abdomen, it is preferable to excisethat scar and make a new incision over it, provided it gives adequate access

for intraabdominal surgery. One must keep in mind that there is fibrosis in theabdominal wall around a scar, and an incision of the same length as the previousone may not provide sufficient access because the cut edges of the incision cannotbe stretched as much as the previous time. A repeat Pfannenstiel incision is technicallymore difficult than a repeat vertical incision because there is extensive fibrosisbetween the rectus sheath and the recti. It is not possible to extend the incisionif required, while a vertical incision can be extended.

E. An obese patient is at significant additional risk for wound complications. In thesecases the incision should not involve the underside of the panniculus, because thatarea is more heavily colonized with bacteria, and is difficult to prepare surgically, toinspect and to keep dry in the postoperative period. A vertical midline or a transverseincision closer to the umbilicus is made. The former is developed periumbilicallyboth above and below the umbilicus. The rectus sheath is closed in these casesplacing sutures at least 1.5 to 2 cm lateral to the cut margins, successive bitesapproximately 1.5 cm apart along the length of the incision.

F. Desire for a cosmetic scar is considered only after the more important factorshave been considered adequately. A Pfannenstiel incision is preferred becauseits scar is not seen (bikini incision). Other advantages of Pfannenstiel incisionare as follows.

1. Postoperative pain is less.2. Postoperative breathing is better owing to less pain in abdomen. Hence the risk

of hypostatic pneumonia and deep vein thrombosis is less.3. The incidence of an incisional hernia with a Pfannenstiel incision is said to be

25% of that with a vertical midline incision. However there is no evidence thatvertical incisions are associated with greater risk of incisional hernia formation,provided they are closed properly.

If a Pfannenstiel incision does not provide adequate access to the peritonealcavity, the recti have to be divided transversely (Maylard incision). That causes aloss of the benefit of Pfannenstiel incision, even if the divided portions of the rectiare sutured back after the surgery.

97

Page 104: Differential Diagnosis and Management Options inuploads.worldlibrary.org/uploads/pdf/20180107071525differential... · 83 Peripartum cardiomyopathy 165 84 Hypertension 167 85 Convulsions

CHOICE OF A LAPAROTOMY INCISION

Assess possibility of need for access to the upper abdomen (A)

Yes No

Vertical midline incision Assess need for rapid entry into the peritoneal cavity (B)

Yes No

Vertical midline incision Consider possibility of a wrong diagnosis (C)

Yes No

Vertical midline incision Check for presence of a previous abdominal scar (D)

Yes No

Same incision as the previous one Check for morbid obesity (E)

Yes No

Vertical midline incision or transverse incision closer to umbilicus Assess desire for a cosmetic scar (F)

Yes No

Pfannenstiel incision Vertical midline incision/ Pfannenstiel incision

98

Page 105: Differential Diagnosis and Management Options inuploads.worldlibrary.org/uploads/pdf/20180107071525differential... · 83 Peripartum cardiomyopathy 165 84 Hypertension 167 85 Convulsions

50. Difficult VaginalHysterectomy

A vaginal hysterectomy is technically more difficult than an abdominalhysterectomy unless there is a significant degree of uterine prolapse. The reason forthis is that the space available is limited, and it cannot be increased as it can be inabdominal surgery by extension of the incision.

A. Normally vaginal surgery is done in lithotomy position. It cannot be givenif the woman has a disease of the hip joint(s). In such a case the position is modified,putting the legs in orthopedic stirrups, as during surgery on the hip joint.

B. If the introitus is too small to permit access to the vagina, as in case ofan unmarried woman undergoing a vaginal hysterectomy, a midline perineotomy isdone. If better access is desired, as in Schauta's radical hysterectomy orabdominoperineal radical hysterectomy, Schuchardt's incision is made. It is like amediolateral episiotomy but extending into the ischiorectal fossa and dividing medialfibers of the levator ani.

C. The pouch of Douglas is opened before opening the uterovesical pouch. Theorder may be changed on individual preference. It may be difficult to open itif it gets dissected off the back of the cervix by too deep dissection. In sucha case a finger is passed into the rectum to define its limits, and the doubleperitoneal fold of the dissected pouch is identified on the surface of the rectum.It is then opened. The pouch cannot be opened if it has been obliterated by adhesions.In that case the operation is completed by the abdominal route.

D. Difficulty may be encountered in opening the uterovesical peritoneal pouch,due to various reasons as follows.

1. The reflection of the peritoneum may be quite high. In such a case, theuterosacral and cardinal ligaments are clamped, cut, and ligated first. Thatincreases descent of the uterus, so that the pouch may be reached.

2. If a high position of the pouch does not seem to be the problem, a bladdersound is passed to define the limit of the bladder, so that it is not injured duringdissection. The pouch may be a little higher than the extent of the dissection. Inthat case the dissection is continued further and the pouch is reached.Sometimes too deep dissection into the front of the cervix results in the pouchgetting dissected off the front of the cervix. Then it needs to be identifiedcorrectly and opened.

3. It may not be possible to identify the pouch despite careful dissection. In thatcase a finger is passed through the open pouch of Douglas, over the uterinefundus, to the front of the uterus. The finger is then seen through theuterovesical pouch, and the peritoneum is cut over it.

4. The pouch may be obliterated by adhesions from past surgery, like a cesarean

section or ventral suspension operation. In such a case the uterine vessels areclamped, cut and ligated first, then dissection is done on both sides lateral to thecorpus of the uterus in the broad ligament to reach above the level of theadhesion between the uterus and the bladder. Then the scar tissue is dividedfrom lateral ends to the midline. If that is not possible the vaginal approach isabandoned and the operation is completed abdominally.

E. The uterine vessels are then clamped, cut, and ligated. A finger is passedbehind the uterus and its fundus is pushed out through the open uterovesicalpouch. It may not be possible due to the following reasons.

1. Uterine leiomyomas: the uterus is bisected in midline till the leiomyomas arereached. They are then enucleated. The uterine bulk is thus reduced and thefundus can then be delivered out.

2. Long uterus: it is bisected in midline. Then one half of the uterus can be pushedup and the cornual structures of the other half can be reached, to be clamped,cut, and ligated.

3. Lack of descent: it is treated by bisection as discussed above.4. Large uterus: it may be treated by any of the following techniques.

a. Bisection: see above.b. Lash procedure: a series of circumferential cuts are made in the uterine

corpus while maintaining traction on the cervix. The uterus gets coredout so that its width reduces, and then it can be delivered.

c. Morcellation: the corpus is removed piecemeal, cutting into it from within.Its bulk gets reduced and then it can be delivered.

d. The corpus is pushed to one side displacing it upwards. Then cornualstructures of one side can be clamped, cut and ligated. The cut portion of theuterus is held with Allis’ forceps and traction is made so that the uterusswings out by rotating around an anteroposterior axis passing through theother cornu. The other cornual structures can then be clamped, cut andligated.

5. Adhesions to the uterine corpus: adherent bowel or omentum is separated bysharp dissection to free the uterus. If that cannot be done, the operation iscompleted by the abdominal route.

F. The cornual structures are then clamped, cut, and ligated. The pelvic peritoneum isclosed by a purse-string suture of No. 2-0 absorbable material. It may be difficult tofind the cut edge of the uterovesical peritoneal pouch. Usually this is due todistension of the urinary bladder with urine, which elevates the peritoneum out ofreach. In that case the bladder is emptied by catheterization so that the peritonealedge can be reached. This problem is avoided by keeping an indwelling catheter inthe bladder throughout the operation. Another useful technique is to make traction onthe cornual pedicles in a downward and lateral direction, so that the peritoneal edgegets stretched between the two and becomes accessible. Suspension and closure of thevault usually pose no difficulties.

99

Page 106: Differential Diagnosis and Management Options inuploads.worldlibrary.org/uploads/pdf/20180107071525differential... · 83 Peripartum cardiomyopathy 165 84 Hypertension 167 85 Convulsions

DIFFICULT VAGINAL HYSTERECTOMY

Lithotomy position (A)Not possible due to disease of the hip joint(s) Possible

Modify lithotomy position with legs in orthopedic stirrups Assess access through the introitus (B)

Inadequate Adequate

Midline perineotomy or Schuchardt’s incision Circumferential vaginal mucosal incision at junction with cervix, open pouch of Douglas (C)

Not possible Possible

Pouch dissected off the cervix Pouch obliterated by adhesions Open uterovesical pouch (D)

Examine rectally, identify pouch and open it Complete hysterectomy by the abdominal route Not possible Possible

High location of the pouch Limits of the bladder not identifiable Pouch dissected off the front of uterus Pouch obliterated by adhesions

Clamp, cut and ligate uterosacraland cardinal ligaments first, then

open the pouch

Define bladder limits by passing abladder sound through the urethra,

then open the pouch

Define bladder limits by passing a bladdersound through the urethra, identify the pouch,

then open the pouch

Dissect laterally in broad ligament to abovethe level of the scar tissue, then divide the

tissue from sides to midline, and open pouch

Possible Not possible

Open uterovesical pouch. Clamp, cut and ligate uterine vessels. Deliver uterine fundus though uterovesical pouch (E) Complete hysterectomy by the abdominal route

Not possible Possible

Uterine leiomyoma Long uterus Lack of descent Large uterus Adhesions to the uterine corpus

Myomectomy Bisection Bisection Bisection, Lash procedure, morcellation or displacement ofcorpus to one side to access cornual structures on that side.

Dissect adherent structure off the uterus

Clamp, cut and ligate cornual structures. Close pelvic peritoneum. Suspend and close vaginal vault. (F)

100

Page 107: Differential Diagnosis and Management Options inuploads.worldlibrary.org/uploads/pdf/20180107071525differential... · 83 Peripartum cardiomyopathy 165 84 Hypertension 167 85 Convulsions

51. MyomectomyA woman with a symptomatic leiomyoma needs myomectomy if she is

young and desires continuation of menstrual and reproductive functions. It isnecessary to rule out contraindications for myomectomy, which are as follows.

1. Pregnancy.2. Puerperium or postabortal period.3. Pelvic inflammatory disease.4. Malignant disease of any part of the genital tract.5. Uterus studded with leiomyomas.6. Patient unwilling for hysterectomy if necessary.7. At least two units of blood compatible with the patient's blood not available.

A. A leiomyomatous polyp in the vagina is removed by polypectomy. If it arisesfrom the cervix, its pedicle can easily be clamped, cut, and ligated. If it is uterinein origin, and its pedicle is accessible, the technique is similar as for cervicalpolyp. If the pedicle cannot be reached easily, a simple rubber catheter is passedaround its base and traction is made so that the polyp descends and the pedicle canbe clamped. If its pedicle is not accessible, and if its widest diameter has passedthrough the cervix, it is grasped with a tenaculum or a bull-dog vulsellum andtwisted until its pedicle breaks. Usually the pedicle does not bleed. If it bleeds, asharp curettage is done, so that it gets shaved flush with the endometrial surface. Thevessels then retract in the myometrium and do not bleed further. Uterine packing isrequired as a last resort to control hemorrhage. The pack is removed after 8 hours. Ifthe widest transverse diameter of the polyp has not passed through the cervix whichis dilated and very well effaced, the cervix is cut in midline (posterior cervicotomy),the polyp is removed as described before, and the cervix is reconstructed. Thewoman is seen every 6 months at least up to the time of menopause for developmentof new leiomyomas.

B. If the leiomyoma arises from the corpus and is subserous or intramural inlocation, a laparotomy is done for performing a myomectomy. A subserous pedunculatedleiomyoma is removed by clamping, cutting, and ligating its pedicle. If it is subserousnonpedunculated, or intramural, the uterine wall has to be cut until the pseudocapsuleof the leiomyoma is divided and the leiomyoma bulges in the incision. It canthen be enucleated with the handle of a scalpel. Bleeding can be prevented usingBonney's myomectomy clamp on the uterine vessels, and one sponge holdingforceps on each infundibulopelvic ligament (application time not exceeding 20minutes). Alternative techniques include tying a rubber catheter as a tourniquetaround the isthmus (Rubin’s method) and injection 5 ml of vasopressin solution (20

units in 20 ml of normal saline) at each of the four locations – around uterine vesselsand at anastomosis of uterine and ovarian arteries on the two sides. The uterineincision is placed on the anterior wall for leiomyomas in the anterior wall. Anyleiomyomas on the side of the first leiomyoma removed are removed throughtunneling incisions placed in the side of the bed of the first one.

C. If the leiomyoma is posterior fundal, it is removed by Bonney’s hood operationthrough a transverse incision placed near the anterior limit of the bulge caused by theleiomyoma. This incision is essentially posterior to the line joining the two cornua.The hood of pseudocapsule and overlying muscle and serosa is used to cover thetumor bed and then the hood is fixed to the top and upper part of the front of theuterus. It is fixed in this position by a series of transverse rows of sutures of No 1-0delayed absorbable suture, placed through the inner surface of the hood and theuterus. The resultant scar is anterior, so that the risk of development of fixedretroversion of the uterus, or adhesions between bowel and/or omentum and theuterus is thus eliminated.

D. If the leiomyoma is low down in the posterior uterine wall, a primary posteriorincision has to be made. Though some workers recommend cutting throughthe anterior uterine wall and the posterior endometrium to reach such a tumor,the risk of rupture of the uterus in a future pregnancy is too high. Hence it isnot recommended.

E. A central cervical leiomyoma may be removed through an anterior transverseincision placed after separation and downward displacement of the urinary bladder.However it may be difficult to do so safely, and it is associated with the riskof transverse division of the cervical canal if it is stretched over the front ofthe leiomyoma. A better method is Rutherford Morrison's operation, in whichthe uterus is bisected in midline to reach the leiomyoma. It has to be reconstructedafter the myomectomy. Blood loss can be severe, since Bonney's myomectomyclamp cannot be used in presence of a cervical leiomyoma.

F. A pseudobroad ligament leiomyoma arises from the side of the uterus. Itdisplaces the ureter downwards and laterally. It has a pseudocapsule, which canbe divided on its upper aspect without harming the ureter. A true broad ligamentleiomyoma does not have a pseudocapsule. It may be on any aspect of the ureter.Hence the ureter has to be isolated before dissecting and removing the leiomyoma.Both of these types are approached by dividing the round ligament and adjacent partof the broad ligament, which are sutured back at the end of the operation.

All the operations done by a laparotomy can also be done by laparoscopy. Theoperative steps are the same, but the access and the instruments used are different.

101

Page 108: Differential Diagnosis and Management Options inuploads.worldlibrary.org/uploads/pdf/20180107071525differential... · 83 Peripartum cardiomyopathy 165 84 Hypertension 167 85 Convulsions

MYOMECTOMY

Determine site of the leiomyoma

Vaginal polyp (A) Uterine corpus (B) Broad ligament (F)

Cervical Uterine Check location in corpus Determine type

Check accessibility of pedicle Subserous Intramural Submucous

Yes No Pedunculated Nonpedunculated Hysteroscopic removal

Clamp, cut and ligatethe pedicle

Check if widesttransverse diameter is

outside the cervix

Laparotomy, clamp,cut and ligate pedicle

Determine site of leiomyoma

Yes No Anterior High posterior (C) Low posterior (D) Central cervical (E)

Twist polyp untilit comes off

Posterior cervicotomy Anterior uterineincision

Bonney’s hoodoperation

Posterior incision RutherfordMorrison’s operation

Check for vaginal bleeding Pseudo True

Yes No Open broad ligament. Enucleate leiomyomaby cutting pseudocapsule.

Open broad ligament. Enucleate leiomyomaby dissection safeguarding ureter.

Sharp curettage

Bleeding controlled Bleeding continues

Uterine packing for 8 hours

102

Page 109: Differential Diagnosis and Management Options inuploads.worldlibrary.org/uploads/pdf/20180107071525differential... · 83 Peripartum cardiomyopathy 165 84 Hypertension 167 85 Convulsions

52. Repair of VesicovaginalFistula

A. A vesicovaginal fistula (VVF) may be caused by cervical carcinoma, genitaltuberculosis, schistosomiasis of the bladder etc. These conditions are not curedspontaneously. If an attempt is made to repair it before the etiological conditionis cured by adequate treatment, it fails and the fistula recurs.

B. The prerequisites for undertaking repair of a VVF are as follows.

1. Any infection in the bladder and vagina should be controlled.2. Any bladder calculus should be removed.3. Any granulations and old sutures in the edge of the fistula should be removed.4. At least 3, 4, and 24 months should have passed since the etiological obstetric

trauma, gynecologic trauma, and irradiation respectively, responsible fordevelopment of the fistula.

5. At least 2 units of blood compatible with the patient's blood should be available.

C. Renal chemistry is studied. It is followed by an excretory urography to assessthe effect of the fistula on the ureters and kidneys, if any. A cystoscopy is doneto assess the proximity of the ureteric orifices to the fistula. If one or both theureters open in the edge of the fistula, a transabdominal extraperitoneal repairis done, at which the ureter(s) is (are) reimplanted into the bladder.

D. If the ureters are not involved in the edge of the fistula, treatment is determinedby the type of the fistula. If it is on the bladder base, a vaginal repair by thelayer method is satisfactory. An incision is made in the vaginal mucosa all aroundthe fistula. The vagina is widely separated from the bladder by sharp dissection.All fibrous tissue is excised. All bleeding other than gentle ooze is controlled.The bladder is closed in two layers. The first layer is a continuous stitch of No. 3-0polyglactin for the entire thickness of the bladder wall starting beyond the openingand ending beyond it at the other end. The second layer is placed burying the first

layer using No. 3-0 polyglactin passed through the bladder the muscle. The sutureline is kept horizontal if the fistula is near the cervix, vertical if it is near the urethra,and vertical if it is central. The vaginal suture line is kept parallel to the bladdersuture line. Overlapping of the two is avoided by using the flap sliding technique.

E. A circumferential fistula is associated with total destruction of the bladderneck and upper urethra. It is plastered to the back of pubic symphysis. It is repairedby the layer method. Relaxing incisions in the vagina on the sides help relievethe tension on the suture line.

F. A small fistula may develop in the vault of vagina after hysterectomy. Itis a little difficult to repair it by the layer method. It is best treated by Latzko'smethod, in which a small square of vaginal mucosa is excised around the fistula.Then it is inverted into the bladder and the opposite halves of the raw area areapproximated with interrupted sutures of No. 3-0 polyglactin.

G. If a woman has a VVF and a rectovaginal fistula (RVF) both, the VVF isrepaired first. The RVF is repaired after 3 months.

H. If the blood supply to the site of the fistula is poor due to multiple attempts atrepair in the past, extensive local fibrosis after obstructed labor and tissue necrosis,or irradiation, the fistula is unlikely to heal after a repair. Its blood supply can beincreased by putting a vascular graft over the suture line in the bladder. Any of thefollowing may be used.

1. Martius operation: a fat pad from a labium majus is passed submucosally to thesite of repair.

2. Ingleman-Sundberg operation: the upper end of the gracillis is detached fromthe pubis and transferred to the site of the repair.

3. Bastiaanse operation: an omental flap is placed between the suture lines in thebladder and the vagina.

I. If the woman has had multiple attempts at repair of her VVF, and it seems that itwill not heal, a urinary diversion procedure is carried out as a last resort.

103

Page 110: Differential Diagnosis and Management Options inuploads.worldlibrary.org/uploads/pdf/20180107071525differential... · 83 Peripartum cardiomyopathy 165 84 Hypertension 167 85 Convulsions

REPAIR OF VESICOVAGINAL FISTULA

Assess for presence of active disease responsible for the fistula (A)

Present Absent

Postpone repair until disease is treated Confirm that prerequisites are satisfied (B)

Excretory urography, cystoscopy.Assess involvement of ureter(s) (C)

Present Absent

Transabdominal extraperitoneal repair with reimplantation of ureter(s)into the bladder

Assess type of fistula

On bladder base (D) Circumferential (E) In vault, followinghysterectomy (F)

Combined (G) Poor local vascularity(H)

Multiple failedattempts (I)

Near cervix Near urethra Central Repair by layermethod, use relaxingincisions if required.

Latzko’s operation Repair VVF firstRepair RVF after 3

months

Vascular graft Urinary diversion

Repair bylayer method,

horizontalsuture line.

Repair bylayer method,

verticalsuture line.Reinforcesphincter.

Repair bylayer method,

verticalsuture line.

104

Page 111: Differential Diagnosis and Management Options inuploads.worldlibrary.org/uploads/pdf/20180107071525differential... · 83 Peripartum cardiomyopathy 165 84 Hypertension 167 85 Convulsions

53. Request for Reversal ofFemale Sterilization

A. Tubal ligation for female sterilization is a minor procedure. However its reversalis a major operation. In modern gynecology, assisted reproduction is preferred overtubal reconstructive surgery because it is far less invasive, can cause fewercomplications and has a good success rate. A woman who requests a reversal of hersterilization operation has to understand this information. She has to understand thata tuboplasty is a major operation and can have a number of complications includingthose of a laparotomy. She should also understand that the reversal procedure mayfail, or it may result in a tubal ectopic pregnancy. If she cannot afford assistedreproduction and is willing to accept all the risks of a tubal reconstructive surgery,then further evaluation is done.

B. Even if the fertility of the woman and potency of her husband have been provedin the past, it is possible that one of them acquired a defect after the sterilizationoperation. Hence it is necessary to evaluate the other causes of infertility (see chapters34 to 37) before subjecting the woman to tuboplasty to reverse the sterilizationprocedure.

C. Hysterosalpingography is done to assess the length of the proximal tubalsegment. If it is zero (cornual block), a tubocornual anastomosis would be requiredfor tubal reconstruction, which does not have such good results as a midsegmentreconstruction with adequate lengths of both segments. The results are not goodif there is a proximal segment which measures less than 2 cm in length. If the womanwithdraws her request for tubal reconstruction because of this, she is offered assistedreproduction or adoption. If she persists with her request despite counseling, or if theproximal tubal segment is 2 cm or more in length, further evaluation is done.

D. Laparoscopy is done to see if the fimbriae are present. If they are absent(as after a fimbriectomy), a tuboplasty is not successful. Such a woman is advisedagainst tuboplasty and is offered assisted reproduction or adoption. If the fimbriaeare present, but the distal tubal segment is less than 3 cm, the chances of successfuloutcome are not as good as when it is 3 cm or more in length. However afteradequate counseling, if the woman insists on a tuboplasty, it may be done. If theproximal segment measures 2 cm or more, the distal segment measures 3 cm or morein length, and fimbriae are present, the conditions are optimum for a tuboplasty, andit may be done.

Potential for Reversal of Tubal Sterilization

Method Potential for reversibility

Spring loaded clip Very good

Tantalum hemoclip Very good

Falope ring Good

Pomeroy Good

Modified Madlener Moderate

Uchida Poor

Wood Very good

Irving Good

Fimbriectomy Poor

Salpingectomy Poor

Cornual resection Poor

Simple ligation Very good

Electrocoagulation Poor

Chemical methods Poor

105

Page 112: Differential Diagnosis and Management Options inuploads.worldlibrary.org/uploads/pdf/20180107071525differential... · 83 Peripartum cardiomyopathy 165 84 Hypertension 167 85 Convulsions

REQUEST FOR REVERSAL OF FEMALE STERILIZATION

Counseling (A)

Opts for assisted reproduction Desires tubal reconstruction

Assisted reproductive technique Assess for other factors causing infertility (B)

Present Present

See chapters 33-36 Hysterosalpingographic assessment of the length of the proximal tubal segment (C)

Nil < 2 cm > 2 cm

Cornual block

Counseling

Withdraws request fortubal reconstruction

Persistent request for tubal reconstructive surgery Laparoscopy (D)

Assisted reproductionor adoption

Fimbriae absent Fimbriae present

Advise against tubal reconstructive surgery Tubal reconstructive surgery

106

Page 113: Differential Diagnosis and Management Options inuploads.worldlibrary.org/uploads/pdf/20180107071525differential... · 83 Peripartum cardiomyopathy 165 84 Hypertension 167 85 Convulsions

54. Postoperative FeverA. Postoperative fever is temperature rising above 380 C on two consecutive daysor above 390 C on one day after an operation. If the woman develops fever within thefirst 24 to 48 hours after an operative procedure or at ay time, while she is receivingintravenous fluids or blood transfusion, it is usually a reaction to pyrogens present inthe fluid being given intravenously, or the blood. Such fever is with rigors, and canbe quite high (up to 40° C). The container of the fluid, the infusion set, and theintravenous cannula are replaced by a fresh set. If it is caused by blood transfusion, itis stopped. The fever is treated by tepid sponging and administration of paracetamol. If itis not, other causes are looked for.

B. If fever develops on day 1-2, the respiratory system is examined. Fever withchills, cough, mucoid expectoration, dyspnea, chest pain, tachypnea, tachycardia,central cyanosis (in advanced cases), dullness to percussion, crepitations, rhonchi,wheezes and reduced intensity of breath sounds suggest pneumonia. Hypostaticpneumonia can develop in old, moribund patients postoperatively.Contributing factors include poor ventilation as a result of abdominal pain onbreathing movements, and a failure to get up and walk. The features are similar inaspiration pneumonia, but the expectoration is putrid due to anaerobic infection.Milder form of pulmonary embolism causes fever, productive cough, wheezing,tachypnea, crepitations, accentuation of second heart sound and edema of lowerlimbs. Such patients are evaluated and treated in joint consultation with a physicianand a pulmonologist. If the respiratory system is normal, other causes of fever arelooked for.

C. The urinary bladder is catheterized prior to all gynecologic major operations,and during all abdominal major procedures. If adequate aseptic precautions are nottaken during catheterization, bacteria are introduced into the urinary tract. If thecatheter is kept in place for some time after the operation, the risk of urinaryinfection is increased, directly proportional to the duration of catheterization.Dysuria, frequency and urgency suggest the presence of cystourethritis. If there isloin pain and renal angle tenderness, the woman is likely to have developed acutepyelonephritis. Microbiologic studies are done on a mid-stream urine sample, andthen appropriate antibiotic therapy is given. If the fever develops on day 2-3 and/orthe patient has complaints of dysuria, urinalysis and microbiologic studies on urineare carried out. Antibiotics are given based on antibiotic sensitivity report of urine.

D. Surgical wound infection is a not uncommon cause of postoperative infection.It usually takes more than 48 hours to set in. The skin punctures made by suturesand the skin edges appear inflamed. Pus may be expressed from the suture

line. If the hemostasis has not been perfect before approximation of the skinedges, a hematoma may form within the wound and get infected to form an abscess.In such a case, the sutures have to be removed to drain the abscess. Microbiologicstudies are made on the pus so that appropriate antibiotics can be given to thepatient. Vault of vagina may show features of sepsis after a vaginal hysterectomy orcolporrhaphy. Pelvic cellulitis, pelvic peritonitis and generalized peritonitis are moreserious forms of surgical site sepsis. These could be due to contamination duringsurgery or inadvertent bowel injury. Fever with chills, abdominal distension,tenderness, guarding and rigidity suggest presence of peritonitis. The are no bowelsounds. The patient is treated continuous nasogastric aspiration through a Ryles’tube, intravenous fluids, broad spectrum antibiotics, and if surgical consultation andmanagement if bowel injury is suspected.

D. Thrombophlebitis is not uncommon if adequate care is not taken while insertingan intravenous cannula and maintaining the intravenous line. The involved veinis tender, firm, cord-like. There is edema surrounding it. The treatment is antibioticsand local application of an anticoagulant cream.

E. If there is no surgical site sepsis, and/or fever developing after day 7, the lowerlimbs are examined. Presence of edema of lower limbs, pain, tenderness on calfcompression and straight leg raising suggest the cause is deep vein thrombosis. Asurgical reference is done and appropriate treatment is given if the diagnosis isconfirmed (see chapter 91).

F. When all causes of postoperativev fever have been excluded, iatrogenic factorsare looked for and treated, such as drug induced fever and reaction to transfusion ofblood and blood products. Administration of the offending drug is stopped.Transfusion reactions are treated in joint consultation with a transfusion medicineconsultant. If these causes are also ruled out, the most likely diagnosis is body’sreaction to trauma. It responds to supportive symptomatic treatment.

F. When fever develops more than a month after surgery, it could be due to viralinfection acquired from blood products (e.g. cytomegalovirus infection, hepatitis orHIV), parasitic infections acquired from transfusion (e.g. malaria, toxoplasmosis),surgical site infection (e.g. infection by more indolent microorganisms likecoagulase-negative staphylococci, delayed cellulitis due to surgical disruption ofvenous or lymphatic drainage, implantation of a foreign body like meshes and tapesused for prolapse repair), or infective endocarditis. Appropriate investigations aredone and treatment given in joint consultation with a physician for medical causes.Surgical site infection is treated as per microbiologic report. If that treatment fails,implanted foreign body will have to be removed.

107

Page 114: Differential Diagnosis and Management Options inuploads.worldlibrary.org/uploads/pdf/20180107071525differential... · 83 Peripartum cardiomyopathy 165 84 Hypertension 167 85 Convulsions

POSTOPERATIVE FEVER

Check interval from operation

Any (A) Day 1-2 (B) Day 2-3 (C) Day 3-7 (D) Beyond day 7 (E)

Check if with rigors Examine respiratory system

Normal

UrinalysisUrine culture

Check lower limbsYes No Aspiration

UTI NormalPyrogens in IV

fluidPneumonia Pulmonary

embolismCheck surgical site Deep vein thrombosis Normal

Change transfusionfluid, IV set and

cannula Abdominalwall/vaginal

infection

Pelvic infection Intraabdominalinfection

Normal

Resolution Fever persists

Check for iatrogenicfactors (F) After > 1 month (G)

Drug Blood/bloodproducts

None Viral from bloodproducts

Parasitic infectionsthrough transfusion

Surgical Site Infection

Reaction to trauma

108

Page 115: Differential Diagnosis and Management Options inuploads.worldlibrary.org/uploads/pdf/20180107071525differential... · 83 Peripartum cardiomyopathy 165 84 Hypertension 167 85 Convulsions

55. Postoperative HemorrhageA. Hemorrhage related to operations is of three types. primary hemorrhage occursduring the operation, reactionary hemorrhage occurs during the first few hours afterthe operation (when the patient recovers from the effect of sympathetic blockadeand the blood pressure rises), while secondary hemorrhage occurs after a fewdays. If the hemorrhage is of significant volume, the first step in the managementis resuscitation. An intravenous line is established with an 18-gauge cannula.Ringer's lactate is infused. Blood is administered as required.

B. The site of bleeding is then determined. It may be from the surgical incisionin the abdominal wall or vagina. A pressure dressing is applied to the bleedingabdominal wound. It usually arrests the bleeding. If it fails to do so, an underrunningsuture is placed over the bleeding site under local anesthesia. In case the bleedingis from the vaginal suture line, a vaginal pack is inserted. If it fails to achievehemostasis, the site of bleeding is examined under anesthesia and underrunningsutures are placed over the bleeding area.

C. If the site of bleeding is vaginal, from above the vault of vagina, the woman'scoagulation profile is checked. If it is abnormal, the coagulopathy is treatedappropriately. If it is normal, then the sutures in the vault are removed underanesthesia, and the source of the bleeding is sought. The bleeding vessels are caught

with a hemostat and ligated. If the bleeding cannot be controlled in this way, theanterior division of the internal iliac artery is ligated. If that also fails to control thebleeding, an umbrella pack is placed over the bleeding pelvic wall(s), and is broughtout vaginally. It is removed after 12 to 24 hours.

D. Intraperitoneal hemorrhage is diagnosed by rising pulse rate, fall in bloodpressure, pallor, increase in the abdominal girth, and shifting dullness. The diagnosisis confirmed by an ultrasonic scan (if it can be done in the ward without wastingtime). It shows presence of free fluid in the peritoneal cavity. An abdominalparacentesis is done, which yields free flow of fresh blood. Such a patient needsan immediate exploratory laparotomy and ligation of the bleeding vessel(s). Ifthe bleeding cannot be controlled in this way, the next step is ligation of theanterior division of the internal iliac artery, and if that fails, placement of anumbrella pack.

E. Secondary hemorrhage develops a few days after the operation. Usually itis due to the development of infection at the site of surgery. It results in infectionof a thrombus in a blood vessel and its erosion, causing hemorrhage. After resuscitationis initiated as required, a pressure pack is applied (e.g. vaginal pack), and broadspectrum antibiotics are administered parenterally. If that fails to control hemostasis,the bleeding vessel is embolized by interventional radiography. If that fails tocontrol the bleeding, the anterior division of the internal iliac artery is ligated.

109

Page 116: Differential Diagnosis and Management Options inuploads.worldlibrary.org/uploads/pdf/20180107071525differential... · 83 Peripartum cardiomyopathy 165 84 Hypertension 167 85 Convulsions

POSTOPERATIVE HEMORRHAGE

Note time after the operation (A)

Few hours Few days (E)

Reactionaryhemorrhage

Secondaryhemorrhage

Resuscitation, if required.Determine site (B)

Resuscitation, if required.Local pressureBroad spectrum antibiotics

Incision Vaginal (above vault)(C)

Intraperitoneal (D)Bleeding controlled Bleeding not controlled

Abdominal Vaginal Coagulation profile Exploratory laparotomy, ligation of bleeding vessel Embolization of bleeding vessel

Compression dressing Vaginal pack Abnormal Normal Bleeding controlled Bleeding not controlled Bleeding controlled

Bleedingcontrolled

Bleeding notcontrolled

Bleedingcontrolled

Correct defect Vaginal exploration, ligationof bleeding vessel

Undermining sutures Bleeding controlled Bleeding not controlled

Ligation of internal iliac artery

Bleeding controlled Bleeding not controlled

Umbrella pack

110

Page 117: Differential Diagnosis and Management Options inuploads.worldlibrary.org/uploads/pdf/20180107071525differential... · 83 Peripartum cardiomyopathy 165 84 Hypertension 167 85 Convulsions

56. Suspicion of IntraoperativeBladder Injury

A. Injury to the urinary bladder is not uncommonly the result of lack of adequatecare during surgery. However it can occur even when great caution is exercisedduring surgery owing to the presence of some conditions. These are as follows.

1. Dense adhesions between the bladder and the uterus due to past operations likecesarean section, ventral suspension etc.

2. Malignancy: cervical carcinoma or ovarian carcinoma with spread to the urinarybladder.

3. Pelvic endometriosis.4. Pelvic inflammatory disease.5. Placenta percreta.6. Cervical ectopic pregnancy.

B. The detrusor is more vascular than pelvic fascia and it bleeds when cut.Thus bladder injury may be suspected when there is unusual degree of bleedingduring dissection in that area. If there is a frank leakage of watery fluid, thediagnosis is almost confirmed. The bladder mucosa has a typical appearance, and itmay be identified if there is a big tear in the bladder wall.

C. A simple rubber catheter is passed into the urinary bladder. Blood stainedurine may be obtained in case the bladder has been injured. There may not beany urine in the bladder in case all of it has escaped through the tear in thebladder wall. The catheter is looked for in the area of dissection. It may be seenthrough the rent in the bladder wall, which confirms the diagnosis of bladderinjury.

D. If the catheter is not seen, sterile methylene blue solution is instilled intothe bladder through the catheter. It escapes through the rent in the bladder wall.Thus the site of injury is also identified. If methylene blue does not escape intothe operative field injury to the urinary bladder is ruled out. If facilities permit,a cystoscopy can be used conveniently for accurate diagnosis of presence andsite of bladder injury.

E. It is essential to diagnose ureteric injury, if any. If there is no injury to anyureter, the rent in the bladder is repaired in two layers. The first layer is a continuousnoninterlocking stitch of No. 3-0 synthetic delayed absorbable suture passed throughthe bladder wall including the mucosa. The second layer is a continuousnoninterlocking stitch of No 3-0 delayed absorbable suture, passed through thedetrusor, burying the first layer.. Adequacy of the repair is confirmed by instillingmethylene blue solution into the bladder. If it leaks out from any site, additionalsutures are placed in that area. The bladder is drained continuously for 15 daysthrough a Foley's catheter. This keeps the bladder decompressed so that the sutureline is rested enough and heals well. The healing is confirmed by a cystogramperformed prior to removal of the cathetrer.

111

Page 118: Differential Diagnosis and Management Options inuploads.worldlibrary.org/uploads/pdf/20180107071525differential... · 83 Peripartum cardiomyopathy 165 84 Hypertension 167 85 Convulsions

Risk factors (A)

SUSPICION OF INTRAOPERATIVE BLADDERINJURY

Clinical suspicion (B)

Pass a catheter into the bladder

Drainage of blood stained urine No urine is obtained

Look for catheter in the operative field

Catheter seen Catheter not seen (C)

Instill methylene blue solution into the bladder through the catheter

Methylene blue solution escapes into the operative field Methylene blue solution does not escape

Bladder injury is confirmed. Bladder injury is ruled out.

Assess involvement of the ureter(s) (D)

Present Absent

Repair injury in two layers. Continuous drainage of bladder for 15 days.

112

Page 119: Differential Diagnosis and Management Options inuploads.worldlibrary.org/uploads/pdf/20180107071525differential... · 83 Peripartum cardiomyopathy 165 84 Hypertension 167 85 Convulsions

57. Suspicion of OperativeInjury to Ureter

The ureter is more likely to be injured during surgery when the normalanatomy has been distorted due to conditions like pelvic endometriosis, pelvicadhesions due to past surgery, broad ligament tumors and pelvic malignancy. It canalso be injured when it runs an aberrant course. A number of ureteric injuries resultwhen adequate attention is not paid to the normal course of the ureter. Common sitesof ureteric injury are as follows.

1. At the point of crossing the bifurcation of the common iliac vessels.2. On the lateral pelvic wall lateral to the infundibulopelvic ligament.3. Below the uterine artery.4. At the site of entry into the urinary bladder.

A. If a clamp is accidentally applied to the ureter, it is removed immediately.If the ureter appears normal, the site is drained extraperitoneally for 48 to 72hours. On the other hand, if it appears to be crushed, it is catheterized by cystoscopyor cystotomy. The catheter is removed cystoscopically after 14 days.

B. If a ligature is found to have been placed accidentally around the ureter,it is removed immediately. A ureteric catheter is passed into it cystoscopicallyor through a cystotomy. It is removed cystoscopically after 14 days.

C. Partial transection of the ureter is repaired over a ureteric catheter withNo. 4-0 polyglactin interrupted sutures. The catheter is removed cystoscopicallyafter 14 days.

D. The treatment of complete transection of the ureter depends on the site ofthe injury. If it is near the urinary bladder, it is implanted into the bladder, usingthe submucosal tunnel technique. If it is not possible to do so, but it is within5 cm of the bladder, a psoas hitch operation is done, in which the bladder ismobilized and hitched from the psoas major muscle on the side of ureteric injury.The ureter can then be implanted into the bladder using the submucosal tunnel

technique. The older Boari flap operation is rarely done in modern gynecologicurologic practice. If the site of transection is farther away from the bladder than this,ureteroureteral anastomosis is done.

E. Ureteric injury may not always be recognized during the operation. If itis a transection, urinary leak begins vaginally soon after the operation. If it hasbeen crushing injury or a ligature around the ureter, it takes a few days to undergonecrosis, and then the leakage starts vaginally. It needs to be differentiated froma vesico-vaginal fistula (VVF). Methylene blue solution is instilled into the urinarybladder through a catheter passed in through the urethra. If the dye appears inthe vagina, it is a VVF. If it does not, it is a uretero-vaginal fistula (UVF). Thediagnosis is confirmed by excretory urography and cystoscopy. In the formertest, a cup-and-saucer appearance is seen with a VVF, and ureteric leakage intothe vagina is seen with UVF. A VVF is clearly seen on cystoscopy. UVF canbe diagnosed on cystoscopy by injecting indigo carmine dye into a peripheralvein. It is seen entering the urinary bladder through an intact ureter, but notthrough one which has developed a fistula with vagina. In case of a VVF, continuousbladder drainage is established with a Foley's catheter for 14 days. If the fistuladoes not heal, it is repaired surgically after 4 months (see chapter 52). If it isa UVF, a double-J stent is passed into the ureter cystoscopically, one end of thestent being placed in the renal pelvis, and the other in the urinary bladder. Itis removed after 6 weeks. If it cannot be passed, a percutaneous nephrostomyis done to save the renal function, and the fistula is repaired after 6 weeks.

F. Ligation of a ureter may cause loin pain, anuria, and fever postoperatively.If not recognized early and treated well, it may lead to the development of aUVF or renal necrosis and atrophy. The patient is evaluated by urgent ultrasonography,excretory urography, and chromocystoscopy. If the ureter is found to be injured,a double-J stent is passed from the renal pelvis to the urinary bladder cystoscopically.It is removed after 6 weeks. If that is not possible and it is less than 48 hourssince the operation, the concerned ureter is explored surgically and the injuryis managed as when diagnosed during the primary operation. If the interval islonger than 48 hours, a percutaneous nephrostomy is done immediately, and theinjury is repaired after 6 weeks.

113

Page 120: Differential Diagnosis and Management Options inuploads.worldlibrary.org/uploads/pdf/20180107071525differential... · 83 Peripartum cardiomyopathy 165 84 Hypertension 167 85 Convulsions

SUSPICION OF INJURY TO URETER

Check nature of injury

Accidental application ofa clamp (A)

Ligature around ureter (B) Partial transaction (C) Complete transaction (D) Postoperative leak of urinevaginally (E)

Postoperative loin pain,anuria, fever (F)

Remove clampimmediately, assess

ureteric damage

Remove ligature Primary repair over aureteric catheter

Note site Intravesical dye instillation test cystoscopy, excretory urography

Renal and ureteric ultrasonography excretory urography, chromocystoscopy

Ureterappearsnormal

Ureterappearscrushed

Bear bladder Away frombladder

VVF UVF

Extraperitonealdrain for 48 h

Assess feasibility ofimplantation into bladder

Ureteroureteralanastomosis

Continuous bladderDrainage for 14 d

Ureternormal

Ureterinjured

Catheterize ureter by cystoscopy or cystotomy, removeit after 14 d cystoscopically.

Fistulaheals

Fistula does notheal(see chapter 51)

Cystoscopic passage of a double-J stent

Possible Not possibleCystoscopic passage of a double-J stent

Ureteric implantationinto bladder

Psoas hitch and uretericimplantation into bladder Not

possiblePossible

Possible Notpossible

Percutaneous nephrostomy,repair fistula after 6 weeks

Remove stentafter 6 weeks

Note time since primary operation

> 48 h < 48 h

Percutaneous nephrostomy, repair fistula after 6 weeks Repair as when diagnosed during primary operation

114

Page 121: Differential Diagnosis and Management Options inuploads.worldlibrary.org/uploads/pdf/20180107071525differential... · 83 Peripartum cardiomyopathy 165 84 Hypertension 167 85 Convulsions

58. Operative Injury to BowelThe bowel may be injured while opening the anterior abdominal wall if it is

adherent to it due to previous exploration, abdominal tuberculosis, or peritonitis. Itmay be injured while being separated from the pelvic structures to which it isadherent due to pelvic inflammatory disease, past pelvic surgery, genital tuberculosis, ormalignancy. It may also be injured during laparoscopy.

A. If the injured segment of the bowel is diseased, it needs to be treated appropriately.It may show conditions such as tuberculosis, ulcerative colitis, regional enteritisand stricture due to any cause. The affected segment is resected if possible followedby either reanastomosis or an ostomy as appropriate..

B. The type of operation responsible for the injury is important in determiningthe treatment. Bowel may be injured during laparoscopy. The injury may be superficialand nonpenetrating. It can be repaired by laparoscopic placement of sutures.If it is cauterized by electrocautery or laser, it is treated like a penetrating injury. Themanagement of penetrating injuries caused during a laparoscopy and a laparotomy isthe same. A nonpenetrating injury caused during a laparotomy is repaired, with anadditional reinforcing suture line over the first suture line.

C. In case of cauterization or penetrating injury, the treatment is different forthe small bowel and the large bowel. If the injury to the small bowel is small,it is repaired in two layers transversely. If the injury is extensive, it is treatedby segmental resection and end-to-end anastomosis.

D. Sometimes the possibility of injury to the bowel during surgery is anticipated

and the bowel is prepared in advance by high bowel washes and locally actingantibiotic like neomycin, combined with erythromycin and metronidazole, as shownin the following table. If the bowel has been prepared and the degree of fecalcontamination of the peritoneal cavity is minimal, injuries to the right colon arerepaired in two layers transversely. In case of an injury to the left colon and/or grosscontamination of the peritoneal cavity, the treatment is extensive peritoneal lavageand a diverting colostomy.

Preoperative Bowel Preparation

On the day before the surgery:

1. Light breakfast in morning2. Noon3. Polyethylene glycol, 238 g in 4L solution PO over 4 to 6 hours4. 238 g PEG + 1.9 L5. Neomycin 1 g PO at 2 pm, 3 pm, and 10 pm plus6. Erythromycin base 1 g PO at 2 pm, 3 pm, and 10 pm or7. Metronidazole 500 mg PO at 2 pm, 3 pm, and 10 pm8. Clear liquids only after the bowel preparation is completed. Nothing orally

after midnight the day prior to surgery.

Postoperative management of all cases of bowel injury includes continuousnasogastric decompression, administration of broad spectrum antibiotics, andmaintenance of fluid and electrolyte balance. Peritoneal sepsis is watched for andtreated early.

115

Page 122: Differential Diagnosis and Management Options inuploads.worldlibrary.org/uploads/pdf/20180107071525differential... · 83 Peripartum cardiomyopathy 165 84 Hypertension 167 85 Convulsions

OPERATIVE INJURY TO BOWEL

Note condition of affected bowel (A)

Diseased Normal

Tuberculosis Ulcerative colitis Regional enteritis Stricture Determine type of operative injury (B)

Appropriate treatment, resection of affected segment of bowel if possible. Laparoscopy Laparotomy

Determine nature of injury Determine nature of injury

Nonpenetrating injury Electrocauterization, laser Penetrating injury Penetrating injury Nonpenetrating injury

Repair Note site Repair, with an additionalreinforcing suture line.

Small bowel (C) Large bowel (D)

Assess extent Check if bowel has been prepared

Small Extensive Yes No

Repair in two layerstransversely

Segmental resection, end-to-end anastomosis Assess degree of fecal contamination

Minimal Gross

Check side of colon

Repair in two layers transversely Right Left Extensive peritoneal lavageDiverting colostomy

116

Page 123: Differential Diagnosis and Management Options inuploads.worldlibrary.org/uploads/pdf/20180107071525differential... · 83 Peripartum cardiomyopathy 165 84 Hypertension 167 85 Convulsions

59. Postoperative AbdominalWound Breakdown

A. A number of factors are responsible for poor wound healing.

1. Glucocorticoid therapy.2. Vitamin C deficiency.3. Chronic renal disease.4. Chronic hepatic disease.5. Poor vascularity of local tissues.6. Lymphatic obstruction.7. Infection.8. Antineoplastic chemotherapy.

B. The abdominal wound breakdown varies from nonapproximation of the skinedges to burst abdomen (in which the peritoneum is also open and the contentsof the peritoneal cavity escape out). If the breakdown involves tissues up to andincluding the anterior rectus sheath, there is no cause for great alarm, becausethere is no threat to the patient's life. Discharge from the wound is obtainedfor microbiologic studies. Broad spectrum antibiotics are administered, whichcan be changed if necessary, depending on the antibiotic sensitivity report. Thewound is dressed twice a day, applying Eusol soaks for half an hour, followedby cleaning with hydrogen peroxide and normal saline, and then applicationof an antiseptic cream. This procedure is continued until healthy granulation tissue isseen in the entire wound. Then the skin edges are freshened and the wound is closedunder local anesthesia using vertical mattress sutures of No. 1-0 monofilamentnylon, each suture being passed through the entire thickness of the wound. Thesutures are removed after 10 days.

C. Burst abdomen is a state of total breakdown of the abdominal wound i.e.

nonhealing of any layer of the abdominal wall. It usually occurs throughout thelength of the incision, though sometimes only a part of the wound may be involved.It may be a total burst, in which the peritoneum has given way and the bowel comesout of the abdomen. It may be an adherent burst, in which the parietal peritoneum isintact, and the underlying bowel is adherent to it. It cannot be sutured for fear ofinjury to the bowel. In cases of adherent burst, the wound discharge is sent formicrobiologic studies and appropriate antibiotics are administered. Sterile paraffinsoaked dressing is applied to the bowel surface and antiseptic dressing is applied tothe wound edges. The wound is allowed to heal by second intention. It may beclosed with monolayer interrupted sutures of No. 1-0 monofilament nylon when thedeeper part of the wound has healed and bowel surface is well buried.

In case of a total burst, it is not only a frightening experience for the woman to findher bowel coming out of her abdominal incision, but is also a serious situationbecause serious and life-threatening infection of the peritoneal cavity may developwith this. Discharge of a large amount of serous or serosanguinous fluid oftenprecedes a burst abdomen, and should alert the clinician to this possibility. Suchcases should be kept in the hospital for observation rather than sent home until thedischarge stops, so that if a burst abdomen does develop, it takes place in the hospitalrather than on the way home or at home. Discharge from the wound is sent formicrobiologic studies. Broad spectrum antibiotics are administered parenterally.These can be changed as required, depending on the antibiotic sensitivity result of themicrobiologic studies. If the abdomen does burst, the bowel is put back into theperitoneal cavity, and a sterile binder is applied to the abdomen while preparationsare made for suturing of the wound. A nasogastric tube is passed and the stomachcontents are aspirated continuously (without suction, connected to a drainage bottle)as well as two hourly (with suction). Monolayer closure is done with No. 1monofilament nylon tension sutures. Each suture is passed through the entirethickness of the abdominal wall. If the patient's condition is unstable, the suturing ispostponed until it is stabilized. Then it is sutured the same way as described before.The sutures are removed after 12 days.

117

Page 124: Differential Diagnosis and Management Options inuploads.worldlibrary.org/uploads/pdf/20180107071525differential... · 83 Peripartum cardiomyopathy 165 84 Hypertension 167 85 Convulsions

POSTOPERATIVE ABDOMINAL WOUND BREAKDOWN

Rule out/treat factors which impair wound healing (A)

Assess depth (B)

Skin Subcutaneous tissue Anterior rectus sheath Burst abdomen (C)

Microbiological study of the discharge Regular dressing Appropriate antibiotics Secondary suturing when health granulation tissue is

present

Check type of burst

Total burst Adherent burst

Assess patient’s condition Microbiological study of the discharge Sterile paraffin dressing over bowel,

antiseptic dressing over sides of thewound

Abdominal binder Secondary suturing when healthy

granulation tissue is present and thebowel in the floor is well covered.

Good Unstable

Microbiological study of the discharge Appropriate antibiotics Monolayer closure of abdominal wall with

tension sutures

Microbiological study of the discharge Reposit bowel Abdominal binder Appropriate antibiotics Monolayer closure of abdominal wall with

tension sutures when condition stabilizes

118

Page 125: Differential Diagnosis and Management Options inuploads.worldlibrary.org/uploads/pdf/20180107071525differential... · 83 Peripartum cardiomyopathy 165 84 Hypertension 167 85 Convulsions

60. Posthysterectomy VaginalEvisceration

A. Posthysterectomy vaginal evisceration is a complication equivalent to totalbreakdown of a laparotomy wound (burst abdomen). It is due to one or moreof the following factors.

1. Infection.2. Vitamin C deficiency.3. Glucocorticoid therapy.4. Antineoplastic chemotherapy.5. Chronic renal disease.6. Chronic hepatic disease.

It is diagnosed when the patient complains of something coming out pervaginum and a local and/or speculum examination reveals bowel in the vagina,coming out through an open vault of vagina.

B. The eviscerated bowel is covered with a sterile mop soaked with sterile normalsaline. A nasogastric tube is passed and the stomach contents are aspirated continuously(without suction, connected to a drainage bottle) as well as two hourly (withsuction). The patient's condition is stabilized as much as possible, and an exploratorylaparotomy is performed as soon as possible. At laparotomy, the bowel and itsmesentery are examined carefully. The chief differences between a burst abdomenand vaginal evisceration responsible for ischemic damage to bowel with the latterare as follows.

1. Burst abdomen is noticed soon after it develops, while vaginal evisceration is

not noticed until bowel comes out of the introitus. The longer interval increasesthe chances of mesenteric vascular occlusion and infection both.

2. The opening in the abdominal wall after it bursts is larger than the narrowopening in the vaginal vault. The latter constricts the mesenteric vessels morethan the former.

3. The mesentery is stretched when the bowel escapes through the vaginal vault.This does not occur with a burst abdomen.

C. The bowel may develop gangrene because the mesenteric vessels get constrictedby the edges of the opening in the vaginal vault, or due to thrombosis of thevessels.

D. The mesentery of the eviscerated segment of bowel may be torn, which alsocan cause vascular damage or thrombosis. The treatment of gangrene of the boweland mesenteric vascular thrombosis is resection of the involved segment of thebowel and anastomosis of the cut ends. The peritoneal cavity is cleaned by lavagewith sterile normal saline.

E. There may be ischemic perforation of the bowel and peritonitis. It is treatedby resection of the involved segment of the bowel, anastomosis of the cut ends,and peritoneal lavage with sterile normal saline. If the bowel and mesenteryare entirely normal, no treatment need be given to them.

F. After the normal or treated bowel is reposited in the peritoneal cavity, thenecrotic portion of the edges of the opening in the vaginal vault is excised. Thefreshened edges are then approximated with interrupted sutures of No. 1-0 delayedabsorbable suture. The rectum and the back of the vaginal vault are thenapproximated with interrupted sutures of No. 1-0 delayed absorbable suture. Anintraperitoneal tube drain is placed if the patient has a perforative peritonitis. Theabdomen is then closed by monolayer technique, using No. 1 monofilament nylon.

119

Page 126: Differential Diagnosis and Management Options inuploads.worldlibrary.org/uploads/pdf/20180107071525differential... · 83 Peripartum cardiomyopathy 165 84 Hypertension 167 85 Convulsions

POSTHYSTERECTOMY VAGINAL EVISCERATION

Rule out/treat factors which impair wound healing (A)

Cover bowel by saline-soaked sterile mop Exploratory laparotomy Examine all bowel and mesentery (B)

Normal, healthy bowel Gangrenous bowel (C) Mesenteric tear with vasculardamage/thrombosis (D)

Perforative peritonitis (E)

Enterectomy with reanastomosis and peritoneal lavage.(F)

Excision of necrotic edges of vaginal defect and closure with interrupted sutures of No. 1-0 delayed absorbable suture. Approximation of front of the rectum and the back of the vaginal wall. Closure of abdomen (with intraperitoneal drainage in cases of perforative peritonitis

120

Page 127: Differential Diagnosis and Management Options inuploads.worldlibrary.org/uploads/pdf/20180107071525differential... · 83 Peripartum cardiomyopathy 165 84 Hypertension 167 85 Convulsions

61. Needle Breaking InsideTissues

A surgical needle is a dangerous foreign body if left inside tissues. It canmigrate and injure important structures nearby. Long term complications includeinfection and nerve irritation. If it has broken in the genital tract, the patient maydevelop anxiety due to fear of injury to her sexual partner or fetus. Leaving a brokenneedle behind amounts to medical negligence. All these complications are bestprevented rather than treated. Selection of a proper needle depends on the type oftissue, its tensile strength, shear strength, weave, penetrability, density, elasticity,and thickness. The needle holder also has to be appropriate for the needle. Theneedle should be held near the swaged eye at the junction of two-thirds and one-thirdof its length, so that the point protrudes after inserting the stitch. That makes it easierto retrieve the needle from the tissue. This is suitable for soft tissues like fat, muscle,and thin rectus sheaths. If the tissue is tough, in order to achieve greatest drivingforce, the needle should be grasped between its midpoint and tip. As the needle isadvanced, the holder is repositioned more and more proximally. In dense tissues, theneedle should not be grasped near its eye. The needle should be graspedperpendicular to the holder. If that is not possible, the least acute or obtuse angleshould be used. A pencil grip is less forceful than finger and thumb-ring grip, whichin turn is less forceful than the palmed grip. Stronger the grip, greater the risk ofbreaking of the needle. So the weakest grip compatible with driving the needlethrough the tissues should be used. A needle usually bends before it breaks. When aneedle is felt to be bending, the force being used should be reduced. If it cannot bepassed through the tissues with that much force, it should be substituted by a thickerand more stout needle. When a needle bends, it should be discarded rather thanstraightened and reused.

A. If a needle breaks in tissues despite preventive measures, it is checkedimmediately if the broken end or the tip of the needle is visible. If it is, it is grasped

by the needle holder and removed by withdrawal or advancing respectively. If theneedle is not seen at all, the location should be pinpointed and marked immediately.For this purpose, the needle holder should be kept where it is and one’s gaze shouldnot be averted from that part of the operative field. All light sources should bedirected to that spot. The area is exposed well by proper retraction by the assistants.The entry and expected exit points of the needle should be marked by application ofradiopaque markers like surgical hemostatic clips of different sizes for the twopoints. Then the wound should be probed gently taking care not to advance thebroken tip any further. If it cannot be felt because the tissue is firm, a small incisionshould be made perpendicular to the expected course of the needle. That oftenexposes the needle.

B. If the surgical probing and dissection does not reveal the needle, a sterile strongmagnet is used to locate the needle. If a magnet fails to locate the needle, a smallhand-held electromagnetic instrument which is used in ophthalmic surgery may beuseful to locate the needle.

C. Ultrasonography using a sector scanner is very useful in localizing the needle. Itsuse is not associated with radiation damage to the patient or personnel.

D. Plain radiographs (anteroposterior and lateral views) do not help in accuratelocalization of the needle. Intraoperative fluoroscopy via C-arm device is thetechnique of choice. Needles are advanced in one or more planes under fluoroscopiccontrol, until their tips lie close to the buried needle. An incision is then made downthe shaft of the needle to expose the broken needle. Alternatively a hand-held lowintensity X-ray imaging device called lixiscope may be used to locate the needlesafely.

When the retained needle is found, it is grasped with a stout hemostat andremoved by advancing in the direction of its original passage if found near heexpected exit point or withdrawn if it is found near its entry point. It cannot be justpulled out. The removed fragments must be reassembled to confirm that the entireneedle has been removed and no fragment is left in the tissues.

121

Page 128: Differential Diagnosis and Management Options inuploads.worldlibrary.org/uploads/pdf/20180107071525differential... · 83 Peripartum cardiomyopathy 165 84 Hypertension 167 85 Convulsions

NEEDLE BREAKING INSIDE TISSUES

Preventive measuresNeedle breaks

Check if the broken end or tip of the needle is visible (A)

Needle is visible Needle is not visible

Remove the needle held by the needle holder Mark entry and expected exit pointsProbe

Incise tissues perpendicular to the path of the needle

Needle is not found Needle is found

Locate the needle with a sterile strong magnet(B)

Needle is not found Needle is found

Locate the needle with ultrasonography usinga sector scanner (C)

Needle is not found Needle is found

Locate the needle with fluoroscopy via C-armdevice (D)

Needle is found

Grasp the needle with a stout hemostat and remove it.

122

Page 129: Differential Diagnosis and Management Options inuploads.worldlibrary.org/uploads/pdf/20180107071525differential... · 83 Peripartum cardiomyopathy 165 84 Hypertension 167 85 Convulsions

62. Suspicion of PregnancyA. Suspicion of pregnancy arises when a patient presents with amenorrhea,morning sickness, giddiness etc. Characteristic findings in the first trimesterinclude cyanosis of vaginal mucosa (Chadwick's sign), softening of the cervix(Goodell's sign), enlargement and softening of the corpus, softness and emptinessof isthmus dissociating corpus from cervix (Hegar's sign).

B. Immunologic tests for pregnancy are based on hCG's antigenic potential.An inhibition of coagulation of sensitized red cells or latex particles coated withhCG in a mixture of anti-hCG and serum or urine indicates presence of a pregnancy.Sensitivity of such tests varies widely (250-1400 mlU/ml). Such tests may befalse positive with proteinuria, hematuria, bacteriuria, semen, drug ingestion,or LH-peak. Beta subunit of hCG ((βhCG) is detectable in urine and serum by9 days after ovulation and fertilization. βhCG radioimmunoassay result is availablein less than 90 minutes and is very accurate (20 mlU/ml). It is not false positive.Enzyme-linked immunosorbent assay (ELISA) involves an enzyme which inducesa color change indicating the presence of hCG. It is a rapid (5 min) and sensitive(1-2.5 mlU/ml) office test. Multiple-antibody sandwich assay (immunoradiometricand immunoenzymetic assays) can detect 25-50 IU/L 8-10 days after fertilization. Ifthe test is negative, a bimanual pelvic examination may be repeated after 2 weeks. Ifthe test had been false negative and an intrauterine pregnancy grew in the weeks, itcan now be diagnosed by a bimanual pelvic examination. Alternatively one may doan ultrasonography if a missed abortion is suspected. If the uterus is empty, it issecondary amenorrhea. If a missed abortion I seen, it is managed appropriately/

C. Ultrasonography can detect a pregnancy as early as the fourth week. Transvaginalultrasonography is more sensitive than the abdominal technique. The first signis a gestational sac. Fetal heart movements are seen by 42 days after conception,always when mean sac diameter is > 25 mm. Ultrasonography is useful in differentiatinga normal pregnancy from a blighted ovum, missed abortion, ectopic pregnancy,multiple pregnancy, and hydatidiform mole.

D. Fetal heart sounds can be heard with Doppler ultrasonic auscultation at 10to 12 weeks from LMP, and with a stethoscope by 18 to 20 weeks.

E. The following tests are obsolete.1. Biologic pregnancy tests : lower accuracy, extensive preparation time, high cost

of animal laboratory.2. Radiography: teratogenic and oncogenic potential.

3. Estrogen-progestin withdrawal bleeding: risk of embryonic maldevelopment.

Differential Diagnosis of Presumptive Symptoms and Signs of Pregnancy

Feature Differential diagnosisAmenorrhea See chapter 3.Nausea, vomiting Pseudocyesis, anorexia nervosa,

gastroenteritis, hepatitis, hiatus hernia,appendicitis, intestinal obstruction, foodpoisoning.

Mastalgia, breast tingling Anovulatory cycle, estrogen therapy,fibroadenosis of breasts.

Urinary frequency Cystourethritis, cystocele, diabetesmellitus, pelvic tumor, anxiety.

Quickening Pseudocyesis, increased peristalsis, gas,free adnexal cyst.

Fatigue Overwork.

Constipation Low residue diet, low fluid intake.Weight gain Overeating.Leukorrhea See chapter 22.Chadwick's sign Vascular anomaly, tumor of cervix or uterus.

Hegar's sign, Ladin's sign Vascular uterine anomaly or tumor.Piskacek's sign Leiomyoma.Enlargement and softening ofcorpus

Cystic degeneration in a leiomyoma.

BBT elevation for > 2 weeks Infection, corpus luteum cyst, progestin orhCG therapy.

Abdominal enlargement See chapter 19.Breast changes Mastitis, premenstrual syndrome,

pseudocyesis, malignancy.Pelvic souffle Pelvic tumor, aneurysm.Chloasma Combination contraceptives, tanning.

Epulis Gingivitis.

123

Page 130: Differential Diagnosis and Management Options inuploads.worldlibrary.org/uploads/pdf/20180107071525differential... · 83 Peripartum cardiomyopathy 165 84 Hypertension 167 85 Convulsions

SUSPICION OF PREGNANCY

Assess history Speculum examination (A) Bimanual pelvic examination (A)

Signs of pregnancy Doubtful findings Normal findings

Pregnancy Secondary amenorrhea(See chapter 3)

Immunological pregnancy test on urine (B)

Positive Doubtful Negative

Pelvic ultrasonography (C) Repeat clinicalexamination after 2weeks if amenorrheapersists

Pelvicultrasonography

Pregnancy No pregnancy

Normal findings Pregnancy

Missed abortion Uterus empty

Secondary amenorrhea(See chapter 3)

124

Page 131: Differential Diagnosis and Management Options inuploads.worldlibrary.org/uploads/pdf/20180107071525differential... · 83 Peripartum cardiomyopathy 165 84 Hypertension 167 85 Convulsions

63. Last Menstrual Period(LMP) Unknown

Expected date of delivery (EDD) is calculated from LMP using Naegele'srule. When the cycle length is 28 days:

EDD = LMP + 9 months + 7 days.To this 2 days are added if the entire February is included in the duration of

the pregnancy, and 1 day is added if it is a leap year in addition. If the menstrualcycles are longer than 28 days, the extra number of days are added to EDD, and ifshorter, as many days are subtracted from the EDD. Naegele's rule cannot be appliedif the LMP is unknown, or if the menstrual cycles are irregular. Even with regularcycles, the EDD by this method is only reliable to within 2 weeks.A. If the day of ovulation is known by symptoms like ovulation pain (mittelschmerz)or midcycle spotting, EDD can be calculated by adding 266 days to it. Its accuracyis similar to that of Naegele's rule. If conception follows a single act of coitus, theEDD is calculated by adding 266 days to it. Quickening is a subjective perceptionof fetal movements (a fluttering sensation) perceived by the gravida. It is felt earlierby a parous woman (16-18 weeks) than by a nulliparous woman (18-20 weeks).B. At 12 weeks, the uterus is just palpable abdominally. Uterine size at variousgestational ages is determined by Bartholomew's rule of fourths. The uterus isjust above pubic symphysis at 12 weeks and just above umbilicus at 24 weeks.The distance between the two points is divided into 3 equal parts by 2 points,which show position of the fundus at 16 and 20 weeks. It is at xiphisternumat 38 weeks, and a little below that at 40 weeks, and still a little below at 36weeks. The distance between 24 and 36 weeks is divided into 3 equal parts by2 points indicating position of the uterine fundus at 28 and 32 weeks.C. Ultrasonography is a useful and accurate technique of determining gestationalage. Simple equations like Median BPD=GA/4. Median AC=GA-5. MedianFL=GA/5 can be used.

1. Gestational sac: 5 to 6 weeks from LMP. The diameter of the gestational sac at4, 5, 6, 7 and 8 weeks from L.M.P. is 2, 5, 10, 20 and 25 mm respectively. Gestationalage in first 2 months (in days) is equal to mean sac diameter (in mm) + 30.

2. Yolk sac: 5.6-10 weeks.3. Crown rump length (CRL): 8 weeks from LMP.

Gestational age (days) = V 8.052 CRL + 23.73.4. Fetal heart movements: 8 weeks from LMP.5. Biparietal diameter (BPD): 16 weeks.

Gestational age (weeks) = BPD2 + 2 (up to 5 completed months)BPD X 4 + 2 (after 5 completed months)

6. Head circumference (HC): 16 weeks.7. Femur length: routinely, or when BPD cannot be obtained, as with a deeplyengaged or high floating head, anencephaly, or collapse of cranial bones with fetaldeath. Accuracy of prediction of gestational age from BPD at 16, 17-26, 27-28, and29-40 weeks is + 7, + 10-11, + 14, and + 21 days respectively.

Table 63.1 Fetal Measurements at Various Gestational AgesGestational age (weeks) BPD HC AC FL

(mm) (mm) (mm) (mm)

14 2.8 9.6 8.1 1.6

15 3.2 11.0 9.3 1.9

16 3.6 12.4 10.5 2.2

17 3.9 13.7 11.6 2.5

18 4.2 15.0 12.8 2.8

19 4.5 16.2 14.0 3.2

20 4.8 17.4 15.1 3.5

21 5.1 18.6 16.2 3.8

22 5.4 19.8 17.3 4.0

23 5.8 20.9 18.4 4.2

24 6.1 22.0 19.5 4.4

25 6.4 23.0 20.6 4.6

26 6.7 24.0 21.6 4.8

27 7.0 25.0 22.7 5.0

28 7.2 25.9 23.7 5.3

29 7.5 26.8 24.8 5.5

30 7.8 27.7 25.8 5.7

31 8.0 28.6 26.8 6.0

32 8.2 29.4 27.8 6.2

33 8.5 30.1 28.7 6.4

34 8.7 30.9 29.7 6.6

35 8.8 31.5 30.7 6.8

36 9.0 32.2 31.6 7.1

37 9.2 32.8 32.5 7.3

38 9.3 33.4 33.4 7.5

39 9.4 34.0 34.4 7.7

40 9.5 34.5 35.2 8.0

(BPD: biparietal diameter; HC: head circumference; AC: abdominal circumference; FL:femur length)D. Fetal ossification centers can be seen with radiography. But that test is not done inmodern obstetrics owing to radiation hazards to the fetus.E. Amniotic fluid analysis for fetal maturity (see chapter 101) tells only aboutmaturity of various parts of the fetus (e.g., lungs, kidney, skin) and fitness fordelivery, rather than EDD.

125

Page 132: Differential Diagnosis and Management Options inuploads.worldlibrary.org/uploads/pdf/20180107071525differential... · 83 Peripartum cardiomyopathy 165 84 Hypertension 167 85 Convulsions

LMP UNKNOWN

Check history (A)

Ovulation spotting Mittelschmerz Quickening None

Add 266 days. Check parity Obstetric examination (B)

0 > 1 Apply Bartholomew’srule of fourths

Ultrasonography (C) Radiography (D) Amniotic fluidanalysis (E)

Add 22-24 weeks Add 20-22 weeks Fetal ossificationcenters

126

Page 133: Differential Diagnosis and Management Options inuploads.worldlibrary.org/uploads/pdf/20180107071525differential... · 83 Peripartum cardiomyopathy 165 84 Hypertension 167 85 Convulsions

64. Uterine Size Less ThanExpected

The commonest cause of the uterine size below expected value is wrongdates. However the diagnosis can be made by exclusion alone.

A. Presence of fetal movements suggests the presence of a live fetus. The diagnosiscan be confirmed by auscultation of fetal heart sounds with a fetal stethoscopeor Doppler ultrasound. If the fetus is dead, uterine enlargement ceases. The amnioticfluid gets resorbed. The fetus softens, loses its tone, and its contour becomesrounded. As a result the uterus feels smaller than expected. Fetal death is diagnosedby ultrasonography (USG). It is discussed in chapter 104.

B. Ultrasonographic fetal biometry is a useful tool for assessment of fetal gestationalage and fetal growth. However a single study is not adequate, because low valuesmay indicate either wrong dates or symmetrical intrauterine growth restriction(IUGR). Hence the test is repeated after 2 weeks. If all the parameters showgrowth by 2 weeks in 2 weeks, the diagnosis is wrong dates. However if thegrowth is less than 2 weeks in 2 weeks, the diagnosis is symmetrical IUGR. It is dueto decreased growth potential, as with any of the following conditions.

1. Genetic disorder of the fetusa. Autosomal: trisomy 13, trisomy 18, trisomy 21, partial trisomies, ring

chromosomes, chromosomal deletions.b. Sex chromosomal: 45, X; XXX, XYY.c. Neural tube defects.d. Miscellaneous: osteogenesis imperfecta, achondroplasia, chondrodystrophy.

2. Fetal infectionsa. Viral: rubella, cytomegalovirus, herpes, varicella zoster.b. Bacterial: listeriosis.c. Protozoal: toxoplasmosis, malaria.

3. Drugs: alcohol, anticonvulsants, tobacco, aminopterin, methotrexate, warfarin.4. Radiation.5. Idiopathic

The diagnosis is made by asking history of exposure to radiation or drugswhich are known to cause IUGR. If the history is not contributory, serological testsare done on the maternal serum for the infections. Presence of IgM antibodies, orrising titer of IgG antibodies suggests the presence of a recent infection responsible forthe IUGR. If an infection is not found, an amniocentesis is done for chromosomalstudies on the amniotic fluid cells. If all tests are negative, it is idiopathic IUGR.

C. If fetal heart sounds are not heard after 24 weeks, it could be dead. The

diagnosis is confirmed by ultrasonography (see chapter 104). If the fetus is alive,amniotic fluid volume is assessed. The fundal height is reduced in oligohydramnios(see chapter 100). It is associated with IUGR too. The diagnosis of IUGR is confirmedby serial ultrasonography for fetal biometry. If all parameters fail to grow intwo weeks by two weeks, it is symmetrical IUGR. If the biparietal diameter and headcircumference are spared while the other parameters are affected, it is asymmetricalIUGR. It is due to restricted growth potential as in the following conditions.

1. Placental disorders: infarction, chronic villitis, chronic abruption, placentaprevia, circumvallate placenta, battledore placenta, placental hemangioma.

2. Maternal disorders: preeclampsia, chronic hypertension, chronic renal disease,malnutrition, cyanotic heart or respiratory disease, hemoglobinopathies. Thecause is found out by a detailed general, medical and obstetric examination,and obstetric ultrasonography.

D. Besides treatment of the cause of IUGR, obstetric management is also important.Fetal well being is closely monitored (see chapter 144). If the fetus is compromised,labor is induced (see chapter 105). Fetal well being is monitored during labor(see chapter 144). Vaginal delivery is achieved if the fetus remains well duringlabor. If it develops distress during labor, an immediate delivery is achieved asappropriate (see chapter 150). Some workers prefer to perform a cesarean sectionrather than induce labor if the nonstress test is nonreactive and a contractionstress test is positive. If Doppler studies show absent diastolic flow or reversal ofdiastolic flow in fetal umbilical vessels, a cesarean section is preferred to aninduction of labor if the fetus is mature enough to survive.

E. An apparently low fundal height may be the result of a transverse lie. Thetransverse width of the uterus is more in such cases. It should also be rememberedthat a small woman tends to have a small baby.

Table 63.1 Uterine Height at Various Gestational Ages

Gestational age (weeks) Fundal height (cm)

12 Just above pubic symphysis

15 Midway between umbilicus and symphysis

20 Just below umbilicus

28 6 cm above umbilicus.

32 6 cm below xiphoid

36 2 cm below xiphoid

40 4 cm below xiphoid

127

Page 134: Differential Diagnosis and Management Options inuploads.worldlibrary.org/uploads/pdf/20180107071525differential... · 83 Peripartum cardiomyopathy 165 84 Hypertension 167 85 Convulsions

UTERINE SIZE LESS THAN EXTPECTED

Check gestational age

< 24 weeks (A) > 24 weeks (C)

Assess fetal movements Assess fetal heart sounds

Present Absent Present Absent

USG for fetal biometry (B) USG Assess amniotic fluid volume (C) Fetal death

Repeat USG after 2 weeks Live fetus Fetal death Normal Reduced

Fetal growth by 2 weeks Fetal growth by < 2 weeks Two USGs 2 weeks apart (D) ? IUGR Oligohydramnios

Wrong dates Symmetrical IUGR Fetal growth by 2 weeks Fetal growth by < 2 weeks USG to confirm diagnosis

Assess exposure to drugs Wrong dates IUGR See chapter 100

Present Absent Symmetrical Asymmetrical

Assess exposure to radiation General examination

Present Absent Hypertension Anemia Malnutrition Normal

Serological tests for infections Systemic examination

Positive Negative Cyanotic heart disease

Cyanotic respiratory disease Renal disease Normal

Look for neural tube defects USG

Present Absent Placentaprevia

Chronicabruption of

placenta

Placentalhemangioma

Normal

Amniocentesis for chromosomal studies Idiopathic

Positive Negative Idiopathic

128

Page 135: Differential Diagnosis and Management Options inuploads.worldlibrary.org/uploads/pdf/20180107071525differential... · 83 Peripartum cardiomyopathy 165 84 Hypertension 167 85 Convulsions

65. Uterine Size More ThanExpected

The commonest cause of uterine size more than expected is wrong dates.However the diagnosis can be made only by exclusion of the other causes.

A. Multiple pregnancy is a fairly common cause of undue enlargement of theuterus. Though the individual fetuses are usually growth restricted, their cumulativesize is more than that of a fetus in a singleton pregnancy. It is essential to beable to palpate four fetal poles so as to make a diagnosis of twins. If only threepoles can be palpated, it could be a conjoined twin, or a singleton pregnancywith a uterine leiomyoma (the third pole). If more than four fetal poles can befelt, it is triplets or more.

B. A vesicular mole is a gestational trophoblastic tumor. It is a bulky tumorbecause of hydropic degeneration of the villi, and proliferation of the trophoblast.The uterine size can be increased further by hemorrhage within it. The uterusis tense and elastic (not doughy as often described). It is bigger than expectedfor the period of amenorrhea in 50% cases, of corresponding size in 30% cases,and smaller than expected in 20% cases. It is small when the mole is dead (asin a missed abortion).

C. The amniotic fluid volume is more than twice the expected volume at thatgestational age in hydramnios. The uterus is large, cystic, and has a fluid thrill(as in an ascites). The fetus is ballottable (when it should not be i.e. beyond 28weeks), and its parts cannot be felt easily. It is often in malpresentation owingto its increased freedom of movement.

D. Trendelenburg position results in upward shift of structures free to do so(e.g. bowel, omentum, and a mobile ovarian tumor). A moderate to large sizedcystic ovarian tumor may get included in the uterine mass during clinical examinationbeyond 24 weeks. Trendelenburg position causes it to move cranially while theuterus cannot do so. As a result a transverse groove is seen in the fundal contour,between the tumor and fundus of the uterus.

E. Uterine leiomyomas often cause irregular enlargement of the uterus. Howeverthe leiomyomas enlarge during pregnancy, become softer, and tend to merge withthe uterine contour. If there is a single, large, intramural leiomyoma, the diagnosiscould be difficult in pregnancy. The diagnosis in either type is difficult whenthe patient presents for the first time during pregnancy. Ultrasonography is diagnosticin such cases.

F. A fetus with a normal rate of growth shows a growth of 2 weeks in 2 weekswhen studied by serial ultrasonographic biometry (see chapter 63). In such acase, the patient's LMP was wrongly stated by her. If the fetus grows by morethan 2 weeks in 2 weeks, it has macrosomia.

129

Page 136: Differential Diagnosis and Management Options inuploads.worldlibrary.org/uploads/pdf/20180107071525differential... · 83 Peripartum cardiomyopathy 165 84 Hypertension 167 85 Convulsions

UTERINE SIZE MORE THAN EXPECTED

Obstetric examination (A)

Four fetal poles > four fetal poles No fetus palpable, no fetal heart sounds, no ballottement (B) Two fetal poles

Twins Triplets or more Vesicular mole Singleton pregnancy

Ultrasonography to confirm diagnosis Assess volume of amniotic fluid

Normal Excessive (C)

Trendelenburg position (D) Hydramnios

See chapter 99Separate contour of ovarian cyst above uterine contour No change in uterine contour

Ovarian cyst with pregnancy Ultrasonography

Ultrasonography to confirm diagnosis Uterine leiomyoma(s)with pregnancy (E)

Single fetus

Repeat ultrasonography after 2 weeks for fetal biometry (F)

Fetal enlargement by 2 weeks Fetal enlargement by > 2 weeks

Wrong dates Fetal macrosomia

See chapter 115

130

Page 137: Differential Diagnosis and Management Options inuploads.worldlibrary.org/uploads/pdf/20180107071525differential... · 83 Peripartum cardiomyopathy 165 84 Hypertension 167 85 Convulsions

66. Failure to Gain WeightA gravida gains about 11 kg during pregnancy, of which 1 kg is gained in

the first trimester, and 5 kg each in the second and third trimesters. She should gainat least 225 g/wk. If she does not gain weight, it means either the fetus is notgrowing, or there is a combination of growth of the fetus and loss of maternalweight.

A. The pointer of a bathroom scale needs to be adjusted at zero prior to weighinganyone. If it has shifted, the weight obtained is wrong. It is the commonest causeof wide fluctuations in the weight reading of gravidas in the antenatal clinic. Use ofan electronic weighing machine will avoid this error.

B. Maternal weight gain is an indirect measure of fetal growth. Serial measurementof fundal height and abdominal girth at the level of the umbilicus (after 24 weeks)are much more accurate.

C. If the fetus is found to be growing, the gravida must have lost weight. Thecommonest cause of such loss of weight is loss of edema fluid (a physiologicalchange). If the gravida did not have edema at the time of the previous visit,the cause of weight loss is loss of body mass. This is quite serious. One shouldlook for conditions like malnutrition, excessive vomiting, malabsorption, or abuseof alcohol, tobacco, or drugs of addiction. An interested reader may refer to a textbook of internal medicine for details of these conditions.

D. The gravida does not gain weight if the fetus has growth restriction, oris dead. In the latter case, the physiological changes of pregnancy reverse, andhence hormone-induced weight gain by the gravida also ceases. See chapter 64

and chapter 104 for details of these conditions.

E. Underweight women are those who weigh under 10% less than their ideal bodyweights. If they do not gain sufficient weight during pregnancy, they are at risk ofdevelopment of pregnancy complications like placental abruption, preterm labor,amnionitis, and intrauterine growth retardation.

Table 65.1 Weight Gain During Pregnancy

Weight gain (g)

1st Trimester 2nd Trimester 3rd Trimester

Breastbloodglands

UterusFetusPlacentaAmniotic fluidWater

545

2005

20 30

20180700300170250

1500*

45405

10503300650800

3750**700***

Note : * At 20 weeks** At 30 weeks*** At 40 weeks

131

Page 138: Differential Diagnosis and Management Options inuploads.worldlibrary.org/uploads/pdf/20180107071525differential... · 83 Peripartum cardiomyopathy 165 84 Hypertension 167 85 Convulsions

FAILURE TO GAIN WEIGHT

Check zero adjustment of the weighing machine (A)

Accurate Inaccurate

Check gain in fundal height and abdominal girth (B) Readjust zero and weigh again

Adequate gain No gain Weight gain

Check previous record of edema (C) Auscultate for fetal heart sounds (D) Follow up

Present Absent Present Absent

Loss ofedema fluid

Loss of maternal weight ? Fetal growth restriction Fetal death(See chapter 104)

Investigate cause Ultrasonographic fetal biometry twice 2 weeks apart

Failure to grow by 2 weeks Growth by 2 weeks

Malnutrition Malabsorption Alcohol Tobacco Assess amniotic fluid volume

Drug abuse Adequate Less

Normal Oligohydramnios(See chapter 100)

132

Page 139: Differential Diagnosis and Management Options inuploads.worldlibrary.org/uploads/pdf/20180107071525differential... · 83 Peripartum cardiomyopathy 165 84 Hypertension 167 85 Convulsions

67. Exposure to TeratogenA. A teratogen is an agent which causes congenital malformation(s) in the fetuswhen the latter is exposed to it in the period of organogenesis. Since organogenesisis complete by the end of the first trimester, any such exposure in the secondor third trimester is safe. Such gravidas are given reassurance and the pregnanciesare continued.B. Some agents cause fetal abnormalities in all the cases, while others are lessconsistently teratogenic. The effect is also influenced by the relative rate of absorption,maternal metabolism, placental transfer, and fetal metabolism of the agent. Thetime during embryogenesis when the cells of a given organ system divide mostrapidly is the period of greatest teratogenic susceptibility. These periods are differentfor different major organ systems (table 66.1).

Table 66.1 Development of Fetal Organ SystemsEmbryonic age (weeks) Embryonic development

1 Implantation, resistance to teratogenesis.

2-3 Cranium, face, CNS, musculoskeletal system.

4 Limb buds, cardiovascular system.

5 Limb bud segments, nose, eyes, ears.

6 Fingers, toes.

7-8 Maxilla, eyelids.

9-10 External and internal genitalia, relative resistance toteratogenesis.

Potential malformations depend on the developmental age at the time of exposure toa teratogen as follows.

1. Three weeks : ectromelia, sympodia, ectopia cordis, omphalocele.2. Four weeks: hemivertebra, ectromelia, omphalocele, tracheo-esophageal fistula.3. Five weeks : nuclear cataract, carpal or pedal ablation, facial clefts,

microphthalmia, tracheo-esophageal fistula, hemivertebra.4. Six weeks : lenticular cataract, microphthalmia, cleft lip, agnathia, carpal or

pedal ablation, aortic anomalies, cardiac septal defects.5. Seven weeks : cleft palate, micrognathia, brachycephaly, epicanthus, digital

ablation, ventricular septal defect, pulmonary stenosis.6. Eight weeks : nasal bone ablation, epicanthus, brachycephaly, digital stunting,

ostium primum defect.C. US FDA has established five categories of drugs with regard to possible adversefetal effects.Category Features

A no fetal risk.B no fetal risk in animal studies, human studies not available.C no adequate studies in animals or humans, or adverse effects in

animals, but no human data.D evidence of fetal risk, but benefits are thought to outweigh the risks.X proven fetal risks which outweigh any benefits.

It is necessary to check the category the drug belongs to, before prescribing the drug

during pregnancy. Fetotoxic effects of some commonly used drugs are as follows.1. Aminoglycosides: Vestibulocochlear nerve damage, ocular damage.2. Amphetamine: withdrawal, transposition of great vessels.3. Androgens: masculinization of female fetus.4. Anticonvulsants: cleft palate, congenital heart disease.5. Antineoplastic drugs: potent teratogenic action.6. Antithyroid drugs: fetal goiter, tracheal obstruction, aortic atresia, hypospadias,

growth retardation.7. Benzothiazides: hyperglycemia, hyperbilirubinemia, thrombocytopenia,

hyponatremia, hypokalemia.8. Chloroquine : retinal damage, vestibulocochlear nerve damage.9. Cocaine : CNS anomalies, sudden infant death syndrome.

10. Diethylstilbestrol: cervical/vaginal adenosis/adenoma/adenocarcinoma, cervicalcockscomb, T-shaped uterine cavity, hypoplastic uterus.

11. Ethanol : mental retardation, facial dysmorphism, microcephaly, congenitalheart disease, renal anomalies (fetal alcohol syndrome).

12. Isoniazid: encephalopathy.13. Lithium: congenital heart disease.14. Narcotics: withdrawal syndrome.15. Phenytoin: facial anomalies, mental retardation, microcephaly, nail hypoplasia,

growth retardation (fetal hydantoin syndrome).16. Sulfonamides: neonatal jaundice.17. Tetracyclines: stained teeth.18. Trimethadione: limb reduction defects.19. Warfarin: nasal hypoplasia, stippling of epiphyses, mental retardation, growth

retardation, bleeding (Conradi-Hunnermann syndrome).Viruses can affect the fetus adversely. Fetal effects of various maternal virus infectionsare as follows.1. AIDS: box-like forehead, microcephaly, flattened bridge of nose, hypertelorism,

patulous lips, triangular philtrum, growth restriction.2. Cytomegalovirus: microcephaly, hydrocephaly, cerebral calcification,

chorioretinitis, deafness, cerebral palsy, mental retardation,hepatosplenomegaly.

3. Herpes genitalis: hydranencephaly, chorioretinitis, encephalitis,thrombocytopenia, hemolytic anemia.

4. Mumps: varied malformations, endocardial fibroelastosis.5. Parvovirus: eye defects, other anomalies, degenerative changes.6. Rubella: microcephaly, cardiac/great-vessel malformations, microphthalmos,

cataract, glaucoma, retinopathy, mental retardation, hepatosplenomegaly,sensorineural deafness.

7. Varicella-zoster: hydrocephalus, microcephaly, microphthalmos, cataract, opticatrophy, Horner’s syndrome, urogenital anomalies, skeletal hypoplasia, mentalretardation.

D. MTP is permitted for exposure to a teratogen. In case the woman desires tocontinue the pregnancy, fetal anomalies may be detected using ultrasonography,MRI, and amniotic fluid analysis. The pregnancy is allowed to continue if noanomaly is detected, though the woman must be explained that all anomalies maynot be detectable by these means.

133

Page 140: Differential Diagnosis and Management Options inuploads.worldlibrary.org/uploads/pdf/20180107071525differential... · 83 Peripartum cardiomyopathy 165 84 Hypertension 167 85 Convulsions

EXPOSURE TO A TERATOGEN

Check time of exposure (A)

First trimester Second trimester Third trimester

Check teratogenic effect (B)Check risk (C)

Reassurance, continue pregnancy. Reassurance, continue pregnancy.

High Low

MTP Counseling

Option for MTP Option for continuing pregnancy (D)

MTP First trimester Second trimester Third trimester

Chorion villus biopsy, vaginosonography Ultrasonography, MRI, amniocentesis Ultrasonography

Anomaly detected Anomaly not detected Anomaly detected Anomaly not detected

Ultrasonography in second trimester MTP

Anomaly detected Anomaly not detected

Continue pregnancy, delivery at term, neonatal management

134

Page 141: Differential Diagnosis and Management Options inuploads.worldlibrary.org/uploads/pdf/20180107071525differential... · 83 Peripartum cardiomyopathy 165 84 Hypertension 167 85 Convulsions

68. Request for PrenatalDiagnosis

A number of techniques have now been developed for prenatal diagnosis ofcertain genetic disorders. It has altered the outlook for families at risk of havingaffected children. The following criteria should be satisfied before undertakingprenatal diagnosis.

1. The disorder should be sufficiently severe to warrant a termination of thepregnancy.

2. There should not be any or satisfactory treatment for the condition.3. Accurate prenatal diagnostic test should be available.4. Genetic risk to the pregnancy should be sufficiently high.5. Termination of pregnancy should be acceptable to the couple.

A. Maternal serum alpha fetoprotein (AFP) level is raised with an open neuraltube defect. However AFP can be raised with other conditions like intrauterinefetal death, omphalocele, gastroschisis, esophageal and bowel atresia, sacrococcygealteratoma, congenital nephrosis, ectopia vesicae, focal dermal hypoplasia, andMeckel's syndrome. An ultrasonographic scan is obtained to differentiate betweenthese conditions. If there is a suspicion of a neural tube defect on ultrasonography,an amniocentesis is done. Features of an open neural tube defect on amnioticfluid analysis include elevation of AFP, glucose, acetyl cholinesterase, and presenceof fetal neuroglial cells. Fetoscopy is rarely done in modern obstetrics. With that,direct visualization of fetal malformations is possible. Conventional fetoscopyis done in the second trimester, using a thin telescope under local anesthesiaand maternal sedation. Fetoscopy can be done less invasively in the first trimesterusing a contact hysteroscope passed through the cervix.

B. Fetal chromosomal errors and inborn errors of metabolism can be diagnosedbetween 8 and 11 weeks of gestation by chorion villus sampling (CVS). Chorionictissue is obtained under ultrasonographic control using a plastic cannula. A samplemay be obtained through an operative contact hysteroscope too. Maternal tissueis removed by observing under a microscope. The villi can be used for the followingtests.

1. DNA studies.2. Chromosome preparations.3. Enzyme studies.

Principal chromosomal indications for CVS are shown in table 68.1 and principalinborn errors of metabolism that can be diagnosed by CVS are shown in table 68.2.

Table 68.1 Chromosomal Indications for CVSAdvanced maternal ageOne parent carrier of a balanced translocationOne parent mosaic for chromosomal abnormalityA previous child with autosomal trisomyFragile X syndromeChromosomal instability syndrome

Table 68.2 Inborn Errors of Metabolism Diagnosed by CVSAcid phosphatase deficiency Hyperammonaemia

Adenosine deaminase deficiency Hypercholesterolemia

Adrenogenital syndrome Hypophosphatasia

Adrenoleukodystrophy Krabbe's disease

Arginosuccinic aciduria Lesch Nyhan syndrome

Citrullinaemia Mannosidosis

Cystic fibrosis Maple syrup urine disease

Cystinosis Methylmalonic aciduria

Fabry's disease Mucolipidosis II

Farber's disease Mucopolysaccharidoses I-VI

Fucosidosis Niemann-Pick disease

Galactosemia Phenylketonuria

Gangliosidosis (generalized) Porphyria

Gaucher's disease Propionic acidemia

G6PD deficiency Refsum's disease

Glutaric aciduria Sandhoffs disease

Glycogenosis types I to IV Tay-Sachs disease

Hb-S disease Thalassemia (beta)

Hemophilia A, B Wolman's disease

Homocystinuria Xeroderma pigmentosum

If CVS fails, or if the woman presents between 11 and 16 weeks ofpregnancy, one should perform an amniocentesis at 16 weeks under local anesthesia,using No. 18 lumbar puncture needle under ultrasonographic control to avoid traumato the fetus or placenta. Conditions shown in tables 68.1 and 68.2 can be diagnosedby amniocentesis too.

C. The approach to fetal hematologic disorders is similar to that described under B,except that fetal blood is obtained by cordocentesis after 16 weeks. Between 8 and11 weeks, techniques like DNA analysis, linkage analysis and RFLPs are used forthe diagnosis of fetal hemoglobinopathies. Various conditions which can bediagnosed in this way are shown in table 68.3.

D. Structural malformations of the fetus are diagnosed by ultrasonography (anomalyscan), fetoscopy (see under A), and MRI.

Table 68.3 Conditions diagnosed by cordocentesis

Thalassemia and other hemoglobinopathies Chronic granulomatous diseaseHemophilia A and B Immune deficiency disordersCongenital rubella or cytomegalovirus infections Fragile X syndromeChromosomal instability syndrome

135

Page 142: Differential Diagnosis and Management Options inuploads.worldlibrary.org/uploads/pdf/20180107071525differential... · 83 Peripartum cardiomyopathy 165 84 Hypertension 167 85 Convulsions

REQUEST FOR PRENATAL DIAGNOSIS

Check indication

Neural tube defect (A) Chromosomaldisorder (B)

Inborn error ofmetabolism (B)

Hematologic disorder (C) Structuralmalformations (D)

Maternal serum alpha fetoprotein assay Check gestational age Check gestational age Ultrasonography

Raised Normal 8-11 weeks 11-16 weeks

Wait up to 16weeks

> 16 weeks 8-11 weeks 11-16 weeks

Wait up to 16weeks

> 16 weeks

Ultrasonography Chorion villus biopsy Amniocentesis Chorion villus biopsy Amniocentesis

Neuraltube defect

Suspicious Normal Successful Failure Successful Failure

Amniocentesis, fetoscopy

Neural tubedefect

Normal

136

Page 143: Differential Diagnosis and Management Options inuploads.worldlibrary.org/uploads/pdf/20180107071525differential... · 83 Peripartum cardiomyopathy 165 84 Hypertension 167 85 Convulsions

69. First Trimester VaginalBleeding

A. It is necessary to rule out the diagnosis of a delayed period when a womanpresents with amenorrhea and vaginal bleeding. If the uterus is of normal sizeand firm, she needs cyclical progestin therapy for regularization of her menstrualcycles (if she does not desire a pregnancy), and clomiphene citrate (if she desiresa pregnancy).

B. The diagnosis of inevitable abortion is made when the cervix is open, andthe products are felt in the vagina, cervix, or uterine cavity. It does not needany investigations for confirmation. If the woman has bled excessively, she needsresuscitation while arrangements are made for a curettage. The bleeding canbe reduced by any of the following means, provided there are no contraindicationsfor their use.

1. Infusion of 10 units of oxytocin in 500 ml of D5W.2. 15-S-15-CH3-PGF2 alpha, 250 microgram intramuscularly.3. Misoprostol 400-800 μg rectally.

The products are evacuated with suction (if the cervix is not excessivelydilated), ovum forceps, and/or blunt curette. If the bleeding is not controlled afterevacuation, she may be given any of the 3 drugs mentioned above, or methylergometrine 0.1 mg intravenously, in addition to bimanual uterine massage.

C. A threatened abortion is associated with uterine bleeding, when there isan intrauterine pregnancy and the cervix is closed. Conservative treatment isindicated only if the fetus is alive. Real time ultrasonography reveals fetal cardiacactivity if the fetus is alive. Such women are given bed rest, though its value has notbeen proved. Serum progesterone assay may be done to detect corpus luteuminsufficiency (see chapter 12). Some workers empirically treat all women sufferingfrom threatened abortion with progestins or hCG. If a retroplacental clot is seen, thewoman needs periodic ultrasonic scans to see if the clot gets resorbed. Withsubchorionic hematoma, abortion rate is 10%. It is 20% when hematoma is 30% of

sac volume. If fetal heart activity is absent, a diagnosis of fetal death due to placentalseparation is made. Such women are treated by rapid dilatation of cervix, suctionevacuation, and blunt curettage.

D. An empty gestational sac below 8 weeks of gestation may not be due toa blighted ovum. However if fetal echoes and fetal cardiac activity are not seenon a repeat scan after 1 week, the woman is treated by rapid cervical dilatationand curettage. If the gestational sac volume is more than 2.5 ml and it is empty,a diagnosis of a blighted ovum (a form of a missed abortion) is made.Diameter of the gestational sac increases by 1.2 mm/d. No growth orgrowth < 0.7 mm/d is of poor prognosis. Failure of a 25 mm sac to increaseby 95% in one week suggests diagnosis of a blighted ovum. Sac > 20 mm withoutyolk sac, > 25 mm without a fetus suggests presence of a blighted ovum. Yolksac > 5.6 mm in diameter is associated with fetal demise. It is treated by rapiddilatation and evacuation of uterine contents.

E. The uterus may be bigger than expected for the period of amenorrhea in a case ofthreatened abortion due to the presence of blood clots (extramembranous orretroplacental). In the presence of such large clots, it is less likely that thefetus would survive. A real time ultrasonography confirms the diagnosis. A vesicularmole is another condition with identical clinical findings. It is diagnosed byserum beta-hCG assay and ultrasonography.

F. The uterine size is smaller than expected if the fetus is dead (missed abortion) orthe uterus is empty. The latter may be due to an extrauterine pregnancy (see chapter71) or a delayed period. With the advent of ultrasonography, the diagnosis of amissed abortion can be made soon after fetal death, and it is no longer necessary towait for a conventional period of 8 weeks before making the diagnosis. Thetreatment is rapid cervical dilatation and curettage, after correction of disseminatedintravascular coagulation (if present).

G. A local condition of the lower genital tract can cause vaginal bleeding at anygestational age, just as in the nonpregnant state. The diagnosis of local trauma can bemade from history as well as local examination. Cervical cancer or cervical erosioncan be diagnosed by a speculum examination.

137

Page 144: Differential Diagnosis and Management Options inuploads.worldlibrary.org/uploads/pdf/20180107071525differential... · 83 Peripartum cardiomyopathy 165 84 Hypertension 167 85 Convulsions

FIRST TRIMESTER VAGINAL BLEEDING

Bimanual pelvic examination, speculum examination (A)

Uterus normal sized,firm

Cervix open, productsfelt (B)

Uterine size corresponding to the period ofamenorrhea, cervix closed (C)

Uterus bigger thanexpected, cervix

closed (E)

Uterus smaller thanexpected, firm (F)

Uterine size asexpected, cervix closed,local lesions on cervix

or vagina (G)Delayed period Inevitable

abortionThreatened abortion βhCG, ultrasonography Ultrasonography

Pelvic ultrasonography Threatened abortion

Vesicularmole

(See chapter 64)

Curettage Disorganized gestational sac,no fetal cardiac activity

Empty uterus

Fetal heart activity seen Fetal heart activity absent Early gestational sac (D) Missed abortion

Conservative treatment Dilatation and curettage Ectopicpregnancy

(See chapter 71)

Delayedperiod

Gestation < 8 weeks Gestation > 8 weeks, Sac volume > 2.5 ml

Repeat ultrasonography after 1 week Blighted ovum Cervical cancer Cervical erosion

Fetal echoes, cardiac activity No fetal echoes, cardiac activityLocal trauma

Threatened abortion Missed abortion Coagulation studies

Normal DIC(See chapter 155)

Dilatation and curettage

138

Page 145: Differential Diagnosis and Management Options inuploads.worldlibrary.org/uploads/pdf/20180107071525differential... · 83 Peripartum cardiomyopathy 165 84 Hypertension 167 85 Convulsions

70. Suspicion of SepticAbortion

Fever, foul vaginal discharge with bleeding in the first trimester indicate thepresence of a septic abortion.

A. A septic abortion may be due to any of the following.1. Criminal abortion: an abortion induced by quacks without aseptic and antiseptic

precautions, using methods like insertion of abortion stick or catheter(s) into theuterine cavity, injection of sclerosant(s) or soap solution into the uterine cavityalmost invariably results into a septic abortion.

2. Dilatation and curettage: an incomplete procedure with infection.3. Pregnancy with IUCD.4. Spontaneous abortion: ascending infection from the lower genital tract.

B. The patient has lower abdominal and pelvic pain, fever with/without chills,foul discharge and bleeding per vaginum. Bimanual pelvic examination showsfeatures of acute pelvic inflammatory disease - vaginal warmth, tenderness ontransverse cervical movements, tenderness in lateral fornices, and sometimestender tubo-ovarian masses. There may be features of peritonitis.

C. Microbiological studies are done on high vaginal swab, cervical swab, andblood. Finding intracellular Gram negative diplococci of gonorrhea helps avoidsurgery. Both aerobic and anaerobic cultures are obtained, because anaerobesand mixed flora are quite common. A hemogram and coagulation profile areobtained. Ultrasonography of the abdomen and pelvis is done to detect retainedproducts in the uterine cavity, tubo-ovarian mass(es), pelvic abscess, foreign body(abortion stick), free fluid in pelvis or abdomen (peritonitis) etc. A radiographof the abdomen (including both domes of the diaphragm) is obtained in standingposition to detect free gas under the diaphragm (due to perforation of the bowelor infection with gas producing organisms), fluid levels (intestinal obstruction),and intraabdominal foreign body.

D. The patient is considered to have mild disease in the absence of complications.Antibiotics are started only after samples are obtained for microbiological studies.A combination of an antibiotic against Gram-positive organisms, one against Gram-negative organisms, and one against anaerobes is started initially. Uterine contentsare removed by dilatation and curettage after 6 hours, when adequate tissue levels ofthe antibiotics are reached. Extreme care must be exercised to avoid perforating the

uterus, which is very soft, so as to avoid spread of the infection to the peritonealcavity. The antibiotic is changed to one indicated by antibiotic sensitivity tests, if thepatient fails to respond to treatment in 48 hours.

E. Patients with complications require aggressive management. Higher antibioticsare administered from the beginning. Low dose hydrocortisone (50 mg IV q6h, or100 mg IV bolus followed by an infusion of 10 mg/h for seven days) is required inseptic shock not responding to antibiotics, IV fluids and vasopressors. It is taperedand stopped when the patient becomes hemodynamically stable. Surgicalintervention is required for the following conditions.

1. Dilatation and curettage for all cases, to evacuate retained products ofconception.

2. Total abdominal hysterectomya. Uterine perforation with peritonitis.b. Pelvic sepsis not responding to curettage and higher antibiotics.c. Large tubo-ovarian masses not responding to treatment.d. Expanding tubo-ovarian masses.e. Rupture of a tubo-ovarian mass.f. Gas gangrene: confined to the uterus.

3. Incision and drainage of a pelvic abscessa. Posterior colpotomy: for an abscess in the pouch of Douglas.b. McBurney's grid iron incision: for an abscess deep in pelvis or pointing

above the inguinal ligament.4. Resection and end-to-end anastomosis of injured small bowel, colostomy

proximal to the site of injury to large bowel.5. Drainage of peritoneal cavity after an exploration in a patient with peritonitis.

Maternal mortality in various stages of septic abortion is shown in table 70.1

Table 70.1 Maternal Mortality

Condition Maternal Mortality

Sepsis without shockSepsis plus shockShock after completesepsis syndrome

13%28%43%

139

Page 146: Differential Diagnosis and Management Options inuploads.worldlibrary.org/uploads/pdf/20180107071525differential... · 83 Peripartum cardiomyopathy 165 84 Hypertension 167 85 Convulsions

SUSPICION OF SEPTIC ABORTION

History (A)Clinical examination (B)Investigations (C)Determine severity of the disease

Mild (D) Severe (E)

Intravenous antibiotics Septicshock

DIC ARDS Uterineperforation

Bowelinjury

Peritonitis Renalfailure

Good response No responseProgression of disease

See chapter155

Medicaltreatment

Resection andanastomosis

Medicaltreatment

Dilatation andcurettage

Change antibioticsbased on antibiotic

sensitivity

IV fluidsHigh antibioticsSteroids

Exploratory laparotomyRepair or hysterectomy

Exploratory laparotomy,hysterectomy, drainage.

Good response No responseProgression of disease

Conservative treatmentDilatation and curettage

Dilatation and curettage

No response Good response

Total abdominal hysterectomy Observation

140

Page 147: Differential Diagnosis and Management Options inuploads.worldlibrary.org/uploads/pdf/20180107071525differential... · 83 Peripartum cardiomyopathy 165 84 Hypertension 167 85 Convulsions

71. Suspicion of TubalEctopic Pregnancy

A fertilized ovum implanted outside the uterine cavity is ectopic pregnancy.The fallopian tube is the commonest site ( > 95%).

A. Risk factors for an ectopic pregnancy are as follows.1. Pelvic infection: endosalpingitis induced tubal narrowing, peritubal adhesions.2. Developmental abnormalities of the tube: diverticula, accessory ostia, and

hypoplasia.3. Previous operations on the tube: tuboplasty and tubal ligation.4. Previous ectopic pregnancy.5. Previous MTPs.6. Tubal distortion by tumors: uterine leiomyomas, ovarian tumors.7. Progesterone-only contraception.9. Cigarette smoking.

10. Assisted reproduction: ovulation induction, IVF-ET, GIFT.

B. Clinical features of an ectopic pregnancy are as follows.1. Amenorrhea: it is usually present. The ectopic pregnancy sometimes causes

irregular vaginal bleeding, and sometimes ruptures prior to missing a period.2. Symptoms of early pregnancy.3. Acute pelvic pain.4. Syncopal episode(s).5. Tenderness on transverse cervical movements.6. Tender mass in a lateral fornix.7. Shoulder tip pain, and features of free fluid in peritoneal cavity (with rupture of

ectopic pregnancy).

C. With rupture of a tubal pregnancy, a hemoperitoneum develops. Generalexamination reveals tachycardia, hypotension, and severe pallor. There is abdominaldistension, and horse-shoe shaped area of dullness and shifting dullness on percussion.

D. The pouch of Douglas is the most dependent part of the peritoneal cavity.Hence free fluid collects there. It is aspirated by colpopuncture. Abdominal paracentesisis an alternative. The blood obtained clots if it is a traumatic tap. It does notclot if it is due to a hemoperitoneum. Microclots are seen if a drop of this bloodis observed under a microscope. This is due to lysis of blood clots by proteolyticenzymes in the peritoneal fluid.

E. An intrauterine gestational sac is seen on transabdominal ultrasonography ifserum βhCG is > 1800 mlU/ml. (1000 mlU/ml with transvaginal ultrasonography). Ifthe cavity is empty, the diagnosis is ectopic pregnancy. If it is < 1800 mlU/ml, andan intrauterine gestational sac is seen, it is an intrauterine pregnancy which is

probably going to abort. If it is < 1800 mlU/ml and the uterine cavity is empty, beta-hCG assay is repeated after 48 hours. If it doubles, the pregnancy is intrauterine (tobe confirmed by ultrasonography). If it is less than double, the pregnancy isextrauterine. When in doubt, a laparoscopy is done to diagnose a tubal ectopicpregnancy.

Table 71.1 Lower Normal Limits for Increase in Serum Beta-hCG Levels withEarly Intrauterine Pregnancy

Sampling interval (days) % Increase in beta-hCG level from initial value1 29

2 66

3 114

4 175

5 255

F. A ruptured tubal pregnancy is treated by resection of the affected segmentof the tube. Removal of the entire tube is not justifiable, in view of a possibleneed for reconstructing the tube at a later date. The round ligament of the sameside is drawn backwards over the stump, and sutured to the uterus behind thecornu (modified Coffey's repair) to prevent adhesions with omentum and/or bowel.

G. Conservative treatment is possible only for unruptured tubal ectopicpregnancy.

1. Expectant management: mere observation may be sufficient if the diameter ofthe ectopic is not > 3.5 cm, βhCG levels are low (below 2000 IU/L) and falling,and there is no intra-abdominal hemorrhage. Close observation is necessary toconfirm resolution and to detect rupture in an early stage.

2. Mifepristone: it is an antiprogesterone. A single oral dose of 600 mg is given toa woman with inclusion criteria as for expectant management. She is closelymonitored with serial beta-hCG assay and ultrasonography for resolution orrupture.

3. Salpingocentesis: the amniotic fluid in the ectopic sac is removed by tappingunder ultrasonographic control or during laparoscopy. Then methotrexate (1mg/kg is the usual dose, though 5 mg may be adequate) or potassium chloride(10%, 2 ml) is injected into the sac. The ectopic embryo dies and gets resorbed.

4. Methotrexate: a single dose of 50 mg/m2 IM is adequate. Its toxicity has to bewatched for, besides monitoring resolution of the ectopic pregnancy. Injectionof 1 mg/kg into the gestation sac under ultrasonic control is effective too, andassociated with less side effects.

5. Conservative surgery: suction of tubal abortion, linear salpingotomy, and linearsalpingostomy are different options available. All of these operations can beperformed by laparotomy or laparoscopy. The results of linear salpingotomyand linear salpingostomy are similar. The aim of conservative surgery is toconserve a functional fallopian tube and thereby preserve fertility.

141

Page 148: Differential Diagnosis and Management Options inuploads.worldlibrary.org/uploads/pdf/20180107071525differential... · 83 Peripartum cardiomyopathy 165 84 Hypertension 167 85 Convulsions

SUSPICION OF TUBAL ECTOPIC PREGNANCY

History of predisposing factors (A) Current illness (B)

General examination (C)

Tachycardia, hypotension, pallor Stable, normal pulse and blood pressure, no pallor

Colpopuncture (D) Pelvic ultrasonography, serum βhCG assay (E)

Free flow of blood Negative Empty uterusβhCG > 1800 mIU/ml

Empty uterusβhCG < 1800 mIU/ml

Intrauterine empty sacβhCG < 1800 mIU/ml

Clots Does not clot Ultrasonographically directed tap of peritoneal fluid Repeat serum βhCG assay after 48 hours Spontaneous abortionis likely to occur

Traumatic tap Ruptured ectopic pregnancy Free flow of bloodwhich does not clot

βhCG < double βhCG doubles Observation

ResuscitationExploratory laparotomyPartial salpingectomy and modified Coffey’s repair

(F)

Unruptured ectopic pregnancy Ultrasonography to confirm diagnosis

Conservative treatment (G) Normal pregnancy

142

Page 149: Differential Diagnosis and Management Options inuploads.worldlibrary.org/uploads/pdf/20180107071525differential... · 83 Peripartum cardiomyopathy 165 84 Hypertension 167 85 Convulsions

72. Repeated AbortionsA woman is said to have habitual abortions when she has had three or

more, spontaneous, consecutive abortions. Most of the first trimester abortions aredue to germ plasm defects. Recurrence occurs if there is a persistent untreatedfactor. Rate of abortion with previous 0, 1, 2, > 3 abortions is 12, 20, 25 and 27%respectively.A. A pattern may be identified with certain causes of repeated abortions.

1. Septate uterus: either first trimester abortions (due to implantation on theseptum) or in second trimester (due to incompetent os).

2. Bicornuate uterus: either first or second trimester abortions at two differentgestational ages randomly (due to random implantation into either horn of variabledegree of development) or in second trimester (due to incompetent os).

3. Hypoplastic uterus: successive abortions at progressively increasing gestationalages from the first trimester, followed by preterm deliveries and finally termdelivery (due to growth of the uterus in each pregnancy).

4. Incompetent os: second trimester abortions at the same or decreasing gestationalages (with progressive damage to the cervix) characterized by painless dilatationand effacement of the cervix, watery discharge per vaginum, and a shortduration of pain before the abortion.

5. Syphilis: second trimester abortions (after 20 weeks of gestation) at progressivelyincreasing gestational ages, followed by preterm deliveries, macerated stillbirths,fresh still births, and delivery of a baby with congenital syphilis.

B. Pelvic ultrasonography is a very useful investigation both in the pregnantas well as in the nonpregnant state. It helps diagnose all anatomical abnormalitiesof the uterus, such as septate uterus, bicornuate uterus, hypoplastic uterus, uterineleiomyomas, cervical incompetence etc. It is difficult to diagnose Asherman'ssyndrome by this test, though transvaginal ultrasonography may prove useful.Ultrasonography also helps to localize placenta, and detect fetal malformationsprior to cervical cerclage.

C. Some chronic infections are believed to cause repeated abortions.1. Toxoplasma gondii: there is inconclusive evidence that it causes abortions. It

burns itself out, and is unlikely to cause repeated abortions.2. Listeria monocytogenes: there is no evidence that it causes abortions.3. Rubella, Cytomegalovirus, Herpes simplex: these agents may cause one time

abortions, but not repeated abortions.4. Ureaplasma urealyticum: it is an important cause, treatable with doxycycline to

both partners. It should not be used during a pregnancy due to risk ofteratogenesis.

5. Mycoplasma hominis: it plays no role.6. Bacterial vaginosis: it may cause midtrimester losses.

D. Antiphospholipid antibodies, including lupus anticoagulant, anticardiolipinantibody, anti-beta-2glycoprotein I antibody and cause recurrent abortions, by acting

against platelets and vascular endothelium. IgG antibodies are more significant,while IgM are important only in high titers. Lupus anticoagulant test may be maskedby pregnancy, while Russell viper venom time is confirmatory. Anticardiolipinantibody is detected and titered by specific solid-phase or enzyme-linkedimmunoassays. The condition is treated by low-dose aspirin (75 mg/d) and low-doseheparin (10000 units SC ql2h). It is more effective than combination of aspirin andprednisone (20-60 mg/d).

E. Maternal failure to develop antipaternal antileukocytic antibodies may causerecurrent abortions. Paternal lymphocyte immunization is claimed to be beneficial,but its long-term consequences are not yet known.

F. Balanced translocation, large chromosome 9, maternal X-chromosome mosaicism,and large long arm of Y-chromosome in parents can cause recurrent abortions in2%–5% of couples with repeated pregnancy wastage. Women are twice as likely tobe affected as men. However there is no treatment for this condition other than theuse of donor gametes.G. Cervical incompetence is a failure of the cervix to function as a sphincterwhen the pregnancy grows beyond 14 weeks. It may be due to any of the followingconditions.

1. Gynecologic cervical trauma: rapid dilatation, conization, amputation, Fothergill'soperation etc.

2. Obstetric cervical trauma: traumatic forceps or vacuum delivery, precipitatelabor, bucket-handle tear (with the use of prostaglandins for second trimesterMTP, or failure to remove cervical cerclage in presence of uncontrollableuterine contractions) etc.

3. Associated with congenital malformations of the uterus: bicornuate or septateuterus, T-shaped uterine cavity (DES exposure in utero).

4. Congenital: preponderance of smooth muscle over fibrous tissue in the cervix.Cervical incompetence is treated by cervical cerclage. The results of

MacDonald's and Shirodkar's procedure are similar. Results of an elective procedureare better than those of an emergency procedure. Abdominal cerclage (Benson andDurfee) is rarely required, when there is no portio vaginalis, forniceal tears, orextensive scarring of the cervix.

H. Maternal medical disorders responsible for recurrent abortions are uncontrolleddiabetes mellitus, smoking, alcoholism, PCOD, and exposure to toxic agents(anesthetic gases, arsenic, lead, formaldehyde, benzene and ethylene oxide).Presence of thyroid antibodies can cause recurrent abortions.

I. Syphilis during pregnancy is most often subclinical, detected by routineserological tests like VDRL test. Since the duration of the disease beyond the stageof a chancre (not noticed in a woman) cannot be diagnosed, all such women aretreated as when the duration is more than one year. The treatment is benzathinepenicillin 2.4 MU IM after a test dose, once a week for 3 weeks. If the patient isallergic to penicillin, she is given erythromycin (stearate, ethyl succinate or base)500 mg PO q6h for 30 days.

143

Page 150: Differential Diagnosis and Management Options inuploads.worldlibrary.org/uploads/pdf/20180107071525differential... · 83 Peripartum cardiomyopathy 165 84 Hypertension 167 85 Convulsions

REPEATED ABORTIONS

Assess pattern (A)

First trimester Second trimester

Early Late Previous painless abortions Painless dilatationand effacement of

the cervix

Serum progesterone assay Pelvic ultrasonography (B)Cervical

incompetence (G)

Low Normal Abnormal Normal

Corpus luteuminsufficiency

Pelvic ultrasonography (B) Not pregnant Pregnant Assess for the following

Abnormal Normal Conservative treatment

Pregnant Not pregnant Cervical culture (C) Antiphospholipid antibodies (D) Antipaternal antibodies (E) Parental karyotype (F)

Medicaldisorders (H)

Rh isoimmunization(see chapter 95)

Antiphospholipidantibodies

Conservative treatment Syphilis (I)

Uterineleiomyoma

Uterine septum Uterus bicornisunicollis

Asherman syndrome

Myomectomy(see chapter 51)

Hysteroscopicresection of septum

Strassman’smetroplasty

Hysterography/hysteroscopy Hysteroscopic lysis of adhesions, insertion of Lippes’ loop,estrogen therapy

144

Page 151: Differential Diagnosis and Management Options inuploads.worldlibrary.org/uploads/pdf/20180107071525differential... · 83 Peripartum cardiomyopathy 165 84 Hypertension 167 85 Convulsions

73. Antepartum HemorrhageAntepartum hemorrhage (APH) is hemorrhage from the genital tract after

28 completed weeks of pregnancy (age of viability) up to the delivery of the baby.

A. Placenta previa (PP) is implantation of the placenta in the lower uterinesegment. It is characterized by painless vaginal bleeding, often at night, oftenwithout any precipitating factor. Abdominal findings are malpresentation, highfloating presenting part, and placental soufflé. A vaginal, rectal, or speculumexamination is contraindicated as it can precipitate or aggravate bleeding.

B. Real time grey-scale ultrasonography confirms the diagnosis and definesthe degree of PP. Magnetic resonance imaging is also useful, but more expensive,and inappropriate in an emergency.

Table 73.1 Degrees of Placenta Previa

Degree Name Location of placenta

I Lateral One wall of uterus, not reaching internal os

II Marginal One wall of uterus, reaching internal os

III Incomplete central Crossing internal os, but not covering it after full cervicaldilatation

IV Complete central Covering internal os even after full cervical dilatation

Now the trend is to recognize only two degrees – complete (internal os completelycovered by placenta) and marginal (leading edge of the placenta is < 2 cm from theinternal os but not covering it).

C. If the fetus is dead, the pregnancy is terminated by amniotomy and oxytocininfusion for degrees I and II anterior PP; other degrees are managed by a lowersegment cesarean section (LSCS). LSCS is required if the fetus is distressed,irrespective of its maturity. If the fetus is well, its maturity is assessed (see chapter63 and 101).A woman with a mature fetus is managed the same way as one with a dead fetus. Ifthe fetus is immature, the treatment is Johnson and Macafee’s regimen, whichconsists of conservatism with bed rest, replacement of all significant blood loss, andkeeping transfusion and LSCS facilities ready at all times. It is continued until thefetus matures, or if any complication develops, such as hemorrhage severe enough tocause shock, or fetal death. Fetal lung maturity is hastened by administering dexa- orbetamethasone to the gravida (see chapter 101)Indications for LSCS in PP are as follows.

1. Malpresentation.2. Previous cesarean section.

3. Degree II posterior – IV placenta previa.4. Acute fetal distress.

D. If the placenta is in the upper segment, a speculum examination can bedone safely. Local lesions like cervical carcinoma, cervical erosion, varicose veinsetc may be found. If there is no local lesion responsible for the hemorrhage,the diagnosis is revealed placental abruption (AP), circumvallate placenta, orrupture of marginal sinus. The last two conditions can be diagnosed only afterdelivery of the placenta. The management of all the three conditions is as ofrevealed abruption. The pregnancy is conserved if the bleeding stops, and thefetus is alive but immature. Labor is induced if the fetus is mature or dead. LSCSis done if the fetus is distressed. Labor is induced if the bleeding is continuousand/or severe. If the bleeding is significant and delivery is not likely to occursoon, LSCS is done.

E. Abdominal pain may be intermittent (due to labor) or continuous. Both PPand revealed AP can be associated with labor pains. The labor is controlled withtocolytics (see chapter 101) if the fetus is immature. Labor is allowedto progress if the fetus is mature. If the pain is constant, it is due to mixed AP,the concealed component causing the pain, and the revealed component beingseen as vaginal bleeding. Ultrasonography shows separation of the placenta andthe membranes near the edge of the placenta from the upper segment, with fluid(blood) underneath. Labor is induced if the fetus is dead, mature, or if the clotis large. A small clot may be managed by continuation of the pregnancy andweekly assessment of the size of the clot by ultrasonography. The clot may undergoresorption or organization. A large clot is managed as follows.

1. Low rupture of membranes: to induce labor, and to reduce intraamnioticpressure, so that the clot may not expand further.

2. Left lateral position: to prevent compression of inferior vena cava, whichreduces venous pressure in placental bed and reduces chances of expansion ofthe clot.

3. Oxytocin infusion: to induce labor.4. Central venous cutdown: to monitor fluid infusion. Blood pressure is not a

reliable guide to fluid therapy, as often the patient is hypertensive, and herblood pressure reaches normal levels with severe hemorrhage.

5. Monitoring of coagulation profile (see chapter 155).6. LSCS is done for the following indications.

a. Acute fetal distress.b. Steady bleeding.c. Failure of labor to begin promptly and effectively with delivery in 12

hours.d. Progression of abruption.e. Progressive fall in plasma fibrinogen level.

145

Page 152: Differential Diagnosis and Management Options inuploads.worldlibrary.org/uploads/pdf/20180107071525differential... · 83 Peripartum cardiomyopathy 165 84 Hypertension 167 85 Convulsions

Resuscitate

ANTEPARTUM HEMORRHAGE

Assess abdominal pain (A)

Absent Present

Ultrasonoographic localization of placenta (B) Check nature

Lower segment Upper segment Labor pains, uterusrelaxed in between

Constant

Placenta previa Speculum examination Show Show

Assess fetal well being (C) Normal Local lesion Assess fetal maturity Mixed AP

Dead Distressed Well Revealed abruptionof placenta

Cervical erosionor carcinoma

Immature Mature Ultrasonography toconfirm diagnosis

Cesareansection

Assess fetalmaturity

Assess fetal wellbeing

Appropriatetreatment

See chapter 101 Management of labor Assess fetal wellbeing

Immature Mature Dead Distressed Well Dead Distressed Well

Johnson and Macafee’sregime

Induction oflabor

Cesareansection

Assess fetalmaturity

Induction of labor Cesarean section Assess size ofretroplacental clot

Check placental position Immature Mature Small Large

Degree I-II anterior Degree II posterior,III, IV

Assess bleeding Assess fetal maturity Induction of laborManagement ofcomplications

Amniotomy, oxytocininfusion

Cesarean section Mild Severe, continuous Immature Mature

Conservative treatment Amniotomy, oxytocin infusion Conservative treatment Induction of labor

146

Page 153: Differential Diagnosis and Management Options inuploads.worldlibrary.org/uploads/pdf/20180107071525differential... · 83 Peripartum cardiomyopathy 165 84 Hypertension 167 85 Convulsions

74. VomitingNausea and vomiting occur in 50 to 85% of uncomplicated pregnancies.

However the diagnosis of vomiting of pregnancy should be made only afterexclusion of other causes, so as not to miss organic conditions with seriousconsequences.

A. Vomiting due to gastrointestinal conditions is usually associated with painin the abdomen. Erosive gastritis may be caused by aspirin, NSAIDs, alcohol,burns, sepsis, shock, or renal, respiratory, or hepatic failure. It causes epigastricpain and hematemesis (See chapter 127). Acute viral gastritis causes severe vomitingand epigastric pain. It may be associated with diarrhea (due to enteritis). Acutepancreatitis causes midepigastric pain radiating to the back, nausea, vomiting,and low-grade fever. Acute cholecystitis causes right hypochondrial or epigastricpain, nausea, vomiting, and fever. There is local tenderness and a mass in 20%of the cases. In acute hepatitis, the patient has malaise, nausea, vomiting, diarrhea,low grade fever, high colored urine, jaundice, and tender hepatomegaly. In appendicitis,the patient has pain (the site of which ascends with gestational age), vomiting,and fever. In case of perforative peritonitis, the abdominal pain is acute andconstant, accompanied by vomiting, fever, tachycardia, abdominal distension,tenderness, guarding, rigidity, and no peristaltic sounds. In a case of intestinalobstruction, the patient has abdominal distension, vomiting, and hyperperistalsis. Theseconditions are investigated and treated appropriately.

B. Drugs known to cause vomiting are digitalis, opiates, and cytotoxic drugs.Digitalis toxicity is treated by withdrawing the drug and maintaining serum potassiumbetween 4.0 and 5.0 mEq/L. Opiate and cytotoxic drug induced vomiting is controlledby administration of antiemetic drugs.

C. If the patient has features of neurologic disease, detailed evaluation is necessary.A patient of meningitis has severe headache, vomiting, fever, neck stiffness, andmeningeal signs. In a case of subarachnoid hemorrhage, the patient has excruciatingheadache, vomiting and progressive third or sixth cranial nerve palsy. With raisedintracranial tension, the patient has persistent headache, nausea, vomiting, diplopia,blurring of vision, sixth cranial nerve palsy, and papilledema. A patient withclassical migraine had unilateral (right or left) throbbing headache, nausea, vomiting,visual scotomas and scintillations.

D. A gravida with acute pyelonephritis has fever with chills, nausea, vomiting,and costovertebral (renal) angle tenderness. Culture of urine grows pathogenicorganisms in colony count more than 105/ml.

E. Acute retrosternal chest pain of constricting type, nausea, vomiting, sweating,and apprehension suggest occurrence of acute myocardial infarction.

F. A number of metabolic disturbances can cause vomiting in pregnancy. Diabetic

ketoacidosis causes anorexia, nausea, vomiting, increase in urine output, abdominalpain, altered consciousness, acidotic breathing and later coma. Uremia causesanorexia, nausea, vomiting (especially in the morning), gastrointestinal bleeding,osteitis fibrosa cystica, and glucose intolerance. Hypercalcemia from any causecauses fatigue, depression, mental confusion, anorexia, vomiting, constipation,increase in urine output, cardiac arrhythmias, and a short QT interval on ECG.

G. Morning sickness of pregnancy is possibly related to high levels of β-hCG,pregnancy specific β-1-glycoprotein, (relatively) reduced levels of progesterone andcortisol, elevated levels of free T4 and reverse T3, endorphins from placenta andamniotic fluid and sensitization of opioid receptors in the hypothalamus to β-hCG.Some psychological component is responsible for the symptoms. Nausea andvomiting occur between 6 and 16 weeks of pregnancy, though in some cases thesymptoms continue to term. Though classically described as morning sickness, thesymptoms may occur at any time during the day, only in the evening, or eventhroughout the day. The incidence of abortion, preterm delivery, and stillbirth islower in gravidas who have morning sickness than in those who do not have it. Thetreatment is reassurance, giving small, dry, carbohydrate rich meals and avoidinglarge volume of drinks in the morning. Supplementation with vitamin K, Vitamin C,and Vitamin B6 is said to be beneficial. Cyclizine, prochlorperazine, promethazine,chlorpromazine have been used as first line, metoclopramide, domperidone andondansetron as second line and hydrocortisone as third line of treatment when thetreatment described above proves to be inadequate. However there are sporadiceffects of teratogenicity with their use, as shown in the following table.

Teratogenic Effects of Antiemetic Drugs

Drug Effect

Meclizine Cleft PalateCyclizine Cleft PalatePromethazine Congenital dislocation of hipProchlorperazine Cardiovascular and other anomaliesDoxilamine succinate Skeletal anomalies, cleft palate, pyloric stenosis,

congenital heart disease.

Hyperemesis gravidarum is vomiting prior to 20 weeks of pregnancyrequiring hospitalization. There is vomiting with every feed and later independent offood. It leads to dehydration, hypokalemia, hypochloremia, and ketoacidosis.Advanced cases may have hepatic dysfunction and jaundice. The treatment iswithholding oral intake, maintenance of fluid and electrolyte balance and acid basebalance, administration of 25% dextrose in water IV to supply extra calories, andsupplementation with vitamins. Total parenteral nutrition is given to severe cases. Ifthe cause is multiple pregnancy or vesicular mole, it is managed appropriately (seechapters 97 and 47).

147

Page 154: Differential Diagnosis and Management Options inuploads.worldlibrary.org/uploads/pdf/20180107071525differential... · 83 Peripartum cardiomyopathy 165 84 Hypertension 167 85 Convulsions

VOMITING IN PREGNANCY

Assess for abdominal pain and other abdominal features (A)

Present Absent

Gastrointestinal disease History of drug therapy (B)

Acute gastritis Acutecholecystitis

Acuteappendicitis

Intestinalobstruction

Acutepancreatitis

Present Absent

Acute hepatitis Perforative peritonitis Assess for features of neurologic disease (C)

Present Absent

Meningitis Raisedintracranial

tension

Migraine Subarachnoidhemorrhage

Assess for acute pyelonephritis (D)

Present Absent

Check for acute chest pain (E)

Present Absent

Acute myocardial infarction Assess for metabolic disturbances (F)

Diabetic ketoacidosis Uremia Hypercalcemia None

Vomiting of pregnancy (G)

Determine type (H)

Morning sickness Hyperemesis gravidarum

148

Page 155: Differential Diagnosis and Management Options inuploads.worldlibrary.org/uploads/pdf/20180107071525differential... · 83 Peripartum cardiomyopathy 165 84 Hypertension 167 85 Convulsions

75. Edema in PregnancyClearly demonstrable pitting edema of the ankles and legs is seen in a large

proportion of women in pregnancy, especially at the end of the day. This is causedby an increase in the venous pressure below the level of the uterus due to partialocclusion of the vena cava by the gravid uterus. A decrease in the interstitial colloidosmotic pressure seen in normal pregnancy also contributes to formation of edemain pregnancy. Such edema may manifest in fingers (tight rings) or face too.However, the diagnosis of physiological edema can be made only by exclusion ofother causes of edema.

A. Pregnancy induced hypertension (PIH) is characterized by hypertension withproteinuria with/without edema. It is a common cause of edema in pregnancy. If thegravida is following-up regularly, an early sign is excessive weight gain, due toaccumulation of body fluid. It is followed by hypertension, and finally by proteinuria.See chapter 84 for further details of PIH.

B. Pedal edema is a sign of congestive cardiac failure. The gravida may givea history of cardiac disease in the past. A detailed examination shows elevationof jugular venous pressure, basal crepitations, enlarged tender liver, and findingsof rheumatic valvular disease, or congenital noncyanotic heart disease. A gravidawith severe anemia can also have congestive cardiac failure (see chapter 76).

C. Deep vein thrombosis (DVT) occurs with a little higher incidence duringpregnancy than in the nonpregnant state, and much more often in the puerperium.Pain, tenderness, and edema of legs are classical features, but may not be presentin all the cases. Asymmetric limb, usually 2 cm greater than the other limb incircumference, may be found. Homan's sign is calf pain on stretching Achilles’tendon. It is positive in 33% symptomatic cases of DVT. Localized warmth anddependent cyanosis are variably present. Impedance plethysmography and pulsedDoppler ultrasound are noninvasive tests which are usually reliable, but theiraccuracy is reduced by physiologic changes of pregnancy. Compressibility of veinsunder real-time ultrasound visualization is a very accurate diagnostic procedure.

Venography is the standard for diagnosis. When the diagnosis is in doubt, normalantithrombin III or fibrinopeptide activity rules out active thrombotic disease. Thetreatment is with heparin by one of the following regimes.

1. Intravenous route: a loading dose of 100 to 110 U/kg, followed by a continuousinfusion of 1000 U/h. Partial thromboplastin time (PTT) is kept 1.5 to 2-foldprolonged over baseline for 7 to 10 days by frequent monitoring. Oncestabilized, PTT is repeated every 24 to 48 h. Subcutaneous therapy is given ina dose of 10000 to 20000 U every 12 h for the remainder of pregnancy and 6weeks postpartum.

2. Subcutaneous route: a loading dose of 150 U/kg is given intravenously,followed by 15000 to 20000 U every 12 h. PTT is checked at the midintervaland maintained at 1.5 times normal. It is continued for up to 6 weeks postpartum.

D. Edema due to renal disease is found mainly in the face. Generalized edemamay be found in conditions like nephrotic syndrome. The gravida may give historyof renal disease antedating pregnancy. See chapter 87 for further discussion ofrenal disease in pregnancy.

E. Edema may be due to hypoproteinemia. The diagnosis can be made by checkingserum proteins. If it is present, its cause is found and treated appropriately. Ifserum proteins are normal, it is physiological edema in pregnancy.

F. Physiological edema of pregnancy does not require any treatment. It is foundthat the birth weight of the baby is higher when the gravida has edema as comparedto when she does not. Reassurance is usually adequate. The gravida is asked to keepher feet on a stool of the height of her chair while sitting down, and elevate herlegs on pillow(s) while sleeping. Left lateral position helps mobilize edema fluid.Normal diet is permitted. She is advised to avoid intake of extra salt in her diet.Diuretics are best avoided, as they can prove harmful to the gravida as well as herfetus. Furosemide can cause maternal hypovolemia, hypokalemia, hyponatremia, andsimilar electrolyte imbalance in the fetus. Thiazides can cause similar electrolytedisturbances, hyperglycemia, hyperuricemia, and thrombocytopenia in the motherand baby both. Such adverse effects far outweigh the relief from edema achieved.

149

Page 156: Differential Diagnosis and Management Options inuploads.worldlibrary.org/uploads/pdf/20180107071525differential... · 83 Peripartum cardiomyopathy 165 84 Hypertension 167 85 Convulsions

EDEMA IN PREGNANCY

Check blood pressure (A)

High Normal

Pregnancy induced hypertension(See chapter 84)

Cardiovascular assessment (B)

Normal Congestive cardiac failure

Test for deep vein thrombosis (C) Valvular heartdisease

Congenitalheart disease

Anemia

Negative Positive Treat cardiac failureTreat cardiac disease (see chapter 80)

See chapter 76

Renal function tests (D) Appropriatetreatment

Abnormal Normal

Chronic renaldisease

Check serum proteins (E)

See chapter 87 Hypoproteinemia Normal (F)

Appropriate treatment Physiologicaledema

ReassuranceElevation of legs

150

Page 157: Differential Diagnosis and Management Options inuploads.worldlibrary.org/uploads/pdf/20180107071525differential... · 83 Peripartum cardiomyopathy 165 84 Hypertension 167 85 Convulsions

76. Suspicion of AnemiaA. Anemia is an extremely common disorder during pregnancy. Hemoglobin(Hb) level is estimated at the first antenatal visit. If it is less than 10 g/dL, thewoman has anemia. Hb at 12 weeks is 2 g/dL higher than that at term, if noiron supplements are given during pregnancy.

B. Though iron deficiency anemia and dimorphic anemia are the commonesttypes of anemia in pregnancy, it is essential to determine the type and to watchfor presence of the rarer types before initiating treatment. Hemogram, peripheralsmear, and blood indices are the basic tests performed on a woman with anemia.The following types are seen with these tests.

1. Iron deficiency anemia: the features are hypochromia, microcytosis, low levelsof MCV, MCH, and MCHC, anisocytosis and poikilocytosis. Serum iron levelsare also low, ferritin levels are < 12 μg/L, TIBC is < 15%, and sideroblasts inbone marrow are < 10%.

2. Chronic diseases like chronic renal disease, suppuration, inflammatory boweldisease, systemic lupus erythematosus, granulomatous infections, malignantneoplasms, and rheumatoid arthritis cause mild to moderate anemia. Peripheralsmear shows mild hypochromia and microcytosis. Serum iron levels are low,ferritin levels are raised, while the bone marrow is unaltered.

3. Megaloblastic anemia: macrocytosis and hypersegmentation of neutrophils areevident. The diagnosis is confirmed by serum folate assay and bone marrowbiopsy.

4. Dimorphic anemia: hypochromia and macrocytosis are seen. MCV is increasedwhile MCH and MCHC are low. Bone marrow biopsy, serum iron estimation,and bone marrow stainable iron reveal combination of iron and folic aciddeficiency anemias.

5. Normocytic anemia: it is due to iron deficiency (low serum iron) orhemoglobinopathy (diagnosed by Hb-electrophoresis).

6. Congenital spherocytosis and elliptocytosis are two forms of hemolytic anemiasdue to abnormal red cell morphology, which is seen on a peripheral smear.

7. Hemolytic anemia: the woman may have jaundice and splenomegaly. Her MCVand MCH are low, while MCHC is normal. Hb electrophoresis differentiatesbetween β-thalassemia, sickle-cell disease, and other rarer types.

8. Aplastic anemia: the woman has pancytopenia.9. Leukemia: the WBC count is very high, while Hb and platelet count are low. A

bone marrow biopsy determines the type.Results of various hematologic laboratory tests are shown in table 76.1

C. Management of anemia in pregnancy is bound by time. It is necessary tohave a Hb of at least 9g/dL by 37 weeks. Hence the next step in the management

after determination of the type of anemia is to determine the severity of the anemiaand the duration of pregnancy at the time of the diagnosis. Mild anemia is Hb 8-10g/dL, moderate anemia is Hb 5-8 g/dL, and severe anemia is Hb < 5 g/dL. Withoptimum therapy (e.g. iron or folic acid therapy in a deficiency state), the rate of riseof Hb is 0.1-0.25 g/dL/d. If there is sufficient time up to term for the desired Hb levelto be reached, appropriate therapy is given. If there is no response to such therapy, or ifthere is not adequate time for the treatment to act, blood or blood component transfusionshave to be given. The pregnant woman has hyperdynamic circulation due to theplacental shunt. Presence of anemia makes it even more hyperdynamic. If the preloadon the heart is increased suddenly, such a woman is likely to go into congestive cardiacfailure. Hence packed cells are preferred to whole blood for transfusion. If the Hb is <2g/dL, even packed cells can precipitate cardiac failure. Such a woman is given modifiedexchange transfusion, in which the patient's blood is removed from one vein and packedcells are transfused through another one. In this way Hb can be corrected to 8 g/dL in45 minutes. It is also useful if a woman with severe anemia presents directly in labor. It isused in sickle cell disease too, to reduce HbS level and rate of its synthesis (to improvetissue oxygenation and prevent sickling crises) and improve adult Hb level.

Women with aplastic anemia and leukemia are referred to a hematologist forspecific treatment.

Table 76.1 Hematologic Tests in Pregnancy

Test Result

Pregnancy Nonpregnant state

Hemoglobin 10-13 g/dl 12-16 g/dlHematocrit 30-39% 37-45%RBC count 3.8-4.4 X lOVmm3 4.2-5.4 X lOVmm3

WBC count 6-16 X lOVmm3 4.7-10.6 X lOVmm3

MCV 70-90 fl 80-100 flMCH 23-31 pg/RBC 27-34 pg/RBCMCHC 32-35 g/dl/RBC 32-25 g/dl/RBCRetic count 1-2% 0.5-1%Serum iron 30-100 μg/dl 50-100 μg/dlTIBC 280-400 μg/dl 250-300 μg/dlTransferrin saturation 15-30% 25-35%Serum B12 75-500 ng/dl 70-85 ng/dlSerum folate 4-10 ng/ml 4-16 ng/ml

151

Page 158: Differential Diagnosis and Management Options inuploads.worldlibrary.org/uploads/pdf/20180107071525differential... · 83 Peripartum cardiomyopathy 165 84 Hypertension 167 85 Convulsions

SUSPICION OF ANEMIA

Hemoglobin estimation (A)

> 10 g/dL 10 g/dL

Hemogram, peripheral smear, blood indices

Hypochromia,microcytosis

Macrocytosis, hypersegmentedneutrophils

Normal RBCs, plus hypochromia,microcytosis, basophilic stippling

Normochromia, Anisocytosismacrocytosis

Low MCV,low MCH,normal MCHC

Low RBC count,leucocytosis,

thrombocytopenia

Plasma ferritin Megaloblastic anemia Retic count, free RBC protoporphyrin Serum iron Hemoglobinopathy Bone marrowexamination

Low High Retic count ↑, free RBCprotoporphyrin ↑

Normal Low Normal Hb-electrophoresisor high

Leukemia

Iron deficiencyanemia

Lead poisoning Bone marrow Iron deficiencyanemia

Normal High HbA2,normal orhigh HbF

HbS

Chronicdiseases

Confirm by serum andurine lead levels

Ringed/abnormalsideroblasts

Anemia of infection

β thalassemia

Sickle celldisease

Systemic examination Sideroblastic anemiaSpherocytes Elliptocytes Spur cells Schistocytes Heinz bodies

Chronic renalFailure

Cirrhosis ofliver

Hypothyroidism Granulomatousinfections

DirectCoombs’ test

Elliptocytosis Spur cell anemia

Traumatichemolyticanemia

Check for enzymedefects in RBCs

Hypogonadism Adrenal insufficiency SLE Negative Positive Laennec’scirrhosis Present Absent

Rheumatoid arthritis Malignancy Spherocytosis Autoimmunehemolytic anemia G6PD

deficiencyUnstable Hb

Pancytopenia

Bone marrow examination

Aplasia Normo- or hypocellular Myelofibrosis Infiltration by other disease cells

Aplastic anemia Myelodysplastic syndrome Metastatic carcinoma Lymphoreticularmalignancy

Tuberculosis

152

Page 159: Differential Diagnosis and Management Options inuploads.worldlibrary.org/uploads/pdf/20180107071525differential... · 83 Peripartum cardiomyopathy 165 84 Hypertension 167 85 Convulsions

77. Subacute/ChronicBreathlessness

An increased awareness of a need to breathe is common in pregnancy. Innonsmokers and others without any respiratory disease, breathlessness may beexperienced as early as the twelfth week of pregnancy, and many women have thissymptom by the thirtieth week. It is due to an increase in the tidal volume whichlowers the PCO2 slightly. It is induced by progesterone and to a lesser extent byestrogen. It does not need nor does it have any treatment. However, the diagnosis ofphysiological cause of breathlessness can be made only by exclusion of organicconditions.

A. Breathlessness may be due to cardiac disease, such as congestive cardiacfailure or left ventricular failure (see chapter 80).

B. Breathlessness may be due to a number of respiratory disorders, due to different

mechanisms.

1. Bronchial asthma: bronchospasm.2. Pneumonia.3. Bronchitis.4. Pleural effusion.5. Interstitial lung disease.

C. Anemia is quite common in Indian women during pregnancy. Severe anemia isnot so common. It is an important cause of breathlessness, partly due to loweredoxygen-carrying capacity of the blood, and partly due to congestive cardiac failure. Itis further discussed in chapter 77.

D. Excessive distension of the uterus, as seen in multiple pregnancy and hydramnios,can cause splinting of the diaphragm and hence breathlessness. Hydramnios can be acomplication of monozygotic twins with twin-to-twin transfusion, which furtheraggravates the symptom. In multiple pregnancy, anemia is another complicating factor,which can contribute to the symptom. Similar situation may arise if there is a largeovarian tumor concurrent with the pregnancy.

153

Page 160: Differential Diagnosis and Management Options inuploads.worldlibrary.org/uploads/pdf/20180107071525differential... · 83 Peripartum cardiomyopathy 165 84 Hypertension 167 85 Convulsions

SUBACUTE/CHRONIC BREATHLESSNESS

Examination of cardiovascular system (A)

Cardiac disease Normal

See chapter 80 Examination of respiratory system

Respiratorydisease (B)

Normal

See chapter 79 Assess for pallor (C)

Present Absent

Anemia Assess uterine size (D)

See chapter 77 Excessive Normal

Multiplepregnancy

Hydramnios Physiological

See chapter 96 See chapter 99

154

Page 161: Differential Diagnosis and Management Options inuploads.worldlibrary.org/uploads/pdf/20180107071525differential... · 83 Peripartum cardiomyopathy 165 84 Hypertension 167 85 Convulsions

78. CoughA. After initial evaluation of the patient's symptoms, the cardiovascular systemis examined for evidence of cardiac disease, which is a serious cause of cough.Cough with blood stained sputum may be the presenting symptom of mitral stenosis.A patient with left ventricular failure or congestive cardiac failure due to anycause may present with cough. Acute left ventricular failure causes cough with pinkfrothy sputum. Cardiac disease is discussed further in chapter 80.

B. Upper respiratory tract infection is the commonest cause of cough duringpregnancy. Pharyngitis causes dry cough with throat soreness and pain. Inspection ofthe throat shows inflammation, confirms the diagnosis.

C. Sputum is an important differentiating point. A swab may be obtained from thethroat for microbiological study. Medical treatment of pharyngitis includes thefollowing.

1. Steam inhalation.2. Cough suppressants

a. Linctus codeine 1 tsf PO q8h.b. Dextromethorphan 30 mg PO ql2h to q6h.c. Guaifenesin 600-1200 mg PO ql2h.

3. Antibiotics: based on microbiologic report. Usually amoxicillin, ampicillin,cloxacillin, and erythromycin give satisfactory results.

Extrinsic pressure on a bronchus is an uncommon cause of dry cough inpregnancy. The cough is brassy. A chest radiograph may be obtained with shieldingof the abdomen. A medical reference is necessary for management.

Presence of purulent sputum indicates the presence of bacterial infection inthe upper or lower respiratory tract. If there is blood in the sputum, pulmonary

tuberculosis is a strong possibility. Examination of the respiratory system reveals thepresence of upper or lower respiratory tract infection. An upper respiratory tractinfection is treated as discussed before. In case of productive cough, fever, chestpain, and dyspnea, with typical findings on chest radiograph, diagnosis of pneumoniais made. The sputum is checked microbiologically. Treatment of pneumonia is asfollows.

1. Hospitalization.2. Antibiotics

a. Erythromycin is the drug of choice, as most of the adult pneumoniasare due to pneumococci or mycoplasma. Dose is 500-1000 mg q6h,intravenously initially, and orally later.

b. Cefotaxime, ceftizoxime, or cefuroxime is given for Staphylococcal orHaemophilus pneumonia.

c. Cotrimoxazole is used to treat Pneumocystis carinii pneumonia.3. Expectorant mixture.4. Mucolytic agent: acetylcysteine is administered by inhalation (1-10 ml 10%

solution q2-6h) for inspissated mucus.

D. If treatment of lower respiratory tract infection fails, or if the sputum has blood init, it should be examined for M. tuberculosis on 2 occasions, by microscopy andGeneXpert test. If it is positive a diagnosis of pulmonary tuberculosis is made. If it isnegative, a chest radiograph is obtained (with abdominal shielding) to diagnose orrule out tuberculosis. In case of pulmonary tuberculosis, a first trimester pregnancy isterminated after 6 weeks of antituberculous therapy, in view of possibility ofteratogenic effects of the treatment on the fetus. If the pregnancy is beyond the firsttrimester, the pregnancy is continued under close supervision, continuingantituberculous therapy (see chapter 4). The woman is counseled to have a test forHIV antigen, since tuberculosis and AIDS are sometimes associated with each other.

155

Page 162: Differential Diagnosis and Management Options inuploads.worldlibrary.org/uploads/pdf/20180107071525differential... · 83 Peripartum cardiomyopathy 165 84 Hypertension 167 85 Convulsions

Check for cardiac disease (A)

COUGH

Check for sore throat, throat pain, dysphagia (B)

Present Absent

Inspection of throat Check for sputum (C)

Pharyngitis Absent Present

Examine respiratory system Check nature of sputum

Upper respiratory tractinfection

Extrinsic pressure onbronchus

Mucoid Purulent Blood-stained

Examine respiratory system

Upper respiratory tract infection Lower respiratory tract infection

Microbiologic tests on sputumAppropriate antibiotics, expectorant, mucolytic agents

Relief No relief (D)

Sputum examination for M. tuberculosis

Positive Negative

Assess gestational age Pulmonary tuberculosis Chest radiography with abdominal shield

First trimester > First trimester Nontuberculous condition

AKT, MTP AKT, antenatal care, neonatal management Appropriate treatment

156

Page 163: Differential Diagnosis and Management Options inuploads.worldlibrary.org/uploads/pdf/20180107071525differential... · 83 Peripartum cardiomyopathy 165 84 Hypertension 167 85 Convulsions

79. Pulmonary DiseaseA. Course of bronchial asthma remains unchanged in pregnancy in 40% cases, whileit worsens in 30% and improves in 30% cases. Women who begin the pregnancywith severe asthma are more likely to experience aggravation of symptoms. A severeattack is diagnosed in presence of respiratory distress at rest, diaphoresis, difficultyin speaking, and use of accessory muscles. Respiratory rate > 30/min, pulse >120/min, and pulsus paradoxus > 18 mm Hg indicate a dangerous state. Somnolenceand agitation suggest impending respiratory failure.Treatment of acute attack

1. Oxygen2. Bronchodilators

a. Metered dose inhaler with a spacer device or reservoir is used to administerterbutaline 200-400 µg, albuterol 90-180 µg or metaproterenol 650-1300µg. Two to four initial puffs are followed by 1-2 puffs q 10-20 min untilimprovement or toxicity is seen.

b. Albuterol (2-5 mg/ml) or metaproterenol (50 mg/ml) 0.2-0.3 ml in 3ml normal saline is given by a nebulizer over 5-15 min, repeated notearlier than 4 hours.

c. Adrenalin 0.3 mg SC ql5-20 min up to 3 doses may rarely be required.d. Aminophylline 6 mg/kg IV over 20 min, followed by 0.5-0.6 mg/kg/h

IV infusion.3. Corticosteroids : methyl prednisolone 0.5-1 mg/kg IV q6h is administered ifsubstantial improvement does not occur within first 1 hour of intensivebronchodilator therapy.

Maintenance therapy1. Avoidance of known precipitating factors.2. Bronchodilators :

a. Terbutaline, albuterol, or metaproterenol 2 puffs q4-6h by inhalation.b. Theophylline 200-400 mg PO ql2h (long acting preparation).

B. Pneumonia : see chapter 77.C. Restrictive lung disease may be due to a variety of causes such as sarcoidosis,rheumatoid arthritis, systemic lupus erythematosus, systemic sclerosis, kyphoscoliosis,and neuromuscular disorders. After a detailed history and pulmonologic evaluation,the patient is treated as follows.

1. Potentially injurious environmental and drug exposures are discontinued.Cardiac failure and infection are treated appropriately.

2. Oxygen supplements are given as required. Nasal intermittent positive-pressureventilation may be required in some cases.

3. Corticosteroids are often required.D. Adult respiratory distress syndrome (ARDS) is a nonspecific pulmonary reactionto a large number of insults e.g. sepsis, major trauma, massive aspiration ofacid stomach contents, and drug overdose. Acute pulmonary injury triggers it. Thediagnosis is made only after exclusion of other causes of pulmonary edema. Clinicalfeatures of ARDS are dyspnea, crepitations, and a variable degree of hypoxemia

Table 79.1 Causes of ARDS in PregnancyObstetric causes Nonobstetric causesPreeclampsia Burns, smoke inhalation.Acute fatty liver of pregnancy SepsisTocolysis with betamimetic drugs ShockAspiration of stomach contents Arsenic poisoningObstetric infections: amnionitis TraumaAmniotic fluid embolism PyelonephritisObstetric hemorrhage, placental abruption Varicella pneumoniawithout cardiomegaly, pleural effusion, or pulmonary vascular redistribution. Chestradiograph shows bilateral diffuse infiltrates. The patient has severe hypoxemiadespite a high FiO2, an increased alveolar-arterial oxygen gradient and an acute onsetof impaired oxygenation such that inspired oxygen does not raise the PO2 above 100mm Hg. The PaO2 to FiO2 ratio is < 200, regardless of positive end-expiratorypressure (PEEP). Lung compliance is diminished, pulmonary artery wedge pressureis <18 mm Hg (which excludes cardiogenic pulmonary edema). Left atrial pressureis not raised. Principles of treatment of ARDS in pregnancy are maintenance ofadequate tissue oxygenation, prevention of secondary infection, and avoidance ofsecondary lung injury and other complications of critical care. The underlying causemust be identified and treated vigorously. Mechanical ventilatory support is requiredfor most cases. It is usually provided initially with volume ventilation with an Fi02

sufficient to provide arterial O2 saturation above 90. PEEP is used to maintainalveolar inflation and improve oxygenation. However PEEP can decrease cardiacfilling pressures, and when it exceeds 10 cm Hg, the cardiac output may be reducedsignificantly. Ventilator support becomes more and more difficult as gas exchangeand lung compliance worsen. Complications like barotrauma, hypoxemia, hypercapnia,and airway injury may develop. Overhydration needs to be avoided. Diuresis may beinduced with diuretics. But they decrease maternal intravascular space and cardiacfilling pressures, and may depress maternal cardiac output and reduce mixed venousoxygenation and uteroplacental perfusion. This may lead to impaired fetaloxygenation and fetal distress. Dopamine is used to preserve cardiac output and renalperfusion in the face of maternal diuresis. In case of low systemic vascularresistance, phenylephrine or norepinephrine may be used, but they may decreaseuteroplacental perfusion and cause fetal compromise. PGE2 cannot beused as it causes uterine contractions. Preterm labor may be caused by seriousmaternal illness. Suppression of uterine contractions reduces oxygen consumptionrelated to uterine contractions and may benefit the patient. NSAIDs are the drug ofchoice because they control preterm labor and also help control ARDS. Use ofcalcium channel blockers is limited by their hypotensive and cardiac effects. Beta-2-sympathomimetic agents are contraindicated as they themselves can cause ARDS inpregnancy. Since continuation of pregnancy causes an increase in maternal demandsin terms of cardiac performance and oxygen consumption, delivery should beachieved if the gestational age is > 37 weeks. It is also done if the maternal conditioncontinues to deteriorate despite adequate therapy at any gestational age, because thefetus is unlikely to do well under such conditions, and delivery also increases thetherapeutic options for the woman.

157

Page 164: Differential Diagnosis and Management Options inuploads.worldlibrary.org/uploads/pdf/20180107071525differential... · 83 Peripartum cardiomyopathy 165 84 Hypertension 167 85 Convulsions

PULMONARY DISEASE

Determine nature

Bronchial asthma (A) Pneumonia (B) Restrictive lung disease (C) ARDS (D)

Assess for bronchospasm Chest radiograph, hemogram, arterialblood gases.

Hemogram, pulmonary function tests, ,arterial blood gases.

Clinical examination, chest radiograph,arterial blood gases. Pulmonary

capillary wedge pressure, determinecause

Absent

Usual treatment

Present Oxygen, antibiotics,promote cough.

Oxygen, , IV fluids, antibiotics, optimumposition.

Obstetric Nonobstetric

Oxygen, adrenergic agents (nebulized),aminophylline, IV fluids. Treat cause

Assess for bronchospasm Determine severity of ARDS

Present Absent Medical treatment of ARDS

IV corticosteroids, arterial line, serial arterialblood gases.

Improvement Deterioration

Determine gestational age

Preterm labor Not in labor

Control preterm labor with NSAIDs Observation Term

Achieve delivery

158

Page 165: Differential Diagnosis and Management Options inuploads.worldlibrary.org/uploads/pdf/20180107071525differential... · 83 Peripartum cardiomyopathy 165 84 Hypertension 167 85 Convulsions

80. Cardiac DiseaseA. Clinical features suggestive of the presence of an organic heart disease areas follows.

1. Symptoms: severe or progressive dyspnea, orthopnea, paroxysmal nocturnaldyspnea, syncope with exertion, hemoptysis, and chest pain with exercise oremotion.

2. Signs: cyanosis, clubbing, elevation of jugular venous pressure, systolicmurmur, sustained arrhythmia, cardiomegaly, and pulmonary hypertension.

A classification of cardiac status is as follows.Class I: uncompromised; no limitation of physical activity.Class II: slightly compromised; slight limitation of physical activity.Class III : markedly compromised; marked limitation of physical activity.Class IV : severely compromised; inability to perform any physical activity.

B. Usually hemogram, electrocardiogram, and 2-D echocardiogram are adequate.A chest radiograph is sometimes necessary. It should be obtained with shieldingof the maternal abdomen to avoid fetal irradiation.

C. The maternal prognosis depends on the nature and severity of the diseaseand presence of complications.

1. Group 1: ASD, VSD, PDA, mitral stenosis (class I and II), tetralogy of Fallot(corrected), pulmonary or tricuspid valve disease, and presence of abioprosthetic valve.

2. Group 2: mitral stenosis (class III and IV), aortic stenosis, coarctation of aorta,tetralogy of Fallot (uncorrected), Marfan syndrome, previous myocardialinfarction, atrial fibrillation, and presence of artificial valve.

3. Group 3: pulmonary hypertension, coarctation of aorta (with valve involvement),Marfan syndrome (with aortic involvement).

Maternal mortality in the three groups is 0-1%, 5-15%, and 25-50% respectively.

D. The following conditions may precipitate cardiac failure during pregnancy.

1. Anemia.2. Infection: pulmonary, urinary, or other.3. Tachyarrhythmia: atrial fibrillation, atrial flutter.4. Rheumatic carditis.5. Thyrotoxicosis.6. Infective endocarditis.7. Pregnancy induced hypertension.8. Exercise or vigorous physical activity.

The patient is at increased risk of cardiac failure at the end of the firsttrimester (maximum increase in cardiac output), after 7 months (maximum increasein circulating blood volume), during labor (physical activity), and fourth stage oflabor (sudden closure of placental shunt and uterine circulation). Cardiac failure iswatched for and treated as follows.

1. Propped up position.2. Moist oxygen by face mask.

3. Furosemide 20 mg IV, repeated as required.4. Digoxin: for cardiac failure accompanied by atrial fibrillation or flutter,

dilatation of left ventricle, and impaired systolic function (third heart sound,low ejection fraction, large cardio-thoracic ratio).a. Loading dose: 0.25-0.5 mg PO or IV initially, followed by 0.25 mg PO

q6h to a total dose of 1-1.5 mg. Evidence of toxicity should be soughtprior to each dose.

b. Maintenance: 0.125-0.375 mg PO qd.

E. Medical termination of pregnancy (MTP) is done only on medical grounds,and only in the first trimester. The risk to the patient after the first trimesteris equal to that with a vaginal delivery. Antibiotic prophylaxis is given for infectiveendocarditis. Drugs like methyl ergometrine and prostaglandin are avoided. Uterinecontractions can be achieved with oxytocin (intramuscularly or intravenously withconstant infusion of a concentrated solution).

F. Continuous prophylaxis for recurrent rheumatic fever is given by one ofthe following regimens.

1. Benzathine penicillin G, 1.2 MU IM q4 weeks.2. Sulphadiazine 1 g PO qd, if the patient is allergic to penicillin.3. Erythromycin 250 mg PO q12h, as an alternative to sulphadiazine.

Infective endocarditis prophylaxis is given as shown in table 79.1.Anticoagulation is required for atrial fibrillation and thromboembolism. Heparin isthe drug of choice.

Labor needs to be induced only for fetal indications, such as restriction ofgrowth. Cervical ripening is done with intracervical Foley’s catheter and not PGE2

gel. Induction or augmentation is done as required with a constant infusion ofoxytocin using a pump (10 units in 100 ml) rather than with conventional solution soas to avoid fluid overload. Methyl ergometrine is avoided after delivery, as it causeshypertension, sudden increase in cardiac preload (due to uterine contraction andretraction) and sustained elevation of venous tone for up to 24 hours. Breast feedingis not contraindicated. Temporary contraception is by barrier method. Permanentcontraception is by vasectomy of the husband, and if that is not possible, byminilaparotomy (tubal ligation) under local anesthesia.

Table 80.1 Infective Endocarditis Prophylaxis

Drug Dosage

Dental ProcedurePenicillin VK 2 g PO 1 hour before, 1 g PO 6 hours afterErythromycin (if allergic to penicillin) 1 g PO 2 hours before, 0.5 g PO 6 hours afterMTPAmoxicillin 3 g PO 1 hour before, 1.5 g PO 6 hours afterLabor, LSCSAmpicillin plus

Gentamicin2 g IV/followed by 0.5 g IV q6h,1.5 mg/kg IV q 8h(last dose 6 hours after delivery/LSCS)

159

Page 166: Differential Diagnosis and Management Options inuploads.worldlibrary.org/uploads/pdf/20180107071525differential... · 83 Peripartum cardiomyopathy 165 84 Hypertension 167 85 Convulsions

Clinical features (A)

CARDIAC DISEASE

Assess cardiac status (C)

Investigations (B)

Decompensated Compensated

Evaluate for factors which precipitate cardiac failure (D)

Present Absent

Appropriate treatment Control cardiac failureAssess gestational age

First trimester Other cases Second/third trimester

MTP for (E) Repeated cardiac failure Rheumatic carditis Infective endocarditis Eisenmenger’s syndrome Primary pulmonary hypertension

Close observation Prenatal care Treatment of septic foci Early recognition and treatment of factors which precipitate cardiac failure Serial assessment of fetal growth and well-being (see chapters 63, 64, and 144) Plan vaginal delivery under antibiotic cover, helped with low forceps or vacuum extraction.

160

Page 167: Differential Diagnosis and Management Options inuploads.worldlibrary.org/uploads/pdf/20180107071525differential... · 83 Peripartum cardiomyopathy 165 84 Hypertension 167 85 Convulsions

81. Cyanotic Heart DiseaseA. The commonest cyanotic heart disease during pregnancy is tetralogy of Fallot(TOF), which consists of right ventricular hypertrophy, a large ventricular septaldefect, and overriding of the aorta. Its clinical features include cyanosis, clubbing,right ventricular heave, and a loud systolic murmur over the precordium and back.Owing to the increase in the cardiac output and decrease in the peripheral resistanceduring pregnancy, the right to left shunt increases, which increases the cyanosis,desaturation, and polycythemia. These women tolerate severe hemorrhage dueto any cause rather poorly, because it reduces systemic blood pressure and furtheraggravates the right to left shunt. Any hemorrhage must be treated energetically,preventing hypotension. Antibiotic prophylaxis must be used in all high risk situationsto prevent infective endocarditis. The incidence of spontaneous abortions and pretermdeliveries is increased. However there is acceleration of fetal lung maturationdue to chronic intrauterine hypoxia, and hence the incidence of development ofrespiratory distress syndrome is lower. Preterm labor is controlled withindomethacin. Nifedipine is not used because it can cause hypotension.Fetal growth can be retarded due to chronic hypoxia. There is an increase in therisk of fetal cardiac malformations. These are diagnosed by fetal echocardiography.The woman is allowed to go into spontaneous labor at term. Management inlabor and puerperium is as for rheumatic cardiac disease. Postpartum hemorrhageis prevented if possible. If it does occur, it is treated energetically.

B. Ebstein's anomaly, a rare condition, is the second commonest cyanoticheart disease during pregnancy. It consists of a downward displacement ofthe tricuspid valve, tricuspid insufficiency, and atrial septal defect. Its clinicalfeatures include cyanosis, prominence of the jugular venous pulsations witha large V-wave, murmur of tricuspid regurgitation, and an early diastolic sounddue to the motion of the displaced tricuspid leaflet. These women do not toleratesevere hemorrhage well, which has to be treated energetically. Frequently there iscoexistence of an accessory bypass tract in the heart (Wolff-Parkinson-White

syndrome or WPW syndrome) and atrioventricular conduction defects. Theseconditions are treated appropriately. Ebstein's anomaly is sometimes hereditary, andthe manifestations in the fetus are more severe than in the mother. Fetalechocardiography confirms the presence of the condition in the fetus. Induction oflabor is avoided except in fetal interest, as in the case of intrauterine fetal growthrestriction (IUGR) with compromise of its well being. Management during labor andpuerperium is as for rheumatic heart disease. Surgical correction of the condition isdone after the puerperium.

C. Eisenmenger's syndrome is reversal of the left to right shunt in a case ofatrial septal defect, ventricular septal defect, or patent ductus arteriosus. It occursdue to the development of pulmonary hypertension. The shunt gets aggravateddue to a fall in the systemic vascular resistance. It worsens with severe hemorrhagedue to any cause, which needs to be treated energetically. If the condition is diagnosedin the first trimester, the patient is advised to undergo a medical termination ofpregnancy (MTP) because maternal mortality with an MTP is only 7%, while thatwith continuation of pregnancy is 30%. If she accepts, MTP is done by rapid cervicaldilatation and suction evacuation under local anesthesia. If she refuses, she isgiven antenatal care. Preeclampsia causes an increase in the pulmonary vascularresistance, decrease in cardiac output, and hence a decrease in the pulmonary bloodflow. The hypertension needs to be treated well to prevent deterioration of thepatient. There is a risk of thromboembolism, for which the patient is offered prophylacticanticoagulation with subcutaneous heparin. Other antenatal complications like IUGRand infective endocarditis are managed appropriately. Spontaneous labor is awaitedat term. If there is deterioration, oxygen and alpha-sympathomimetic amines areadministered.

D. Tricuspid atresia, pulmonary atresia, and transposition of great vessels aretreated surgically in infancy. A pregnancy may be allowed depending on the degreeof residual cyanosis and adequacy of pulmonary blood flow. Individualized treatmentis given if the woman is already pregnant.

161

Page 168: Differential Diagnosis and Management Options inuploads.worldlibrary.org/uploads/pdf/20180107071525differential... · 83 Peripartum cardiomyopathy 165 84 Hypertension 167 85 Convulsions

CYANOTIC HEART DISEASE

Clinical examinationEchocardiography

Tetralogy of Fallot (A) Ebstein’s anomaly (B) Eisenmenger’s syndrome (C) Other (D)

Watch for development of complications Check trimester

Hemorrhage Abortion Preterm labor IUGR First Second/third

Endocarditis Fetal cardiacmalformations

Advise MTP

Labor at term Watch for development of antenatal complications

Accepts Declines Watch for antenatalcomplications

Hemorrhage Infectiveendocarditis

WPW syndrome Ebstein’s anomalyin fetus

Rapid dilatation andsuction evacuation

Offer prophylactic anticoagulation

AV conductiondefects

IUGR Accepts Refuses

Labor at term Labor at term

162

Page 169: Differential Diagnosis and Management Options inuploads.worldlibrary.org/uploads/pdf/20180107071525differential... · 83 Peripartum cardiomyopathy 165 84 Hypertension 167 85 Convulsions

82. Coarctation of AortaA. Most of the cases of severe coarctation are diagnosed and treated surgically ininfancy. Symptomatic coarctations are frequently associated with congenital heartdefects, particularly of the aortic valve. These defects are also treated surgically, sothat a girl is normal when she enters the reproductive age. She is reassessed beforeshe becomes pregnant. If the correction of her coarctation is maintained, a pregnancyis permitted. If there is a recurrence of the coarctation, she is treated by percutaneoustransluminal angioplasty (balloon dilatation).

B. If coarctation of the aorta is diagnosed for the first time in adult life when thewoman is not pregnant, it is treated surgically. A pregnancy is permitted after sixmonths if the woman is hemodynamically stable and her cardiovascular function isnormal.

C. If a coarctation of the aorta is diagnosed during the first trimester, the risk ofcontinuing the pregnancy is explained to the woman. If she is willing, the pregnancy isterminated by rapid cervical dilatation and suction evacuation under local anesthesia.The coarctation is corrected by elective surgery more than 3 weeks after the abortion.If she desires to continue the pregnancy, she is treated the same way as a womandiagnosed to have the condition beyond the first trimester.

D. Antenatal care is offered to these women. They are examined for the presence ofupper limb hypertension. If present, aggressive antihypertensive therapy is given.Chronic hypertension during pregnancy is discussed elsewhere. The ascending aortais assessed for dilatation by 2-D echocardiography. If it shows gross widening, it issuggestive of intrinsic disease in the aorta. Such a woman is observed closely for

features of development of dissection of the aorta. If dissection does not develop, sheis treated by an elective lower segment cesarean section at 37 weeks of gestationbecause she is at a great risk of dissection of the aorta during labor.

E. The risk of dissection of the aorta is increased during pregnancy due tohyperdynamic circulation of pregnancy and degeneration of the media of the aortadue to hormonal influences of the pregnancy. An intimal tear develops and blooddissects into the media. In type A (DeBakey type I and II) the ascending aorta isinvolved, while in type B (DeBakey type III) the descending aorta and abdominalaorta are involved. With the development of dissection the patient experiencesstabbing, cutting, or tearing type of pain in the chest. Hypotension or cardiac arrestmay occur. About 25% of the cases develop stroke, visceral infarction, renalinfarction, or limb ischemia. A cardiogram is usually normal. A chest radiograph isobtained with shielding of the abdomen. It shows mediastinal widening,cardiomegaly, disparity in the size of the ascending and decending aorta, anddisplacement of the intimal calcification. Aortography is diagnostic in 95% of thecases, but causes fetal irradiation. CT and MRI are diagnostic in 90% of the cases.Transesophageal echocardiography can provide a rapid diagnosis of the dissection ofthe aorta. After stabilization of the blood pressure, type A dissection is treated bysurgical repair of the aorta with placement of a prosthetic graft. The aortic valve isresuspended, repaired, or replaced as required. Type B dissection is treated medicallyunless it ruptures, there is persistent pain, hypertension is resistant to medicaltreatment, or ischemia or infarction of the kidneys, viscera, or a limb is present. Suchcases are treated by surgical repair. If aortic dissection does not develop, labor isallowed at term and delivery is achieved by a low forceps operation.

163

Page 170: Differential Diagnosis and Management Options inuploads.worldlibrary.org/uploads/pdf/20180107071525differential... · 83 Peripartum cardiomyopathy 165 84 Hypertension 167 85 Convulsions

COARCTATION OF AORTA

Diagnosed in infancy Diagnosed later in life

Surgical correction Check for pregnancy

Reassess in adult life Not pregnant Pregnant

Correction maintained Recurrence Surgical treatment Check trimester

Allow a pregnancy Percutaneous transluminal angioplasty First Second/third

Counseling, Check desire

MTP Continuation of pregnancy

MTP under local anesthesia Check for upper limb hypertension

Surgical correction Present Absent

Antihypertensive treatment Assess degree of widening of ascending aorta (D)

Gross Mild/nil

Elective LSCS at 37 weeks Watch for aortic dissection

Dissection develops Dissection does not develop

Surgical repair Labor at term, low forceps delivery

164

Page 171: Differential Diagnosis and Management Options inuploads.worldlibrary.org/uploads/pdf/20180107071525differential... · 83 Peripartum cardiomyopathy 165 84 Hypertension 167 85 Convulsions

83. Suspicion of PeripartumCardiomyopathy

Peripartum cardiomyopathy (PPCM) refers to unexplained heart failureduring pregnancy following a exclusion of known causes. Its incidence varies fromapproximately 1:15000 pregnancies in the United States to 1:299 in Haiti and 1:100in a small region of sub-Saharan Africa. PPCM usually occurs in the third trimester,and even more commonly in the puerperium. Its mortalities range from 18% to 56%.Even when the woman survives, she may not recover her myocardial function fully(70% cases) and may require chronic therapy for cardiac failure or may need acardiac transplantation.

A. Causes of or factors aggravating congestive cardiac failure like heart disease(valvular, rheumatic, or coronary), anemia, chronic hypertension (chronic,gestational, preeclampsia), thyrotoxicosis, diabetes, dislipidemia, sexuallytransmitted diseases, arrhythmias, current or past alcohol or drug use, collagenvascular disease, sleep disorders, prior chemotherapy or mediastinal radiation, andfamily history of cardiomyopathy or sudden death are looed for. Any condition foundis treated appropriately.

B. The patient has dyspnea, orthopnea, cough, palpitations, and chest pain. Theremay be paroxysmal nocturnal dyspnea, nocturnal cough, new regurgitant murmurs,pulmonary crackles, increased jugular venous pressure, or hepatomegaly. Diagnosticcriteria describe by The National Heart, Lung, and Blood Institute are as follows.

1. Development of heart failure in the last month of pregnancy, or within 5months of delivery;

2. Absence of an identifiable cause for the heart failure;3. Absence of recognizable heart disease prior to the last months of pregnancy;4. Left ventricular systolic dysfunction demonstrated by typical

echocardiographic findings, such as depressed shortening fraction (e.g.,ejection fraction less than 45%, M-mode fractional shortening less than 30%,or both, and an LV end-diastolic dimension of more than 2.7 cm/m2).

Investigations should include hemogram, urinalysis, serum electrolytes, fastingglucose, hemoglobin A1C, TSH, liver function tests, lipid profile, 12-leadelectrocardiogram, posterior-anterior and lateral chest radiographs, 2Dechocardiography, and MRI. MRI permits assessment of morphology and function,and displays myocardial fibrosis.

C. PPCM may be caused by viral myocarditis (including HIV), abnormal immuneresponse to pregnancy, increased apoptosis of myocytes, stress (by activation ofcytokines), inadequate response to the hemodynamic stresses of pregnancy,hyperprolactinemia, malnutrition, genetic predisposition, myocardial ischemia,increased adrenergic tone, prolonged tocolysis (terbutaline, ritodrine, salbutamol,isoxsuprine, and magnesium sulfate), and abuse of alcohol or cocaine, rheumatologic

disease, amyloidosis, and pheochromocytoma.

D. Therapy consists of treatment for congestive cardiac failure. Sodium and fluidintake is restricted. Diuretics are given to reduce preload. Blood pressure iscontrolled to reduce afterload, using hydralazine or another vasodilator. Angiotensin-converting enzyme inhibitors are avoided because of marked fetal adverse effects.They can be given postpartum, weighing the risk-benefit ratio. β-blockers (sustained-release metoprolol succinate, carvedilol, and bisoprolol) reduce mortality and arerecommended for all stable patients except when they are contraindicated.Metoprolol is considered safe during breast feeding, but the exposed neonates shouldbe watched for its adverse effects like bradycardia, hypoglycemia, and growthrestriction. Digoxin is given except when complex arrhythmias are present, keepingserum levels between 1 and 1.2 ng/dL or less. Heparin is recommended to preventthromboembolism. Oxygen is administered to relieve hypoxemia. Drugs known toaffect cardiac failure adversely (nonsteroidal anti-inflammatory agents, manyantiarrhythmic drugs, and nondihydropyridine calcium channel blockers) should beavoided whenever possible. When the ejection fraction is 35% or less on optimaltherapy and the expected survival is more than 1 year, implantable cardioverterdefibrillator therapy is useful for primary prevention of sudden cardiac death. If thepatient has widened QRS on ECG and class 3 or 4 symptoms despite optimalmedical therapy, cardiac resynchronization therapy is recommended. LV-assistdevices and transplantation can be considered for the most critical patients. Antenatalcare includes standard care plus serial clinical assessment of blood pressure, heartrate, weight, volume status, the ability to perform routine activities of daily living,serum electrolytes, and renal function. Ejection fraction and structural heart changesshould be assessed again at intervals, in patients who have had a change in clinicalstatus, and before delivery.

E. Spontaneous labor at term is permitted if the patient is stable and wellcompensated. Induction of labor before term is required if the patient’s conditionremains unstable despite treatment. Prostaglandins are safe for cervical ripening,Oxytocin is administered as a constant infusion of concentrated solution to preventvolume overload.. An early epidural analgesia helps to minimize sympathetic output.Delivery is done with the use of low forceps or a vacuum extraction. Cesareansection is done only for obstetric indications. Invasive monitoring is done if volumestatus is problematic. Sequential compression devices are used forthromboprophylaxis.

F. Follow up is 1 week after delivery to assess her cardiovascular status and adjustdosage of medications. Prognosis is good for women who regain normal ventricularfunction within 6 months. Women with persistent ventricular dysfunction developcongestive heart failure during a subsequent pregnancy. Women with apparentresolution of cardiomyopathy develop cardiac failure in 20% cases, though it isusually less severe. A woman with PPCM is counseled not to become pregnant untilher ventricular ejection fraction increases to above 50%, and at least 1 year haspassed after the illness to permit remodeling of cardiac muscle.

165

Page 172: Differential Diagnosis and Management Options inuploads.worldlibrary.org/uploads/pdf/20180107071525differential... · 83 Peripartum cardiomyopathy 165 84 Hypertension 167 85 Convulsions

SUSPICION OF PERIPARTUM CARDIOMYOPATHY

Check for other causes of congestive cardiac failure (A)

Present Absent

Appropriate treatment Clinical features, investigations (B)

Peripartum cardiomyopathy

Check for predisposing factors (C)

Present Absent

Appropriatetreatment

Antenatal care (D)

Unstable, decompensated Stable, compensated

Induction of labor(E)

Labor at term (E)

Follow up (F)

166

Page 173: Differential Diagnosis and Management Options inuploads.worldlibrary.org/uploads/pdf/20180107071525differential... · 83 Peripartum cardiomyopathy 165 84 Hypertension 167 85 Convulsions

84. HypertensionA. If the blood pressure (BP) of a gravida has not been checked prior to thepregnancy or during early pregnancy, one has to differentiate between pregnancyinduced hypertension (PIH) and chronic hypertension due to a medical cause. PIHis a disease usually of the third trimester. It does not develop prior to 20 completedweeks of pregnancy, except with a vesicular mole or multiple pregnancy. See chapter65 for the diagnosis of these two conditions.

B. PIH is diagnosed when the BP is > 140/90, with proteinuria, with or without edema.If the gravida has hypertension that develops in the third trimester, without edemaor proteinuria, it is gestational hypertension. It is managed like PIH. Chronic hypertensionis managed with antihypertensive agents (table 84.1). The patient is given prophylaxisfor superadded PIH as follows.

1. Low dose aspirin : 50-75 mg PO qd.2. Calcium : 1.2 g PO qd. Its value has not been proved.3. Rest in left lateral position for 2 hours in the afternoon and 8 hours at night.

Complications of chronic hypertension are managed appropriately. The fetus maydevelop restriction of growth (see chapter 64). The cause of chronic hypertension ismanaged appropriately.

Causes of Chronic Hypertension

Essential hypertension Renal vascular hypertensionGlomerulonephritis Nephrotic syndromePyelonephritis Lupus nephritisPeriarteritis nodosa SclerodermaDiabetic nephropathy Polycystic kidney diseaseCoarctation of aorta PheochromocytomaPrimary aldosteronism

Table 84.1 Drag Therapy of Chronic Hypertension

Drug DoseBeta-adrenergic antagonists:

Propranolol 80 mg PO q12hAtenolol 50-100 mg PO qdOxprenolol 80 mg PO q12hVasodilators:

calcium antagonistsNifedipine 20 mg PO q12hDiltiazem 60 mg PO q8hVerapramil 40-80 mg PO q8hAmlodipine 5-10 mg PO qdVasodilators:

alpha-1-adrenergic antagonistsPrazosin 5-20 mg PO qdLabetalol 300-600 mg PO qdVasodilators:

ACE inhibitors

Captopril 50-150 mg PO qdLisinopril 10-20 mg PO qdEnalapril 10-20 mg PO qdVasodilators:

vascular actionHydralazine 25-50 mg PO q8hMinoxidil 10-50 mg PO qdVasodilators:

centrally acting drugsMethyl dopa 250-500 mg PO q8hClonidine 0.1-0.3 mg PO qd

C. PIH may be mild or severe (table 84.2)

Table 84.2 Severity of PIH

Parameter MM Severe

Diastolic BP (mmHg) < 110 > 110Proteinuria < + 1 > + 2Headache Absent PresentVisual disturbances Absent PresentEpigastric pain Absent PresentOliguria Absent PresentConvulsions Absent Present (in eclampsia)Pulmonary edema Absent PresentThrombocytopenia Absent PresentHyperbilirubinemia Absent PresentElevation of liverenzymes

Absent Present

(Note All parameters listed under severe disease are not necessarily present in a givenpatient.)

D. A patient with severe PIH is treated just like a patient with eclampsia.1. Magnesium sulfate (Pritchard's regimen): 20 ml 20% solution IV over 5 min,

monitoring pulse rate, respiratory rate and tendon jerks, followed by 20 ml 50%solution IM 10 ml into each buttock. If convulsions occur 15 min after this, 10ml 20% IV slowly. Maintenance dose is 10 ml 50% solution deep IM q4h,provided respiratory rate is > 16/min, urine output is > 60 ml/2h and tendonjerks are present. Treatment is continued for 48 hours in severe PIH. If severePIH recurs on stopping the treatment, the regimen is started again and labor isinduced. In case of eclampsia, labor is induced after stabilizing the patient, andadministration of drug is continued for 24 hours after delivery in severe PIH aswell as eclampsia.

2. Hydralazine: 5 mg IV over 5 min, to be repeated in a dose of 5-10 mg IV ql5-20min, so as to maintain diastolic BP at 96 mm Hg.

3. Nifedipine: in case parenteral hydralazine preparation is not available, nifedipineis given 5 mg SL, to be repeated as required.

E. Patient's BP is controlled with a combination of alpha methyldopa, hydralazine,labetalol, etc., as required. The pregnancy is continued monitoring fetal well being(see chapter 144) until the fetal lungs mature, the fetus is compromised, BP startsrising despite treatment, or any complication of PIH develops. Then the patient isdelivered by induction of labor, or by LSCS as appropriate.

167

Page 174: Differential Diagnosis and Management Options inuploads.worldlibrary.org/uploads/pdf/20180107071525differential... · 83 Peripartum cardiomyopathy 165 84 Hypertension 167 85 Convulsions

HYPERTENSION

Assess gestational age (A)

< 20 weeks > 20 weeks

Assess for multiple pregnancy and vesicular mole

Absent Vesicular mole(See chapter 65)

Multiple pregnancy(See chapter 97)

Assess for proteinuria and edema (B)

Medical cause of hypertension Proteinuria with/without edema Both absent

PIH Medical causeof hypertension

Assess severity (C)

Mild Severe (D)

Bed rest MgSO4, antihypertensive drugs

Improvement Deterioration Improvement Failure to improve

Ambulation Deterioration Assess fetalmaturity

Condition remainsstable

Immature Mature Induction of labor

Close follow up PIH remains controlled Stop MgSO4 PIH recurs Restart MgSO4

Induction of labor

Assess fetal growth and well being serially Induction of labor at 37 weeks or if fetus gets compromised,whichever is earlier

168

Page 175: Differential Diagnosis and Management Options inuploads.worldlibrary.org/uploads/pdf/20180107071525differential... · 83 Peripartum cardiomyopathy 165 84 Hypertension 167 85 Convulsions

85. ConvulsionsA. What a patient's relatives call a fit may not be convulsions, but syncope, hysteria,or effect of hyperventilation. A careful history helps differentiate true convulsionsfrom pseudoseizures. The latter condition is managed appropriately.

B. Epilepsy is a chronic disorder, found in 0.3-0.5% of pregnant women. Pregnancyaggravates the disease in 45% cases, ameliorates it in 5% cases, while there isno effect on 50% cases. The diagnosis is confirmed by EEG. Management duringa convulsion is as follows.

5.Tongue pad to prevent tongue bite.6.Left lateral position to prevent aspiration.7.Suction of mouth and pharynx after the convulsion to prevent aspiration of

accumulated secretions.8.Oxygen for hypoxia.9.Respiratory support as required.10. Anticonvulsant therapy

a. Phenytoin: 18-20 mg/kg IV at a rate not exceeding 50 mg/min, monitoringBP and cardiac rhythm, followed by maintenance dose of 4-8 mg/kg PO/IV qd.

b. Phenobarbitone: it is an alternative for first-line treatment of status epilepticus,or when phenytoin fails to control convulsions. The dose is 10 mg/kg IVat a rate of 100 mg/min. Maintenance dose is 1-5 mg/kg PO qd.

c. Diazepam: it is used to control prolonged generalized convulsionsor frequent, repetitive convulsions. The dose is 5-10 mg IV at a rate< 1-2 mg/min. It is not useful for maintenance, which is achieved witha long-acting drug.

d. Paraldehyde: it is used for status epilepticus not responding to phenytoinor phenobarbitone. The dose is 3-5 ml deep IM.

e. Pentobarbital: it is used when the preceding drugs fail. Loading doseis 15 mg/kg IV, and maintenance dose is 1 mg/kg/h. Endotrachealintubation and supervision by an anesthetist are necessary.

Control of seizure is better with older anticonvulsant drugs like carbamazepine,

phenytoin and valproic acid, compared with newer anticonvulsant drugs likelamotrigine, evetiracetam, and topiramate. Levetiracetam is associated with fewerseizure-affected pregnancies than lamotrigine. Valproate should be avoided inpregnancy due to its teratogenic action and adverse neurodevelopmental outcomes.Valproate and carbamazepine should be avoided with a family history of neural tubedefects in the offspring. Folic acid supplements should be given to all gravidas.Enzyme inducing antiepileptic drugs can lower fetal vitamin K levels, and increaserisk of perinatal hemorrhage by causing deficiency of coagulation factors II, VII, IX,and X. Vitamin K1 10-20 mg PO qd should be given to the gravida in the last monthof pregnancy and 1 mg IM to the infant at birth.

Table 85.1 Congenital Malformations Due to Anticonvulsants

Drug Malformations

Phenytoin Fetal hydantoin syndrome : microcephaly, facial clefts, facialdysmorphism, limb malformations, nail hypoplasia, distal phalangealhypoplasia.

Phenobarbitone Cleft lip, cleft palate, cardiac malformations.Trimethadione Trimethadione syndrome : IUGR, craniofacial, limb, and cardiac

malformations, mental retardation.Valproic acid Face-heart-limb syndrome, lumbosacral spina bifida.Carbamazepine Syndrome like fetal hydantoin syndrome.

C. Eclampsia is almost completely preventable with good antenatal care. However, itremains the most common cause of convulsions in the third trimester, due toinadequate antenatal care in the developing countries. It is diagnosed when theconvulsing patient has features of PIH. It is discussed in chapter 84.

D. A very careful neurologic assessment is necessary in all cases, because thecause of the convulsions could be a serious disease. Besides the drug treatmentto control convulsions, specific therapy of the underlying cause is required toreduce maternal morbidity and mortality.

169

Page 176: Differential Diagnosis and Management Options inuploads.worldlibrary.org/uploads/pdf/20180107071525differential... · 83 Peripartum cardiomyopathy 165 84 Hypertension 167 85 Convulsions

CONVULSIONS

Assess nature (A)

True convulsions Pseudoconvulsions

Assess past history of convulsions (B) Reassurance andsupport

Present Absent

Epilepsy Check gestational age

First two trimesters Third trimester (C)

Check blood pressure, proteinuria, edema

Neurological and medical assessment (D) No PIH Eclampsia

Meningitis Epilepsy Cerebrovascularaccident

Metabolicdisorder

Head injury Hypertensiveencephalopathy

Intracranialtumor

Electrolyte imbalance Cerebral sinus thrombosis Drug withdrawal

170

Page 177: Differential Diagnosis and Management Options inuploads.worldlibrary.org/uploads/pdf/20180107071525differential... · 83 Peripartum cardiomyopathy 165 84 Hypertension 167 85 Convulsions

86. Asymptomatic BacteriuriaUrine culture is said to be positive if a midstream clean-catch specimen

shows > 105 cfu/ml of ≥1 bacterial species or a catheterized specimen shows > 103

cfu/ml of ≥1 bacterial species. Asymptomatic bacteriuria is presence of persistent,actively multiplying bacteria within the urinary tract without symptoms. Itsincidence varies from 2 to 7%, depending on race, parity, and socioeconomic status.If not treated, it may lead to the following complications.

11. Acute symptomatic urinary infection (25% cases): cystitis, acutepyelonephritis.

12. Preterm labor: possible. .13. Intrauterine growth restriction: possible.14. Persistence after delivery, and association with any of the following.

a. Chronic renal infection.b. Chronic obstructive lesions of the urinary tract.c. Congenital urinary abnormalities.

If an initial culture is negative, < 1.5% develop urinary infectionsubsequently in pregnancy.

A. A screening test at first visit is cost-effective if the incidence is high in thatpopulation. A colony count above 105/ml is diagnostic. If it is 104-105, the test isrepeated. If it is < 105/ml, the test is repeated every month. When > 105/ml, anunderlying cause is sought by renal ultrasonography, renal function tests, and fastingblood sugar assay. Any underlying cause is treated as appropriate. The bacteriuria istreated with appropriate antimicrobial agent for 10 days, based on the antimicrobialsensitivity tests.

B. After completion of a course of an antimicrobial agent, urine culture is obtainedafter a week. If it shows significant growth of bacteria, an alternative antimicrobial isadministered. If the culture is negative, it is repeated every month up to term.

Recurrence rate for all regimes is about 30%. A recurrence is treated as before. Aftercure, the patient is given nitrofurantoin prophylactically, in a dose of 200 mg/d up toterm. More recently, single-dose antimicrobial therapy is used with up to 80%success rate.

C. If the count is < 104/ml, and there is pyuria, the patient is evaluated for urinarytract tuberculosis. If absent, she is evaluated for renal parenchymal and urinarycollecting system abnormalities with an excretory urography after puerperium.

Table 86.1 Drug Therapy of Asymptomatic Bacteriuria

Drug Dosage

Cotrimoxazole 160/800 mg ql2hNitrofurantoin 100 mg q6hAmoxicillin 500 mg q8hAmoxicillin + clavulanate 500 mg q8hAmpicillin 500 mg q6hCephalexin 500 mg q6hNorfloxacin 200 mg q8h

Table 86.2 Single Dose Antimicrobial Therapy

Drug Dose

Amoxicillin 3 gAmoxicillin,with probenicid

2 g1 g

Sulfisoxazole 2 gCephalexin 3 gCephalexin,with probenicid

2 g

Cotrimoxazole 320/1600 mg

171

Page 178: Differential Diagnosis and Management Options inuploads.worldlibrary.org/uploads/pdf/20180107071525differential... · 83 Peripartum cardiomyopathy 165 84 Hypertension 167 85 Convulsions

ASYMPTOMATIC BACTERIURIA

Urine culture, colony count, antibiotic sensitivity (A)

> 105/ml 104-105/ml < 104/ml

Pyuria (C)

Repeat culture Present Absent

> 105/ml < 105/ml Evaluate for urinarytract tuberculosis

Continue prenatalcare

UltrasonographyRenal function testsFasting blood sugar

Repeat every month

Antibiotic therapy based on sensitivity report

Repeat culture (B)

> 105/ml < 105/ml

Alternative antibiotic therapy Repeat culture every monthAssess for IUGR

172

Page 179: Differential Diagnosis and Management Options inuploads.worldlibrary.org/uploads/pdf/20180107071525differential... · 83 Peripartum cardiomyopathy 165 84 Hypertension 167 85 Convulsions

87. Chronic Renal DiseaseA. Renal function may be normal with chronic renal disease, or impaired to avariable degree. A list of various chronic renal disorders follows.

1. Chronic glomerulonephritis.2. Membranous nephropathy.3. Proliferative glomerulonephritis.4. Diabetic glomerulosclerosis.5. Systemic lupus erythematosus.6. Amyloidosis.7. Chronic interstitial nephritis.8. Chronic pyelonephritis.9. Arterial nephrosclerosis.

The disease may manifest with hypertension, proteinuria, or abnormalurinary sediment. With passage of time, there is progressive decrease in glomerularfiltration rate and development of uremia. Even if renal function is normal, thepregnancy outcome may still be abnormal. Fetal loss rate is up to 15% even withoutimpairment of renal function, early or severe hypertension, or severe proteinuria.

B. The results of renal function tests may be altered in pregnancy.

1. Blood urea nitrogen and serum creatinine levels are lowered in pregnancy.Unless prerenal factors are present, if serum creatinine is persistently above 0.9mg/dL (75 μmol/L), the presence of intrinsic renal disease is suspected.

2. Creatinine clearance is very useful to assess renal function, provided thatcomplete urine collection is done over several hours.

3. Urine concentration tests are not very useful. A pregnant woman tends toaccumulate water during the day. This fluid is mobilized at night, and is excreted,producing dilute urine. Urine may not be concentrated even after withholdingfluids for 18 hours.

4. Urinalysis interpretation is essentially as in the nonpregnant state, with a fewvariations. Glycosuria occurs in 15% women due to lowered renal threshold.

Proteinuria in the nonpregnant state, the first trimester, and the second and thirdtrimesters is 100 mg/d, 150 mg/d, and 200 mg/d respectively. It should be > 500mg/d to be considered abnormal. If contamination during urine collection is ruledout, red cells in urine indicate the presence of a urinary tract disease.

5. Ultrasonography is done to assess renal size, architecture, and presence ofoutflow tract obstruction.

6. Excretory urography is rarely required. One or two radiographs are obtained ifnecessary.

C. A pregnancy is permitted only if the following criteria are satisfied.

1. Serum creatinine < 250 μmol/L.2. Absence of significant hypertension.

If these criteria are not satisfied the pregnancy is terminated. If they aresatisfied, the pregnancy is continued with a close watch on maternal renal functionand fetal well being (see chapter 144). If renal function is normal and the bloodpressure is not elevated, the woman is given low dose aspirin therapy (50-75 mg POqd) for prevention of pregnancy induced hypertension, and the pregnancy iscontinued, monitoring fetal well being periodically. If the blood pressure is elevated,it is managed as discussed in chapter 84.

In case of a woman with renal transplant, besides other factors mentionedunder chronic renal disease, the dose of prednisone should be < 15 mg PO qd, andthat of azathioprine < 2 mg PO qd.

If pregnancy is continued, renal function tests and tests for fetal well beingare done serially. If the kidney disease is due to diabetes mellitus, maternal bloodsugar is controlled meticulously. ACE inhibitors or angiotensin receptor blockers(ARB) are used for controlling hypertension. In nondiabetic women withhypertension, if the proteinuria is not more than 0.5 g/d, any antihypertensive agentsmay be used. If it is more than 0.5 g/d, ACE inhibitors or ARBs are preferred forcontrolling hypertension, though non-dihydropyridine calcium channel blockers canalso be used. Supportive treatment is continued until after delivery, in jointconsultation with a nephrologists.

173

Page 180: Differential Diagnosis and Management Options inuploads.worldlibrary.org/uploads/pdf/20180107071525differential... · 83 Peripartum cardiomyopathy 165 84 Hypertension 167 85 Convulsions

CHRONIC RENAL DISEASE

Assess nature of disease (A)

UrinalysisRenal function tests (B)

Normal Impaired (C)

Check blood pressure Assess threat to maternal life

Normal Elevated Low High

Assess fetal well being See chapter 84 Antenatal careSerial renal function tests

Serial assessment of fetal well being

Terminate pregnancy

See chapter 144 Check type of kidney disease

Diabetic Nondiabetic

Control blood sugarsACE inhibitors or ARBs for controlling hypertension

Check blood pressure

Normal High

Check degree of proteinuriaACE inhibitors or Angiotensin receptor blockers preferredNon-dihydropyridine Calcium Channel Blocker > 0.5 g/d < 0.5 g/d

Supportive treatment Any antihypertensive

174

Page 181: Differential Diagnosis and Management Options inuploads.worldlibrary.org/uploads/pdf/20180107071525differential... · 83 Peripartum cardiomyopathy 165 84 Hypertension 167 85 Convulsions

88. Jaundice in PregnancyA. A family and/or past history of jaundice, or history of ingestion of some drugprior to the onset of the jaundice, anemia and splenomegaly suggest hemolyticanemia. In cases of hepatocellular jaundice, the patient has flu syndrome, history ofcontact with jaundice, administration of drugs or blood transfusion, alcoholism orchronic liver damage. In cholestatic jaundice, there may be right hypochondrial pain,itching, dark urine, pale stools, hepatomegaly and palpable gall bladder.B. In all cases, urinalysis and liver function tests are performed. In hemolyticanemia, serum unconjugated bilirubin is markedly raised, liver enzymes are normal,and urinary urobilinogen is high. In hepatocellular jaundice serum conjugated bilirubinis raised, AST and ALT are markedly raised, alkaline phosphatase is raised, urinarybilirubin is raised and urobilinogen is markedly raised. In cases of cholestaticjaundice, serum conjugated bilirubin is markedly raised, AST and ALT are elevated,alkaline phosphatase is very high, and urinary bilirubin is very high.C. The cause of hemolytic anemia is found by performing a hemogram, peripheralsmear, sickle cell test, osmotic fragility test, and Hb-electrophoresis. From these, onecan distinguish between spherocytosis, Hb-S disease, Hb-SC disease andthalassemia. If the anemia is precipitated by tiredness, dehydration and depletion ofcalories, and is waxing and waning, the diagnosis is Gilbert syndrome. If the testresults are normal, tests are done for extrinsic factors causing hemolysis, and one ofthe following conditions is diagnosed: warm-antibody or cold-antibody autoimmunehemolytic anemia, drug/chemical-induced hemolytic anemia, mismatchedtransfusion, malaria, G6PD deficiency, autoantibodies. The condition is treatedjointly with a physician. See chapter 75 for management of anemia in pregnancy. .D. If the liver in hepatocellular jaundice is smaller than normal, with attenuationof echoes, it is probably acute fatty liver of pregnancy. Serum bilirubin is < 10 mg%(mainly conjugated), AST is between 300-500 U/L, PT and PTT are raised, plateletcount is 100000-150000/cmm, neutrophil count 13000-30000/ cmm, FDP are raised,antithrombin III is lowered, creatinine is raised, uric acid is quite high, and there isacidosis. A liver biopsy may be done when in doubt. There is panlobular effect withperiportal sparing showing hepatic steatosis with microdroplets of fat within the livercells, intrahepatic cholestasis is seen, including canalicular plugs of bile and an acutecholangiolitis, extramedullary hemopoiesis and hyperplastic collections of Kupffercells. Distinction from acuter viral hepatitis and alcoholic hepatitis might beimpossible, especially in mild cases, because several histopathological featuresoverlap, and cytoplasmic ballooning might mask microvesicular steatosis of acutefatty liver of pregnancy. Treatment is supportive measures, early delivery and freshfrozen plasma for obstetric hemorrhage due to coagulopathy.E. In case the liver is normal-sized or enlarged, evaluation is done with clinicalfeatures as well as liver function tests, viral markers, and coagulation studies. Inviral hepatitis, there are constitutional symptoms for 1-2 weeks, serum AST 400-4000 U/L, bilirubin 5-20 mg%, and is rising when the enzymes are falling. Usingvarious viral markers, one can distinguish between hepatitis A, B,C, D and E, CMV,HSV, and Epstein Barr virus hepatitis. The treatment is supportive and symptomatic.There is no indication for a medical termination of pregnancy, induction of labor, orcesarean section. Postpartum hemorrhage may occur due to hepatic dysfunction, and

is treated with fresh frozen plasma. Infants born to mothers who developed acutehepatitis A infection within 2 weeks of delivery should be given 0.5 ml immuneserum globulin IM. Hepatitis B, C, E can be transmitted to the fetus vertically,primarily through maternal blood during childbirth. Acute and chronic hepatitis Binfections of the infant are prevented by giving hepatitis B immune globulin 0.5 mlIM, and hepatitis B vaccine 0.5 ml (10μg) IM on day 0, and after 1 and 6 months. InHELLP syndrome, there are features of PIH. Serum AST and LDH are raised, PTand PTT are normal, and platelet count is 50000-150000/cmm. A liver biopsy isusually not required. There is a deposition of fibrin in the periportal regions andsinusoids (which is also seen in PIH without HELLP, some cases of viral hepatitis,or drug-induced liver damage) as an early feature. Distinction between it, acute fattyliver of pregnancy and PIH may only be possible by using immunofluorescenttechniques. Treatment is dexamethasone and early delivery. Hemorrhage due tothrombocytopenia and hepatic dysfunction is managed by platelet transfusion anduse of fresh frozen plasma respectively. In PIH, there are hypertension, proteinuria,serum bilirubin 2-4 mg%, AST 200-500 U/L, PT normal, and platelet count normal.Liver biopsy is very rarely required. It shows periportal hemorrhage, hepatocellularnecrosis, deposition of fibrin, thrombi in portal tract capillaries, and noinflammatory infiltrate. Early delivery is indicated. Hepatic involvement may occurin advanced cases of hyperemesis. Serum bilirubin and AST are raised, PT andplatelets are normal. Liver biopsy is normal and is not required for diagnosis. Seechapter 74 for management of hyperemesis gravidarum. In septicemia, there aretypical clinical features. Blood culture shows causative organisms. See chapter 69for management of septicemia. Leptospirosis may present with jaundice inpregnancy. The diagnosis is confirmed by sensitized RBC lysis test, triDot test.Treatment is given jointly with a physician. If the patient has microangiopathichemolytic anemia, renal failure, neurological disturbances, elevation of serumbilirubin, AST and low platelet count, the diagnosis is thrombotic thrombocytopenicpurpura or hemolytic uremic syndrome. The distinction between the two is difficult.Treatment is given in joint consultation with a physician.F. Ultrasonography is done in a case of obstructive jaundice. If dilated bile ducts areseen, the patient is referred to a surgeon for management. Investigations likepercutaneous cholangiography, endoscopic retrograde cholangiopancreaticographyare associated with the risk of fetal irradiation. If there are no dilated ducts onultrasonography, a liver biopsy is done to differentiate between hepatocellular jaundice(See under ‘D’) and intrahepatic cholestasis, including intrahepatic cholestasis ofpregnancy (IHCP). IHCP is characterized by generalized pruritus and mild jaundice,usually after 30 weeks of pregnancy, increasing in severity progressively and gettingrelieved in 48 hours after delivery (maximum 2 months). Serum bilirubin is < 8.4mg%, AST, ALT and alkaline phosphatase are raised not more than twice the normallevel. PT may be raised. Liver biopsy is usually not required. It shows acinarcholestasis, bile plugs in bile canaliculi, no hepatocellular necrosis, no inflammatoryinfiltrates. Preterm labor, fetal distress and intrauterine fetal death may occur. Fetalwell being is monitored closely. Pruritus is treated by administration of cholestyramine20g PO qd, or ursodiol 300 mg PO ql2h (or 10-15 mg/kg/d) after meals. Vitamin Kis given to prevent vitamin K deficiency due to both IHCP and cholestyramine. Thebaby is delivered when its well being is compromised. Postpartum hemorrhage dueto deficiency of vitamin K is managed by administration of fresh frozen plasma.

175

Page 182: Differential Diagnosis and Management Options inuploads.worldlibrary.org/uploads/pdf/20180107071525differential... · 83 Peripartum cardiomyopathy 165 84 Hypertension 167 85 Convulsions

JAUNDICE IN PREGNANCY

Check symptoms and signs (A)

Family history, drug history, anemia,splenomegaly.

Flu syndrome, contact with jaundice, infections, blood transfusions,drug history, alcoholism, features of chronic liver damage.

Right hypochondrial pain, itching, dark urine, palestools, hepatomegaly, palpable gall bladder.

Urinalysis, liver function tests (B) Urinalysis, liver function tests (B) Urinalysis, liver function tests (B)

Unconjugated bilirubin ↑↑, liver enzymesnormal, urinary urobilinogen ↑↑.

Conjugated bilirubin ↑, AST↑↑, ALT↑↑, AP↑, urinary bilirubin ↑.urobilinogen ↑↑.

Conjugated bilirubin ↑↑, AST↑, ALT↑, AP↑↑↑,urinary bilirubin ↑↑.

Hemolytic jaundice (C) Hepatocellular jaundice (D) Obstructive jaundice (F)

Peripheral smear, sickle cell test, osmotic fragility test, Hb electrophoresis. Ultrasonography

Spherocytosis Hb-S Hb-SC Thalassemia Ultrasonography No dilated ducts Dilated ducts

Precipitated by tiredness, dehydration, caloriedepletion, waxing and waning.

Normal Liver small, withattenuation of echoes

Liver normal sized orenlarged (E)

Liver biopsy ERCP, Percutaneouscholangiography,

Gilbert disease Test for extrinsic factors causing hemolysis LFT, hemogram LFT, coagulationstudies

Hepatocellular jaundice IHCP

Warm Abautoimmune HA

Cold Abautoimmune

HA

Drugs,chemicals

Mismatchedtransfusion

Acute fatty liver ofpregnancy

HELLP PIH

G6PD deficiency Malaria Hyperemesisgravidarum

SepticemiaPortal pyemia

Leptospirosis

Autoantibodies Viral hepatitis Thromboticthrombocytopenic purpura

Hemolytic uremicsyndrome

Check viral markers

IgM anti-HAV IgM anti-HBcAg IgM anti-HC IgM anti-Delta IgM anti-HE IgM anti-CMV IgM anti-HSV

Hepatitis A Hepatitis B Hepatitis C Hepatitis D Hepatitis E CMV hepatitisEpstein Barr

virusHepatitis

HSV hepatitis

176

Page 183: Differential Diagnosis and Management Options inuploads.worldlibrary.org/uploads/pdf/20180107071525differential... · 83 Peripartum cardiomyopathy 165 84 Hypertension 167 85 Convulsions

89. Pruritus In PregnancyPruritus from all causes occurs in about 15% gravidas. It is a disturbing

symptom. It may be due to conditions found in the nonpregnant state as well asconditions peculiar to pregnancy. The distinction is important, because the formerare cured by the same treatment as in the nonpregnant state, while the latter are givensymptomatic treatment and are cured after delivery.

A. If the gravida has pruritus all over, but there is no skin rash, the diagnosis ispruritus gravidarum. It is due to an alteration in the hepatic function. It is a mildvariant of recurrent cholestasis of pregnancy (See chapter 88). It usually begins inthe third trimester. Some cases have pruritus of only the abdomen. Serum bilirubin isnormal or slightly raised. Serum alkaline phosphatase and transaminases may beelevated. Mild cases are treated by application of calamine lotion. Severe cases aregiven cholestyramine 4 mg PO q8h and antihistamines. The condition resolves afterdelivery, but tends to recur in subsequent pregnancies and with the use ofcombination contraceptive pills.

B. Dermatoses are diagnosed depending on the nature of the skin rash. Those notpeculiar to pregnancy include scabies (see chapter 23), pediculosis corporis (seechapter 23), tenia (see chapter 23), urticaria, atopic dermatitis, and neurodermatitis.Urticaria is of abrupt onset. There are severely pruritic pink edematous papules andplaques with normal overlying epidermis, scattered randomly all over the bodyincluding the face and scalp. Edema of lips, hands and face is common. Old lesionsdisappear in less than 24 hours, while new ones appear. If the triggering factor canbe identified, it is removed. Cool baths or compresses give temporary relief.Adrenaline 1:1000, 0.3 to 0.5 ml subcutaneously gives fast relief in an acute episode.Antihistamines give temporary relief.

Adult atopic dermatitis is a chronic condition. There are lichenified lesionsin flexures or on face and hands. Flares and remissions occur. The skin is dry, flaky,and irritated easily. Palms are hyperlinear. Family history of the same or other atopicdiseases is usually present. Treatment is topical corticosteroids. Oral antibiotics aregiven for secondary infection. Antihistamines are an useful adjunct. Irritation of skinis prevented by adequate but not excessive bathing, application of an oil, cream orointment after a bath, avoiding unlined wool and polyester clothing, avoidingexcessive contact with grease, dirt, solvents and hot, humid climates.

The lesion of neurodermatitis is a thick, lined, well-circumscribed

lichenified patch on the nape and sides of the neck, arms, perineum, lateral andmedical sides of ankles and lower legs. It is the result of an unexplained itch-scratchcycle, worse in times of stress. It is a tension releasing mechanism. Its treatment istopical or intralesional corticosteroid. Occlusion with plastic film or steroidimpregnated tape is used if the former treatment fails.

C. Dermatoses specific to pregnancy include pemphigoid gestationis, polymorphiceruption of pregnancy, prurigo of pregnancy, and pruritic folliculitis of pregnancy.Pemphigoid gestationis can occur in pregnancy as well as gestational trophoblasticneoplasms. Initially there are pruritic erythematous urticated papules and plaques,target lesions, and annular weals. Vesicles develop in a few days and enlarge to formtense bullae. The lesions first appear in the periumbilical region and then spread tothe thighs, breasts, palms and soles. The bullous lesions usually resolve within amonth after delivery, while the urticated plaques may persist for up to one year. Mildcases are treated with antihistamines and topical fluorinated corticosteroids. Systemicsteroids are given for severe disease with bullae. Prednisolone 40 mg PO qd is the initialdose, which is reduced quite rapidly to 10 mg PO qd, and increased again afterdelivery to prevent a flare up in the puerperium. Plasmapheresis helps very severe cases.Dapsone is not used because it can cause hemolytic disease of the newborn.

Polymorphic eruption of pregnancy usually affects primigravidas in the last5 weeks of pregnancy. Pruritic urticarial eruption develops in and around abdominalstriae. Plaques, polycyclic weals, and vesicles up to 2 mm in diameter may develop,but bullae do not develop as in pemphigoid gestationis. The lesions may remainconfined to the abdomen with relative sparing of the umbilicus, or may spread to thebuttocks, shoulders and limbs. Treatment is antihistamines and mild topical steroids.Systemic steroids are given for severe disease. The lesions usually resolve in a fewweeks.

Prurigo of pregnancy is characterized by development of groups of pruriticexcoriated papules on the extensor surface of limbs and abdomen between 25 and 30weeks of pregnancy. The lesions may spread widely. The pruritus settles afterdelivery, while the papules may persist for months. The disease may recur insubsequent pregnancies. It is treated with antihistamines and topical corticosteroids.

Pruritic folliculitis begins after fourth month of pregnancy and resolves in 2weeks after delivery. There is an acneiform eruption with widespread follicularerythematous papules. It is treated with topical 10% benzoyl peroxide and 1%hydrocortisone.

177

Page 184: Differential Diagnosis and Management Options inuploads.worldlibrary.org/uploads/pdf/20180107071525differential... · 83 Peripartum cardiomyopathy 165 84 Hypertension 167 85 Convulsions

PRURITUS IN PREGNANCY

Check for skin rash (A)

Present Absent

Determine nature (B) Pruritusgravidarum

Dermatoses not specific to pregnancy Dermatoses specific to pregnancy (C)

Scabies Urticaria Tenia Pemphigoidgestationis

Polymorphiceruption ofpregnancy

Prurigo ofpregnancy

Pruriticfolliculitis ofpregnancy

Pediculosis Atopicdermatitis

Neurodermatitis

178

Page 185: Differential Diagnosis and Management Options inuploads.worldlibrary.org/uploads/pdf/20180107071525differential... · 83 Peripartum cardiomyopathy 165 84 Hypertension 167 85 Convulsions

90. Out-of-Wedlock PregnancyA. An out-of-wedlock pregnancy may occur in an unmarried girl, a widow,or a married woman. In case of a married woman, continuation or interruptionof the pregnancy is the woman’s option. The gynecologist is not ethically ormorally bound to disclose the fact to the woman’s husband or relatives that thepregnancy is a result of extramarital relationship. In fact, he is liable to getsued for breach of confidentiality. An unmarried girl may get pregnant due toany of the following.

1. Failure to use contraception.2. Failure of contraception.3. Breach of promise of marriage.4. Rape.5. Sexual promiscuity.6. Prostitution without adequate contraception.

Marriage is a possible option only in causes “1” and “2”. All others, andusually a pregnant widow do not have that option.,

B. Medical termination of pregnancy (MTP) is permitted by the MTP act onlyup to 20 completed weeks of pregnancy. See chapter 165 for various methods ofMTP.

C. The pregnancy has to be continued if diagnosed after 20 completed weeks ofpregnancy. Such women need to be explained the reason for continuation, andadvised against any attempts at MTP (criminal abortion) in view of high complicationrates. Antenatal care is given to them as to any gravida, with special attention topossible problems like anemia (see chapter 76), sexually transmitted diseases (in

cases of rape, sexual promiscuity, and prostitution), and preeclampsia (more inprimigravidas). They also need psychological support. Their families needcounseling as well, so as to reduce tensions, anxiety, and rejection behavior. Thesewomen should also be given health and sex education, and contraceptive advice.

D. These women need to be away from relatives and friends so that their pregnanciesremain secret. If possible, they should stay in homes maintained by social organizationsfor destitute women. If not, they can stay in hospitals. Other indications forhospitalization are medical, or when they go in labor.

E. Labor is managed strictly on obstetric principles, without any regard for the socialproblem. There should be no dilemma when a cesarean section is required. The fearthat the woman may hide the fact that she has had a cesarean section for a premaritalpregnancy and may suffer from a rupture of the scar during labor in future should notmake the obstetrician avoid a cesarean section. She can be told that she can tell herfuture obstetrician her obstetric history in confidence, and that she is protected bylaw from disclosure of such information.

F. Unless the woman wishes to rear the baby, it is sent for adoption. Lactation issuppressed using physical methods like no handling of breasts, tight breast support,and drugs like bromergocriptine, cabergoline or pyridoxine (in high doses).Rehabilitation of unmarried mothers is difficult. They are sometimes rejected bytheir families. Even otherwise, unless the social problem which led to pre- orextramarital sex is solved, the woman is not really helped. Help may be obtainedfrom the social workers of the hospital, and social organizations operative in the areaof residence of the woman. In case the woman is likely to continue her sexualactivity, she must be given adequate contraceptive advice, and also protection fromacquiring HIV infection and other sexually transmitted infections.

179

Page 186: Differential Diagnosis and Management Options inuploads.worldlibrary.org/uploads/pdf/20180107071525differential... · 83 Peripartum cardiomyopathy 165 84 Hypertension 167 85 Convulsions

OUT-OF-WEDLOCK PREGNANCY

Consider marriage (A)

Possible Not possible

MarriageAntenatal care

Assess gestational age

< 20 weeks (B) > 20 weeks

Medical termination of pregnancySee chapter 165)

CounselingAntenatal careHealth educationSex educationPsychological support

Hospital admission (D)

After 24 weeks for social reasons In labor, if kept at home for destitute women orher own home

Preeclampsia Anemia

Management of labor (E)

Normal delivery Operative delivery for obstetric reasons

Suppression of lactationAdoption of babyRehabilitationContraceptive advice

180

Page 187: Differential Diagnosis and Management Options inuploads.worldlibrary.org/uploads/pdf/20180107071525differential... · 83 Peripartum cardiomyopathy 165 84 Hypertension 167 85 Convulsions

91. Venous ThrombosisThe incidence of venous thromboembolism during pregnancy varies from

0.5 to 0.7 per 1000 deliveries, though one study reported an incidence of 7 per 1000deliveries. The left lower limb is affected more often than the right one. The calfveins and iliofemoral veins are affected most commonly.

A. Clinical features vary with the extent of the vascular occlusion, existing collateralcirculation, and the associated inflammatory response. The features are as follows,though they are not present in a large number of cases. The clinical diagnosisis further made difficult by the physiologic edema of pregnancy.

1. Pain in the affected limb.2. Swelling of the affected limb.3. Tenderness of the affected limb.4. Redness of and warmth over the affected limb.5. Engorgement and varicosities of the superficial veins.6. Pitting edema of the affected limb.7. Palpable cord in the limb.

B. Impedance plethysmography measures the changes in the electrical resistance inrelation to the venuos volume through two electrodes wrapped around the calf. It isvery accurate during the nonpregnant state. Its accuracy during pregnancy has notbeen tested adequately. Its interpretation may be compromised during pregnancybecause of compression of the inferior vena cava and iliac veins by the gravid uterus.This effect may be minimized by using the left lateral position during testing.

C. Real time B-mode ultrasonography is used to assess venous compressibility. It isvery accurate in detecting proximal deep vein thrombosis. Serial compressionultrasonography is a more recent version of the same and it yields somewhat betterresults. Duplex and color Doppler ultrasonography combine real time B-modeultrasonography with Doppler flow imaging using high resolution, high frequency (5to 10 MHz) transducers. Duplex ultrasound criterion for thrombosis is failure of thevascular lumen to collapse completely with gentle probe pressure. Though it ishighly accurate, its false positive rate is 30%. After the first trimester, the uterus hasto be displaced laterally to reduce the false positive rate.

D. Venography evaluates the entire venous system of the lower limb. It is morereliable than the noninvasive tests described before in differentiation of intraluminalfilling defect from external venous compression. Its side effects include chemicalphlebitis, swelling of leg, pain in the leg, necrosis of skin due to extravasation of

dye, and thrombosis of superficial veins. Fetal radiation is as shown in the followingtable.

Table 91.1 Fetal Absorbed Dose of Radiation

Procedure Fetal absorbed dose (Gy)

Unilateral venography without abdominal shield 0.00314Limited venography < 0.0005Angiography via femoral route 0.00221-0.00374Angiography via brachial route < 0.0005Perfusion lung scan using Tc99m, 0.00006-0.000121 to 2 mCi ventilation lung scan

Tc99m SCTc99m-DTPAXe133

0.00001-0.000050.00007-0.000350.00004-0.00019

Chest radiograph, posteroanterior view < 0.000001

E. Rapid and prolonged anticoagulation is required for preventing extension of thethrombus, restoring venous patency, and reducing the risk of pulmonary embolism.After an initial bolus of 5000 U intravenously, the protocol for unfractionatedheparin (UFH) therapy is as in the following table.

Table 91.2 Protocol for Heparin Therapy

APTT (sec) Repeat bolus New rate of infusion Repeat APTT assay

<50 5000U Add 2800 U/hr 6 hours50-59 - Add 2800 U/hr 6 hours60-85 - No change Next morning86-95 - Reduce by 1920 U/hr 6 hours96-120 - Stop for 30 minutes

Reduce by 1920 U/hr6 hours

> 120 - Stop for 30 minutesReduce by 3840 U/hr

6 hours

Low moleclar weight heparin (LMWH) and heparinoids (eg, danaparoid sodium) areat least as effective and safe as UFH. LMWHs have a longer plasma half-life and amore predictable dose response than UFHHence it is more suitable for once-daily,weight-adjusted subcutaneous dosing. Enoxaparin is given in a dose of 1 mg/kg SCq12h or 1.5 mg/kg SC qd (at same time each day).

181

Page 188: Differential Diagnosis and Management Options inuploads.worldlibrary.org/uploads/pdf/20180107071525differential... · 83 Peripartum cardiomyopathy 165 84 Hypertension 167 85 Convulsions

SUSPICION OF DEEP VEIN THROMBOSIS (A)

Impedance plethysmography (B)Ultrasonography (C)

Positive Equivocal Negative

Repeat testing Observation

Positive Negative

Abdominal shielded venography (D)

Positive Equivocal Negative

Unshielded venography

Positive Equivocal Negative

Anticoagulant therapy (E) Observation

182

Page 189: Differential Diagnosis and Management Options inuploads.worldlibrary.org/uploads/pdf/20180107071525differential... · 83 Peripartum cardiomyopathy 165 84 Hypertension 167 85 Convulsions

92. Pulmonary EmbolismThe incidence of pulmonary embolism (PE) during pregnancy is low. But

when it does occur, it tends to be massive. Increased coagulability of blood due tohormonal changes of pregnancy and venous stasis in the deep veins of the lowerlimbs due to increased pelvic blood flow and pressure of the gravid uterus on theiliac veins are responsible for increase in risk of deep vein thrombosis (DVT) andPE during pregnancy as compared to the nonpregnant state. Other predisposingfactors are congestive cardiac failure, prolonged bed rest, and obesity. The condition issuspected when a gravida develops dyspnea (75%), pain in chest (65%), hemoptysis(25%), and altered sensorium (20%). She has tachycardia (60%), tachypnea (40%),crepitations on auscultation of chest (40%), and fever (45%).

A. A chest radiograph is obtained with shielding of the abdomen. It may showdecrease in pulmonary vascularity. However it is most often nondiagnostic. If it isnormal, a ventilation-perfusion lung scan is obtained. If the scan is normal in the first48 hours, PE is excluded. If it is equivocal, pulmonary arteriography and pressuremeasurements are obtained. If the scan is positive, it is diagnostic of PE fn 90% ofthe cases. If a scan cannot be obtained, venography or MRI is done. If it showspelvic or femoral thrombosis, anticoagulation is started. An electrocardiogram isabnormal in 10 to 20% of the cases. It may show arrhythmias, ST-T changes, andenlargement of P wave. Arterial blood gases show low Pa02 in 90% of the cases.

B. If the gravida does not have cardiovascular collapse, she has stable hemodynamics.She is then given complete bed rest, elevation of legs, compression bandage on thelower limbs, and heparin for anticoagulation (see chapter 91). Thrombolytic therapyis given to patients who are extremely symptomatic, who have mild to moderatehemodynamic stability, who have large (more than two lobes) emboli, and in whomanticoagulation is not contraindicated. It is given as follows.

Drug Loading dose Maintenance doseStreptokinase 250,000 units IV 100,000 units/hr IV for 12-24 hrs.Urokinase 300,000 units IV 300,000 units/hr IV for 12-24 hrs.

If the condition resolves, observation is adequate. Anticoagulation iscontinued. Precautions are taken to prevent DVT and PE during labor andpuerperium. If anticoagulation is contraindicated or it fails, pulmonary embolectomyis done.

C. Recurrent PE is prevented by interruption of inferior vena cava. Vena cava filtercan be inserted percutaneously under angiographic control. However smaller embolican pass through these filters, or emboli can propagate around them throughcollateral venous channels. Ligation or clipping of inferior vena cava is an alternativeto placement of a filter. Ovarian veins may have to be ligated too. Indications forinterruption of the inferior vena cava are as follows.

1. Recurrent PE despite adequate anticoagulation.2. Pulmonary hypertension due to recurrent PE.3. Intolerance to heparin: sensitivity, heparin-induced thrombocytopenia.4. Septic PE.5. Recent pulmonary embolectomy.

D. If the patient is hemodynamically unstable, a Swan-Ganz catheter is passed andpulmonary arterial pressure monitoring is done. Beta-agonists like dopamine,dobutamine, and epinephrine are used. Mechanical ventilation is done for respiratorysupport. If the patient becomes hemodynamically stable, she is treated like a patientwho was initially stable. If she remains unstable, thrombolytic therapy is given orpulmonary embolectomy is done.

E. If all forms of treatment fail, the pregnancy is terminated by induction of labor(See chapter 105).

183

Page 190: Differential Diagnosis and Management Options inuploads.worldlibrary.org/uploads/pdf/20180107071525differential... · 83 Peripartum cardiomyopathy 165 84 Hypertension 167 85 Convulsions

SUSPICION OF PULMONARY THROMBOEMBOLISM

Clinical features and investigations (A)

Normal Pulmonary embolism

Observation Assess hemodynamics

Stable (B) Unstable (D)

Check if anticoagulation is contraindicated Supportive therapy

Yes No Improvement Failure

Bed restLeg elevationAnticoagulation

Thrombolytic therapyPulmonary embolectomy

Persistence Resolution Intolerance to heparin Failure Improvement

Pulmonary embolectomy Observation Observation

Observation Remains well Recurrence

Remains well Recurrence Inferior vena cava filter, ligation or clipping

Success Failure

Termination of pregnancy

184

Page 191: Differential Diagnosis and Management Options inuploads.worldlibrary.org/uploads/pdf/20180107071525differential... · 83 Peripartum cardiomyopathy 165 84 Hypertension 167 85 Convulsions

93. Systemic LupusErythematosus

A. It is diagnosed when four or more of the following criteria are present (AmericanCollege of Rheumatology): malar rash, discoid rash, photosensitivity, oral ulcers,non-erosive arthritis, pleuritis or pericarditis, renal disease (proteinuria >500 mg/day,3+ urine protein, and/or cellular casts on urinalysis), neurological abnormalities(including seizures or psychosis), hematological abnormalities (including hemolyticanemia with reticulocytosis, platelet counts < 100000/mm3 in the absence ofoffending drugs, or two or more occurrences of leukopenia less than 4000 cells/mm3

or lymphopenia less than 1500/mm3), laboratory findings (including anti-dsDNA,anti-Sm, or positive finding of antiphospholipid antibodies (anticardiolipin antibodyIgG or IgM, lupus anticoagulant, or a false-positive RPR) and positive antinuclearantibody test in the absence of drug therapy.

B. Hemogram, platelet count, 24-hour urine protein and creatinine clearance, serumcreatinine, hepatic transaminases (ALT, AST), lupus anticoagulant andanticardiolipin antibody, antinuclear antibody and anti-deoxyribonucleic acid (DNA)titers, complement factors 3 and 4, anti-SSA (Ro)/Anti-SSB (La) antibody screen areobtained. Obstetric ultrasonography is done for early biometry and serially forassessment of fetal growth.

C. Arthralgia and serositis are managed by aspirin and other NSAIDs, avoiding ifpossible therapeutic doses after 24 weeks to prevent premature closure of the fetalductus arteriosus. Severe disease is managed with prednisone 1 to 2 mg/kg/d,tapered to 10-15 mg each morning after the disease is controlled. Azathioprine isavoided during pregnancy unless life-threatening complications develop. If skindisease is controlled with hydroxychloroquine, it may be continued due to its longhalf life and risk of a lupus flare on its discontinuation. If the condition isuncomplicated, fetal well being is monitored regularly (see chapter 144). If there isany indication of fetal well-being, the fetus is delivered appropriately. If the fetusremains well, delivery is achieved at term. These women are also at risk of preterm

labor and preterm premature rupture of membranes. See chapters 101 and 102 fortheir management.

D. Antiphospholipid antibody syndrome management is discussed in chapter 95.

E. Anti-SS-A (Ro) and anti-SS-B (La) antibodies may cause diffuse myocarditis andfibrosis in the region between the atrioventricular node and bundle of His, causingdamage the fetal heart and conduction system and neonatal death. If fetalcardiomyopathy and complete heart block are detected, the woman is administereddexamethasone in high doses. Ultrasonography is done weekly to detect fetalanasarca. If heart block persists, fetus is delivered at 31 weeks. If anasarca develops,delivery is achieved. If heart block resolves, delivery is achieved at term. A neonatewith a complete heart block requires a pacemaker. Neonatal mortality is 5% in theabsence of and 20-30% in the presence of structural cardiac anomalies.

F. See chapters 64 and 144 for management of fetal growth restriction.

G. A woman with lupus nephritis is delivered at term if she is normotensive and thefetus remains well. In presence of hypertension and mild decline in C3, C4, CH50levels, normal level of soluble FMS-like kinase (sFLT-1) (sFLT-1) suggestspreeclampsia and higher level suggests lupus flare.

H. Generalized SLE flare is treated with methylprednisolone 1000 mg/d for 3 days,then tapered after the disease is controlled (C). Additional plasmapheresis (withIVIG or fresh frozen plasma) is required if there is membranous or diffuseglomerulonephritis.

I. Women with lupus nephritis, preeclampsia, lupus flare; or fetal complete heartblock, cardiomyopathy, and/or growth restriction are delivered immediately ifgestation is beyond 34 weeks. They are delivered immediately after hastening fetallung maturation with betamethasone between 28-34 weeks if the disease deterioratesand at 34 weeks if it remains stable. Those between 20-28 weeks are treatedmedically up to 28 weeks if the disease does not deteriorate and then as describedbefore for 28-34 weeks group. Medical termination of pregnancy is done below 20weeks of gestation.

185

Page 192: Differential Diagnosis and Management Options inuploads.worldlibrary.org/uploads/pdf/20180107071525differential... · 83 Peripartum cardiomyopathy 165 84 Hypertension 167 85 Convulsions

SUSPICION OF SYSTEMIC LUPUS ERYTHEMATOSUS (A)

Investigations (B)

UncomplicatedSLE (C)

Antiphospholipid antibodysyndrome (D)

Anti SSA (Ro)antibodies (E)

Fetal growthrestriction (F)

Lupus nephritis (G)

monitor blood pressure

Generalized SLE flare (H)

Check type of renal pathology

Nonstress testweekly from 30thweek

Low-dose aspirinHeparin + IVgamma globulin

Monthly fetalechocardiographyfrom 18th week

Normal

Deliver at term

RisesLupus

nephritisMembranous or diffuseglomerulonephritis

Reactive Nonreactive Normal Fetal complete heart blockOr cardiomyopathy

C3, C4, CH50, sFLT-1 assay Prednisone Prednisone, plasmapheresis(with IVIG or fresh frozenplasma)

Appropriateaction

Dexamethasone Mild decline in C3, C4,CH50, normal sFLT-1

> Mild decline in C3,C4, CH50, high sFLT-1

Deliver at termPreeclampsia Lupus flare

Check gestational age (I)

< 20 weeks 20-28 weeks 28-34 weeks > 34 weeks

MTP Medical treatmentHasten fetal lung maturation

Medical treatmentHasten fetal lung maturation

Deterioration Stabilization Deterioration Stabilization

Deliver Deliver Delay delivery beyond 34 weeks unlesscondition deteriorates

Deliver

186

Page 193: Differential Diagnosis and Management Options inuploads.worldlibrary.org/uploads/pdf/20180107071525differential... · 83 Peripartum cardiomyopathy 165 84 Hypertension 167 85 Convulsions

94. SclerodermaA. It is a rare autoimmune disorder characterized by progressive fibrosis andvasculitides primarily affecting the skin. It is called systemic sclerosis when there aresystemic manifestations. Clinical features include Raynaud's phenomenon,sclerodactyly, telangiectasia, cardiomyopathy, myocardial infarctions, cardiacconduction defects, calcinosis, dysphagia and renal failure.

B. The investigations include those for hypertension, renal function, fetal growth andfetal well-being. Urinalysis, 24-hour urine collection for creatinine clearance andtotal protein, serum creatinine are done in each trimester. Early datingultrasonography, serial scans for growth, and non-stress tests and/or biophysicalprofiles weekly beginning at 36 weeks in uncomplicated cases or 28 weeks and laterin the presence of fetal growth restriction are obtained. Antiphospholipid antibodiesand anti-SSA antibodies are checked for. These two conditions are discussedelsewhere (See chapterd 93 and 95).

C. In the absence of antibodies, severity of the disease and the presence ofcomplications are checked. Complications include malignant hypertension with orwithout renal failure, pulmonary hypertension, myocardial fibrosis, fetal growthrestriction, and fetal death.

D. Conservative treatment is given in the absence of clinical deterioration orcomplications. Physiotherapy is given for hand contractures. Antihypertensivetherapy is given with calcium channel blockers, avoiding ACE inhibitors.

Prednisone is given for concomitant myositis. Antacids and metoclopramide to usedto manage severe reflux esophagitis. Dialysis may be required for renal failure.

E. If the patient has malignant hypertension, intensive care is required. Cardiacoutput, pulmonary capillary wedge pressure, pulmonary artery pressure and systemicvascular resistance are assessed by placing a Swan Ganz catheter if there is nocoagulopathy. Electrocardiogram is obtained. Cardiac rhythm is checked with acardiac monitor. Pulse oximeter is attached and an arterial line is placed. Bloodpressure is monitored continuously. Foley’s catheter is placed to monitor urineoutput every hour. Input and output are charted. Oxygen is administered by a facemask. Hypertension is managed by administration of hydralazine 5-10 mg IV every20 minutes. An alternative is labetalol 20 mg IV over 2 minutes, followed by 40-80mg at 10-minute intervals, up to a total of 300 mg. Nitroglycerine drip is used if thistreatment fails to control hypertension. Maintenance therapy may be given withlabetalol 200-300 mg PO q6h. Coagulation studies are done and compatible blood iskept ready for use as required. If the gestation age is below 34 weeks, betamethasoneis given to hasten maturation of fetal lungs. Fetal growth and well-being aremonitored. Dialysis is done for renal failure. If the pregnancy is less than 20completed weeks in duration, medical termination of pregnancy is done for renaldisease, pulmonary hypertension, and/or myocardial fibrosis.

F. Obstetric complications include increased risk of abortion, preterm delivery,stillbirth, and increase in perinatal mortality. These conditions are managed asdiscussed elsewhere. Risk of inheritance by the baby is low. Genetic counseling isdone. accordingly.

187

Page 194: Differential Diagnosis and Management Options inuploads.worldlibrary.org/uploads/pdf/20180107071525differential... · 83 Peripartum cardiomyopathy 165 84 Hypertension 167 85 Convulsions

SUSPICION OF SCLERODERMA (A)

Perform investigations (B)

Antiphospholipid antibody syndrome Anti SSA antibodies Absence of antibodies

See chapter 95 See chapter 93

Assess severity of disease and presence of complications (C)

No deterioration orcomplications

Malignanthypertension (E)

Preterm labor Preeclampsia Fetalcompromise

Conservativetreatment (D)

Check renal function See chapter 101 See chapter 84 See chapter 144

Renal failure No renal failure

Dialysis Intensive care

188

Page 195: Differential Diagnosis and Management Options inuploads.worldlibrary.org/uploads/pdf/20180107071525differential... · 83 Peripartum cardiomyopathy 165 84 Hypertension 167 85 Convulsions

95. Antiphospholipid Antibody SyndromeA. The antiphospholipid syndrome (APS) is diagnosed when specific clinical andlaboratory criteria (Sydney criteria) are fulfilled. It requires a history of thrombosisin any tissue or organ except superficial veins, at least one fetal death at or beyondthe tenth week of gestation, at least one premature birth at or before the thirty-fourthweek due to preeclampsia or placental insufficiency, or at least three consecutivespontaneous abortions before the tenth week. There must also be either lupusanticoagulant (LAC) or moderate to high (>40 GPl or MPL units) levels ofanticardiolipin antibodies (ACA) or >99th percentile levels of anti-(β-2-glycoprotein-l IgG or IgM levels on two occasions at least 12 weeks apart. Other features of thecondition include thrombocytopenia, Coombs'-positive hemolytic anemia, otherwiseunexplained transient ischemic attacks or amaurosis fugax, livedo reticularis, chorea,and chorea gravidarum.

B. APS is managed by administration of heparin 15000-20000 units ofunfractionated heparin or equivalent (prophylactic dose) subcutaneously qd in 2divided doses and aspirin 75 mg PO qd. For APS with prior fetal death or recurrentpregnancy loss, calcium and vitamin D supplements are given in addition. APS withprior thrombosis or stroke is managed by therapeutic administration of heparin. The

management of presence of antiphospholipid antibodies without APS is uncertain.Therapeutic options include no treatment, low-dose aspirin, or combination ofprophylactic doses of heparin and low-dose aspirin. The patient is counseledregarding symptoms of thrombosis and thromboembolism. Glucocorticoids are notused in the management of primary disease. If APS develops in a patient of SLE,lowest possible doses of glucorticoids are used to prevent a flare. Steroids can causeosteopenia, osteoporosis, pathological fractures, impairment of wound healing,gestational and overt diabetes mellitus. Immunoglobulin therapy is used for overtdisease, heparin induced thrombocytopenia. and when other first-line therapies havefailed, especially with the development of preeclampsia and fetal growth restriction.The dose of immunoglobulin is 0.4 g/kg IV qd for 5 days (total dose of 2 g/kg). It isadministered monthly in this way or as a single dose of 1 g/kg each month.C. Though the patient is under treatment, she can still develop complications of APS.Or the patient may present for the first time with one or more of the complications.The complications are treated appropriately, as discussed elsewhere in this book. Seechapter 69 for the management of abortion, chapter 73 for placental abruption,chapter 84 for preeclampsia, chapter 65 for fetal growth restriction, chapter 104 forfetal death, and chapter 92 for venous thromboembolism.D. If the patient has no complications, then aspirin administration is stopped a weekbefore due date and heparin is stopped when the patient goes in labor. The patient’scoagulation studies and platelet count are obtained. Platelets are transfused asrequired. Fresh frozen plasma and/or cryoprecipitate are administered as required toprevent postpartum hemorrhage.

189

Page 196: Differential Diagnosis and Management Options inuploads.worldlibrary.org/uploads/pdf/20180107071525differential... · 83 Peripartum cardiomyopathy 165 84 Hypertension 167 85 Convulsions

SUSPICION OF ANTIPHOSPHOLIPID ANTIBODY SYNDROME (APAS)

Check historyPerform investigations (A)

No APAS APAS

Antenatal care as appropriate Medical treatment (B)

Check for development of complications (C)

Abortion Placental abruption Preeclampsia Fetal growthrestriction

Fetal death Venousthromboembolism

None (D)

See chapter 69 See chapter 73 See chapter 84 See chapter 65 See chapter 104 See chapter 92 Manage laborstopping aspirin andheparin atappropriate time

190

Page 197: Differential Diagnosis and Management Options inuploads.worldlibrary.org/uploads/pdf/20180107071525differential... · 83 Peripartum cardiomyopathy 165 84 Hypertension 167 85 Convulsions

96. Rh-negative GravidaRh-isoimmunization is a process of development of Rh-antibodies in an Rh- negativeperson due to introduction of Rh-antigen from outside. It may be due to any of thefollowing.

1. Mismatched blood transfusion.2. Fetomaternal hemorrhage (Rh-negative mother, Rh-positive fetus): it can occur

with all types of abortions, evacuation of a vesicular mole, ectopic pregnancy,antepartum hemorrhage, amniocentesis, version, normal or operative delivery,manual removal of placenta etc.

3. Maternofetal hemorrhage (Rh-positive mother, Rh-negative female fetus).4. Twin-twin transfusion.

A. The problem of Rh-isoimmunization can be totally avoided if an Rh-negativewoman marries an Rh-negative man. However that is not always possible, sinceonly 5% of the people are Rh-negative. If the father of the fetus is Rh-negative, thepregnancy is treated as a normal pregnancy. If he is Rh-positive, the presence of Rh-antibodies in the maternal circulation is checked by an indirect Coomb's test. Unlessthe woman has been sensitized prior to this pregnancy, the fetus is unaffected.

B. If this is the first pregnancy of the woman, the fetus is less likely to be affected.However she could still be affected by a factor other than pregnancy. The Rh-antibody titer is checked in primigravidas and all previously unaffectedmultigravidas. The pregnancy is not at risk at titers from 0 to 1:32 (varies fromcenter to center). The titer is repeated every 4 weeks up to 24 weeks and then every2 weeks up to term. Since maternal sensitization can take place prior to delivery, 300μg Rh-immune globulin is administered intramuscularly at 28 weeks. It preventsisoimmunization up to term. These women are delivered at term.

C. Umbilical cord blood is tested after delivery of the baby. If the baby is Rh-negative, no further management is required. If it is Rh-positive and a direct Coomb'stest on its blood is negative, the mother is given 300 μg of Rh-immune globulin IMfor prevention of Rh-isoimmunization, so that subsequent pregnancies are notaffected. If the fetomaternal hemorrhage has been more that 15 ml, as judged byKleihauer Betke test, additional dose of Rh-immune globulin is required (20 μg perml of fetal RBCs).

D. A titer above 1:32 needs assessment of fetal involvement by serial fetal middlecerebral artery (MCA) peak velocity (if available) or amniotic fluid bilirubin content(produced by fetal hemolysis due to maternal Rh-antibodies). The tests are started 4-8 weeks prior to the gestational age at which a previous fetus was affected. MCApeak velocity > 1.5 MoM requires intervention. Amniotic fluid spectrophotometry isdone to find out optical density between 375 nm and 525 nm. Deviation of the graph

at 450 nm from interpolated baseline (ΔOD450) is calculated. The value is plotted inLiley's graph (fig. 96.1). If it is in zone I or II, the test is repeated after 1-2 weeks. Ifit remains so, the fetus is delivered at term. If the value falls by 0.01 or more, the testis repeated weekly. If it is rising, steady, or falling by < 0.01 per week, interventionis necessary. All patients in zone III need intervention. The intervention is fetalcordocentesis to determine its hematocrit. Hematocrit below 30% necessitatesintrauterine fetal transfusion (IUT) below 32 weeks. In earlier pregnancy, thistransfusion is given intraperitoneally since vascular access is difficult. In laterpregnancy, fetal intravascular transfusion is given. If the pregnancy is above 32weeks, fetal lung maturity is assessed. If the lungs are mature, the fetus is delivered. Ifthe lungs are immature, fetal transfusion is given under ultrasonographic control withRh-negative RBCs, compatible with the maternal serum. Betamethasone isadministered to the woman to hasten maturation of fetal lungs. IUT is repeated up to32 weeks of gestation as required, and the fetus is delivered when its lungs mature.

GESTATIONAL AGE WEEKS)Fig. 96.1 Liley's graph for assessment of fetal hemolysis in Rh-isoimmunization.

191

Page 198: Differential Diagnosis and Management Options inuploads.worldlibrary.org/uploads/pdf/20180107071525differential... · 83 Peripartum cardiomyopathy 165 84 Hypertension 167 85 Convulsions

Blood group of father, indirect Coomb’stest on gravida

Rh-NEGATIVE GRAVIDA

Check gravidity and previously affected pregnancy (B)

Primigravida Multigravida, previously unaffected pregnancy Multigravida, previously affected pregnancy

Rh-Antibody titer Serial fetal MCA Doppler studies or amniotic fluid ΔOD450 assay every1-2 weeks starting 4-8 weeks before gestational age at which previous

pregnancy was affected (D)

< 1:32 > 1:32 (D)

Repeat titter every 4 weeks up to 24 weeksand every 2 weeks thereafter

Serial fetal MCA Doppler studies or amniotic fluid ΔOD450 assay every 1-2 weeks

< 1:32 Peak MCA velocity < 1.5 MoM or ΔOD450 in Liley zone I or II Peak MCA velocity > 1.5 MoM or ΔOD450 in Liley zone III

Continue serial assessment of peak MCA velocity or amniotic fluidΔOD450 assay every 1-2 weeks

Fetal cordocentesis to determine hematocrit

> 30% < 30%

Deliver when fetal lungs mature Check fetal lung maturity

Deliver at term Mature Immature

Management afterdelivery (C)

Intrauterine transfusion, maternal steroid administration to hasten fetal lung maturation, serialassessment of peak MCA velocity and amniotic fluid ΔOD450 assay every 1-2 weeks

Peak MCA velocity < 1.5 MoM or ΔOD450 in Liley zone I or II Peak MCA velocity > 1.5 MoM or ΔOD450 in Liley zone III

Deliver when fetal lungs mature Intrauterine transfusion if fetal lungs immature, deliver when fetal lungs mature

192

Page 199: Differential Diagnosis and Management Options inuploads.worldlibrary.org/uploads/pdf/20180107071525differential... · 83 Peripartum cardiomyopathy 165 84 Hypertension 167 85 Convulsions

97. Multiple PregnancyMultiple pregnancy is the development of more than one fetus at a time in a

gravid uterus. It occurs in India in 14.3 per 1000 births. The incidence in the worldvaries between 5.6 and 46 per 1000 births. Etiological factors for multiplepregnancy are as follows.

1. Maternal age between 35 and 39 years.2. Multiparity.3. Family history on the maternal side.4. Induction of ovulation: more with gonadotropins than with clomiphene citrate.5. Assisted reproductive techniques: IVF-ET, GIFT, ZIFT etc.

A. The first step in the management of multiple pregnancy is assessment of gestationalage. It is easy if the date of the last menstrual period is known accurately. If it is notknown, obstetric examination is not very useful because the uterus is larger thanexpected due to the presence of multiple babies, and in some cases hydramnios.Ultrasonography gives a reasonably accurate idea of the gestational age (see chapter63). Those with 4 or more embryos are called grand multifetal pregnancies.

B. If the gestational age is less than 12 weeks and the number of fetuses is more than3, embryo reduction is done so as to reduce the number to 2 or 3. This is necessarybecause successful outcome of pregnancy is unlikely with so many fetuses. It shouldbe done by a person highly experienced in amniocentesis and high-resolutionultrasonography. It can be done either by transcervical suction, or fetal thoracicinjection of potassium chloride. Prior to such a procedure, the following informationshould be provided to the couple, so that an informed decision can be made.

1. Risk of abortion of the remaining fetus(es).2. Retention of genetically or structurally abnormal fetus(es) after a reduction in

number.3. Damage without death of a fetus and continuation of the pregnancy.4. Preterm labor.5. Maternal hemorrhage.6. Maternal infection.7. Disseminated intravascular coagulation due to retention of a dead fetus.

C. If major congenital malformations are present, the pregnancy is best terminatedby appropriate method (see chapter 165). If the fetuses are normal, the pregnancyis continued under supervision. Bed rest is given. Iron and folic acid are given toprevent anemia. Complications are watched for and treated appropriately. If onefetus dies before 18 weeks, in 38% cases the other one dies in 2 weeks. Other twinmay develop multiple cystic lesions in brain and kidneys.

D. Indications for cesarean section are as follows.

1. Umbilical cord presentation/prolapse.

2. Severe pregnancy induced hypertension.3. Placenta previa.4. First fetus in transverse lie.5. Uterine dysfunction.6. Number of fetuses > 3.7. Gross discordance in the size of the fetuses, with the smaller fetus presenting

first.8. Locking of twins.

E. When the gravida goes in labor at term, the presentation of the first of the twins isassessed. A breech, face, or brow presentation is managed as discussed in chapters107, 109, and 110. A vertex presenting fetus is allowed to deliver vaginally.

F. Usually uterine contractions cease for some time after the delivery of the firstfetus. The well-being of the second of twins needs to be monitored closely, becausethere is the risk of development of placental abruption due to a 50% reduction in theuterine contents and a corresponding retraction of the uterine wall. The umbilicalcord of the first of twins is clamped immediately after delivery so as to prevent twin-to-twin transfusion. The presentation of the second of twins is then assessed. If thefetus is in a transverse lie, an external cephalic version is done. The fetal head is thenpushed into the pelvis, the membranes are ruptured, and an oxytocin infusion isstarted so as to start labor contractions again. If the external version fails, an internalpodalic version is done. If it succeeds, assisted breech delivery is carried out. If itfails, a cesarean section is done. If the fetus is any other malpresentation, it ismanaged as discussed in chapters 107, 109, and 110. If it is in vertex presentationand labor contractions do not start in 15 minutes, the fetal head is pushed into thepelvis, membranes are ruptured, and an oxytocin infusion is started to initiate uterinecontractions again. Indications for cesarean section of the second of twin are asfollows.

1. Uncorrectable transverse lie.2. Umbilical cord prolapse in presence of reformed cervix.3. Compound presentation: vertex with hand prolapse.

Maternal Complications of Multiple Pregnancy

Pregnancy induced hypertensionHydramniosPreterm laborAnemiaPlacenta previaMalpresentationsSupine hypotensionAbortionPostpartum hemorrhage

193

Page 200: Differential Diagnosis and Management Options inuploads.worldlibrary.org/uploads/pdf/20180107071525differential... · 83 Peripartum cardiomyopathy 165 84 Hypertension 167 85 Convulsions

MULTIPLE PREGNANCY

Assess gestational age (A)

< 12 weeks > 12 weeks

Assess number of fetuses Ultrasonography for major fetal malformations (C)

> 4 2 or 3 Present Absent

Embryo reduction Ultrasonography for major fetalmalformations (B)

Antenatal careLabor at term

Absent Present Check number of fetuses

Antenatal care Medical termination of pregnancy 2 > 3

Assess for presence of complicating factors (E) Cesarean section

Placenta previa Cord presentation Previous cesarean section Malpresentation None

Cesarean section Vaginal deliveryAssess presentation of second fetus (F)

Breech Face Brow Vertex Compound Transverse

See chapter 107 See chapter 109 See chapter 110 Oxytocin augmentation if labor does not start in15 minutes

See chapter111

External cephalic version

Succeeds Fails

Vaginal delivery Succeeds Internal podalic version

Cesarean section Fails

194

Page 201: Differential Diagnosis and Management Options inuploads.worldlibrary.org/uploads/pdf/20180107071525differential... · 83 Peripartum cardiomyopathy 165 84 Hypertension 167 85 Convulsions

98. Umbilical Cord ProlapseA. A gravida may present at any time after prolapse of the umbilical cord, theinterval varying from a few minutes to a couple of hours. Most women just complainof leaking, and do not realize that the cord has prolapsed, even if it is outsidethe vulva. As soon as the diagnosis is made, it should be reposited in the vagina,because that elevates the cord’s environmental temperature to body temperature.This reduces thermal stimulation of the cord and relieves cord spasm. Umbilicalcord should be gently palpated for feeling its pulsations, which indicate that thefetus is still alive.

B. If cord pulsations are absent, the fetus is either dead, or its heart is beatingso feebly that it cannot produce pulsations in the umbilical arteries. This differentiationcan be made by performing a real time ultrasonography. If fetal cardiac activityis seen, the fetus is alive but likely to die soon. If it is absent the fetus is dead.The dying fetus needs to be delivered immediately.

C. If the cervix is fully dilated and fully effaced, an immediate delivery canbe achieved. If the presentation is vertex and the head is engaged, a vacuum orforceps delivery is achieved. If it is a breech presentation, a breech extractionis done.

D. If the cervix is not fully dilated and fully effaced, an immediate vaginal deliveryis not possible. Such a patient needs a cesarean section in order to save the baby’slife. It takes some time for preparation for a cesarean section. The following measuresare used to keep the fetus alive in this time.

1. Cystodistension: a Foley's catheter is passed into the urinary bladder and itsballoon is inflated with 5 ml of distilled water. The bladder is distended with200 to 300 ml of normal saline. The full bladder acts like a water-bed andelevates the presenting part up, thereby relieving compression of the umbilicalcord. This method of management is far superior to getting a person to elevatethe presenting part with two fingers in the vagina until the cesarean section isdone. It is not only embarrassing to the patient and tiring to the person doing it,but also less efficient because the upward pressure by two fingers is much less

constant and less uniform than cystodistension.2. Tocolysis is done (see chapter 101) to control uterine activity, to reduce the risk

of umbilical cord compression further.3. Oxygen is administered by a face mask.

The urinary bladder is emptied and tocolysis is stopped at the time of thecesarean section. One must confirm that the fetus is alive before starting theoperation, because it is likely that it might have died in the meantime.

E. If the fetus is dead, a vaginal delivery is achieved. Before allowing the labor toprogress, the cause of the umbilical cord prolapse needs to be found out. Variouscauses of cord prolapse are as follows.

1. Malpresentation: transverse lie, breech (0.5% in frank breech, 5% in completebreech, 15% in knee and footling presentation), face, and brow presentation.

2. Compound presentation.3. Prematurity.4. Multiple pregnancy.5. Hydramnios.6. Contracted pelvis.7. Long umbilical cord.8. Placenta previa.9. Iatrogenic:

a. Uncontrolled drainage of amniotic fluid after artificial rupture ofmembranes, if the presenting part is not engaged or fixed in the pelvic inlet.

b. Version.c. Manual rotation of fetal head.

If the fetus is in a longitudinal lie, the labor is allowed to progress. If it is in atransverse lie, it is converted into a longitudinal lie and then labor is allowed toprogress so as to achieve a vaginal delivery. Since the fetus is already dead, there isno urgency in management of such cases, and labor can be allowed to follow its owncourse.

195

Page 202: Differential Diagnosis and Management Options inuploads.worldlibrary.org/uploads/pdf/20180107071525differential... · 83 Peripartum cardiomyopathy 165 84 Hypertension 167 85 Convulsions

UMBILICAL CORD PROLAPSE

Reposition in the vaginaFeel for cord pulsations (A)

Absent Present

Immediate real time ultrasonography for fetal cardiac activity (B) Assess feasibility of immediate vaginal delivery (C)

Present Absent Possible Not possible

Assess feasibility of immediate vaginal delivery (C) Forceps/vacuumdelivery or breech

extraction

Assess feasibility of immediate internal podalicversion/breech extraction

Possible Not possible Vaginal delivery by appropriate method (E) Possible Not possible (D)

Instrumental vaginaldelivery or breech

extraction

Cystodistension,tocolysis up to thetime of immediatecesarean section

Internal podalicversion/breech

extraction

Cystodistension, tocolysis, oxygenation up todelivery

Immediate cesarean section

196

Page 203: Differential Diagnosis and Management Options inuploads.worldlibrary.org/uploads/pdf/20180107071525differential... · 83 Peripartum cardiomyopathy 165 84 Hypertension 167 85 Convulsions

99. HydramniosHydramnios is a condition in which the amniotic fluid volume is more than

twice the expected volume for that gestational age. If it is more than 2 L at any time,it is also called hydramnios.

A. Ultrasonography is the most important investigation in a case of hydramnios. Itconfirms the diagnosis and rules out rare cases of ascites with pregnancy, which canbe confused for hydramnios. Amniotic fluid index is calculated by finding the sum ofdepth of the largest pocket of amniotic fluid in each of four quadrants of the anteriorabdominal wall. A value more than 24 cm is diagnostic of hydramnios. Finding at leastone 8 cm pocket of amniotic fluid is also diagnostic of hydramnios. Severity ofhydramnios is assessed as shown in table 99.1.

Table 99.1 Severity of Hydramnios

Severity Deepest vertical pocket of amniotic fluid

MildModerateSevere

8-12 cm12-15 cm> 15 cm

Ultrasonography is also useful to find out the cause of the hydramnios as follows.

1. Anencephaly: hydramnios is due to excessive exudation of cerebrospinal fluidfrom exposed choroid plexus, failure of the fetus to swallow amniotic fluid, anddiuresis due to absence of antidiuretic hormone. The pregnancy is terminatedby induction of labor (see chapter 105).

2. Open spina bifida: the pregnancy is allowed to progress to term. A carefulvaginal delivery is allowed, because it has been shown that performing acesarean section does not improve the prognosis in any way.

3. Esophageal atresia: if the diagnosis is made early, the pregnancy is bestterminated.

4. Multiple pregnancy: see chapter 97.5. Chorioangioma of the placenta: it is a vascular tumor under the amnion which

exudes a lot of fluid into the amniotic cavity. The treatment is conservative andthe fetal prognosis is good.

6. Hydrops fetalis: edema of the placental cotyledons causes an impairment ofplacental circulation and hence hydramnios. The treatment is as that of theetiological condition.

7. Neurologic impairment of swallowing: trisomy 18, muscular dystrophies.

B. If there are no fetal abnormalities responsible for the hydramnios, maternaldiabetes is checked for. Fasting and postglucose (100 g) blood sugar levels areestimated. If necessary, a glucose tolerance test is done. If the gravida has diabetesmellitus, the hydramnios disappears when the diabetes is controlled.

C. If the gravida does not have diabetes mellitus, she is assessed for Rh-isoimmunization. Hydrops fetalis due to the Rh-disease can cause hydramnios.It is discussed in chapter 96.

Complications of Hydramnios

Pregnancy induced hypertensionPreterm laborMalpresentationUmbilical cord presentation/prolapseUterine inertia during laborAbruptio placentaePostpartum hemorrhage

D. If a cause is not found for the hydramnios, it still needs to be treatedin order to prevent complications. Medical treatment of hydramnios is indomethacinin a dose of 2 mg/kg PO qd in 3 divided doses. It reduces urine output by the fetalkidneys. Such therapy may cause intrauterine closure of ductus arteriosus (whichshould be watched for by periodic ultrasonography), fetal cerebral vasoconstriction,impairment of fetal renal function, neonatal necrotizing enterocolitis and neonatalintracranial hemorrhage. Sulindac 200 mg PO ql2h is safer and as effective. It doesnot cause closure of fetal ductus arteriosus. If the patient has respiratoryembarrassment due to hydramnios, amniotic fluid is drained by amniocentesis. Therate of drainage is maintained at about 500 ml/hr, and a total of 1500 to 2000 ml iswithdrawn. Such slow rate of drainage does not increase the risk of placentalabruption. Amniocentesis may cause entry of microorganisms into the amnioticcavity and development of chorioamnionitis. This complication should be watchedfor.

197

Page 204: Differential Diagnosis and Management Options inuploads.worldlibrary.org/uploads/pdf/20180107071525differential... · 83 Peripartum cardiomyopathy 165 84 Hypertension 167 85 Convulsions

HYDRAMNIOS

Ultrasonography (A)

Anencephaly Spina bifida Esophagealatresia

Multiplepregnancy

Normal fetus (B) Chorioangioma of placenta

Glucose tolerancestudies

Antenatal care MTP up to 20 weeks,vaginal delivery atterm if > 20 weeks

See chapter 97 Glucose tolerancestudies

Antenatal care, deliver at term.

Normal Abnormal Abnormal Normal

MTP up to 20 weeks,vaginal delivery atterm if > 20 weeks

Treat diabetesmellitus

Treat diabetesmellitus

Assess for Rh-isoimmunization (C)

Present Absent

Assess for respiratory difficulty (D)

Present Absent

Amniocentesis to relieve pressure effect

Maternal indomethacin therapy up to 32-35 weeks

198

Page 205: Differential Diagnosis and Management Options inuploads.worldlibrary.org/uploads/pdf/20180107071525differential... · 83 Peripartum cardiomyopathy 165 84 Hypertension 167 85 Convulsions

100. OligohydramniosOligohydramnios is diagnosed when the amniotic fluid volume is less than

300 ml at term, or less than half that expected at the given gestational age. Clinicallythe diagnosis is made when the fetus appears to fill the entire uterus, and there is nofluid palpable around it.

A. After fetal death, production of amniotic fluid by the fetus (urine, pulmonaryfluid, buccal gland secretion) stops, while its absorption through the amnion intomaternal circulation continues. The result is progressive reduction in amniotic fluidvolume.

B. In a postdated pregnancy, the amniotic fluid volume is reduced due to relativeplacental insufficiency.

C. Premature rupture of membranes causes drainage of amniotic fluid andoligohydramnios. If the site of rupture seals up, the leakage stops. In such cases theamniotic fluid volume may become normal again. However if the leakage continues,the result is severe oligohydramnios. The diagnosis is made by examination of thepatient's clothes and a pad worn by her on the vulva. It is confirmed by examinationof vaginal fluid (see chapter 102).

D. The diagnosis of oligohydramnios is made on ultrasonography by a failureto find even one pocket of amniotic fluid measuring at least 3 cm in diameter.Severity of oligohydramnios is assessed as shown in table 100.1

Table 100.1 Severity of Oligohydramnios

Severity Deepest vertical pocket of amniotic fluid

Mild 2-3 cmModerate 1-2 cm

Severe < 1 cm

E. The oligohydramnios tetrad, initially called Potter's syndrome, consists ofrenal agenesis resulting in oligohydramnios, pulmonary hypoplasia, spade-likehands and feet, peculiar facies with a beaked nose, and creases under the lowereyelids. The fetus, often small for gestational age, is usually in a breech presentation,does not tolerate the stress of labor, and may be stillborn. If liveborn, it developsrespiratory distress and dies of respiratory failure long before it can die of renalfailure. This condition is incompatible with life. If it is diagnosed before 20 weeks,medical termination of pregnancy can be done. In other cases, vaginal delivery isachieved when the woman goes in labor, avoiding a cesarean section except formaternal indications.

F. Posterior urethral valves (PUV) are found usually in male fetuses. Urogenitaland cloacal abnormalities may present similarly in female fetuses. Outflow tractobstruction causes backpressure changes like progressive cystic dilatation of thebladder, ureters, and if unrelieved, destruction of the kidneys. If renal functionis present (as judged by fetal urine analysis) and irreversible pulmonary hypoplasiais absent, ultrasonography-guided fetal vesico-amniotic shunt may be done. It isas yet in an experimental stage.

G. Bilateral pelviureteral junction obstruction presents with oligohydramniosand bilateral hydronephrosis. The fetal bladder is always empty.

H. Fetal infantile polycystic kidney disease (IPKD) is characterized byultrasonographic appearance of enlarged, solid kidneys, empty urinary bladder, andoligohydramnios.

I. Multicystic dysplastic kidney disease (MDKD) is characterized by enlarged andmacrocystic kidneys on ultrasonography, and oligohydramnios. Both IPKD andMDKD are incompatible with life. If detected prior to 20 weeks of gestation, asecond trimester MTP is done. If detected late, spontaneous onset of labor is awaited.

J. Use of indomethacin for control of preterm labor over prolonged period may causeoligohydramnios. This effect is reversible on stopping the drug administration.

199

Page 206: Differential Diagnosis and Management Options inuploads.worldlibrary.org/uploads/pdf/20180107071525differential... · 83 Peripartum cardiomyopathy 165 84 Hypertension 167 85 Convulsions

OLIGOHYDRAMNIOS

Auscultate for fetal heart sounds (A)

Present Absent

Check expected date of delivery (B) Intrauterine fetal death

Postdatism At/before term See chapter 104

See chapter 103 History of rupture of membranes (C)

Absent Present

Ultrasonography (D). See chapter 102

Intrauterinegrowth restriction

Oligohydramniostetrad

(E)

Posteriorurethral valves

(F)

Bilateral pelviureteraljunction obstruction

(G)

Fetal infantilepolycystic kidney

disease(H)

Multicysticdysplastic kidney

disease(I)

See chapter 65 Assess gestational age Sample fetal urine ultrasonographically, assess fetalrenal function

Assess gestational age

< 20 weeks > 20 weeks Good Poor < 20 weeks > 20 weeks

Medical termination ofpregnancy

Counseling, antenatalcare, await spontaneous

onset of labor

Fetal surgery Counseling, antenatalcare, await spontaneous

onset of labor

Medical termination ofpregnancy

Counseling, antenatalcare, await spontaneous

onset of labor

200

Page 207: Differential Diagnosis and Management Options inuploads.worldlibrary.org/uploads/pdf/20180107071525differential... · 83 Peripartum cardiomyopathy 165 84 Hypertension 167 85 Convulsions

101. Preterm LaborA. Preterm labor is occurrence of labor contractions prior to 37 completed weeks ofgestation after the age of viability. The American Academy of Pediatrics and theAmerican College of Obstetricians and Gynecologists (1997) had proposed thefollowing criteria to document preterm labor:

1. Contractions of four in 20 minutes or eight in 60 minutes plus progressivechange in the cervix.2. Cervical dilatation greater than 1 cm.3. Cervical effacement of 80 percent or greater.

Subsequently they have withdrawn these criteria. We diagnose it whenthere are at least two painful uterine contractions in 10 minutes. Presence of cervicaldilatation, effacement, dislodgement of the cervical mucus plug with some blood(show) and formation of bag of waters are late signs. Preterm labor should bediagnosed long before these signs develop, so that its control is effective. Presence offetal fibronectin in cervicovaginal secretions confirms the diagnosis. Women at riskof preterm labor can be monitored at home with a device using the guard-ringprinciple. The contraction sensor is belted about the abdomen. It is connected to asmall electronic recorder worn at the waist. It is used to transmit uterine activity viatelephone to the receiving center every day. This method is extremely expensive, andunlikely to find a place in the developing countries on a large scale.

Causes of Preterm LaborSpontaneous rupture of membranesAmniotic fluid infectionMultiple pregnancyHydramniosCervical incompetencePrevious preterm deliveryFetal death

Anomalies of conceptionUterine anomaliesPlacental abruptionPlacenta previaSerious maternal diseaseRetained IUCDMaternal periodontal disease

B. Contraindications for control of preterm labor include the following.

1. Intrauterine fetal death.2. Fetal malformations incompatible with life.3. Chorioamnionitis.4. Continuation of pregnancy poses serious risk to the maternal life.

a. Abruptio placentae.b. Placenta previa.c. Severe pregnancy induced hypertension and eclampsia.

5. Fetal lungs are mature.6. Severe fetal growth restriction.7. Erythroblastosis fetalis.8. Cervical dilatation is more than 4 cm.

C. Tocolytic therapy is used to control preterm labor until fetal lungs mature under

the effect of betamethasone. The value of bed rest has not been proved.

Tocolytic Therapy

Drug Dosage

Nifedipine 20 mg loading dose repeated every 30 min twice ifcontractions persist, followed by 10-20 mg PO q6h

Indomethacin 50 mg PR followed by 25 mg PO q8h, not more than 200 mgin 24 hours.

D. Betamethasone is used to hasten the maturity of fetal lungs up to 37 weeks ofgestation. Its dose is 12 mg IM, repeated after 24 hours. It is repeated after 12 hoursin preterm premature rupture of membranes because less time is available. It iseffective more in cases of female fetuses, and only if at least 24 hours pass betweenthe therapy and the delivery of the baby. It achieves a significant reduction innecrotizing enterocolitis, early systemic neonatal infection, respiratory distress,cerebroventricular hemorrhage, and neonatal death. Dexamethasone is not as useful,and other preparations of glucocorticoids are not useful at all. Besides other adverseeffects of glucocorticoids, there is risk of maternal pulmonary edema whenglucocorticoid therapy is combined with tocolytic therapy in any form. Use ofantibiotics does not help control preterm labor. Control of periodontal disease isuseful in preventing preterm labor. Administration of 17 α hydroxyprogesteronecaproate 250-500 mg deep IM every week up to 37 completed weeks preventspreterm labor.

E. Conduct of preterm labor needs some extra precautions in the interest of the fetus,as compared to normal labor.

1. It is preferable to transfer the patient to a center with facilities for neonatalintensive care, rather than deliver her and then transfer the preterm neonate. Theuterus is the best incubator for this purpose.

2. Fetal heart rate monitoring should be done, preferably by continuous electronicmonitoring. Tachycardia, especially in the presence of ruptured membranes,suggests chorioamnionitis and fetal infection.

3. Labor analgesia is best given in the form of a continuous epidural block.Narcotics and other CNS depressant drugs should be avoided, as they depressthe neonate.

4. A generous episiotomy should be made to avoid rapid compression anddecompression of the fetal head during delivery. The use of “cage forceps” toprotect the head is not very popular. Use of conventional forceps for thispurpose is often detrimental to the fetus. Parallel type of forceps, in which thedistance between the blades is maintained, and fetal head cannot be compressed,would be ideal for this purpose. If vacuum extraction is required, a maximumnegative pressure of 0.5 kg/cm2 should not be exceeded.

5. Neonatal resuscitation should be done as required, exercising more gentlenessthan in the case of a term infant.

201

Page 208: Differential Diagnosis and Management Options inuploads.worldlibrary.org/uploads/pdf/20180107071525differential... · 83 Peripartum cardiomyopathy 165 84 Hypertension 167 85 Convulsions

Confirm diagnosis (A)

PRETERM LABOR

Assess cervical dilatation and effacement

< 4 cm, < 50% > 4 cm, > 50%

Assess for contraindications for control of preterm labor (B) Permit continuation of labor (E)

Absent Present Conventional management of laborAvoid narcotic analgesiaGenerous episiotomyIntensive care of the newborn

Tocolytic therapy (C)Other therapy (D)

Successful Not successful

Progesterone therapyup to 37 weeks

202

Page 209: Differential Diagnosis and Management Options inuploads.worldlibrary.org/uploads/pdf/20180107071525differential... · 83 Peripartum cardiomyopathy 165 84 Hypertension 167 85 Convulsions

102. Vaginal Leakage of FluidA. Clothes soaked with fluid is the best evidence of vaginal leakage of fluid.It is likely to have been there for some time and will have an odor of urine ifthe woman has passed urine involuntarily. However a gravida is quite likely tochange her clothes before presenting to a doctor, and if she does not have a continuedleakage, the clothes will be dry. It is not necessary to do a vaginal examinationat this time since she does not have labor contractions. In fact it is likely to introducebacteria into the vagina, and may cause chorioamnionitis.

B. A sterile pad is applied to her vulva. It is examined periodically. If it remainsdry over 24 hours, she may be sent home with instructions to avoid sexual intercourseand report in case she gets leakage of fluid again.

C. If fluid is found to be leaking from the cervix, the diagnosis of prematurerupture of membranes (PROM) is confirmed. Other signs of PROM are as follows.

1. Leakage of fluid from cervix on fundal pressure.2. Finding lanugo hair, vernix, and/or meconium in the vaginal fluid.

The fluid in the vagina is collected and one or more of the following tests are done toconfirm that it is amniotic fluid.

1. Litmus test: pH of vaginal secretions is 4.5 to 5.5, while that of amniotic fluid is7 to 7.5. The latter turns a red litmus paper blue.

2. Nitrazine test: nitrazine paper becomes blue (pH > 6.5).3. Ferning: it is due to high estrogen content of amniotic fluid.4. Nile blue sulfate test: it stains fetal squamous cells orange.5. Presence of fetal lanugo hair, vernix, or meconium particles on microscopy.6. Evaporation test: a drop is heated on a slide. If a white residue is seen, it is

PROM. If it is brown, the membranes are intact.7. Biochemistry: levels of glucose, fructose, prolactin, alfafetoprotein, diamine

oxidase and fetal fibronectin are high in the amniotic fluid.8. Injection of a dye (Evans blue, methylene blue) into amniotic sac by transabdominal

amniocentesis and its recovery in vaginal fluid is an unnecessarily invasive testand not recommended.

Demonstration of oligohydramnios on ultrasonography supports the diagnosis ofPROM.

D. Gestational age is assessed (see chapter 63). A full term pregnancy isterminated by induction of labor (see chapter 105).

E. Fetal lung maturity is assessed by amniotic fluid analysis, as follows.1. Shake test: a mixture of equal parts of absolute alcohol and the amniotic fluid isshaken in a test tube for 15 seconds. If a ring of bubbles is formed and remainsstable for 15 minutes in a dilution of 1:2 or more, the fetal lungs are mature.2. L:S ratio: if it is > 2:1, the fetal lungs are mature.3. Foam stability index: a positive test at an ethanol concentration of 47% suggests

lung maturity.

4. Tap test: 1 ml amniotic fluid, 1 drop of 6N HCL, and 1.5 ml diethyl ether aremixed in a test tube and tapped briskly. If fetal lungs are mature, bubbles rise tothe surface and break.

5. Amniotic fluid creatinine: values > 2 mg % indicate maturity of fetal kidneys.6. Nile blue sulfate test: percentage of fat laden fetal epithelial cells staining

orange with Nile blue sulfate at different gestational ages is as follows.

Gestational age Cells (%)

< 32 032-34 0 - 134-37 1 - 1037 - 40 10 - 50

> 40 > 50

F. If the fetal lungs are not mature, it is necessary to continue the pregnancy in fetalinterest. Ascending infection and chorioamnionitis is a risk of such treatment. Vaginaland speculum examinations are not repeated, after the first examination at whichcervical swab is obtained for gonococcal culture. Clinical parameters suggestingdevelopment of chorioamnionitis include fetal tachycardia, maternal pyrexia,discharge of foul smelling amniotic fluid vaginally, tenderness over the uterus etc.However these are late indicators. Laboratory tests indicative of chorioamnionitis areas follows.

1. Leukocytosis (> 12000/cmm) and shift to left.2. Elevation of C-reactive protein (> 0.9 mg/dL)3. Amniotic fluid tests

a. Gram stain and/or culture.b. Leukocyte esterase assay: elevation.c. Glucose: lowering.

4. Biophysical profile: nonreactive NST and loss of breathing and gross bodymovements are associated with chorioamnionitis. The test has to be done everyday.

G. Conservative treatment includes bed rest, sterile vulvar pad, avoiding anyspeculum and vaginal examinations, and watching for signs of chorioamnionitis.Prophylactic antibiotics reduce maternal infection rate, but not fetal morbidity.Betamethasone is administered to hasten maturation of fetal lungs at gestational agesbelow 34 weeks in a dose of 12 mg IM repeated after 12 hours. Local injection offibrin to plug the hole in membranes, or use of PROM fence, though promising, needfurther studies before being recommended for routine use.

Complications of PROM include chorioamnionitis, abruptio placentae, fetal distressdue to cord compression, hyaline membrane disease, pulmonary hypoplasia and fetaldeformities.

203

Page 210: Differential Diagnosis and Management Options inuploads.worldlibrary.org/uploads/pdf/20180107071525differential... · 83 Peripartum cardiomyopathy 165 84 Hypertension 167 85 Convulsions

VAGINAL LEAKAGE OF FLUID

Examine underclothes (A)

Dry Wet

Sterile pad to vulva (B) Check odor

Pad remains dry Pad gets wet Collect fluid by speculum examination toconfirm diagnosis (C)

Urinary

Observation PROM Not PROM Involuntaryloss of urine

Assess gestational age (D)

> 37 weeks 34-37 weeks 28-34 weeks < 28 weeks

Assess for chorioamnionitis Assess for chorioamnionitis Assess degree of leak

Present Absent Absent Present Small,stops

Large,does not

stop

Tocolysis Tocolysis, betamethasone Observation

Deliver after 24 hours

Induction of labor

204

Page 211: Differential Diagnosis and Management Options inuploads.worldlibrary.org/uploads/pdf/20180107071525differential... · 83 Peripartum cardiomyopathy 165 84 Hypertension 167 85 Convulsions

103. Prolonged PregnancyA postdated pregnancy is one which has continued for more than 2 weeks beyond theexpected date of delivery. Such a condition leads to functional placental insufficiencybecause the fetus outgrows the placenta. Such a fetus is compromised and may die inthe uterus before labor or during labor.

A. Wrong dates are far more common than truly postdated pregnancy. It is surprisinghow often a woman remembers the date but not the month of her last menstrualperiod. Addition of 9 months and 7 days to the date would give a wrong expecteddate of delivery. Another source of error is a failure to check the cyclicity of hermenstruation. If her cycles were longer than 28 days, the calculation of the due dateby the formula quoted is wrong. See chapter 63 for accurate calculation of due date.

B. It is preferable that one does not wait for 2 weeks beyond the due date, but startsevaluation 1 week after the date, because some fetal compromise is likely to occurafter this time. Daily fetal movement count charting (subjective or Cardiff count to10 method) is a useful method to monitor fetal well-being. It involves the gravida inthe management of her pregnancy, which she appreciates too. A nonstress test (NST)is the most popular method of assessment of fetal well being. If there are at least 2episodes of fetal movements, each associated with an acceleration of the fetal heart rate(FHR) by at least 15 bpm lasting for at least 15 seconds, the NST is said to bereactive and the fetus is said to be well. A fetus usually remains well for 1 week aftera reactive NST. Fetal biophysical profile is another useful method of monitoringfetal well being. However its value over the NST as a screening test remains to beproved.

C. If the NST is nonreactive, it is repeated after 2 hours to rule out temporary factorslike sleep. If it is still nonreactive, a stress test is done to find out if the fetus is trulycompromised, or the test is false positive. A vibroacoustic stimulation test is anoninvasive stress test. If the fetus shows an acceleration of its heart rate of >15 bpmlasting for > 15 sec on application of vibroacoustic stimulus of 2000 Hz, it isconsidered to be well. If it does not, its well-being is said to be compromised. Acontraction stress test is more invasive. Uterine contractions are established bymonitored infusion of oxytocin in this test. If repetitive late decelerations are seenwith a contractility of 3 per 10 minutes or less, each contraction lasting for 60seconds or less, the test is said to be positive, and the fetus compromised. Such a

fetus is managed by induction of labor and continuous monitoring of fetal well beingduring labor by electronic FHR monitoring and scalp blood analysis as required.Some obstetricians prefer to perform a cesarean section rather than induce labor,claiming that a fetus who cannot tolerate uterine contractions of the test cannot standthe stress of the labor too.

D. If the NST becomes nonreactive while it was reactive a week ago, the fetus ispresumed to be compromised. Whether the NST is reactive or not, labor is induced at42 weeks because nothing is gained by continuing the pregnancy further. Well beingof both fetuses needs to be monitored intensively during labor, though the latter ismore likely to develop intrapartum fetal distress.

Causes of Postdatism

Advanced maternal agePrimigravidityDelayed ovulationHeredityPostdatism in a past pregnancyAnencephaly without hydramniosPlacental sulfatase deficiencyTrisomy 16-18PG synthetase inhibitor therapy

Features of Postmaturity

MacrosomiaHead: large, hard skull bones, narrow sutures, small fontanelles, thick marginsof cranial bones.Nails: longSkin: absence of vernix caseosa, deep sole creases, meconium staining.Birth asphyxiaDehydrationHypoglycemiaTemperature instabilityRespiratory distress

205

Page 212: Differential Diagnosis and Management Options inuploads.worldlibrary.org/uploads/pdf/20180107071525differential... · 83 Peripartum cardiomyopathy 165 84 Hypertension 167 85 Convulsions

Rule out wrong dates (A)

PROLONED PREGNANCY

Nonstress test at 41 weeks (B)

Reactive Nonreactive (C)

Repeat after 1 week Repeat after 2 hours

Reactive Nonreactive

Reactive Nonreactive (D) Vibroacoustic stimulation test / contraction stress test

Induction of laborIntrapartum monitoring of fetal heart rateScalp blood analysis as required

Positive VASTNegative OCT

Negative VASTPositive OCT

Fetus remains well Fetus unwell Repeat NST and if necessary VAST/OCT after1 week

Cesarean section

Vaginal delivery Immediate delivery

206

Page 213: Differential Diagnosis and Management Options inuploads.worldlibrary.org/uploads/pdf/20180107071525differential... · 83 Peripartum cardiomyopathy 165 84 Hypertension 167 85 Convulsions

104. Reduced Fetal MovementsA. Fetal movements are first perceived between 16 and 18 weeks by a multipara, andbetween 18 and 20 weeks by a nullipara. The movements increase up to 32 weeks,then they plateau and then there is reduction in movements up to the onset of labor.A healthy fetus maintains its own rhythm of movements. Fetal movements arediminished with compromise of fetal well-being. Loss of fetal movements indicatesthat the fetus is either dead, or unwell and likely to die in the next 24 hours. Hencewhen a gravida presents with loss of fetal movements, auscultation is done for fetalheart sounds. If heart sounds are present, the fetus is alive, but needs tests forassessment of its well being (see chapter 144).

B. If the heart sounds are not heard, a diagnosis of intrauterine fetal death (IUFD) ismade. It is confirmed by real time ultrasonography, which shows an absence of fetalheart movements. The cause of the fetal death is assessed. Various causes of IUFDare as follows.

1. Obstetric: pregnancy induced hypertension, abruptio placentae, placenta previa,true knots in umbilical cord.

2. Medical: hypertension, chronic nephritis, diabetes mellitus, severe anemia,severe cardiorespiratory disease, syphilis, toxic drugs.

3. Fetal: postdatism, Rh-isoimmunization, congenital malformations.4. Iatrogenic: injury to vital organ during amniocentesis or intrauterine

transfusion.

C. Radiographic features of IUFD are as follows.1. Gas appears in fetal heart, great vessels, umbilical vessels, cerebral ventricles,

and subarachnoid space as early as 12 hours after fetal death.2. Overlapping of cranial bones (Spalding sign) is seen 7 to 10 days after IUFD,

due to maceration of the brain. The Spalding sign can be seen onultrasonography too.

3. Accumulation of fluid in subaponeurotic space causes an elevation of a thin,dark fat line around the head (halo sign).

4. Due to the loss of muscle tone, the fetus becomes hyperflexed (ball sign).

D. Disseminated intravascular coagulation (DIC) and consumptive coagulopathy areseen due to release of thromboplastic material from the placenta into the maternalcirculation. It does not occur if the IUFD is recent. It is rare up to 5 weeks fromIUFD. Most of the cases are seen after 5 weeks.

E. If the patient has DIC as shown by abnormal results of the coagulation studies,she is assessed for the presence of active bleeding from any site. If the continuity ofthe vascular tree is broken anywhere, she bleeds uncontrollably from that site. Insuch cases the clotting abnormality is corrected by administration of fresh blood,fresh frozen plasma, cryoprecipitate, and platelets as required. Then the pregnancy isterminated.

F. See chapter 105 for induction of labor. Dead products of conception serve as anexcellent culture medium for bacteria. Hence surgical methods of induction of laborare contraindicated in IUFD, and only medical methods are used.

Table 104.1 Coagulation Studies In Pregnancy

Test Nonpregnant First trimester Second trimester Third trimesterBleeding time (min) 1-9 1-9 1-9 1-9Clotting time (min) 8-15 8-15 8-15 8-15Platelet (x109/L) 165-415 174-391 155-409 146-429D-dimer (μg/mL) 0.22-0.74 0.05-0.95 0.32-1.29 0.13-1.7Fibrinogen (mg/dL) 211-496 244-510 291-538 301-696Prothrombin time (sec) 12.7-15.4 9.7-13.5 9.5-13.4 9.6-12.9International Normalized Ratio 0.9-1.04◊ 0.86-1.08 0.83-1.02 0.80-1.09Partial thromboplastin time, activated (sec) 26.3-39.4 23.0-38.9 22.9-38.1 22.6-35.0

207

Page 214: Differential Diagnosis and Management Options inuploads.worldlibrary.org/uploads/pdf/20180107071525differential... · 83 Peripartum cardiomyopathy 165 84 Hypertension 167 85 Convulsions

REDUCED FETAL MOVEMENTS

Auscultate for fetal heart sounds (A)

Present Absent

Assess fetal well being Ultrasonography

See chapter 144 Live fetus IUFD

Assess cause (B)

Assess time since fetal death by ultrasonography and radiography (C)

Recent < 5 weeks > 5 weeks

Coagulation profile (D)

Induction of labor Normal DIC

See chapter 105 Control DIC, induction of labor

208

Page 215: Differential Diagnosis and Management Options inuploads.worldlibrary.org/uploads/pdf/20180107071525differential... · 83 Peripartum cardiomyopathy 165 84 Hypertension 167 85 Convulsions

105. Induction of LaborA. The Bishop score is used to assess the likelihood of the patient going into labor byinduction.

Table 105.1 Bishop Scoring System

Factor Score

0 1 2 3 Total

Dilatation (cm)Effacement (%)StationConsistency of cervixPosition of cervix

Closed0-30

-3Firm

Posterior

1-240-50

-2Medium

Mid

3-460-70-1,0Soft

Anterior

> 5> 80

+1,+2--

0-30-30-30-20-20-130-13

A score of 9 or more is associated with a successful outcome, while a score of 5 orless is considered unfavorable. In case of a score above 8, the interval to labor isabout 3 days, while if the score is below 2, the interval is more than 3 weeks.

B. If the Bishop score is up to 8, the cervix needs to be ripened before inductionof labor. Dinoprostone gel (0.5 mg) is inserted into the posterior fornix of the vagina.After application she remains in bed for at least 30 minutes. It is a prostaglandin E2

derivative. It results in structural changes in the cervix, which causes improvement inBishop score. Such results are obtained in 6 to 12 hours. A variable proportion ofgravidas go into spontaneous labor with this technique. If the score improves, butstill remains below 9, doses may be repeated every 6 hours, with a maximum ofthree doses recommended in 24 hours. This method should not be used when there isa scar on the uterus. Alternatively one may insert a Foley's catheter through thecervix and inflate the balloon to its maximum capacity.

C. Foley's catheter ripens the cervix by stripping the membranes off the lowersegment, which causes local synthesis and release of prostaglandins. The catheter iseither spontaneously expelled in 4 to 6 hours, or is removed. It causes cervicaldilatation to about 3 cm, though the effacement is usually poor. These gravidasusually do not go into labor spontaneously, and they need artificial low rupture ofmembranes followed by administration of an oxytocin infusion. Other methods ofcervical ripening include the following.

1. Misoprostol 25 μg PV q4h, maximum 6 doses. It may cause uterine rupturein a case of previous cesarean section.

2. Nipple stimulation: it is not recommended, because it is done by the patientherself, and control is difficult.

3. Intracervical laminaria tents: it is useful, but not very popular.4. Mifepristone: 200 mg PO has shown promising results.5. Oxytocin infusion: it has been shown to be ineffective as a cervical

ripening method.

D. Oxytocin is the most commonly used drug for induction of labor. It should not bestarted less than 6 hours from the last dose of dinoprostone gel or less than 4 hoursafter the last dose of misoprostol. A solution of 10 units of oxytocin in 1 L ofRinger's lactate is infused intravenously, starting with a dose of 0.5 mU/min, andincreasing the dose by 1 mU/min every 30-40 minutes (low dose) or starting with adose of 6 mU/min, and increasing the dose by 6 mU/min every 20-40 minutes (highdose) until a uterine activity of 3 per 10 minutes is established, each contractionlasting for about 60 seconds. Advantages of the high dose regimen are a shortermean induction-to-delivery time, fewer failed inductions, fewer forceps deliveries,fewer cesarean deliveries for dystocia, and decreased intrapartum chorioamnionitisor neonatal sepsis. Alternative methods for induction of labor include the following.

1. Oral dinoprostone tablets: 0.5 mg tablet is administered every 1 hour, monitoringall parameters as with oxytocin infusion.

2. Stripping of membranes: the membranes are separated from the lower segmentwith fingers passed in through the cervix. This causes local synthesis andrelease of prostaglandins. This method needs cervical dilatation of 1 to 2 cm,and is likely to be successful only if the Bishop score is above 8.

3. Artificial rupture of membranes: low rupture of membranes is achieved withthe use of a membrane hook or Smyth's amniotomy forceps. It acts by causinglocal synthesis and release of prostaglandins. Different workers recommendadministration of oxytocin infusion 6, 12, or 24 hours later if labor does notstart. It may cause cord prolapse, dry labor, and chorioamnionitis. It iscontraindicated in cases of intrauterine fetal death from fear of fulminantinfection. It is not recommended because safer and more effective methods areavailable. It may however be used to augment labor if required. Artificial highrupture of membranes with Drew Smyth's catheter is obsolete.

4. Oil-enema-bath : a combination of 30 ml castor oil orally at 6 am, a simpleenema at 8 a.m. and a refreshing bath at 9 a.m. has been conventionally used. Itmay induce labor when the Bishop score is above 8 in a few cases. However itcannot be relied on for successful induction.

5. Nipple stimulation: it causes release of oxytocin from the posterior pituitaryreflexly, which causes labor. It is not recommended because it is done by thegravida herself, and overstimulation is possible. It is preferable to give the endproduct (i.e. oxytocin) itself in a well monitored manner.

209

Page 216: Differential Diagnosis and Management Options inuploads.worldlibrary.org/uploads/pdf/20180107071525differential... · 83 Peripartum cardiomyopathy 165 84 Hypertension 167 85 Convulsions

INDUCTION OF LABOR

Assess Bishop score (A)

0-4 5-8 > 9

Intracervical PGE2 gel (B) Oxytocin infusion

Labor begins Bishop score < 8 Bishop score > 9 Labor begins Labor does not begin

Observation Intracervical Foley’s catheter (C) Observation Overnight rest followed by oxytocin infusionnext morning

Bishop score > 9 Bishop score < 8 Artificial low ruptureof membranes ifprogress is slow

Labor does not begin

Oxytocin infusion Artificial low rupture ofmembranes followed by oxytocin

infusion

Vaginal delivery Artificial low rupture of membranes pluscontinuation of oxytocin infusion

Labor begins Labor begins Labor begins Labor does not begin

Vaginal delivery Cesarean section

210

Page 217: Differential Diagnosis and Management Options inuploads.worldlibrary.org/uploads/pdf/20180107071525differential... · 83 Peripartum cardiomyopathy 165 84 Hypertension 167 85 Convulsions

106. OccipitoposteriorPosition

A fetus in a vertex presentation is said to be in occipitoposterior (OP)position when its occiput lies posterior to the 3 or 9 o'clock position. Causes of sucha position are as follows.

1. Reduced transverse diameters of the pelvis.2. Prominent ischial spines.3. Android pelvis.4. Anthropoid pelvis, often with sacralization of a lumbar vertebra.5. Increased convergence of the lateral pelvic walls or a straight sacrum.6. Extension of fetal head.7. Premature rupture of membranes and dry labor.8. Uterine inertia.9. Premature use of conduction anesthesia.10. Excessive lumbar lordosis.11. Anterior placenta.

A. Occipitoposterior position is a variant of normal, and need not cause any concernif diagnosed antenatally. The patient may be advised to sleep in the opposite lateralposition to convert it to occipitoanterior position. If the patient presents in pretermlabor, it is treated as in chapter 101.

B. If the gravida presents in labor with the fetus in OP position, and the pelvis isfound to be contracted to a major degree, a cesarean section is done. A trial of laborcan be given if there is a borderline pelvic contraction (see chapter 113).

C. Ambulation during labor makes the uterus fall forwards a little, so that thefetal long axis comes more in line with the axis of the inlet, and an anteriorrotation of the occiput is promoted.

D. Omen's criteria for probable anterior rotation and spontaneous vaginal deliveryare as follows.

1. Early fixation of fetal head in a pelvis of good size and shape.2. Membranes intact until the cervix is 2/3 dilated.3. Anterior shoulder not too far in the flank.4. Normal sized head.5. Bituberous diameter not less than 10 cm.6. Good driving forces.

The progress of labor is observed. A normal delivery is achieved if the occiputrotates anteriorly. If it rotates posteriorly and the pelvis roomy, a face to pubis delivery canoccur. A generous episiotomy helps prevent perineal tears in such cases. If there isOccipitoposterior arrest, a cesarean section is done.

E. If there is an arrest of rotation in the transverse position of the occiput and thepelvis is adequate, vaginal delivery can be achieved with vacuum extraction atstations below zero. For higher stations, cesarean section is done. Various types offorceps rotations have been described in literature. Forceps rotation is notrecommended because of the possible morbidity for the mother and the fetus. Othermethods for management of arrest of rotation are as follows.

1. Digital rotation.2. Manual rotation

a. Hollandb. Wilsonc. Pomeroyd. Puddicomb

3. Maughan's maneuver.

These methods are rarely used in modern obstetrics.

F. Neglected OP position in labor with arrest of progress is not common withgood antenatal and intrapartum care. It may still be seen in developing countrieswhere health care may not be readily available to every one. Various possibilitiesin such a situation are as follows.

1. Acute fetal distress: it is managed by a cesarean section.2. Intrapartum fetal death: it is due to intrapartum asphyxia. There may or may not

be threatened rupture of the uterus. Signs of threatened rupture are tachycardia,hematuria, tenderness over lower uterine segment, progressively rising retractionring of Bandl etc. If such features are absent, a craniotomy is done so as toreduce the size of the fetal head and permit a safe vaginal delivery. If such signsare present, craniotomy is contraindicated as it can precipitate uterine rupture.A cesarean section is done even if the fetus is dead.

3. Uterine rupture: after resuscitation of the patient, an exploratory laparotomy isdone. The fetus is removed. Then either the uterine injury is repaired or ahysterectomy is done if the former is not possible. In case of the former, it ispreferable to sterilize the woman so as to prevent uterine rupture in a futurepregnancy.

211

Page 218: Differential Diagnosis and Management Options inuploads.worldlibrary.org/uploads/pdf/20180107071525differential... · 83 Peripartum cardiomyopathy 165 84 Hypertension 167 85 Convulsions

OCCIPITOPOSTERIOR POSITION

Assess gestational age (A)

Preterm Term

Assess features of labor Assess pelvic capacity (B)

Present Absent Contracted Normal

See chapter 101 Continue antenatal careAdvise to sleep in opposite lateral position

Cesarean section Ambulation in labor until head is engaged (C)Observation during labor (D)

Conversion to occipitoanteriorposition

Posterior rotation of occiput Occipitotransverse arrest (E) Neglected, obstructed labor (F)

Normal delivery Face to pubisdelivery

Occipitoposteriorarrest

Assess station Fetal distress Fetal death Uterine rupture

Above 0 0 to +2 Cesarean section Exploratorylaparotomy

Vacuumextraction

Assess for threatened rupture of uterus

Fails Succeeds Absent Present

Cesarean section Craniotomy Cesarean section

212

Page 219: Differential Diagnosis and Management Options inuploads.worldlibrary.org/uploads/pdf/20180107071525differential... · 83 Peripartum cardiomyopathy 165 84 Hypertension 167 85 Convulsions

107. Breech PresentationA. Breech presentation is the presentation of the podalic extremity of a fetuscomprising of buttocks with/without knee or foot, the fetus being in longitudinal lie.The incidence is 1 in 30 at term, and 20 to 30% before 32 weeks. Most of the fetusesspontaneously turn and correct their presentation and hence no action need be takenif a breech presentation is diagnosed prior to 32 weeks of pregnancy.

B. After 32 weeks, a breech presentation can be converted into vertex presentationby external cephalic version (ECV), unless there is a contraindication for the same.Contraindications for ECV include the following.

1. Multiple pregnancy.2. Antepartum hemorrhage: placenta previa, abruptio placentae.3. Severe pregnancy induced hypertension, eclampsia.4. Hydrocephalus.5. Major degree of pelvic contraction.6. Scar on the uterus: cesarean section, myomectomy, metroplasty.7. Rh-isoimmunization.8. Placental insufficiency.9. Preterm labor.

10. Threatened rupture of uterus.11. Bicornuate uterus.

C. External cephalic version is best done between 34 and 36 weeks. It can bedone even later, up to the time of rupture of membranes in labor (if the breechis floating), but is less likely to succeed then due to relatively larger size of thefetus and less volume of the amniotic fluid. It can fail between 32 and 36 weeksdue to fetal macrosomia, oligohydramnios, obesity, uterine leiomyomas, uterineanomalies (septate uterus, unicornuate uterus) etc. Its complications are as follows.

1. Preterm labor.2. Abruptio placentae.3. Looping of the umbilical cord around the fetus.4. Umbilical cord presentation.5. True knots in the umbilical cord.6. Compound presentation.7. Fetomaternal hemorrhage.8. Intrauterine fetal death.9. Premature rupture of membranes.

If the patient is very uncooperative, ECV may be done under sedation. It is neverdone under anesthesia, because the patient cannot complain of pain and too muchforce may be used, causing complications.

D. If a gravida presents in labor with the fetus in breech presentation, and themembranes are still intact, an ECV may be attempted provided there are nocontraindications. Indications for a cesarean section are as follows.

1. Pelvic contraction.

2. Large fetus.3. Elderly primigravida.4. Previous cesarean section.5. Placental insufficiency.6. Placenta previa.7. Cord presentation or prolapse.8. Extension of fetal head.9. Uterine dysfunction.10. Previous perinatal death or children suffering from birth trauma.

All other cases are observed in labor and managed by assisted breechdelivery. No intervention is done until the umbilicus is born. A loop of cord is drawndown to confirm adequate length for delivery. No further action is taken exceptdelivering the lower limbs, until the inferior angles of the scapulae are seen. Theupper limbs are then delivered. Further delivery is observed until the occipital hairline is seen. Then the aftercoming head is delivered by Mauriceau-Smellie-Veitmethod (jaw flexion-shoulder traction) or by outlet forceps (Piper). The othermethods are not as convenient as these two.

E. Breech extraction is a procedure in which intervention is done before theumbilicus is born, or any intrauterine manipulation is done. The procedure isrequired for the following situations.

1. Arrest of breech on the perineum: if a generous episiotomy and augmentationof labor as required do not work, the breech is delivered by groin traction. Ifthat fails in case of a frank breech, the lower limbs are delivered by Pinard'smaneuver.

2. Delay in the delivery of shoulders: dorsal or nuchal displacement of arms ismanaged by Lovset’s maneuver or Munro Kerr-Chassar Moir maneuver. Thesemaneuvers need rotation of the fetus, and like other breech extractionmaneuvers, uterus relaxing general anesthesia.

Besides the situations described above, breech extraction is required for the followingindications.

1. Following internal podalic version at full dilatation of the cervix.2. Acute fetal distress.3. Maternal distress.4. Maternal complications contraindicating bearing down, e.g. heart disease (classIII and IV), severe pregnancy induced hypertension, advanced pulmonarytuberculosis.

F. A baby delivered as a breech needs extra care. The following complicationsshould be looked for and treated of present.

1. Birth asphyxia.2. Other birth injuries: intracranial hemorrhage, cervical spinal cord injury,

Erb’s palsy, Klumpke’s palsy, hepatic tear, splenic tear, adrenal injury,fracture of humerus fracture of femur, dislocation of shoulder,sternomastoid tumor etc.

3. Congenital anomalies.

213

Page 220: Differential Diagnosis and Management Options inuploads.worldlibrary.org/uploads/pdf/20180107071525differential... · 83 Peripartum cardiomyopathy 165 84 Hypertension 167 85 Convulsions

Ultrasonography

BREECH PRESENTATION

Assess gestational age (A)

< 32 weeks > 32 weeksRule out contraindications (B)

Continue antenatal care External cephalic version (C)

Succeeds Fails

Repeat under sedation

Manage as in a vertex presentation Succeeds Fails

Continue antenatal care

Check membranes when patient presents in labor (D)

Membranes intact Membranes ruptured

External cephalic version Cesarean section for specific indications Assisted breech delivery

Succeeds Fails Assisted breech delivery Succeeds Fails

Normal delivery Care of neonate delivered by breech (F) Breech extraction (E)

214

Page 221: Differential Diagnosis and Management Options inuploads.worldlibrary.org/uploads/pdf/20180107071525differential... · 83 Peripartum cardiomyopathy 165 84 Hypertension 167 85 Convulsions

108. Transverse LieA. A fetus is said to be in a transverse lie or an oblique lie when its long axiscrosses the long axis of the mother, usually obliquely and rarely transversely.It occurs 1 in 150 to 200 deliveries. Causes of a transverse lie are as follows.

1. Contracted pelvis.2. Placenta previa.3. Fundal implantation of placenta.4. Uterine malformations: subseptate uterus, arcuate uterus.5. Pelvic tumors: leiomyoma, ovarian tumor.6. Hydramnios.7. Lax abdominal wall and anteflexion of the uterus.8. Multiple pregnancy.9. Prematurity.

10. Advanced abdominal ectopic pregnancy.

B. Gestational age of the fetus is assessed by ultrasonography (see chapter 63).It is also useful to determine the cause of the transverse lie, and the presenceof any congenital malformations.

C. Owing to the relatively small size of the fetus and greater volume of theamniotic fluid, the fetal presentation keeps changing prior to 32 weeks of gestation,and hence no attempts are made at its correction. After 32 weeks, if there are nocontraindications, an external cephalic version (ECV) is done (see chapter 107).

D. If the gravida presents in labor with the fetus in transverse lie, and the membranesare still intact, ECV may still be attempted. But it is less likely to work at this time.Indications for a cesarean section are as follows.

1. Umbilical cord prolapse.2. Insuperable obstruction to vaginal delivery.3. First of twins in transverse lie.4. Failure of ECV.5. Failure of internal podalic version (IPV).6. Acute intrapartum fetal distress.7. Threatened rupture of uterus.8. Previous cesarean section.

E. An (IPV) for a transverse lie is done in modern obstetrics for very fewindications, such as the following.

1. Second of twins in transverse lie, and ECV fails.2. Congenital malformation of the fetus incompatible with life.3. Fetal death.4. Fetus too preterm to survive.

Contraindications for IPV are as follows.

1. A scar on the uterus: cesarean section, myomectomy, metroplasty.2. Threatened rupture of the uterus.3. Multiple pregnancy.4. Congenital malformation of the uterus: subseptate uterus, bicornuate uterus.5. Major degree of pelvic contraction.6. Placenta previa degree III and IV.

IPV is done under uterus relaxing general anesthesia. A hand is passed intothe uterus and a lower limb of the fetus is drawn out in such a way that the backbecomes anterior. The fetus is then delivered as a breech presenting fetus (seechapter 107). One must keep in mind that IPV is the most dangerous operation inobstetrics. Its complications include uterine rupture, traumatic or atonic postpartumhemorrhage, puerperal sepsis, and fetal injuries.

F. In developing countries, health care may not be available to all women readily, sothat some gravidas may present with obstructed labor. See chapter 106 for featuresof threatened rupture of the uterus. If such features are absent, a decapitation is done.Under uterus relaxing general anesthesia, the fetal neck is cut off with decapitationsaw, decapitation knife, Blond-Heidler's saw, or decapitation hook and embryotomyscissors. The trunk and lower limbs are then delivered, followed by the head held bya bulldog vulsellum. In case there are features of threatened rupture of the uterus, acesarean section is done, as a decapitation operation may precipitate the rupture. Ifthe uterus has already ruptured, the patient is resuscitated and then an exploratorylaparotomy is done. The fetus is removed, and the uterine injury is repaired ifpossible. This is combined with a sterilization operation so as to avoid a uterinerupture in a future pregnancy. If it cannot be done, a total abdominal hysterectomy isdone.

215

Page 222: Differential Diagnosis and Management Options inuploads.worldlibrary.org/uploads/pdf/20180107071525differential... · 83 Peripartum cardiomyopathy 165 84 Hypertension 167 85 Convulsions

Assess cause(A)

TRANSVERSE LIE

Assess gestational age (B)

< 32 weeks > 32 weeks

Rule out contraindications (C)

Continue antenatal care External cephalic version

Succeeds Fails

Repeat under sedation

Manage as vertex presentation Succeeds Fails

Continue antenatal care, Check membranes in labor (D)

Intact Ruptured

External cephalic version Internal podalic version for (E) Cesarean section for Obstructed labor (F) Uterine rupture

Succeeds Fails Second of twinFetus with malformationsincompatible with lifeDead fetusFetus too preterm to survive

Cord prolapseNormal fetusContracted pelvisFetal distressObstructed laborThreatened rupture of uterus

Assess for threatened ruptureof uterus

Exploratorylaparotomy

Normal delivery Cesarean section Absent Present

Decapitation Cesarean section

216

Page 223: Differential Diagnosis and Management Options inuploads.worldlibrary.org/uploads/pdf/20180107071525differential... · 83 Peripartum cardiomyopathy 165 84 Hypertension 167 85 Convulsions

109. Face PresentationWhen the presenting part of a fetus in a longitudinal lie is the face (i.e. the area of thecephalic pole between the chin at one end, the nasion and the supraorbital ridges atthe other, and the malar prominences on either side), it is called face presentation. Itis found in 1 in 500 labors at term.

A. Ultrasonography confirms the diagnosis and the position. It also helps find thecause of the face presentation. Anencephaly should really be diagnosed in the secondtrimester by an ultrasonography at 16 weeks, and then the pregnancy should beterminated. However a routine scan of every gravida is not possible in a developingcountry owing to lack of resources, and some cases do present at term. They areallowed to go in labor and deliver, after correction of diabetes mellitus, if found to bepresent. Other causes of face presentation are as follows.

1. Pendulous belly.2. Obliquity of the uterus.3. Pelvic contraction.4. Uterine leiomyomas.5. Ovarian tumor in the pouch of Douglas.6. Placenta previa.7. Iniencephaly.8. Reversal of flexor dominance.9. Neck mass: goiter, branchiocele, cystic hygroma.

10. Fetal macrosomia.

B. If face presentation is associated with a normal fetus before term, antenatal care iscontinued. There is no place for conversion maneuvers like Thorn's or Schatz'smaneuver in modern obstetrics. Such maneuvers usually fail. If a maneuver worksonly partly, the face presentation changes into a brow presentation which is evenworse. If the woman is in preterm labor, it is controlled (see chapter 101).

C. If the pelvis is contracted to any degree, a cesarean section is done. If it isadequate, a vaginal delivery is permitted. A mentoanterior face presentation hasbetter prognosis for vaginal delivery than mentoposterior face presentation. If laborprogresses, a vaginal delivery is achieved. If there is an arrest of progress of labor, acesarean section is done. Vacuum delivery is contraindicated because it would cause

severe damage to the facial structures, especially eyes. Rotation forceps operation isnot recommended for the same reasons as in vertex presentation (see chapter 106).

D. Indications for cesarean section are as follows.

1. Pelvic contraction.2. Mentotransverse or mentoposterior arrest.3. Arrest of progress of labor.4. Umbilical cord prolapse.5. Fetal distress.6. Obstructed labor with threatened rupture of uterus.

E. Owing to inadequate health care facilities in some parts of the developing world,some gravidas present with obstructed labor and intrapartum fetal death due toasphyxia. If features of threatened rupture of the uterus are absent, a craniotomy is donethrough fetal orbits. If features of threatened rupture are present (see chapter 106), acesarean section is done even if the fetus is dead, because a craniotomy in such acondition can precipitate a rupture of the uterus.

F. If the patient presents after uterine rupture due to obstructed labor, resuscitationis done. Then an exploratory laparotomy is done. The fetus is removed. Theuterus is repaired if possible. If that is not possible, total hysterectomy is done.If the uterus is repaired, a sterilization operation should be combined with itso as to prevent uterine rupture in a future pregnancy.

G. A neonate which has been delivered in face presentation can have certainproblems which need special care.

1. Caput succedaneum forms over the face. The effusion is usually sanguinolent,giving the appearance of considerable bruising. It gets totally absorbed in a fewdays. Reassurance to the parents is the only treatment required.

2. Severe swelling of the lips and cheeks may make feeding extremely difficult, sothat tube feeding may have to be resorted to for a few days.

3. Edema of the glottis and fracture of the larynx due to pressure of the sacrummay be seen in mentoposterior arrest.

4. Birth asphyxia may be seen due to umbilical cord prolapse or obstructed laborand intrapartum asphyxia.

217

Page 224: Differential Diagnosis and Management Options inuploads.worldlibrary.org/uploads/pdf/20180107071525differential... · 83 Peripartum cardiomyopathy 165 84 Hypertension 167 85 Convulsions

FACE PRESENTATION

Ultrasonography (A)

Anencephaly Normal fetus

Labor at term to achieve vaginal delivery Assess gestational age (B)

Preterm Term

See chapter 101 Assess pelvic capacity (C)

Normal Contracted

Monitor during labor Cesarean section (D)

Normal progress Cesarean section for (D) Neglected obstructed labor with fetal death (E) Uterine rupture (F)

Vaginal delivery Assess for threatened rupture of uterus Exploratory laparotomy

Present Absent

Neonatal care (F) Cesarean section Craniotomy

218

Page 225: Differential Diagnosis and Management Options inuploads.worldlibrary.org/uploads/pdf/20180107071525differential... · 83 Peripartum cardiomyopathy 165 84 Hypertension 167 85 Convulsions

110. Brow Presentation

A fetus in a longitudinal lie and cephalic presentation is said to be in a browpresentation when the presenting part is the area of the fetal head bounded by thesupraorbital ridges and nasion at one end, anterior fontanelle and the coronal sutureat the other end, and longitudinal lines passing through the frontal eminences on thesides. It occurs in 1 in 600 labors at term. Many of these are transient. Persistentbrow presentation is found in 1 in 1000 deliveries.

A. Ultrasonography is an important investigation in brow presentation. It confirms thediagnosis and the position of the fetus. It helps find the cause of the browpresentation and diagnose the presence of congenital malformations, if any. Thecauses of a brow presentation are similar to those of face presentation (see chapter106).

B. If the fetus has a major congenital malformation which is not compatible with life,spontaneous labor is awaited. The presentation changes to either vertex or facepresentation and the fetus delivers vaginally. If it persists as a brow presentation, acraniotomy is done through one of the frontal bones.

C. If the gravida presents in labor and the fetus is normal, preterm labor is controlledwith tocolytic therapy (see chapter 101), while term labor is allowed to progress ifthe pelvis is adequate. Conversion of the brow presentation into a vertex or facepresentation is watched for. If it takes place, a vaginal delivery is anticipated. If thebrow presentation persists, a cesarean section is done. Other indications for acesarean section are as follows.

1. Contracted pelvis.2. Umbilical cord presentation or prolapse.3. Fetal distress.

4. Fetal neck tumors.5. Large baby.6. Obstructed labor with threatened rupture of the uterus.

There is no place for conversion maneuvers, Kielland’s forceps operation oruse of a vacuum extractor in the management of a brow presentation.

D. Owing to inadequate health care facilities in some parts of the developingcountries, some gravidas may present with obstructed labor due to brow presentation.If the fetus is alive but distressed, a cesarean section is done. If the fetus is alreadydead due to intrapartum asphyxia, features of threatened rupture of the uterus arelooked for (see chapter 106). If those are absent, a craniotomy is done through afrontal bone. If those are present, a cesarean section is done even if the fetus is dead,because a craniotomy could precipitate a rupture of the uterus in such a situation. Ifthe uterus has already ruptured by the time the gravida presents herself to the laborroom, resuscitation is done followed by an exploratory laparotomy. The fetus is thenremoved and the uterine injury is repaired if it is possible todo so. This should be combined with a sterilization operation so as to avoid uterinerupture in a future pregnancy. If it is not possible to repair the uterus, a totalhysterectomy is done.

E. The fetus in a brow presentation is liable for the following complications, whichneed to be watched for and treated appropriately.

1. Intracranial hemorrhage.2. Birth asphyxia.3. Fracture of the trachea and/or larynx due to compression of the neck against thematernal pubis.

(Note: it is risky to perform an embryotomy operation like craniotomy withoutadequate experience. It may be better to perform a cesarean section even if the fetusis dead or has a malformation incompatible with life.)

219

Page 226: Differential Diagnosis and Management Options inuploads.worldlibrary.org/uploads/pdf/20180107071525differential... · 83 Peripartum cardiomyopathy 165 84 Hypertension 167 85 Convulsions

BROW PRESENTATION

Ultrasonography (A)

Major malformations incompatible with life (B) Normal

Await spontaneous labor Assess gestational age

Conversion to vertexpresentation

Conversion to facepresentation

Persistence of browpresentation

Term Preterm

Normal delivery Vaginal delivery Craniotomy Assess pelvic adequacy (C) See chapter 101

See chapter 109 Contracted Adequate

Cesarean section Monitor during labor

Conversion to vertexpresentation

Conversion to facepresentation

Persistence of browpresentation

Fetal distress Cord prolapse Neglected obstructed labor(D)

Normal delivery See chapter 109 Fetal distress Fetal death Uterine rupture

Assess for threatened rupture of uterus Exploratory laparotomy

Present Absent

Cesarean section

Neonatal management (E)

220

Page 227: Differential Diagnosis and Management Options inuploads.worldlibrary.org/uploads/pdf/20180107071525differential... · 83 Peripartum cardiomyopathy 165 84 Hypertension 167 85 Convulsions

111. Compound PresentationA fetus is said to be in compound presentation when an extremity prolapses

alongside the presenting part (hand or foot with cephalic presentation, hand withbreech presentation), with both entering the maternal pelvis simultaneously.

A. Ultrasonography is done to confirm the diagnosis and to find out the cause of thecompound presentation. Various types of compound presentation are shown in table111.1.

Table 111.1 Types of Compound Presentation

Type Incidence (%)

Arm with head 75Foot with head 11Arm with breech 11Arm, foot, and head 3

Causes of compound presentation are as follows.

1. High maternal parity.2. Contracted pelvis.3. Malpresentations.

4. Multiple pregnancy.5. Congenital fetal malformations.6. Intrauterine fetal death.7. External cephalic version.

B. In case of a normal fetus, if the maternal pelvis is of adequate size, a vaginaldelivery may be anticipated. Signs suggesting good chances of vaginal deliveryinclude the following.

1. Roomy pelvis.2. Absence of fetopelvic disproportion.3. Fetal limb appears to rise into the uterus with a uterine contraction.

If the cervix is fully dilated, the limb may be pushed into the pelvis. In caseof a vertex presentation, a forceps delivery may then be done. Other alternativesinclude knee-chest position, Trendelenburg position, version and extraction etc.None of these is recommended. The gravida is best left alone. Maternal and fetaldeath rates rise three-fold when active management is routinely followed.

C. A cesarean section is done for the following indications.

1. Pelvic contraction.2. Arrest of progress of labor.3. Umbilical cord prolapse.4. Acute fetal distress.

221

Page 228: Differential Diagnosis and Management Options inuploads.worldlibrary.org/uploads/pdf/20180107071525differential... · 83 Peripartum cardiomyopathy 165 84 Hypertension 167 85 Convulsions

COMPOUND PRESENTATION

Ultrasonography (A)

Malformed fetus Dead fetus Normal fetus (B)

Vaginal delivery Assess pelvic adequacy

Adequate Contracted

Observe progress of labor

Normal Arrest Complications

Vaginal delivery Cesarean section (C)

222

Page 229: Differential Diagnosis and Management Options inuploads.worldlibrary.org/uploads/pdf/20180107071525differential... · 83 Peripartum cardiomyopathy 165 84 Hypertension 167 85 Convulsions

112. High Floating PresentingPart at Term

A. A placenta in the lower segment prevents the presenting part from entering thepelvis. Usually a patient of placenta previa bleeds long before term, and high floatingpresenting part is not the usual presenting feature. But it is necessary to rule it out, ifnecessary by ultrasonography.

B. A full bladder is a frequent cause of a high floating presenting part at term. Ifemptying of the bladder permits entry of the presenting part into the pelvis, nofurther evaluation is necessary. The gravida should be instructed to pass urinefrequently, because filling of the bladder can push the presenting part out of thepelvis, if it is small and does not fit well in the pelvis.

C. Clinical pelvimetry (internal) is done to diagnose pelvic contraction. After digitalpalpation of all pelvic walls, Muller-Kerr’s test is done. The station to which the fetalhead can be made to descend by abdominal pressure is assessed by the vaginalfingers, while the degree of overlap of the head with respect to the front of the pubicsymphysis is assessed by the thumb placed over it. The interpretation of the test is asfollows.

Station Overlap Diagnosis0 Nil Adequate pelvis

Above 0 Flush with front of pubis Borderline pelvic contractionAbove 0 In front of front of pubis Major pelvic contraction

D. Presence of a tumor in the pelvis prevents entry of the presenting part into thepelvis. It may be a cervical leiomyoma, or an ovarian tumor or subserouspedunculated leiomyoma in the pouch of Douglas. Rarely it may be a pelvic kidneyor a bony tumor of the pelvis. All of these conditions are managed by a cesareansection. An ovarian tumor and a subserous pedunculated leiomyoma are removed atthe same time. A cervical leiomyoma has to be operated on at a later date (after thepuerperium). A bony tumor of the pelvis cannot be removed.

E. Borderline pelvic contraction includes mild and moderate degrees of pelviccontraction. If the fetus is in a breech presentation or transverse lie, an externalcephalic version is attempted. Trial of labor is given for vertex presentation, if thereare no contraindications. All other patients are treated by a cesarean section.

Contraindications for Trial of Labor

Elderly primigravidaMalpresentationMedical disorder: hypertension, cardiac disease, renaldisease.PostdatismUterine scar: cesarean section, myomectomy,metroplasty.Previous failed trial of laborPrevious stillbirth/spastic childLack of facilities for cesarean section

In a trial of labor, spontaneous labor is allowed to start, with a view ofobserving the effect of good uterine contractions on the progress of labor, so as toachieve a safe vaginal delivery. If the trial fails, it must be terminated at a time atwhich the mother and the fetus are not affected adversely. Indications fortermination of the trial include fetal distress, nonprogress of labor, onset of maternaldistress, and dysfunctional labor. Easy vacuum delivery or outlet forceps delivery isconsidered as a successful trial of labor.

F. If the head is deflexed, or the fetus is in face or brow presentation, and thematernal pelvis is adequate, labor is awaited. A deflexed vertex may get well flexedand enter the pelvis. A brow can get converted into a vertex or face presentation anddeliver vaginally.

G. Hydrocephalus is diagnosed by ultrasonography, which shows enlargementof the head, dilatation of the cerebral ventricle(s), and thinning of the brain tissueover the ventricle(s). See chapter 117 for management of fetal hydrocephalus.

223

Page 230: Differential Diagnosis and Management Options inuploads.worldlibrary.org/uploads/pdf/20180107071525differential... · 83 Peripartum cardiomyopathy 165 84 Hypertension 167 85 Convulsions

HIGH FLOATING PRESENTING PART AT TERM

History of antepartum hemorrhage (A)

Present Absent

? Placentaprevia

Repeat examination after emptying bladder (B)

See chapter 73 No change Presenting part enters pelvis

Pelvic examinationClinical pelvimetry (C)

Normal

Major pelviccontraction

Cervicalleiomyoma

(D)

Ovariantumor

(D)

Tumor ofpelvis

(D)

Deflexedvertex

(F)

Brow/facepresentation

(F)

Normal Hydrocephalus(G)

Cesarean section Borderline pelvic contraction(E)

Await labor See chapter 116

Vertex Breech / transverse lie Face/brow

External cephalic version

Trial of labor Succeeds Fails Cesarean section

224

Page 231: Differential Diagnosis and Management Options inuploads.worldlibrary.org/uploads/pdf/20180107071525differential... · 83 Peripartum cardiomyopathy 165 84 Hypertension 167 85 Convulsions

113. Suspicion of PelvicContraction at Term

A. Pelvic contraction can be suspected from the following.

1. Obstetric historya. Cesarean section ( >1 ) for small birth passage.b. Failed trial of forceps or failed forceps.c. Obstructed labor and uterine rupture.d. Embryotomy.

2. Short stature.3. History of pelvic fracture.4. History of orthopedic disorders during the developmental years: rickets,

kyphoscoliosis, lameness.5. Osteomalacia.

B. Features of pelvic contraction on obstetric examination include the following.

1. High floating presenting part.2. Malpresentation: breech, face, brow, or transverse lie.3. Pendulous belly.4. Inability to push the presenting part into the pelvis.

Features of pelvic adequacy on clinical internal pelvimetry are as follows.1. Sacral promontory cannot be reached.2. Front of the sacrum has gentle curvatures from above downwards and from side

to side.3. Sacrosciatic notches are wide and shallow.4. Lateral pelvic walls are parallel.5. Ischial spines are not prominent, and both spines cannot be reached at the same

time.6. Retropubic angle is wide.7. Subpubic angle admits 3 fingers.8. A closed fist can be placed between ischial tuberosities.

See chapter 112 for Muller-Kerr’s test. If the pelvis is found to be adequate,nothing further need be done. See chapter 112 for a discussion on trial of labor.

C. Pelvic contraction as a cause of a high floating presenting part is far less commonthan is usually believed. See chapter 112 for the management of a floating

presenting part at term when the pelvis is adequate.

D. Major pelvic contraction is best managed by an elective cesarean section at term.

Causes of Pelvic Contraction

Underdevelopment of a normally shaped pelvisDeformity due to excessive malleability of pelvic bones: rickets, osteomalacia.Deformity due to spinal abnormality: kyphosis, scoliosis, spondylolisthesis.Deformity due to misapplication of femoral pressure : uni- or bilateral lameness,poliomyelitis, congenital dislocation of hip.Dwarf pelvis: true, hypoplastic, chondrodystrophic, cretin.Congenital anomalies: Naegele, Robert, Otto.

E. Borderline pelvic contraction is minor or moderate degree of pelvic contraction.The treatment of both is the same. If the fetus is in a breech presentation ortransverse lie which cannot be corrected by an external cephalic version, a cesareansection is done.

F. If a borderline pelvic contraction is suspected clinically, one has to decide if a trialof labor is to be given. If it is contraindicated (see chapter 112), one has to performX-ray pelvimetry to confirm or rule out the diagnosis of borderline pelviccontraction. Critical diameters of the pelvis are shown in table 112.1.

Table 113.1 Critical Pelvic Diameters

Plane Anteroposterior Transverse Posterior sagittal

(cm) (cm) (cm)Inlet 10.0 12.0 .Midplane 11.5 9.5 4.0Outlet 11.5 10.0 7.5

Elimination of magnification error is quite complicated on X-raypelvimetry. CT pelvimetry yields accurate results without any need for suchcalculations, and without any extra irradiation to the mother and the fetus. If thepelvis is confirmed to be borderline contracted, a cesarean section is done. On theother hand if it is found to be adequate, labor is awaited, so as to achieve a vaginaldelivery. Uterine inertia is managed by administration of oxytocin infusion, and if itfails to achieve progress of labor, a cesarean section is done. Nonprogress of labor inthe first stage is managed by a cesarean section. When the second stage is prolongedand the station is at or below +3, a vacuum extraction or obstetric forceps delivery isdone as appropriate. When it is +2 or higher, a cesarean section is done.

225

Page 232: Differential Diagnosis and Management Options inuploads.worldlibrary.org/uploads/pdf/20180107071525differential... · 83 Peripartum cardiomyopathy 165 84 Hypertension 167 85 Convulsions

SUSPICION OF PELVIC CONTRACTION AT TERMClinical suspicion (A)

Obstetric examination, clinical pelvimetry, Muller-Kerr’s test (B)

Presenting part engaged,pelvis adequate

Presenting part floating,pelvis adequate

(C)

Major pelvic contraction(D)

Borderline pelvic contraction(E)

Suspected borderline pelviccontraction

(F)

No pelvic contraction See chapter 112 Malpresentation Vertex Contraindications to trial of labor

Vaginal delivery Postdatism Heart disease Previous cesarean section Normal Absent Present

Cesarean section Trial of labor X-ray pelvimetry

Normal progress of labor Uterine inertia Nonprogress of labor Prolonged second stage Normal pelvis Borderline pelvis

Vaginal delivery Oxytocin Cesarean section Check station Await labor Cesarean section

+3 or lower +2 or higher

Vacuum extraction orforceps delivery

Cesarean section

226

Page 233: Differential Diagnosis and Management Options inuploads.worldlibrary.org/uploads/pdf/20180107071525differential... · 83 Peripartum cardiomyopathy 165 84 Hypertension 167 85 Convulsions

114. Nonprogress of LaborA. A pelvic contraction causes dystocia. A nulliparous woman's uterus responds bydeveloping hypotonic dysfunction or inertia. A multiparous woman's uterus respondsby developing hypertonicity. A clinical internal pelvimetry is done (see chapter 113).If there is pelvic contraction, a cesarean section is done. If there is no fetopelvicdisproportion then the type of dysfunction is sought.

B. Various types of abnormalities of labor are as follows.

1. Prolonged latent phase: more than 20 hours in a nullipara and more than 14hours in a multipara.

2. Protracted active phase: rate of cervical dilatation is less than 1.2 cm/hr in anullipara and less than 1.5 cm/hr in a multipara.

3. Protracted descent: less than 1 cm/hr in a nullipara and less than 2 cm/hr inmultipara.

4. Prolonged deceleration phase: more than 3 hours in a nullipara and more than 1hour in a multipara.

5. Arrest of dilatation: cessation of dilatation for 2 hours or more in the activephase of labor.

6. Arrest of descent : cessation of descent for 1 hour or more in the active phase oflabor.

C. Optimum uterine activity is the presence of 3 contractions per 10 minutes, eachcontraction lasting for about 60 seconds, the uterus being nonindentable during themiddle portion of the contraction. Hypotonic dysfunction is characterized byinsufficient force (<25 mm Hg) or reduction in frequency or duration or all threeparameters of uterine contractions. It may be due to any of the following causes.

1. Full bladder and/or rectum.

2. Excessive or too early (before active phase) use of sedation or narcotic analgesia.3. Excessive stretching of the uterus: multiple pregnancy, hydramnios.4. Uterine wall abnormalities: leiomyomas, adenomyosis, congenital

malformations of the uterus.5. Psychological: nervousness, fear, anxiety.6. Dystrophia dystocia syndrome.

These etiological factors are corrected appropriately. If hypotonicdysfunction persists, and the woman is in the latent phase of labor with themembranes intact, observation with or without sedation is sufficient because it doesnot distress the mother, nor does it harm the fetus. But if the membranes haveruptured, oxytocin infusion is used to augment labor because otherwise all the liquormay drain out and cause dry labor, constriction ring dystocia, or chorioamnionitisdue to ascending infection. If there is secondary arrest of dilatation beyond the latentphase, or arrest of progress in second stage of labor, oxytocin infusion is used toaugment labor and achieve a vaginal delivery.

D. Hypertonic dysfunction in presence of a malpresentation is managed by acesarean section. If it is a vertex presentation, the patient with intact membranes inthe first stage of labor is given pethidine and promethazine to relieve pain andsuppress contractions. She goes to sleep, and often on waking up goes into goodlabor and progresses well. If the membranes have ruptured, a continuous epiduralblock is administered. It relieves pain and related anxiety. If labor progresses well,observation is sufficient. If not, a cesarean section is required. If hypertonicdysfunction is encountered in late first stage of labor, a cesarean section is requiredin fetal interest. If it occurs in the second stage of labor, vacuum extraction orobstetric forceps delivery is achieved at stations at or below +3, and cesarean sectionfor stations +2 or above.

227

Page 234: Differential Diagnosis and Management Options inuploads.worldlibrary.org/uploads/pdf/20180107071525differential... · 83 Peripartum cardiomyopathy 165 84 Hypertension 167 85 Convulsions

NONPROGRESS OF LABOR

Assess for fetopelvic disproportion (A)

Present Absent

Cesarean section Assess for type of dystocia (B)

Hypotonic (C) Hypertonic

Check phase of labor Assess for malpresentation

Latent Beyond latent Present Absent

Observation Oxytocin Cesarean section Assess stage of labor

First (early) First (late) Second

Membranes intact Membranes ruptured Check station

Pethidine plus promethazine, Awaitresumption of normal labor after a night’s rest

Continuous epidural block + 2 or above +3 or below

Relief from dystocia No relief from dystocia Cesarean section Vacuum/forcepsdelivery

Monitor progress of labor, vaginaldelivery

228

Page 235: Differential Diagnosis and Management Options inuploads.worldlibrary.org/uploads/pdf/20180107071525differential... · 83 Peripartum cardiomyopathy 165 84 Hypertension 167 85 Convulsions

115. Soft Tissue DystociaA. A woman may have a pelvic tumor prior to getting pregnant, either asymptomatic,or symptomatic but not treated surgically for different reasons. Examples of suchtumors are cervical leiomyoma, subserous pedunculated leiomyoma and ovariantumor. The latter two can be in the pouch of Douglas, below the level of thepresenting part. If not treated, such a condition leads to obstructed labor and uterinerupture. In case of an ovarian tumor, other outcomes are possible too. The tumormay rupture (if cystic), or get extruded through the floor of pouch of Douglas (byrupturing it), or through the anus (by rupturing anterior wall of the rectum). All suchoutcomes are undesirable, and the gravida should be treated by a cesarean section. Acervical leiomyoma cannot be treated surgically at this time, but the other twotumors should be removed.

B. If the cervix fails to dilate despite good uterine contractions, it is called cervicaldystocia. It may be due to any of the following factors.

1. Primary or functional: it is seen more often in nulliparas than in multiparas.The cervix dilates very slowly. Its margins feel abnormally firm and tense.

2. Organica. Stenosis of the cervix following cervical amputation, conization, or

deep cauterization.b. Atresia of the cervix.c. Pinhole os.d. Hypertrophic elongation of the cervix (with vaginouterine

prolapse).e. Congenital elongation of the cervix.f. Invasive carcinoma of the cervix.

In case of a pinhole os (conglutination of cervix), the cervix gets 100%effaced, but the external os remains closed. Gentle probing of the external os (seen asa dimple) results in it opening out, and subsequent dilatation is rapid. Invasivecarcinoma of the cervix is a contraindication for vaginal delivery. A gravida withcervical invasive cancer is treated by an upper segment cesarean section. A lower

segment operation is avoided because it is close to the cancer and may result incutting through tumor bearing tissue, or get infected readily. All other cases ofcervical dystocia are observed during labor. If the cervical dilatation does notprogress, a cesarean section is done. There is no place for cervical incisions asadvocated by Duhrssen or O'Sullivan. Such incisions can extend upwards into thelower segment during delivery of the baby, and necessitate an exploratorylaparotomy for repair, which can be quite difficult.

C. The obstruction may be in the vagina. It may be due to any of the following.1. Longitudinal vaginal septum: it is usually thin and gets torn by the pressure of

the presenting part if not treated in time. It is treated by division between twoclamps, and replacement of the clamps by ligatures. The divided septum isexcised electively after the puerperium.

2. Transverse vaginal septum: if the septum has a hole in it, the woman menstruatesthrough that, spermatozoa travel upwards through that, and the presence of theseptum may not even be suspected. If a speculum and vaginal examination is notcarried out during pregnancy, it may only be diagnosed when it causesobstruction during labor. Adequate excision is not possible then, and thewoman is delivered by a cesarean section. The septum is excised electively at alater date, though partial excision may be required in early puerperium if itshole does not permit adequate drainage of lochia.

3. Stricture of the vagina is managed by a cesarean section for delivery of thebaby, and plastic surgery for the stricture after puerperium.

4. Condyloma acuminata may be extensive and fill the entire vagina. Such awoman needs a cesarean section. Adequate antibiotic cover must be given toprevent puerperal sepsis, since the warts usually have secondary infection too.

D. Obstruction may be present at the level of the vulva too. Obstruction due tostenosis or elephantiasis can be managed by an episiotomy. It is less likely to healwell in cases of elephantiasis due to lymphatic block. Extensive condylomaacuminata occluding the vulva necessitate a cesarean section. A stricture andcondyloma acuminata are treated appropriately at a later date.

229

Page 236: Differential Diagnosis and Management Options inuploads.worldlibrary.org/uploads/pdf/20180107071525differential... · 83 Peripartum cardiomyopathy 165 84 Hypertension 167 85 Convulsions

Rule out pelvic contraction

SOFT TISSUE DYSTOCIA

Assess for pelvic tumor (A)

Present Absent

Cesarean section, treatment of pelvic tumor. Assess cervix (B)

Abnormal Normal

100% effaced,remains closed

Congenitalelongation

Supravaginalelongation

Fibrosis Invasivecarcinoma

Assess vagina (C)

Cervicalconglutination

Monitor progress of labor Upper segmentcesarean section

Digital probing Progress occurs No progress Normal Abnormal

Vaginal delivery Cesarean section Assess vulva (D)

Stenosis Elephantiasis Condylomaacuminata

Stricture Excessivecondylomaacuminata

Transversevaginal septum

Longitudinalvaginal septum

Wide episiotomy Cesarean section Division of septum,vaginal delivery

230

Page 237: Differential Diagnosis and Management Options inuploads.worldlibrary.org/uploads/pdf/20180107071525differential... · 83 Peripartum cardiomyopathy 165 84 Hypertension 167 85 Convulsions

116. Fetal MacrosomiaA. Uncontrolled maternal diabetes mellitus causes fetal macrosomia. It is due to fetalhyperglycemia, hyperinsulinemia, and raised levels of growth hormone.

B. Other causes of a large fetus are fetal gigantism, generalized fetal edema,heredity, and postmaturity. See chapter 146 for management of generalized fetaledema due to hydrops fetalis, and chapter 103 for postmaturity. In other cases,antenatal care is continued. At term, assessment is done for fetopelvic disproportion.If there is any disproportion, trial of labor is given for borderline disproportion incase of a vertex presentation, and a cesarean section is done for a major disproportionand any malpresentation.

C. If there is no fetopelvic disproportion, spontaneous labor is awaited at term. Iflabor progresses normally, a vaginal delivery is achieved. The following problemsmay be encountered in labor.

1. Dystocia: it is managed by a cesarean section.2. Prolonged labor: it is managed by a cesarean section.3. Shoulder dystocia: the head delivers, but the shoulders get arrested. The

head then gets pulled back tightly against the maternal perineum. Unlessdelivered promptly, the fetus dies of asphyxia.

D. Shoulder dystocia is diagnosed when the head delivers and further progress ofdelivery is arrested. It may be due to failure of the bisacromial diameter to engage inthe pelvic inlet (high: both shoulders do not engage; low: one, usually anterior,shoulder does not engage) or failure of internal rotation of the anterior shoulder atthe level of the midpelvis. The former is associated with the turtle sign, in whichrestitution and external rotation do not occur. In the latter, the sign is absent, andsome restitution takes place. If the turtle sign is present, the patient is placed inWalcher position, which increases anteroposterior diameter of pelvic inlet, andreduces its angle of inclination to 10°. Suprapubic pressure is applied in the direction

of nearest front end of an oblique diameter of the inlet, and later in dorsal and caudaldirection. Then the patient's position is changed to McRobert's, which is exaggeratedlithotomy position. That increases the anteroposterior diameter of the outlet, and thesymphysis glides around the anterior shoulder, engaging it into the pelvis.

E. McRobert's position cannot be given before engagement of bisacromial diameter asit reduces the anteroposterior diameter of the inlet. But if shoulder dystocia is due tofailure of internal rotation, this position is given right away, combined withsuprapubic pressure. Vaginal delivery is done if it succeeds. An attempt is made todeliver the posterior arm if McRobert's maneuver fails. A hand is passed into thesacral hollow and the posterior arm is swept out across the front of the baby's chest.If it succeeds, the anterior shoulder and then the baby can be delivered. If it fails,Wood's maneuver is carried out under general anesthesia.

F. In Wood's maneuver, the fetus is rotated clockwise when the back is on the rightside, and counterclockwise when it is on the left side. The posterior shoulder thenpasses under the pubic symphysis and gets delivered as the anterior shoulder. Thefetus can then be delivered easily.

G. If Wood's maneuver fails, fetal heart rate is checked. If the fetus is dead, acleidotomy is done to reduce the bisacromial diameter. It is not done on a live fetusbecause it may cause serious injury to the subclavian vessels under the clavicle. If itis possible to perform a cesarean section immediately, Zavanelli maneuver is done.The baby's head is put back into the vagina and the baby is delivered by a lowersegment cesarean section. If it is not possible to perform a cesarean sectionimmediately, one has to choose between symphysiotomy and cleidotomy. If one doesnot have adequate experience of performing symphysiotomy, one may cause seriousmaternal morbidity in the form of injury to the birth passage, vesico-vaginal fistula,damage to sacroiliac joint and pelvic instability. Cleidotomy on a live fetus isassociated with the risk of injury to subclavian vessels and brachial plexus. But itmay be done if all other methods have failed and the fetus will die if not deliveredimmediately. It is the treatment of choice when the fetus is dead.

231

Page 238: Differential Diagnosis and Management Options inuploads.worldlibrary.org/uploads/pdf/20180107071525differential... · 83 Peripartum cardiomyopathy 165 84 Hypertension 167 85 Convulsions

FETAL MACROSOMIA

Glucose tolerance test (A)

Diabetes mellitus Normal

Appropriate treatment Antenatal care, assess for fetopelvic disproportion (B)

Disproportion No disproportion (C)

Check presentation Await spontaneous onset of labor at term

Malpresentation Vertex Dystocia Prolongedlabor

Prolongedsecond stage

Shoulder dystocia (D) Normaldelivery

Cesarean section Assess degree of disproportion Cesarean section Check station Check for the presence of turtle sign

Major Borderline +2 or above +3 or below Present Absent

Cesarean section Trial of labor(See chapter 112)

Cesarean section Vacuum or forceps delivery

Vaginal delivery Succeeds

Walcher’s position +suprapubic pressure

Fails

McRobert’s position +suprapubic pressure (E)

Fails Succeeds

Assess fetal heart sounds (G) Fails Wood’s maneuver (F)

Check feasibility if immediatecesarean section

Fetus alive Fetus dead Succeeds

Possible Not possible Cleidotomy

Zavanelli maneuver,cesarean section

Symphysiotomy Vaginal delivery

232

Page 239: Differential Diagnosis and Management Options inuploads.worldlibrary.org/uploads/pdf/20180107071525differential... · 83 Peripartum cardiomyopathy 165 84 Hypertension 167 85 Convulsions

117. Fetal HydrocephalusFetal hydrocephalus develops due to obstruction to the flow of the

cerebrospinal fluid from the ventricular system into the subarachnoid space. Theventricles get dilated due to accumulation of cerebrospinal fluid. It occurs in about0.3 to 0.8 in 1000 births, and accounts for about 12% of all severe malformationsfound at birth. The head circumference at term may be up to 80 cm (32-38 cm in anormal fetus). The volume of the fluid in the head is usually 500 to 1500 ml, thoughit may reach 5 L.

A. Ultrasonography can make a diagnosis of hydrocephalus in the second and thirdtrimesters. Lateral ventricular ratio (i.e. lateral ventricular width divided by thehemispheric width) may be normally as high as 71% at 15 weeks, but decreases to33% by 24 weeks. A ratio greater than 50% after 24 weeks is considered diagnostic ofhydrocephalus. Diameter of the atrium of the lateral ventricle is relatively constantbetween 6 and 9 mm, between 18 and 35 weeks. If it is greater than 10 mm, itsuggests ventriculomegaly. Dangling choroid plexus, i.e. its separation from themedial ventricular wall is also diagnostic of a hydrocephalus. Ultrasonography alsoreveals associated anomalies which are important, especially spina bifida.

B. If the gestational age of the fetus is less than 20 weeks, a medical termination ofpregnancy is done (see chapter 165). It is not permitted by Indian MTP act after 20completed weeks of pregnancy. In such cases, genetic amniocentesis is done todetermine fetal karyotype. It is important because 85% of hydrocephalic fetuses haveintracranial or extracranial malformations or chromosomal abnormalities. If thekaryotype is abnormal, spontaneous onset of labor is awaited at term. If it is normal,ventricular distension is reassessed.

C. If ventricular distension is mild, it is reassessed after 2 weeks. If it is static, thepregnancy can be continued with periodic reassessment for progress of the condition.If the condition is progressive or severe at any time, the pregnancy is terminated, ifthe fetal lungs are mature. If the lungs are immature (see chapter 101 for tests) aventriculoamniotic shunt is a possible option. Under ultrasonographic control, adouble pigtail catheter with a unidirectional valve is inserted to connect the dilatedventricle to the amniotic cavity, permitting flow from the former to the latter.

However the results are not very encouraging, and a large proportion of fetuses sotreated are born with severe neurologic damage.

D. If the disease remains static, the pregnancy is continued. When the biparietaldiameter of the fetus reaches 10 cm, fetal lung maturity is assessed. If it is mature,labor is induced and a vaginal delivery is achieved. If it is immature, betamethasonemay be used (12 mg IM qd for 2 days) to mature the lungs, and then labor is induced.If the ventricles are quite large, ventriculocentesis is done under ultrasonographiccontrol prior to induction of labor.

E. If the gravida presents for the first time at term or during labor and thebiparietal diameter is up to 10 cm, a vaginal delivery is possible. If the fetal head isvery large, it may be above the pelvic brim or in the uterine fundal area (in one-thirdcases). If there are other severe anomalies associated, the former is managed bytransvaginal ventriculocentesis through dilated cervix during labor, using a 17 gauge,long needle. A vaginal delivery can be achieved when the head collapses due todrainage of the fluid within it. Such a tap can also be done suprapubically afteremptying the bladder (so as to avoid traumatizing it). If the fetus is in a breechpresentation, the tapping is postponed until it delivers up to the head. If it has anopen spina bifida in the cervical region, a Drew Smyth catheter or a female urinarymetallic catheter is passed up the defect and the fluid is drained. An alternative is totap it suprapubically, which is now easier because the head is fixed by delivery of thetrunk. Ventriculocentesis may cause fetal intracranial hemorrhage and fetal death orsevere damage and even neonatal death. If there are no associated anomalies, thefetus is delivered by a cesarean section. Even in these cases, the head may need to betapped to prevent extensions of the uterine incision. The neonate needs treatment forhydrocephalus, including a shunt, by a specialist.

F. A gravida may present with obstructed labor. The uterus is more likely to rupturein these cases, because the uterine wall is already overstretched and thinned down.The head is tapped as described before. If the uterus has already ruptured, anexploratory laparotomy is done after resuscitation. The fetus is removed. The uterus isrepaired if it is possible to do so, along with a sterilization operation to preventuterine rupture in a future pregnancy. If that is not possible, a total abdominalhysterectomy is done.

233

Page 240: Differential Diagnosis and Management Options inuploads.worldlibrary.org/uploads/pdf/20180107071525differential... · 83 Peripartum cardiomyopathy 165 84 Hypertension 167 85 Convulsions

Diagnosis (A)

FETAL HYDROCEPHALUS

Assess gestational age (B)

< 20 weeks > 20 weeks At term, during labor (E) Obstructed labor (F)

Medical termination ofpregnancy

Amniocentesis for fetal karyotyping Check presentation and other anomalies Assess for uterine rupture

Abnormal Normal None Threatenedrupture

Rupture

Termination ofpregnancy at term

Ultrasonographic assessment of severity ofventricular distension (C)

Cesareansection

Exploratorylaparotomy

Severe Mild Cephalic, no major anomalies Breech, no major anomalies Cephalic, seriousanomalies

Breech, seriousanomalies

Induction of labor whenfetal lungs mature

Repeat assessment after 2 weeks Check biparietal diameterVaginal delivery, if necessary after

ventriculocentesis

Progressive disease Static disease < 10 cm > 10 cm

Assess fetal pulmonary maturity Continue antenatal care, periodicultrasonography (D)

Vaginal delivery Cesarean section

Immature Mature Assess pulmonary maturity when biparietal diameter reaches 10 cm

Ventriculoamnioticshunt

Assess biparietaldiameter

Mature Immature

>10 cm < 10 cm Betamethasone

Cesarean section Induction of labor

234

Page 241: Differential Diagnosis and Management Options inuploads.worldlibrary.org/uploads/pdf/20180107071525differential... · 83 Peripartum cardiomyopathy 165 84 Hypertension 167 85 Convulsions

118. Previous Cesarean SectionA. The following points are of importance in the obstetric history of the gravida.

1. Number of cesarean sections: if she has had one cesarean section, she musthave a hospital delivery. About 80% of such women deliver vaginally and 20%need a cesarean section. If she has had 2 cesarean sections, the cesarean sectionrate rises to 35%. With previous 3 or more cesarean sections, it is preferablethat she undergoes a cesarean section along with a sterilization operation.

2. Nature of the cesarean section: if she underwent the operation before going inlabor, the healing of the uterine incision is better than if she had been in laborfor some time at the time of the operation.

3. Extension of the lower segment incision: an extension to the upper segmenteither on the lateral aspect or in midline (inverted T) amounts to an uppersegment incision. That is likely to rupture and a trial of labor is not warranted.

4. Upper segment incision: a primary upper segment incision is rarely done inmodern obstetrics. Such a patient needs elective cesarean section at 37 weeks.

5. Puerperal sepsis: it impairs the healing of the uterine incision and weakens thescar.

6. Duration between the previous cesarean section and next pregnancy: the risk ofrupture of the scar in the pregnancy is increased if the woman conceived in lessthan 6 months after the cesarean section.

B. A gravida may have a persistent indication (e.g. pelvic contraction, pelvic bonytumor) or a nonpersistent indication (e.g. uterine dysfunction). Women with apersistent indication require a repeat cesarean section and they cannot be given a trialof labor.

C. A malpresentation is an indication for a repeat cesarean section. That is sobecause an external cephalic version for a transverse lie or breech presentation, andan internal podalic version for a transverse lie are relatively contraindicated inpresence of a uterine scar for fear of rupture of the same. An assisted breech deliveryis contraindicated too, because a breech extraction may sometimes be required insuch cases, and that involves intrauterine manipulations which can rupture theuterine scar. There is some evidence now that a version may be done even if there isa scar on the uterus.

D. The placenta is localized by ultrasonography. That is important because a uterinescar in the lower segment predisposes the woman to the development of a placentaprevia, and such a placenta may weaken the scar due to trophoblastic invasion. Theincidence of morbidly adherent placenta is also raised. A gravida with placenta

previa is treated by a cesarean section at term, unless she bleeds vaginally beforeterm. See chapter 73 for management of placenta previa. A gravida with placentaprevia and morbid adhesion of the placenta is treated by cesarean section if theinvasion is focal and cesarean hysterectomy if it more than focal.

E. Pelvic adequacy is assessed by clinical internal pelvimetry. If the pelvis isadequate, a vaginal delivery is permitted. If there is any suspicion of pelvic contraction,X-ray or CT pelvimetry is done to confirm or rule out the diagnosis. There isno place for a trial of labor in a gravida with previous cesarean section. An electivecesarean section is done in such cases.

F. With two or more cesarean sections in the past, the risk of uterine scar ruptureduring labor is greatly increased and it is preferable to perform a cesarean sectionrather than give a trial of labor. However there are reports of successful vaginaldelivery with two or more cesarean sections in the past.

G. A gravida with a uterine scar needs close monitoring during labor. It is desirableto monitor the fetal heart rate continuously with an electronic monitor, and to insertintraamniotic catheter for continuous intrauterine pressure monitoring. Often fetalbradycardia and a drop in the intrauterine pressure indicate rupture or dehiscence ofthe scar. Clinical features suggesting threatened rupture of scar are maternaltachycardia, scar tenderness, and hematuria. These should be watched for closely. Ifthe woman needs augmentation of labor, it can safely be done with oxytocininfusion, provided the monitoring is continued as intensively as before. An outletforceps delivery may be done to prevent the gravida from bearing down, which isbelieved by some workers to increase the risk of rupture of the uterine scar. If thereis an arrest of rotation, a vacuum extractor is used. If that fails, a cesarean section isdone. Rotation forceps operation is contraindicated due to the risk of rupture of theuterine scar by a blade of the forceps. After a vaginal delivery, the uterine scar ispalpated vaginally, aseptically, and gently to see if it has ruptured. If there is a smalldefect which does not bleed and does not communicate with the peritoneal cavity, itis left alone. Its behavior in a future pregnancy has not been studied well. If it is a bigdefect which bleeds or communicates with the peritoneal cavity, an exploratorylaparotomy is done and the defect is repaired, along with repair of any associatedinjury.

H. If the pregnancy progresses beyond term, labor is induced (see chapter 105).Cervical ripening is done with intracervical Foley’s catheter. Use of dinoprostone gelor misoprostol tablets is contraindicated because of risk of rupture of the scar.Induction of labor is done with oxytocin infusion.

I. A postdated pregnancy is best managed by a cesarean section.

235

235

Page 242: Differential Diagnosis and Management Options inuploads.worldlibrary.org/uploads/pdf/20180107071525differential... · 83 Peripartum cardiomyopathy 165 84 Hypertension 167 85 Convulsions

History(A)

PREVIOUS CESAREAN SECTION

Check site of uterine incision

Upper segment Lower segment

Elective cesareansection

Check indication for previous cesarean section (B)

Persistent Nonpersistent

Assess for contraindications for trial of labor after a cesarean section

Malpresentation (C) Placenta previa (D) Any degree of pelviccontraction (E)

Two or more cesareansections in past (F)

None

Cesarean section at term Await spontaneous onset of labor at term (G)

Labor at term Pregnancy progresses beyond due date (H) Postdatism (I)

Monitor labor, watch for scar tenderness and other features ofthreatened rupture of uterine scar

Induction of labor Cesarean section

Present Absent

Cesarean section Vaginal delivery, palpate scar vaginally, check for defect in the scar and vaginal bleeding

No scar defect,no vaginal bleeding

Small scar defect,no vaginal bleeding

Scar defect of any size,significant vaginal bleeding

Large scar defect,vaginal bleeding of an degree

Observation Observation Exploratory laparotomy, repair of scar defect and any other associated injury

236

Page 243: Differential Diagnosis and Management Options inuploads.worldlibrary.org/uploads/pdf/20180107071525differential... · 83 Peripartum cardiomyopathy 165 84 Hypertension 167 85 Convulsions

119. TraumaMaternal abdominal trauma is becoming more common with urbanization

and vehicular accidents. Sometimes the trauma is due to physical assault. The uterusand the fetus are well protected by the maternal pelvis in the first trimester. Theamniotic fluid around the fetus absorbs shock and thus protects the fetus. Apenetrating or perforating injury (e.g. stab wounds, bullet wounds) is likely to injurethe uterus and even the fetus.

A. Maternal life and health are of primary concern. If the injuries are serious and lifethreatening, the fetus is temporarily ignored, and the gravida is given life supportivetreatment. Management of surgical and orthopedic trauma take precedence over otherthings. A nasogastric tube is passed to aspirate stomach contents continuously and 2hourly, to prevent aspiration of stomach contents. Oxygen is given to maintain oxygensaturation above 95%. Two intravenous catheters (size 14 or 16) are passed.Intravenous fluids and blood are transfused as required. The vital parameters aremonitored closely. A central venous or Swan Ganz catheter is inserted if required.Vasopressors are avoided as far as possible because they can have deleterious effecton fetal circulation. If the woman needs to be given a left lateral tilt to avoidaortocaval compression by the gravid uterus, the maternal spine needs to be securedfirst. If the patient has to be transferred to another center using military anti-shocktrousers, the abdominal portion should not be inflated as it may reduce placentalperfusion. Abdominal CT scan should not be avoided for fear of fetal irradiation. Ifmaternal benefits outweigh fetal risks, even gadolinium-based contrast agents maybe used. Coagulation studies should be done. Intraperitoneal hemorrhage can beassessed by focused abdominal ultrasonography. If an intercostal drain is needed, itshould be placed 2 spaces higher than in the nonpregnant state.

B. If there has not been any trauma to the abdomen, no further obstetric management isrequired. In all cases of nonvehicular trauma, domestic violence should be lookedfor. In case of blunt trauma, the gestational age of the fetus is assessed. If it is lessthan 12 weeks, the fetus cannot have been injured, since the uterus is still a pelvicorgan. Beyond 12 weeks, the fetus needs to be assessed. Fetal heart sounds can beauscultated beyond 24 weeks. If those are absent, and for fetuses between 12 and 24weeks, fetal heart activity is confirmed by real-time ultrasonography. If the fetus isdead, the pregnancy needs to be terminated after stabilization of the patient andmanagement of other injuries. If the fetus is alive, the pregnancy is continued withusual antenatal care. Preterm labor is managed appropriately (see chapter 101). Insuch cases, the placental site is studied ultrasonographically for retroplacentalhematoma due to traumatic separation of the placenta. If it is present, appropriatetreatment is given (see chapter 73). Fetomaternal hemorrhage should be assessed byKleihauer-Betke test if the woman is nonsensitized Rh-negative gravida and herspouse is Rh-positive. It should be managed appropriately (see chapter 96). Amnioticfluid embolism is an uncommon complication. It should be managed appropriately(see chapter 157).

C. Penetrating abdominal injury is far more serious than blunt trauma. Afterresuscitation, all such patients are subjected to exploratory laparotomy, unlesssurgical treatment of some other condition is required first (e.g. cardiac tamponade).Intraabdominal injury is treated appropriately. If presence of the gravid uterus limitsaccess to intraabdominal structures, it is emptied by a cesarean section irrespectiveof the maturity of the fetus. If the uterus is traumatized, it is emptied by a cesareansection and then the injury is repaired. If the fetus is injured, it is treatedappropriately.

237

Page 244: Differential Diagnosis and Management Options inuploads.worldlibrary.org/uploads/pdf/20180107071525differential... · 83 Peripartum cardiomyopathy 165 84 Hypertension 167 85 Convulsions

Resuscitation (A)

TRAUMA

Assess for abdominal trauma (B)

Blunt None Penetrating (C)

Assess gestational age Appropriate treatment of other trauma Exploratory laparotomyTreatment of trauma

< 12 weeks 12-24 weeks > 24 weeks Assess for uterine injury

Reassure, appropriatetreatment of trauma

Check fetal heartsounds

Present Absent

Absent Normal Cesarean section repair uterine injury

Assess gestational age

Real time ultrasonography to detect fetal heart activity < 28 weeks > 28 weeks

Absent Present Check forcomplications

Continue pregnancy Cesarean section if uterusinterferes with exploration of

abdominal contents

Termination of pregnancyafter treatment of trauma

Treatment of trauma,tocolysis, observation

Preterm labor Placental abruption Amniotic fluid embolism

See chapter 101 See chapter 73 See chapter 157

238

Page 245: Differential Diagnosis and Management Options inuploads.worldlibrary.org/uploads/pdf/20180107071525differential... · 83 Peripartum cardiomyopathy 165 84 Hypertension 167 85 Convulsions

120. Pelvic or AbdominopelvicLump in Pregnancy

Pelvic lumps and a large number of abdominopelvic lumps areasymptomatic, and are diagnosed only on examination. Sometimes the patient isdiagnosed to have the lump prior to the pregnancy.

A. The lump is palpated to determine if it is cystic or solid, which gives an idea aboutthe diagnosis. Ultrasonography is performed in all the cases to differentiate theconditions.

B. A corpus luteum cyst or hematoma needs only observation. If it ruptures and thereis intraperitoneal hemorrhage, an exploratory laparotomy is done to excise it andreconstruct the ovary. A unilocular ovarian cyst, benign cystic teratoma, or acystadenoma are observed up to labor. If they undergo torsion, they are excised byan exploratory laparotomy. Conditions like mesenteric cyst, encysted ascites, hydatidcyst, hemihematometra, and hydronephrosis are observed up to term and duringlabor.

C. The lump may be situated below the level of the presenting part, causing a failureof engagement of the presenting part, a malpresentation, or nonprogress of labor.Such patients are treated by a cesarean section and surgical treatment of the lump, ifpossible. If the presenting part gets engaged and labor progresses normally, vaginaldelivery is achieved, and the lump is treated appropriately afterwards.

D. A solid lump may be a uterine leiomyoma, an ovarian tumor, a chronic ectopic

pregnancy, tubo-ovarian mass, pelvic kidney, or bony tumor of the pelvis.A malignant ovarian tumor is treated appropriately (see chapter 46) irrespective ofthe pregnancy, unless waiting for just a few weeks would achieve a live baby with agood chance of survival. Patients with the other conditions are treated as describedunder (C).

E. Some patients may not have any antenatal care, and may present for the first timeduring labor. Morbidity is likely to be higher in such cases, if the lump is situated inthe pelvis, below the level of the presenting part. The patient may then suffer fromnonengagement of the presenting part or a malpresentation. If she presents beforedevelopment of any complications, a cesarean section and surgical treatment of thelump (if possible) are carried out without any increase in the morbidity. If shepresents with obstructed labor or threatened rupture of the uterus the treatment issimilar, but the maternal morbidity is much higher. The fetus may be badlyasphyxiated or dead. Sometimes the patient's uterus ruptures while trying to overcomethe obstruction. Such a patient has intraperitoneal hemorrhage and even shock. Thefetus is dead. The treatment is an exploratory laparotomy, removal of the fetus,repair of the uterine rupture or hysterectomy, and surgical treatment of the lump, ifpossible. In rare cases, a cystic lump may rupture, and vaginal delivery may occur.Such patients are treated by an exploratory laparotomy and excision of the rupturedmass. Rarely, an ovarian tumor may get delivered through a rupture of the posteriorvaginal fornix and then the fetus may deliver vaginally. Such patients require anexploratory laparotomy to ligate the torn pedicle of the mass, and suture the tears inthe peritoneum of the pouch of Douglas and the posterior vaginal fornix. Rarely thetumor may tear into the anterior wall of the rectum and deliver per rectum, and thenthe fetus delivers vaginally. An exploratory laparotomy is done to ligate the tornpedicle of the tumor and treat the rectal injury surgically in conjunction with asurgeon.

239

Page 246: Differential Diagnosis and Management Options inuploads.worldlibrary.org/uploads/pdf/20180107071525differential... · 83 Peripartum cardiomyopathy 165 84 Hypertension 167 85 Convulsions

PELVIC OR ABDOMINOPELVIC LUMP IN PREGNANCY

Past history

Determine nature of the lump (A)

No antenatal diagnosis, presents forthe first time during labor (E)

Cystic (B) Solid (D)

Ultrasonography Ultrasonography

Corpus luteum cyst orhematoma

Unilocularovarian cyst

Benign cysticteratoma

Cystadenoma Uterineleiomyoma

Chronic ectopic pregnancy, tuboovarianmass, pelvic kidney, bony tumor of pelvis.

Ovariantumor

Observation Observation Mesenteric cyst, encysted ascites, hydatid cyst,hemihematometra, hydronephrosis

Determine nature

Torsion No torsion Observation Benign Malignant

Exploratory laparotomyand excision

Reassess at term and during labor (C) Definitive treatment of cancer, may wait for a few weeks if near term

Failure of engagementof presenting part

Malpresentation Nonprogressof labor

Engagement of presenting partand normal progress of labor

Obstructed labor Threatened rupture ofthe uterus

Cesarean section Vaginal delivery, treatment of the lumpAfter delivery

Cesarean section and treatment of the lump

Uterine rupture Rupture of lump Delivery of lump by rupture ofposterior vaginal fornix

Delivery of lump by rupture of anteriorwall of the rectum

Exploratory laparotomy, repair of uterus orhysterectomy, treatment of lump.

Exploratory laparotomy, excision of the lump.

Exploratory laparotomy, ligation ofpedicle of the lump and repair of

vaginal tear.

Exploratory laparotomy, ligation ofpedicle of the lump and treatment of

rectal injury.

240

Page 247: Differential Diagnosis and Management Options inuploads.worldlibrary.org/uploads/pdf/20180107071525differential... · 83 Peripartum cardiomyopathy 165 84 Hypertension 167 85 Convulsions

121. Burns During PregnancyAccidental, suicidal or homicidal burns are not uncommon in pregnancy.

The risk to the woman is higher during pregnancy than in the nonpregnant state dueto obstetric complications of burns.

A. The percentage of the body surface area (BSA) burned is estimated by the rule ofnines (head and upper limb, each 9%, front of trunk, back of trunk, and lower limb,each 9x2 i.e. 18%, genitals 1%). The depth of burn may be difficult to assessimmediately after the occurrence. The areas of importance obstetrically are theanterior abdominal wall and genitals, because a cesarean section incision would beover a burnt area and vaginal delivery would occur through a burnt area, if these siteswere involved. Inhalation burns substantially increase maternal mortality.

B. All gravidas with burns are hospitalized. The duration of hospitalization is about 1day per 1% of body surface area burnt. Intravenous fluids are administered to eventhose with less than 15% burns, because pregnant women are at risk of developingdeleterious effects on the fetus even with transient hypovolemia. The volume ofRinger's lactate to be transfused during the first 24 hours is given by the followingformula.

Volume (ml) = BSA burnt (%) x Body weight (kg) x 4

Half this volume is transfused intravenously during the first 8 hours after the burn.Colloids are not given until serum albumin falls below 1.8 g/dl. Therapy ismonitored by placing a central venous line by a venous cutdown, or jugular orsubclavian cannulation. A Swan Ganz catheter is passed in critically ill patients.Urine output is monitored by passing a Foley's catheter. Carboxyhemoglobin level ischecked, and if it is above 20%, 100% oxygen is given until it falls to below 5% orsensorium clears. Acute renal failure is watched for and treated appropriately if itdoes develop. Tetanus toxoid is given if the gravida is not already immunized. Broadspectrum antibiotics are given prophylactically, avoiding those which can harm thefetus and also those which are nephrotoxic or are excreted in urine.

C. The fetus may die due to hypoxia secondary to maternal hypoxia in case of

inhalational burns, or maternal hypovolemia. Fetal death in early pregnancy isdiagnosed by real time ultrasonography (see chapter 64). In late pregnancy, thediagnosis is clinical, confirmed by ultrasonography. If the fetus is dead, thepregnancy is terminated when the maternal condition is stabilized (see chapter 105).If the fetus is alive, conservative treatment is given.

D. Hypercatabolism, protein wasting, and loss of weight are common after majorburns. These can have a deleterious effect on both the mother and fetus. Henceenteral nutrition with high protein content is started on the first day, and increased astolerated by the patient. If the feeding tube traverses the pylorus, H2 receptorblocking agents and antacids are given to prevent stress ulcers. Silver sulfadiazine isan effective topical antimicrobial agent. It is applied to the burnt areas afterdebridement, twice a day. Biologic dressings may be used for superficial partialthickness burns e.g. amniotic membrane. Deep partial thickness burns that wouldrequire more than 3 weeks for healing and full thickness burns are treated by earlyexcision and skin grafting. Blood transfusions are given when hemoglobin drops dueto massive hemolysis.

E. Owing to high fever and release of prostaglandins, the patients are at risk ofcomplications like abortion and preterm labor. If threatened abortion develops, it istreated conservatively (see chapter 69). Inevitable abortion is treated by evacuationof the uterine contents. The woman is observed for the development of postabortalsepsis, and if it develops, is treated appropriately. Tocolysis is given for pretermlabor which is controllable (see chapter 101). Nonsteroidal antiinflammatory agentsare the best for this purpose. Beta-mimetic agents cannot be used if the woman's bloodpressure is low or drops with therapy. Steroids cannot be used to enhance fetal lungmaturation because of the risk of existing wound infection or septicemia flaring up.If preterm labor is not controllable or if tocolytic therapy fails, delivery is achieved(see chapter 101). Labor at term is managed as usual. Cesarean section is done ifthere are vulvar burns, and for obstetric indications. If there are burns on the anteriorabdominal wall, the incision is placed at a site away from the burn, the site isolatedby isolation drapes. Antibiotics are administered to prevent and/or treat woundinfection and puerperal sepsis. Breastfeeding is not contraindicated unless there areburns on the nipples.

241

Page 248: Differential Diagnosis and Management Options inuploads.worldlibrary.org/uploads/pdf/20180107071525differential... · 83 Peripartum cardiomyopathy 165 84 Hypertension 167 85 Convulsions

BURNS DURING PREGNANCY

Clinical examination (A)

Emergency treatment (B)

Obstetric assessment and ultrasonography (C)

Fetus dead Fetus alive

Termination of pregnancy(See chapter 165)

Conservative treatment (D)

Observation (E)

Threatened abortion Inevitable abortion Preterm labor Labor at term

Conservative treatment Tocolysis Vaginal delivery Cesarean section forobstetric indications

Resolution Inevitableabortion

Fails Succeeds

Uterine evacuationObservation

Preterm delivery(See chapter 101)

242

Page 249: Differential Diagnosis and Management Options inuploads.worldlibrary.org/uploads/pdf/20180107071525differential... · 83 Peripartum cardiomyopathy 165 84 Hypertension 167 85 Convulsions

122. Goitre in PregnancyWhen a gravida presents with a goiter, the first step is to check the thyroid function.

A. Hypothyroidism during pregnancy is usually mild, since severe disease causesinfertility due to hyperprolactinemia caused by TRH stimulation. Fetal loss inuntreated women is twice that in normal women. The condition is suspected whenthe gravida has excessive weight gain despite poor appetite, cold intolerance,roughening of skin, muscle aches and stiffness, carpal tunnel syndrome, slow pulserate and delayed relaxation of tendon reflexes. Serum T4 and free T4 (FT4) are lowfor pregnancy. T3 resin uptake (T3RU) shows a large number of vacant binding sites.Free thyroxine index (FTI) is low. If the gravida has no other disease, especiallyheart disease, replacement therapy is started without any delay. The dose ofthyroxine is 1-2 μg/kg qd, starting at 0.1 mg every morning. FTI or FT4 and TSH areestimated after one month of therapy and the dose is adjusted by 25 to 50 μgincrements until normal TSH values between 0.5 and 2.5 mU/L are reached. Thetests are repeated every month because as the gravida gains weight, her T4

requirements increase. She is delivered at term. The child may have hypothyroidismdue to maternal thyroid blocking antibodies which cross the placenta. It is treatedappropriately. Breastfeeding is allowed.

B. Most of the cases of hyperthyroidism are already being treated when they getpregnant. Some gravidas develop hyperthyroidism between 10 and 15 weeks ofpregnancy, or even later. Severe disease is usually not found in pregnancy, since itleads to infertility. In untreated cases, fetal loss is up to 50%. The condition isdiagnosed when the gravida fails to gain weight despite good appetite and diet,exophthalmos, lid lag, persistent tachycardia, heat intolerance, palpitations, palmarerythema, and emotional lability. The thyroid may show uniform enlargement,solitary toxic adenoma, autonomously functioning multinodular goitre, and rarely deQuervain's thyroiditis. T4, FT4, and FTI are elevated. Fasting serum TSH is in thenormal range (0-8 mU/L) and it rises to more than 3.4 mU/L but less than 20 mU/Lat 20 minutes after an intravenous injection of 200 μg of thyrotropin releasinghormone (TRH). Serum TSH by immunoradiometry is low. Treatment is started withpropylthiouracil (PTU) in a dose of 150 mg PO q8h. Once the disease is controlled,

the dose is gradually lowered to 150 mg PO qd, so that the risk of fetal goitre andhypothyroidism due to transplacental transfer of the drug is reduced. Thyroidfunction tests are repeated once a month up to the time of delivery, and the dose ofPTU is adjusted. PTU is preferred to carbimazole and methimazole because they arereported to cause scalp defects (aplasia cutis) in the fetus. Some workers prefer tocombine therapy with PTU with physiological replacement doses of T4 so as to avoidhypothyroidism due to overtreatment, and neonatal goitre and cretinism. If there areno complicating factors, the gravida is delivered at term. Fetal goitre may cause facepresentation which is managed appropriately. The neonate is examined for goitre,hyper- or hypothyroidism. Cord blood is collected for FT4 and TSH assay. If thegravida is taking PTU in a dose of up to 150 mg qd, breastfeeding is allowed,provided the infant's thyroid function is checked periodically. A very large goitre or onecausing pressure effects is treated by subtotal thyroidectomy, preferably during thesecond trimester. A malignant goitre is treated by total thyroidectomy and cervicallymph node dissection. T4 is given postoperatively as required. Obstetricmanagement subsequently is as of a medically treated patient. Radioiodine therapy iscontraindicated in pregnancy because the iodine gets concentrated in the fetal thyroidfrom twelfth week onwards, ten times more avidly than in the maternal thyroid, andablates it.

C. A simple, nontoxic goitre is not uncommon during pregnancy. Most of the casesof endemic goitre are not hypothyroid because low levels of T4 caused by deficiencyof iodine lead to elevation of TSH and growth of the thyroid, which increases iodineuptake by the gland and secretion of T3 in preference to T4. Fetal loss is 30% if thewoman is hypothyroid. Even if she is euthyroid, iodine deficiency in the first 3months of gestation may cause neurological cretinism in the neonate. Iodinedeficiency in the second and third trimesters causes hypothyroid type of cretinism inthe neonate. Both are prevented by administration of iodized salt in the diet oriodized oil 2-4 ml IM every 4 to 5 years. A sporadic goitre due to iodine deficiencyis treated similarly. If it is due to thyroiditis or partial enzyme defect (diagnosed byhigh TSH level, with or without normal T3, T4 levels, and absence of thyroidantibodies), replacement T4 therapy is given, monitoring thyroid function everymonth. Delivery is allowed at term. The neonate is checked for goitre and thyroiddysfunction. Breastfeeding is allowed.

243

Page 250: Differential Diagnosis and Management Options inuploads.worldlibrary.org/uploads/pdf/20180107071525differential... · 83 Peripartum cardiomyopathy 165 84 Hypertension 167 85 Convulsions

GOITRE

Assess thyroid function

Hypothyroidism (A) Hyperthyroidism (B) Euthyroidism (C)

Serum TSH assay Determine type Determine type

Elevated Not elevated Grave’sdisease

Solitary toxicadenoma

Multinodulartoxic goiter

de Quervain’sthyroiditis

Endemic Sporadic

Primaryhypothyroidism Antithyroid drugs, assess complicating features

Check for thyroid antibodies

Replacement therapy Absent Present

None Very large goitre Pressure effects Malignancy Iodinedeficiency

Partial enzymedefect

Autoimmune thyroiditis

Surgical treatment Goitrogen in diet

Iodized salt or oil Adjust diet Replacement therapy

Deliver at term

244

Page 251: Differential Diagnosis and Management Options inuploads.worldlibrary.org/uploads/pdf/20180107071525differential... · 83 Peripartum cardiomyopathy 165 84 Hypertension 167 85 Convulsions

123. Thyroid Nodule inPregnancy

A. A solitary thyroid nodule discovered during pregnancy is a potentially serious problem,since it could be a carcinoma. The risk is greater if the gravida has received radiation tothe neck in childhood. About 25% of persons who have received such irradiation have athyroid nodule. Of these, about 8% have cancer of the thyroid. If the gravida has featuressuch as a recent change in voice (indicating infiltration of the recurrent laryngeal nerve bythe cancer), fixation of the nodule (due to infiltration into the surrounding tissues),cervical lymphadenopathy (due to malignant metastases), and Horner's syndrome (dueto involvement of the cervical sympathetic trunk), the nodule is quite likely to be a cancer.The diagnosis has to be confirmed by investigations like fine needle aspiration cytology orbiopsy.

B. Free T3, free T4, and TSH are assayed by immunoradiometric method. High levels of freeT3, free T4, and low level of TSH are diagnostic of a toxic adenoma. It is discussed inchapter 122.

C. Ultrasonography of the thyroid gland helps differentiate between cystic and solidlesions. A cystic lesion measuring less than 4 cm in diameter is usually benign. Thecontents of a cystic lesion are aspirated with a needle and examined for malignant cells. Abenign lesion is observed. Ninety percent of cysts disappear after aspiration. Cysts thatpersist after 2 or 3 aspirations have to be excised. However the excision is done afterdelivery. A malignant cystic lesion is treated like a malignant solid lesion (discussedlater). A solid lesion is subjected to fine needle aspiration cytology or biopsy. A benign

lesion can be observed until after delivery. It is treated by administration of T4, 0.1 to0.15 mg PO qd. If the lesion is tender, its appearance has been preceded by sore throatand systemic upset, the ESR is very high, and aspiration cytology shows multinucleatedgiant cells, the diagnosis is subacute (de Quervain's, viral) thyroiditis. These patientsinitially may have mild thyrotoxicosis, and later hypothyroidism. But almost all casesrecover spontaneously. T4 has to be administered during the phase of hypothyroidism.

D. If the patient has a low free T4 level and high TSH level, the diagnosis isHashimoto's thyroiditis. It is confirmed by the presence of thyroid antibodies in theblood. It is treated by administration of T4.

E. A malignant cystic solitary nodule without spread beyond its limits is treated by totallobectomy during the second trimester. Alternatively the patient can be put onsuppressive doses of T4 until after delivery, since that lowers serum TSH levels, and mostof the carcinomas occurring in the reproductive age are TSH dependent.

Thyroid Function Tests

Tests Nonpregnant Pregnant

BMR -10 to +10 + 20PBI(μg %) 4 - 8 6.2 - 11.2T4 by column (μg %) 3.2 - 6.4 5.5 - 10.5Free T4 (μg %) 1.5 - 2.1 1.5 -2.1T3 resin uptake (%) 25-35 < 21T3 red cell uptake (%) 11 - 17 5-10TBG (μg %) 1 0 - 2 6 15 - 35TSH (μg /ml) < 8 < 8

245

Page 252: Differential Diagnosis and Management Options inuploads.worldlibrary.org/uploads/pdf/20180107071525differential... · 83 Peripartum cardiomyopathy 165 84 Hypertension 167 85 Convulsions

THYROID NODULE IN PREGNANCY

Assess for features of malignancy (A)

Present Absent

Fine needle aspiration cytology/biopsy Thyroid function tests (B)

Carcinoma (E) Benign lesion Toxic adenoma Nontoxic lesion Low free T4High TSH

Ultrasonography ofthyroid (C)

Thyroid antibody test(D)

Cystic Solid Hashimoto’sthyroiditis

Fine needle aspiration cytology/biopsy Fine needle aspiration cytology/biopsy

Malignant (E) Benign Benign Malignant (E) Multinucleated giantcells

Observation T4 therapy Subacutethyroiditis

Surgery or T4 suppressive therapy

246

Page 253: Differential Diagnosis and Management Options inuploads.worldlibrary.org/uploads/pdf/20180107071525differential... · 83 Peripartum cardiomyopathy 165 84 Hypertension 167 85 Convulsions

124. Nipple Discharge DuringPregnancy

Nipple discharge is not an uncommon complaint of women in thereproductive age group. Though a large number of these cases are due to benigndisease, some are due to malignant disease. Hence this symptom needs to beevaluated carefully during pregnancy.

A. The nipple is examined for a lesion which may be the source of the discharge. Ifthere is a lesion, it is biopsied. It may be an intradermal squamous cell carcinoma(Bowen's disease), ductal carcinoma, or duct ectasia. In Bowen's disease, there isbrownish induration with a well-defined edge. Histopathology shows large clearcells as in Paget's disease of the nipple. Invasive carcinoma develops sooner or later.Hence wide excision is recommended. Paget's disease of the nipple is a persistenteczema. There is slow erosion of the nipple, spreading on to the areola later. It is aslowly growing carcinoma of the duct. Histopathology shows large vacuolated cellswith small, deeply staining nuclei in the epidermis, and malignant changes in theducts. It is treated by mastectomy. Duct ectasia is discussed later.

B. If there is no lesion on the nipple, the breast is examined for the presence of alump. See chapter 125 for discussion on breast lumps. If there is no lump in thebreast, the nature of the discharge is assessed.

C. Milky discharge from one or both the nipples is galactorrhea. It is discussed inchapter 33. It must be remembered that serum prolactin levels are physiologicallyelevated during pregnancy and much more so during lactation. Radiologicalinvestigations should be done with caution and that too with shielding of thematernal abdomen to reduce irradiation of the fetus.

D. During the second trimester, tan-colored watery discharge is seen from thenipples. It is called colostrum. It is a physiological response of the breasts to the

hormonal changes of pregnancy. It continues to term. In some women it is profuseenough to wet garments. Reassurance is the only treatment required. The gravida iswarned not to express it vigorously because excessive stimulation of the nipple maylead to preterm labor.

E. Purulent discharge from the nipple is due to a breast infection, the infected partcommunicating with a duct. Antibiotic and NSAID therapy is given to early cases. Ifthe infection does not get controlled and suppurates or if it is already a well formedabscess, it is incised and drained surgically.

F. A clear, serous discharge is usually due to a retention cyst, secondary tofibroadenosis of the breast. If the discharge increases on pressure on the involvedduct, the ductal tissue is excised or explored through a microdochectomy. If not, thegravida is observed.

G. Blood-stained discharge is due to a duct papilloma, duct carcinoma, or ductectasia. It is examined for the presence of malignant cells. If these are present, ductcarcinoma is diagnosed. However their absence does not rule out a carcinoma. Incase of a duct papilloma, a cystic swelling is sometimes felt under the areola,pressure upon which causes discharge from the opening of the affected duct. Thepapilloma is treated by microdochectomy. It is often difficult to distinguish betweena duct papilloma and a duct carcinoma. In case of a carcinoma, a small lump may bepalpable behind the nipple or areola, and there may be a sector-shaped area ofinduration emerging from it. When in doubt, the lactiferous duct is explored up to thetumor. If the diagnosis is confirmed, the treatment is radical mastectomy without anydelay. The prognosis is quite good because the diagnosis is made early.

H. Duct ectasia is characterized by a dilatation of the major ducts of the breast. It is acommon cause of discharge which may be creamy, green or brown-colored, orbloody. It contains debris and lipoid material. There is an associated periductalmastitis. It is treated by removal of the affected segment of the breast and ductsthrough a paraareolar incision.

247

Page 254: Differential Diagnosis and Management Options inuploads.worldlibrary.org/uploads/pdf/20180107071525differential... · 83 Peripartum cardiomyopathy 165 84 Hypertension 167 85 Convulsions

NIPPLE DISCHARGE DURING PREGNANCY

Check for a lesion on the nipple (A)

Present Absent

Biopsy Check for a lump in the breast (B)

Duct ectasia(H)

Squamous cellcarcinoma

Ductalcarcinoma

Absent Present(See chapter 125)

Appropriate treatment Assess nature of discharge

Creamy, green or brown

Milky (C) Tan (D) Purulent (E) Serous (F) Bloody (G)

Duct ectasia (H)

Galactorrhea(See chapter 33)

Colostrum Breast infection Local pressure Local examination,malignant cells in

discharge

Reassurance Antibiotic, NSAIDs No discharge Increase

Resolution Abscess Observation Ductal excision or microdochectomy

Observation Incision anddrainage

Ductectasia (H)

Duct papilloma Ductcarcinoma

Excision of ducts and affected segments of the breast Microdochectomy Radicalmastectomy

248

Page 255: Differential Diagnosis and Management Options inuploads.worldlibrary.org/uploads/pdf/20180107071525differential... · 83 Peripartum cardiomyopathy 165 84 Hypertension 167 85 Convulsions

125. Solitary Breast Mass inPregnancy

A. Benign and malignant conditions of the breast commonly found in the reproductiveage group are also found in pregnancy. Owing to the endocrine changes in pregnancy,both the amount of fat and the glandular tissue of the breast are increased. Thebreasts are also more vascular. These changes make clinical detection of the breastlump difficult. A breast lump is detected by the gravida herself in about 70% of thecases, and by the physician in the remaining cases. Mammography is not a verygood test in these patients. It can miss or misinterpret lesions in the tail of the breast,near the nipple, near the sternum, small peripheral lesions, infected cysts, abscessesand necrotic fatty tissue. This higher rate of error as compared to the nonpregnantstate is due to the presence of dense breast tissue and little or no radiolucent fat.Despite shielding of the abdomen, some irradiation of the fetus does take place.Hence when in doubt, an ultrasonography of the breast is done. It confirms thepresence of a mass, and distinguishes between cystic and solid masses.

B. The contents of a cystic mass are removed by aspiration through a 22-gaugeneedle. The fluid is examined on gross as well as by cytology. If it is benign anddoes not recur in 4 to 6 weeks of observation, no further treatment is required exceptfollow-up examinations. If it is benign, but fills up again any time, it can bereaspirated, and the fluid examined again for the presence of malignant cells. If itrecurs a second time, it is excised. Bleeding can be more than in the nonpregnantstate due to increased vascularity of the breast. Hemostasis has to be achieved verycarefully. If the fluid shows malignant cells, it is treated as a carcinoma of the breast(discussed later).

C. A lesion which is partly cystic and partly solid is more likely to be malignant thana purely cystic swelling. The fluid is aspirated with a needle as in the case of a cysticswelling. Even if there are no malignant cells in it, it could still be malignant. Theresidual mass is subjected to fine needle aspiration cytology (FNAC) or if it is smalland superficial, excision biopsy. If it is benign, no further treatment is requiredexcept observation. If there are malignant cells in the fluid aspirated or on FNAC, orif the excision biopsy report shows malignancy, the gravida is treated for carcinomaof the breast (discussed later).

D. A solid lump in the breast may be benign or malignant. Fibroadenoma is the mostcommon benign solid lump in the breast. It is a discrete, firm, freely mobile tumor. Itmay have been known to be present from before the pregnancy. In such cases, it canbe observed and removed surgically after delivery. However there can be an error ofup to 25% in diagnosis, and hence all solid masses are evaluated by FNAC, coreneedle biopsy, or if the lump is small and superficial, excision biopsy. A lumpdiagnosed to be benign by FNAC or needle biopsy may be observed until afterdelivery and then removed surgically. However that may interfere with lactation. It isnot worthwhile to postpone the operation until after weaning. Hence it may bepreferable to remove the lump surgically under local anesthesia during pregnancy.Surgery during the second trimester and early part of the third trimester is safe, andthe wound heals before delivery, so that breastfeeding can be undertaken safely. Ifthe lesion is malignant, it is treated appropriately depending on its stage. Thetreatment is not postponed until after delivery, unless the woman is at term becausethe tumor grows and spreads in the meantime. If the condition is diagnosed in thefirst trimester, the couple is given the option of having a medical termination ofpregnancy before treatment of the breast cancer. However it must be borne in mindthat termination of the pregnancy does not improve the prognosis in any way. If thewoman survives, she can plan a pregnancy later. Noninvasive intralobular carcinomais treated by local excision and annual follow-up subsequently. Noninvasive intraductalcarcinoma is treated by total mastectomy. The alternative form of treatment by wide localexcision and postoperative irradiation is not acceptable during pregnancy because ofassociated fetal irradiation. Stage I and IIA carcinomas are treated by modifiedradical mastectomy. Alternative forms of treatment by wide local excision, axillarylymph node dissection, and postoperative irradiation are not used as they cause fetalirradiation. Cases of stage IIB and resectable IIIA carcinomas are treated by modifiedradical mastectomy and systemic adjuvant chemotherapy. Chemotherapy is notcontraindicated beyond the first trimester, though the neonate has to be examined fortoxic effects of the chemotherapy. Gravidas with non resectable stage IIIA and IIIBcarcinomas are treated by neoadjuvant chemotherapy, followed by mastectomy ifresectable and lumpectomy followed by irradiation if nonresectable. Stage IVcarcinomas are treated palliatively. Breastfeeding is not permitted because thatwould keep the lymph and capillary bed of the breast dilated on the other side, whichmay also contain a cancer.

249

Page 256: Differential Diagnosis and Management Options inuploads.worldlibrary.org/uploads/pdf/20180107071525differential... · 83 Peripartum cardiomyopathy 165 84 Hypertension 167 85 Convulsions

SOLITARY BREAST MASS IN PREGNANCY

Clinical examinationUltrasonography of the breast (A)

Cystic (B) Partly cystic, partly solid (C) Solid (D)

Aspiration, cytology of contents, observation Aspiration FNAC, needle biopsy, excision biopsy

Benign, doesnot recur

Benign,recurs

Malignant Benign Malignant Benign Malignant

Observation Reaspiration FNAC or excision of mass Observation/removal Determine type

Excision if it recurs again Benign

Observation

Malignant

Benign Malignant

Noninvasive Invasive

Determine stage

Intralobular Intraductal I IIA IIB IIIA resectable IIIAnonresectable

IIIB

IV

Local excision Total mastectomy Modified radical mastectomy Modified radical mastectomy plusadjuvant systemic chemotherapy

Neoadjuvantchemotherapy,

surgery

Palliation

250

Page 257: Differential Diagnosis and Management Options inuploads.worldlibrary.org/uploads/pdf/20180107071525differential... · 83 Peripartum cardiomyopathy 165 84 Hypertension 167 85 Convulsions

126. Breast PainThe breasts respond to estrogen and progesterone. This pain in the breasts can be

functional. This pain varies depending on phase of the menstrual cycle andpregnancy. However organic conditions causing breast pain can be serious and mustbe diagnosed if present.

A. Both breasts are enlarged equally, tense and nodular in pregnancy. Superficialveins are prominent. A few drops of milk may be expressed from the nipple. Thesechanges are preparatory to lactation. Reassurance should be given that these changesare physiological, and a good sign rather than a cause for worry.

B. A cracked nipple is usually unilateral. There is a crack or ulceration of the nipple,with local redness and tenderness. Breastfeeding should be continued on the otherside. Milk should be expressed manually on the affected side until the lesion heals.An emollient, antiseptic cream should be applied to the cracked nipple. If the crack isdue to biting by the baby whose teeth have erupted, weaning is advised. Otherwisegauze pads are worn over the nipples so that skin of the nipples does not get pulledoff (being stuck to the bra) when the bra is removed.

C. Acute mastitis is characterized by fever with chills, acute pain in the breast,localized tender swelling of the breast, and possibly expression of pus through thenipple by causing pressure on the swelling. The nipple may be cracked. Besides thetreatment of cracked nipple, the woman is given antibiotics and an NSAID.

D. A breast abscess is usually preceded by acute mastitis. There is a tender, cystic,fluctuant swelling in the breast. The skin over the swelling is red and edematous. Theabscess may point and burst open on the skin surface. The entire breast may be a bag

of pus. The treatment is incision and drainage of the abscess. Broad spectrumantibiotics are given initially. They are changed if required, based on the results ofthe microbiologic study of the pus.

E. Breast tenderness and engorgement premenstrually is premenstrual syndrome(PMS). Serum prolactin level is estimated in the symptomatic phase of PMS. If it ishigh, bromergocriptine is given. If it is normal, progestin (oral micronizedprogesterone, medroxyprogesterone acetate, or megestrol) therapy is given. If it fails,general measures are prescribed as follows.

1. Exercise2. Restriction of salt, sugar, caffeine and alcohol intake.3. Cessation of smoking.4. Vitamin B6 50-100 mg PO qd.

F. Fibroadenosis occurs in women aged 35 to 50 years. There is localized or diffuseinvolvement of both breasts, in the form of rubbery hardening which achespremenstrually. The lumps are felt with the fingers but not the flat of the hand.Axillary lymph nodes may be enlarged. They are tender and not hard. A cyst mayform. Treatment is cyclical progestin therapy (dydrogesterone) and an NSAID.

G. A carcinoma of the breast is painful only in advanced stages. It is then large, hard,attached to the skin and the chest wall. Axillary lymph nodes are enlarged, hard, andoften fixed. The treatment is palliative.

H. If no disease is found in the breasts, and there is no cause of referred pain (e.g.indigestion, herpes zoster, intercostal neuralgia etc.), the cause is somatoform pain.There may be cancerophobia. The patient is treated in joint consultation with apsychiatrist.

251

Page 258: Differential Diagnosis and Management Options inuploads.worldlibrary.org/uploads/pdf/20180107071525differential... · 83 Peripartum cardiomyopathy 165 84 Hypertension 167 85 Convulsions

BREAST PAIN

Check menstrual and reproductive condition

Pregnancy (A) Lactation Normally menstruating woman

Examination of breasts Check relation of pain to menstruation

Cracked nipple(B)

Acute mastitis(C)

Breast abscess(D)

Premenstrual No relation

Examination of breasts Examination of breasts

Premenstrual enlargement andtenderness, no lumps

Premenstrual enlargement and tenderness,rubbery nodules in breast

Acutemastitis

(C)

Breastabscess

(D)

Carcinoma(G)

Normal

Premenstrual syndrome(E)

Fibroadenosis(F)

Assess for source of referred pain (H)

Serum prolactin assay Present Absent

High Normal Appropriatetreatment

Somatoformpain

Bromergocriptine Progestin

252

Page 259: Differential Diagnosis and Management Options inuploads.worldlibrary.org/uploads/pdf/20180107071525differential... · 83 Peripartum cardiomyopathy 165 84 Hypertension 167 85 Convulsions

127. Hematemesis inPregnancy

Hematemesis is not a common disorder during pregnancy. However itneeds to be treated energetically because it can have serious consequences duringpregnancy, such as hypovolemic shock, intrauterine fetal death due to fetalhypoxemia in maternal shock, and maternal anemia.

A. Intravenous infusion of Ringer's lactate is started through a No. 18 cannula.Blood compatible with the patient's blood is kept ready and administered as required.A No. 32 to 36 Fr nasogastric tube is passed and the stomach contents are aspirated.However absence of blood in the stomach does not exclude bleeding distal to thepylorus. A gastric lavage is given with normal saline. Famotidine is given 20 mg IV,and the same dose is repeated 12 hourly. A nonparticulate antacid like lactate isuseful in case of slow bleeding from the stomach.

B. Esophagogastroscopy is done to find the cause of hemorrhage and treatit appropriately. The site of bleeding can be identified accurately in about 90%of the cases. Two or more sites of bleeding are found in about 30% of the cases.The procedure can be carried out safely under local anesthesia during pregnancy.

C. Mallory-Weiss tear is a linear laceration of the gastric mucosa just below theesophagogastric junction. It may be seen with forceful vomiting in hyperemesisgravidarum or ingestion of alcohol. Bleeding stops spontaneously in most of thecases. Endoscopic coagulation of the bleeder is an effective method of treatment.Other forms of treatment include selective arterial infusion of vasopressin and gastricinflation of a Sengstaken-Blakemore tube. The former is not preferred because of therisk of fetal irradiation.

D. A bleeding duodenal ulcer is an important cause of hematemesis. In addition tothe medical treatment described before, it can be treated with endoscopicelectrocoagulation or Nd-YAG laser. Angiographic embolization is not recommendedduring pregnancy due to the risk of fetal irradiation. This form of treatment is usuallyadequate in most of the cases. Surgical treatment is required if this treatment fails, orif an actively bleeding vessel is seen on endoscopy. The bleeding vessel is oversewn.Then an acid reducing procedure is done, e.g. vagotomy with pyloroplasty. This isnot difficult in the first two trimesters. The access may be limited in the thirdtrimester. In such cases, the uterus may have to be emptied by a cesarean section.However, it is preferable to perform the operation after delivery, if it is at all possibleto wait until then.

E. The treatment of a gastric ulcer is as discussed for a duodenal ulcer. If surgicaltreatment is required, the ulcer is resected, but vagotomy is not done as it increasesthe risk of rebleeding.

F. About 95% of the cases of esophageal varices are due to portal hypertension fromcirrhosis of liver. There are some reports in which some patients had other causes ofhematemesis. If esophagoscopy shows bleeding varices, sclerotherapy is given. It isusually adequate. Endoscopic ligation is also possible. If this treatment fails,vasopressin is given 0.2-0.4 units/min IV. When bleeding stops, the dose is reducedby 0.1 unit/min ql2h. It is successful in 50% of the cases. Somatostatin 100 μg SCq8h is an alternative to vasopressin with fewer side effects. If it fails, balloontamponade is done with Sengstaken-Blakemore tube. It is effective in up to 80% ofthe cases. If that fails, surgical treatment is given as portasystemic shunt orgastroesophageal devascularization. This is difficult in advanced pregnancy due tolimited access. Radiologic embolization may be a useful alternative in such cases,despite the risk of fetal irradiation. Once bleeding is controlled, measures are taken toprevent hepatic encephalopathy and later, recurrence of bleeding which occurs inmore than 10% of the cases in 6 weeks. Vitamin K is given when the woman goesinto labor. Blood compatible with her blood, fresh frozen plasma, cryoprecipitate,and platelets are kept ready. Bearing down is avoided by use of obstetric forcepsbecause it increases portal venous pressure and may precipitate a bleed.

G. Gastritis may occur with ingestion of drugs like NSAIDs and alcohol. Thetreatment is as discussed under 'A'.

H. Stress ulcers are multiple superficial erosions in gastric mucosa due to sepsis,shock and burns. Hematemesis occurs 3 to 10 days after the insult. The condition isdiagnosed by esophagogastroscopy. It is treated by administration of antacids,endoscopic laser photocoagulation, electrocoagulation, or local injection of epinephrine.Surgical treatment is usually not required. Stress ulcers are better prevented by usingthe drugs shown in the table 127.1.

Table 127.1 Drug Therapy For Stress Ulcers

Drug DosageAntacids 30-60 ml NG qlhCimetidine 300 mg IV q6hRanitidine 50 mg IV q8hFamotidine 20 mg IV ql2h

Sucralfate 1 g in 10 ml NG q4hOmeprazole 20 mg PO qd

253

Page 260: Differential Diagnosis and Management Options inuploads.worldlibrary.org/uploads/pdf/20180107071525differential... · 83 Peripartum cardiomyopathy 165 84 Hypertension 167 85 Convulsions

HEMATEMESIS

ResuscitationMedical treatment (A)

Esophagogastroscopy (B)

Mallory-Weisstear(C)

Duodenal ulcer(D)

Gastric ulcer(E)

Esophageal varices(F)

Gastritis(G)

Stress ulcer(H)

Appropriate treatment Endoscopic sclerotherapy or ligation Appropriate treatment

Bleeding does not get controlled Bleeding gets controlled

Vasopressin

Bleeding does not get controlled Bleeding gets controlled

Sengstaken-Blakemore tube

Bleeding does not get controlled Bleeding gets controlled Prevention of recurrence

Surgical treatment

254

Page 261: Differential Diagnosis and Management Options inuploads.worldlibrary.org/uploads/pdf/20180107071525differential... · 83 Peripartum cardiomyopathy 165 84 Hypertension 167 85 Convulsions

128. Epistaxis in PregnancyEpistaxis is a not infrequent problem in pregnancy. Elevated levels of

progesterone tend to increase the vascularity of the nasal mucosa. This may lead totransient and mild epistaxis. However it is a diagnosis made only after excludingmore serious causes.

A. Nasal trauma may be in the form of picking of the nose, which injures themucosa. Such bleeding is usually minor and all that is required is reassurance andadvice not to pick at the nose again. More serious trauma is possible in the case ofvehicular accidents or a fall on the face. If there are no fractures, conservativetreatment is usually satisfactory. The patient is asked to pinch the nose just distal tothe junction of the cartilage and bone for 15 to 20 minutes. If that fails, furthertreatment is given as discussed later.

B. If the patient suffers from a coagulation disorder, epistaxis may occur. Variouscoagulation disorders during pregnancy and their treatment are discussed in chapter141. Local treatment to these patients is application of oxidized cellulose or gelatinizedfoam to the site of bleeding. An advantage of these hemostatic agents is that they donot have to be removed later. Bleeding is controlled with treatment of thecoagulopathy.

C. The tendency for epistaxis during pregnancy is greatly increased by hypertension.Pregnancy induced hypertension and hypertension due to other causes are discussedin chapter 83. Treatment of epistaxis due to hypertension is by local treatment asdiscussed later, and control of hypertension is as discussed in chapter 83.

D. Local examination of the nose requires a nasal speculum, a good light source, andsuction. Local anesthesia may have to be given for adequate examination. Bleedingwhich has not been controlled by local pressure needs to be controlled by

electrocauterization if there are multiple bleeding points or if arterial or venousbleeding is seen. Bleeding from superficial ulcers in Kiesselbach's area is treated byapplication of silver nitrate or 50% trichloroacetic acid after making the area dry bysuction. If electro- or chemical cauterization controls bleeding, observation andtreatment of the cause are adequate. If bleeding continues, anterior nasal packing isdone. If the bleeding stops, the patient is sent home and called back for removal ofthe pack after 2 to 3 days. If the bleeding continues, she is hospitalized and posteriornasal packing is done. The cause is treated simultaneously. If the bleeding stillcontinues, the internal maxillary artery is ligated by the transantral approach, or isembolized by interventional radiography. The latter is associated with the risk offetal irradiation, which is reduced by shielding of the maternal abdomen.

E. Tumors which can cause epistaxis in pregnancy include polyps and angiofibroma.Angiomas tend to increase in size during pregnancy and the risk of epistaxis ishigher than in the nonpregnant state. The treatment is as in the nonpregnant state.

F. Vascular anomalies that can cause epistaxis in pregnancy are arterial aneurysm,hereditary hemorrhagic telangiectasia, and arteriosclerosis. The risk of hemorrhage isgreater than in the nonpregnant state due to increased vascularity of the nasalmucosa. Such patients are referred to an otorhinolaryngologist for further treatment.

G. A meningocele may present with epistaxis in pregnancy. The diagnosis isconfirmed by radiography and CT or MRI of the head. These patients are referred toa neurosurgeon for further management.

H. Inflammatory conditions that can cause epistaxis in pregnancy include upperrespiratory tract infections, allergic rhinitis and rheumatic fever. Local infections aretreated by antibiotics and nonsteroidal antiinflammatory agents. Antihistamines aregiven for allergic rhinitis. Patients with rheumatic fever are referred to a cardiologist.Local treatment is required for all the cases as discussed before.

255

Page 262: Differential Diagnosis and Management Options inuploads.worldlibrary.org/uploads/pdf/20180107071525differential... · 83 Peripartum cardiomyopathy 165 84 Hypertension 167 85 Convulsions

EPISTAXIS

History of trauma (A)

Present Absent

Traumatic History of coagulation disorder (B)

Coagulation disorder Present Absent

Check blood pressure (C)

Hypertension High Normal

Local examination (D)

Multiplecapillary

bleeding points

Superficial ulceration ofKiesselbach’s area

Venous orarterialbleeding

Tumor(E)

Vascularanomaly

(F)

Meningocele(G)

Inflammation(H)

Electrocauterization Chemical cauterization Electrocauterization Appropriate treatment Check cause

Bleeding stops Assess result Bleeding continues Allergic rhinitis Upper respiratoryinfection

Rheumaticfever

Remove pack after2-3 days

Bleeding stops Nasal packing Bleeding continues

Transantral ligation of internal maxillary artery or embolization

Bleeding continues Posterior nasal packing Bleeding stops

256

Page 263: Differential Diagnosis and Management Options inuploads.worldlibrary.org/uploads/pdf/20180107071525differential... · 83 Peripartum cardiomyopathy 165 84 Hypertension 167 85 Convulsions

129. Hernias in PregnancyThe incidence of hernias during pregnancy is low, though the exact

incidence is not well defined. The majority of hernias are asymptomatic,undiagnosed, or unreported. Internal hernias like diaphragmatic hernia areunderdiagnosed because their symptoms are masked by functional disturbances ofpregnancy and one is unwilling to perform radiological tests from fear of irradiationof the fetus. As the uterus enlarges, the intraabdominal pressure rises. As a resultthere is an increase in the outward pressure on the abdominal contents which mayherniate out through hernial orifices. Stretching of the abdominal wall stretches andenlarges the hernial orifices. Thus hernias which were not noticed before apregnancy may become symptomatic for the first time during a pregnancy. Thistendency is increased further by conditions which cause excessive stretching of theabdominal wall, e.g. multiple pregnancy and hydramnios. Small hernias whichbecome symptomatic only during pregnancy may disappear postpartum. Anysignificant hernia in a woman should be repaired surgically before a pregnancy.However if a woman presents with a hernia during a pregnancy, conservativetreatment is given and surgery is resorted to only in the event of a complication.

A. The infraumbilical hernias like inguinal, femoral, and umbilical hernias areusually not symptomatic except in the first-trimester. As the uterus enlarges duringthe second and third trimesters, the contents of these hernias return to the abdominalcavity and get pushed towards the upper abdomen. Thus the chances of incarcerationreduce as the pregnancy advances. An inguinal hernia has to be differentiated fromthe following conditions.

1. Hydrocele of the canal of Nuck: it is a soft, irreducible, sausage-likeswelling which does not change as the pregnancy advances.

2. Benign tumor of the round ligament: a fibroma or leiomyoma is a firm,round, irreducible mass which does not change as the pregnancy advances.

3. Inguinal varicosities: it is a soft, reducible swelling that appears in thesecond half of pregnancy, is usually associated with vulvar varicosities, anddisappears postpartum.

If a hernia does not develop incarceration or obstruction, the gravida is allowed tocontinue the pregnancy to term and deliver. Low forceps delivery is done to avoidstraining. Reduction in uterine size in puerperium permits abdominal contents toenter the sac, and that too with greater ease due to enlargement of the ring. Thus thehernia may become worse. It is repaired as soon as feasible, because complicationslike incarceration, twisting, and strangulation may develop.

B. Hiatal hernia is the commonest of the supraumbilical hernias encountered inpregnancy. About 15 to 20% gravidas have hiatus hernias in the third trimester. Theyare found more often in older multiparas. They cause epigastric discomfort,

heartburn, vomiting, and occasionally hematemesis. These symptoms are also foundwith functional disturbance of pregnancy, and a definitive diagnosis can be madeonly with a barium swallow in extreme Trendelenburg tilt. This test is anyway notdone because the treatment with metoclopramide and antacids is satisfactory andmost of the hernias disappear after delivery. The gravida is asked to sleep in aposition with the head high and avoid abdominal straining. Low forceps delivery isdone to avoid straining in labor. Incarceration is rare, found in case of a large hernia.Operative treatment is required for incarceration, ulceration or stricture.

C. Diaphragmatic hernia through foramen of Bochdalek, congenital or posttraumaticdefect in diaphragm is larger than paraesophageal hiatus hernia. All supraumbilicalhernias get progressively worse in the second half of pregnancy due to pressure ofthe enlarging uterus. About 90% of those which develop obstruction and strangulation areposttraumatic. Mortality in such cases is 25% or more. Prompt surgical treatment isnecessary to save the woman's life. A woman with a known significant diaphragmatichernia should be delivered by elective cesarean section, keeping the stomachdecompressed using a nasogastric tube. Postoperative paralytic ileus is to be avoided.If intestinal obstruction develops postoperatively, a laparotomy is done and thehernia is repaired immediately.

D. Umbilical hernias are usually small and asymptomatic. If an umbilical hernia isreducible, its contents get pushed up by the enlarging uterus and there is no problemafter the sixth month of pregnancy. If it has adherent omentum, increasing upperabdominal pain and discomfort occur due to traction on it as pregnancy advances. Ifit has adherent bowel, it may get strangulated when the uterus enlarges and pusheson it. Hence all irreducible hernias are operated before the uterus enlarges to theirlevel. A reducible hernia is supported with a maternity corset. Labor is allowed atterm. The hernia is protected from trauma by taping the abdomen. Low forcepsdelivery is done to avoid straining. If there are signs of threatened rupture of the skinover the hernia or of incarceration, a cesarean section is done immediately. In case ofthe latter, the hernia is repaired too. If the woman delivers vaginally, the hernia isrepaired prior to the next pregnancy.

E. An incisional hernia often appears for the first time during pregnancy due tostretching of the poorly healed scar. A known hernia enlarges with advancingpregnancy. Obstruction or strangulation of the hernia is rare due to large size of thering. A large hernia may contain the gravid uterus, which goes out of it as it grows.

F. Hysterocele is prolapse of gravid uterus into the sac of abdominal hernia.It is usually found in an incisional or umbilical hernia in a multipara with laxityof the abdominal wall. All irreducible hernias and all inguinal hernias are treatedby elective cesarean section at term. The incisional hernia may be repaired atthe same time or at a later date. There are not adequate data to recommend oneover the other.

257

Page 264: Differential Diagnosis and Management Options inuploads.worldlibrary.org/uploads/pdf/20180107071525differential... · 83 Peripartum cardiomyopathy 165 84 Hypertension 167 85 Convulsions

HERNIA IN PREGNANCY

Determine type

Inguinal (A) Femoral (A) Hiatus (B) Diaphragmatic(C)

Umbilical (D) Incisional (E) Hysterocele(F)

ObservationWatch for incarceration or obstruction

Head high position,metoclopramide,

antacids.

Observation Check reducibility Determine type

None Present Reducible Irreducible Incisional Umbilical Inguinal

Surgical treatment Support Surgical repair Check reducibility Cesareansection

Low forceps deliveryat term

Controlled Complication Labor atterm

Reducible Irreducible

Surgical treatment Uneventful Threatenedrupture ofskin over it

Incarceration Vaginaldelivery

Cesareansection

Controlled Obstruction Lowforcepsdelivery

Cesareansection

Cesareansection plusrepair ofhernia

Elective cesareansection at term,

observation

Repair beforeNext pregnancy

Recovery Obstruction

Laparotomy and repair of hernia

258

Page 265: Differential Diagnosis and Management Options inuploads.worldlibrary.org/uploads/pdf/20180107071525differential... · 83 Peripartum cardiomyopathy 165 84 Hypertension 167 85 Convulsions

130. Varicose Veins inPregnancy

Varicose veins are the result of elongation and dilatation of veins causingtheir tortuosity. They occur in the lower limbs on one or both the sides, and involvethe vulva in 20% of the cases. Vulvar varicosities are usually larger on one side thanon the other.

A. Varicose veins of pregnancy usually appear during the second or third month.They become progressively larger until the last trimester. They are due to relaxationof the smooth muscle in the media of the veins under the action of progesterone,increased blood flow in the pelvic veins causing relative inadequacy of the channelfor blood flow from the lower limbs, and pressure of the gravid uterus on the iliacveins.

B. Inherent defects, especially a missing or defective femoral sentinel valve,predispose to primary varicose veins. Deep venous thrombosis causes incompetenceof the perforating veins connecting the superficial and deep veins, leading tosecondary varicose veins.

C. Tourniquet tests are useful in diagnosis of incompetence of saphenofemoral andpopliteal valves, and of perforator valves. Noninvasive tests like photo-plethysmography (PPG) and Doppler ultrasound tests are much more accurate thanclinical tests. PPG diagnoses insufficiency of the venomotor pump and venousreflux. Correction by a tourniquet applied over upper thigh indicates saphenofemoralincompetence, and at a lower level diagnoses saphenopopliteal or perforatorincompetence. Doppler studies localize incompetent valves and sites of deep venousobstruction.

D. Varicose veins are treated conservatively during pregnancy because of a numberof reasons.

1. Most of the varices recede spontaneously after delivery. Thus there is noindication for surgical treatment.

2. Though it is technically possible to operate on varicose veins in pregnancy,the procedure can be difficult due to distortion of veins.

3. The results of operative treatment are superior when carried out in thenonpregnant state.

Elastic bandages are used to compress the varices during day time when the gravidais ambulatory. An elastic bandage is preferred to an elastic stocking because thestocking stretches with use and is not adjustable. A 10 cm bandage is applied fromtoes to knee, and a 15 cm bandage is applied from knee to upper thigh, in a lyingdown position with the veins emptied by elevation of the leg. If there are large vulvarvarices, a special support is used for their soft compression. It has a foam rubbercompression pad which is self-shaping to suit individual patients. Prolongeddependency of the legs is avoided. Rest is given periodically, elevating the legs tohip level. If it is not possible to elevate the legs while sitting, the toes and feet areflexed and extended to improve venous return. During labor, trauma to the varices isavoided by stirrups in lithotomy position. Vulvar varicosities may bleed profusely iftraumatized during childbirth. It is usually possible to find a place on one side or theother, for making an episiotomy, avoiding varicose veins. Sometimes vulvarvaricosities are associated with varicosities of the vagina, which need to beprotected. Bleeding from traumatized varices is controlled by giving steepTrendelenburg tilt and local compression.

E. Many of the varicose veins that develop during pregnancy resolve spontaneously ina few days after delivery, and definitely by the end of the puerperium. Howeversusceptible women have progressively larger varicosities with each succeedingpregnancy, which also appear at an earlier gestational age. The woman is informedabout these facts in case she plans to have more children. If there are residualvarices, surgical treatment is given if they are large and symptomatic, even if sheplans to have more children. Otherwise it is preferable that she completesher family before the surgery, since there is a risk of recurrence in the next pregnancydespite adequate surgical treatment.

F. Complications of varicose veins may develop during pregnancy in the same wayas in the nonpregnant state.

1. Superficial thrombophlebitis is the result of mechanical injury, venousstasis, bacterial invasion, or increase in coagulability of blood. It is treatedby application of elastic bandages and nonsteroidal antiinflammatory agents.Anticoagulants are used if it is resistant to treatment or very extensive.

2. Deep vein thrombosis: see chapter 91.3. Stasis ulcers may develop due to chronic leg edema, often precipitated by

local trauma. Venous stasis delays healing. Application of medicated elasticadhesive bandage achieves healing of ulcers due to chronic edema in 5 to 8weeks. The bandage is changed weekly. Activity is not restricted. Radicalexcision of the ulcerated area and skin grafting are usually not required.

259

Page 266: Differential Diagnosis and Management Options inuploads.worldlibrary.org/uploads/pdf/20180107071525differential... · 83 Peripartum cardiomyopathy 165 84 Hypertension 167 85 Convulsions

SUSPICION OF VARICOSE VEINS

Check history of varicose veins prior to pregnancy

Absent (A) Present (B)

Check history of deep vein thrombosis

Absent Present

Primary varicose veins Secondary varicose veins

Tourniquet tests, photoplethysmography, Doppler ultrasound tests (C)

Saphenofemoral incompetence Saphenopopliteal incompetence Perforator incompetence

Assess for presence of complications

Absent Present (F)

Conservative treatment (D) Superficialthrombophlebitis

Deep vein thrombosis Stasis leg ulcers

Observation after delivery (E)

Definitive treatment

Resolution Persistence

Counseling Counseling

260

Page 267: Differential Diagnosis and Management Options inuploads.worldlibrary.org/uploads/pdf/20180107071525differential... · 83 Peripartum cardiomyopathy 165 84 Hypertension 167 85 Convulsions

131. Cytomegalovirus InfectionA. Cytomegalovirus (CMV) is the commonest cause of intrauterine infection(0.2%- 2.2% of live births). A secondary infection may be due to reactivation of anendogenous virus or exposure to a new virus strain. Usually primary CMVinfection is asymptomatic and has no long-term sequelae. There may be amononucleosis-like syndrome with persistent fever, malaise, myalgia, cervicallymphadenopathy, and sometimes pneumonia and hepatitis. There is atypicallymphocytosis but the Monospot test is negative.

B. Testing is done on clinical suspicion of the disease or as a screening when thereis an exposure to known CMV infected individual or abnormalities are found onroutine antenatal ultrasound. Screening should be performed prior to a plannedpregnancy or at the beginning of pregnancy. Serological testing for CMV IgG andIgM is the primary method of evaluation. When both IgG and IgM are negative,there is no CMV infection. The patient should be reassured, and repeat testing isdone only if there is high risk of infection in future. It is best managed bypreventive measures like avoiding intimate contact with urine and saliva of youngchildren and careful hand washing after wiping secretions and changing diapers.IgG –ve, IgM +ve indicate CMV infection. The test is repeated after 2 weeks forIgG +ve, IgM –ve. An increase in IgG indicates CMV secondary infection. Nochange in IgG level indicates an old CMV infection. CMV IgM can persist inmaternal blood for months after primary infection or reappear with secondaryinfection. A false positive result occurs with herpes viruses and autoimmunedisorders.

C. CMV specific IgG avidity assay is done when both IgG and IgM are +ve. It isbased on the fact that IgG of low avidity is produced in the initial months afteronset of infection and IgG antibody of increasingly higher avidity is generated laterwhen a maturation process occurs. A low avidity index (< 30%) indicates aprimary infection of duration < 3 months, a high avidity index (> 60%) indicates apast or secondary infection, while an intermediate index (30-60%) indicates aprobable recent primary infection.

D. Isolation of CMV from amniotic fluid is the gold standard for the diagnosis offetal CMV infection. Amniocentesis is done at least 7 weeks after the onset ofmaternal infection (because it takes 5-7 weeks for a detectable quantity of the virusto be secreted to the amniotic fluid) and after 21 completed weeks of gestation(because the virus is replicated in the fetal kidney and then excreted in fetal urine).It can be done by culture on fibroblasts or with the shell vial technique. The latteruses monoclonal antibodies to the early protein p72. It permits detection of thevirus 16-24 hours after amniocentesis. The test is combined with PCR for CMV

nucleic acid detection. Sensitivity of diagnosis by CMV culture and PCR is poor ifdone at < 20 weeks and 80-100% if done at ≥ 21 weeks.

E. Positive results of viral culture and PCR cannot discriminate between infantswho will be symptomatic at birth and those who will not and also they cannotpredict the degree of fetal damage. Quantitative PCR may be able to identifyinfected fetuses at risk of symptomatic disease but not very reliably. Hence fetalevaluation is done using USG. USG abnormalities include IUGR, intracranialcalcifications, cerebral ventriculomegaly, microcephaly, oligohydramnios, pleuraleffusion, ascites, hydrops fetalis, intracranial calcifications, hyperechogenic boweland hepatic calcifications. After making a diagnosis of fetal CMV infection, USGshould be repeated every 2-4 weeks to look for features of CMV infection. Thereare limitations to the use of USG. USG findings are useful but not diagnosticbecause other intrauterine infections and other fetal diseases can have similarfindings. Less than 25% of infected fetuses show such abnormalities while morethan 75% do not. MRI may be used to assess the fetal abnormalities, especially ofCNS.

F. Assessment of the fetal risk helps determine management of at risk pregnancies.The rate of transmission of CMV to the fetus is 30-40% with primary and 1% withsecondary maternal infection. In case of the former 10-15% have symptomaticcongenital CMV infection while 85-90% have asymptomatic congenital CMVinfection. In case of secondary maternal infection, these rates are < 1% and > 99%respectively. Risk of fetal infection decreases with time after maternal infection,being highest during the first year. Positive findings at birth include IUGR,hepatosplenomegaly, microcephaly, jaundice, thrombocytopenia, petechiae,anemia and chorioretinitis. Neurological abnormalities are more likely to occurwhen primary infection occurs in the first half of pregnancy. Hepatitis,thrombocytopenia and pneumonia are more likely when primary infection occurslate in the pregnancy. About 20-30% of such babies die from DIC, superaddedbacterial infection and hepatic dysfunction. About 5-15% of asymptomatic infectedbabies subsequently develop visual impairment, sensorineural hearing loss anddelay in psychomotor development. If the baby's development by 12 months isnormal, a normal long term development is more likely, and progression ofabnormalities after the second year of life is uncommon.

When fetal CMV is diagnosed, MTP may be offered, after counseling about fetalprognosis. If the couple do not want an MTP, expectant observation is done.Treatment with CMV-hyperimmune globulin intravenously in a dose of 200 U/kgof the maternal body weight seems promising. There are a few studies showingsome improvement in hearing and less deterioration of hearing in infants treatedwith ganciclovir.

261

Page 268: Differential Diagnosis and Management Options inuploads.worldlibrary.org/uploads/pdf/20180107071525differential... · 83 Peripartum cardiomyopathy 165 84 Hypertension 167 85 Convulsions

Suspicion of CMV Infection or Screening for CMV (A)

Serological tests (B)

Viral culture in blood, urine or salivaNucleic acid detection by PCR (D)

IgG –ve, IgM –ve IgG +ve, IgM +ve IgG –ve, IgM +ve IgG +ve, IgM –ve Negative Positive

No CMV CMV specific IgG avidity(C)

CMV infection Repeat test No CMV infection CMV infection

Low (< 30%) Intermediate (30-60%) High (> 60%) Increase in IgG No change in IgG

Recent primaryinfection

Probable recentprimary infection

Old/secondaryinfection

Secondary infection Old infection

Preventive measures Fetal evaluation (E) Assessment of fetal risk (F)

USG, MRI Amniocentesis

See under (D)

Check type of maternal infection

Primary infection Secondary infection

262

Page 269: Differential Diagnosis and Management Options inuploads.worldlibrary.org/uploads/pdf/20180107071525differential... · 83 Peripartum cardiomyopathy 165 84 Hypertension 167 85 Convulsions

132. Dengue In PregnancyA. Dengue is an acute febrile illness with severe headache, myalgia, arthralgia andrashes. It is caused by Dengue virus transmitted by Aedes egypti. It has 3 phases.The febrile phase lasts for 2-7 days. It has features like facial flushing, headache,myalgia, arthralgia, diffuse blanching erythema of the skin, nausea and vomiting,sometimes sore throat, pharyngeal congestion, diarrhea and conjunctival injection,sometimes mild hemorrhagic manifestations, leucopenia (WBC <5000 cmm) mildthrombocytopenia (<150000/cmm). The critical phase occurs on the 5th to 6th dayof the illness. It lasts for 24-48 hours. It shows plasma leakage into the peritonealand pleural spaces and resultant hemoconcentration and a decrease in serumalbumin (<3.5g/dl). The convalescent (recovery) phase lasts for 2-5 days. It showsimprovement in general wellbeing and appetite, convalescent rash, generalizeditching, hemodynamic stability, diuresis, rise in white cell and platelet counts and astabilization of hematocrit. Sometimes there are unusual manifestations in dengue,like involvement of liver, kidneys, brain or heart. Diagnosis of dengue duringpregnancy may be difficult. Warning signs of severe dengue may be confused forhyperemesis during the first trimester. Hemolysis, elevation of liver enzymes andthrombocytopenia may be confused for HELLP syndrome. A gravid uterus maymake detection of pleural effusion and ascites difficult. Pregnancy related vaginalbleeding may be confused for dengue related hemorrhage.

B. Dengue is diagnosed by viral detection, genome detection, NS1 detection,Dengue IgG and IgM detection. These tests are in descending order of specificityand increasing order or accessibility. Once a diagnosis is made, vital parametersshould be checked 2 hourly. Fluid and electrolyte balance is checked every day. Ahemogram liver and renal function tests should be done. USG should be done todetect pleural effusion, ascites and gallbladder wall edema.

C. There are 4 grades of dengue.

Grade FeaturesI Fever, constitutional symptoms, a positive tourniquet test and/or easy

bruising.II The manifestations of Grade I patients plus skin or other hemorrhages.III The manifestations of Grade II patients plus a rapid, weak pulse and

narrowing of pulse pressure, hypotension, restlessness and cold, clammyskin.

IV The manifestations of Grade III patients plus severe shock withundetectable pulse or blood pressure.

D. Hospitalization is recommended for all cases of dengue in pregnancy. It is

especially indicated for the presence of the following features.

1. Recurrent vomiting2. Pleural effusion/ ascites/ gall bladder edema3. Hemorrhage: hematemesis, hemoptysis, hematuria, vaginal bleeding, bleeding

gums.4. Abdominal pain5. Hepatomegaly, hepatic dysfunction.6. Palpitations, breathlessness7. Decreased urinary output8. Hematocrit >45%9. Thrombocytopenia: count 100000-150000/cmm and dropping rapidly.10. Rapid pulse11. Hypotension12. Cold clammy extremities

E. High risk factors include the following. The patient needs to be watched for thedevelopment of severe disease in these cases.

1. Obesity2. Elderly3. G6PD deficiency4. Peptic ulcer diseases5. Coronary artery disease6. Thalassemia7. Diabetes mellitus8. Chronic pulmonary diseases9. Hypertension10. Patients on drugs: steroids, antiplatelet agents, anticoagulants.11. HIV infected persons / immunocompromised state.

F. Obstetric complications of dengue include bleeding, low birth weight,premature birth and risk of vertical transmission. Severe bleeding may complicatechildbirth and/or surgical procedures done during the critical phase.

G. Medical management is done as in the nonpregnant state. Conservativemanagement is the treatment of choice. Excessive fluid replacement should beavoided. LSCS or induction of labor should be avoided during the critical phase.Platelet count should be kept above 50000/cmm if obstetric procedures are to beundertaken or patient goes in labor. For surgical procedures like LSCS, the plateletcount should be at least 80000/cmm. When platelet transfusions are to be given, asingle donor platelet transfusion is preferred. There is no indication for MTP.

263

Page 270: Differential Diagnosis and Management Options inuploads.worldlibrary.org/uploads/pdf/20180107071525differential... · 83 Peripartum cardiomyopathy 165 84 Hypertension 167 85 Convulsions

SUSPICION OF DENGUE (A)

Laboratory diagnosis (B)

Positive Negative

Check for grade of the illness (C)

Check for indication for hospitalization (D)

Present Absent

Hospitalization Conservative treatment

Check for presence of high risk for development of complications (E)

Watch for development of obstetric complications

Management

264

Page 271: Differential Diagnosis and Management Options inuploads.worldlibrary.org/uploads/pdf/20180107071525differential... · 83 Peripartum cardiomyopathy 165 84 Hypertension 167 85 Convulsions

133. Ebola Virus DiseaseEbola virus is an RNA virus of filoviridae family. Its reservoir is in fruit bats. Itsspread to humans occurs by consumption or handling of meat from infectedanimals like rodents. Human-to-human transmission occurs by a direct contact withthe secretions and excretions of infected symptomatic patients through mucosalsurface or breaks in the skin, contact with bedding and clothing heavilycontaminated with blood, vomitus and/or feces, and by needle stick injuries. Thevirus is found in breast milk. Breast milk may still be infectious for 15 to 40 days.Semen may still contain the virus up to 7 weeks post-recovery.

A. Ebola virus disease (EVD) is suspected when the patient gets fever of suddenonset, fatigue, headache, muscle pain, sore throat, nausea, vomiting, diarrhea,abdominal pain, maculopapular rash and injection of conjunctiva. At the end of thefirst week petechia, bruising, and gastrointestinal bleeding, and blood loss fromvenipuncture sites may occur. Severe hypovolemic shock and multi-organ failuremay occur between days 6 and 16. Seizures and coma usually occur before death.Conditions like preeclampsia with HELLP syndrome, abruptio placentae andgestational thrombocytopenia should be ruled out.

B. Positive laboratory findings include leukopenia, thrombocytopenia andelevation of liver enzymes. Prothrombin time and partial thromboplastin time maybe prolonged and fibrin degradation products may be elevated. The most definitivediagnostic test is reverse transcription PCR. It gives a positive result in 3 days fromthe onset of symptoms. The test may be negative in the first 3 days. So if it isnegative, it should be repeated. If it is still negative, the patient does not have EVD.If the patient is at or near term when the initial test is negative and there issuspicion that she has possibly been infected by Ebola virus but the fetus is not yetinfected, induction of labor or cesarean section is done so as to deliver the baby

before development of maternal viremia and fetal infection. If the woman becomesPCR +ve subsequently, the neonate is isolated from the mother and breastfeeding isavoided so as to avoid neonatal infection.

C. A patient with EVD is isolated so as to prevent spread of the infection.Personnel treating the patient should be trained to use universal precautions,including use of personal protective equipment. There is no specific curativetreatment of EVD. Intravascular volume is maintained. Electrolyte imbalance iscorrected. Metabolic abnormalities are treated. Coagulation abnormalities aremanaged appropriately. Antibiotics are given to prevent secondary bacterialinfections.

D. Fetal outcomes are very poor. Most patients present with first or secondtrimester abortions or intrauterine fetal death in the third trimester. In case ofspontaneous abortion, the products of conception are evacuated after stabilizationof her condition and correction of any abnormality of coagulation. In case of fetaldeath, labor is induced after similar treatment.

E. If the fetus is alive and the woman is in labor, labor is allowed to progress,monitoring maternal and fetal well being noninvasively, and giving appropriatetreatment. Antibiotics are given to avoid chorioamnionitis and puerperal sepsissubsequently. Cesarean section is done for obstetric indications only. Membranesare ruptured late. Episiotomy or perineal tear repair is done using instruments onlytechnique. Postpartum hemorrhage is prevented and if it does occur, it is managedusing uterotonic drugs rather than surgical methods as far as possible. If the womanis not in labor or is preterm, conservative treatment is given, allowing spontaneousonset of labor. Intrapartum infection of the fetus has not been documented.

Condoms, oral contraceptive pills or injectable contraceptives are used forcontraception for 3 months or longer after survival from EVD.

265

Page 272: Differential Diagnosis and Management Options inuploads.worldlibrary.org/uploads/pdf/20180107071525differential... · 83 Peripartum cardiomyopathy 165 84 Hypertension 167 85 Convulsions

SUSPICION OF EBOLA VIRUS DISEASE

Check clinical features (A)

Negative Positive

Reverse transcription PCR (B)

No EVD Negative

Negative Repeat after 3-4 days Positive (C)

At term, fetus is not yet infected

Induction of labor or caesarean sectionIsolate neonate of mother gets EVD.

Isolate the patient.Ensure personnel are trained in use of personal protective equipment.Supportive treatment, antibiotics.Check fetal well being (D)

Fetal death Fetus alive (E)

Evacuate products of conception or induce laboras appropriate.

Check gestational age

Term Preterm

Check if the woman is in labor

Yes No

Conduct labor monitoring maternal and fetal being. Conservative treatmentAllow spontaneous onset of labor, preferably after maternal recovery.

Check neonate by PCR at birth 3 days of life and any time within the first 21 days if symptomatic or unwell.Advise contraception for 3 to 6 months.

266

Page 273: Differential Diagnosis and Management Options inuploads.worldlibrary.org/uploads/pdf/20180107071525differential... · 83 Peripartum cardiomyopathy 165 84 Hypertension 167 85 Convulsions

134. Group B StreptococcusAbout 25-33% women carry group B streptococci (GBS) in the vagina and rectum.These can infect the amniotic fluid even with intact membranes and the babyduring childbirth, causing neonatal pneumonia or meningitis. Early onset GBSdisease (EOGBS) occurs in the first week after delivery, 90% within 12 hours ofbirth. Colonization rate of infants born to colonized women is up to 70%, butEOGBS occurs at a rate of less than 1 per 1000 live births. The risk of infectionand mortality is much higher with very low birth weight infants - up to 3% and30% respectively. Late onset GBS disease (LOGBS) begins 7-9 days after delivery.It is less often fatal than EOGBS.

A. A gravida should be screened for Group B streptococcal (GBS) colonization at35-37 weeks. When facilities permit, this screening should be done for all gravidas.But when resources are limited, at least high-risk women should be screened,including those with previous infant having early onset GBS disease (EOGBS),GBS in urine (any colony count), spontaneous onset of labor before 37 completedweeks gestation, intrapartum fever ≥ 380C and rupture of membranes for 18 hoursor more. GBS bacteriuria is associated with more frequent and heavier colonizationof the neonate with GBS, and higher risk of EOGBS. Swabs should also be takenfrom women scheduled to undergo elective cesarean section, as the risk of laborbefore term or premature rupture of membranes remains unknown in them.However, 25-30% of cases are not associated with maternal risk factors and wouldescape detection by this approach of screening high-risk women. Culture should betaken from one swab passed first high in the vagina and then through the analsphincter. Alternatively separate swabs may be taken. Selective enrichment broth(e.g. Todd-Hewitt broth supplemented with either colistin and nalidixic acid orwith gentamicin and nalidixic acid) is more sensitive than standard solid media.

PCR based rapid tests are highly sensitive and may be useful, but cannot replaceculture. Chromogenic agars can detect beta-hemolytic GBS, but not nonhemolyticstrains.

B. Intrapartum chemoprophylaxis is given if the screen shows GBS positive result.It is very effective in reducing colonization of the neonate with GBS andpreventing EOGBS. It also delays LOGBS and achieves a milder presentation ofLOGBS. Benzyl penicillin is given as a 3 g IV loading dose followed by 1.8 g IVq4h, from the onset of labor until delivery. If the gravida is hypersensitive topenicillin, alternative treatment is clindamycin 600 mg IV q8h or vancomycin 1 gIV q12h. Clindamycin resistance is being reported in up to 20% cases.Erythromycin is not considered useful any more.

C. If the woman is GBS negative, presence of obstetric risk factors is checked for(see under ‘A’). If any risk factor is present, intrapartum chemoprophylaxis isgiven. If no risk factor is present, the woman is checked for features of intrauterineor genital tract sepsis. If sepsis is found, she is given amoxycillin 2 g IV q6h,gentamicin 4-6 mg/kg IV qd and metronidazole 500 mg IV q12h. If sepsis is notfound, intrapartum chemoprophylaxis is not given.

D. GBS status is said to be unknown when the screening fails, the woman refusesto give a swab, or the result of the test is not available. Such gravidas are managedthe same way as a GBS negative woman (see under ‘C’).

E. GBS is known to be excreted in the breast milk. The role of infected breast milkin neonatal infection is not known, and hence concrete recommendations onbreastfeeding cannot be made in such cases. The neonate needs to be watched forfeatures of GBS sepsis and managed appropriately.

267

Page 274: Differential Diagnosis and Management Options inuploads.worldlibrary.org/uploads/pdf/20180107071525differential... · 83 Peripartum cardiomyopathy 165 84 Hypertension 167 85 Convulsions

SCREENING FOR GBS (A)

Positive (B) Negative (C) Unknown (D)

Check for obstetric risk factors

Give intrapartum chemoprophylaxis Present Absent

Check for features of intrauterine or birth canal sepsis

Neonatal management (E) Present Absent

No intrapartum chemoprophylaxis

268

Page 275: Differential Diagnosis and Management Options inuploads.worldlibrary.org/uploads/pdf/20180107071525differential... · 83 Peripartum cardiomyopathy 165 84 Hypertension 167 85 Convulsions

135. Hepatitis B In PregnancyHepatitis B is caused by an hepadna virus (HBV). Owing to non-vaccination of thepopulation, it is found in 400 million people in the world. Hepatitis B surfaceantigen (HBsAg) carriage rate is up to 20% in south and east Asia. Chroniccarriage rate may be up to 8%, being higher in injecting-drug users and femalesex workers. It is transmitted sexually and parenteral contact with blood and bloodproducts. HBV is much more transmissible than HIV. Four phases of chroniccarriage of HBV are: immune tolerant hepatitis B e antigen (HBeAg) +ve phase,immune active eAg (HBeAg) +ve phase, inactive hepatitis B carrier HBsAg +ve,HBeAg -ve phase and HBeAg -ve chronic active hepatitis phase. The secondand fourth may progress to cirrhosis and liver cancer.

A. All gravidas should be checked for HBsAg. All gravidas with icterus, all HIVgravidas and all patients commencing immunomodulatory therapies andchemotherapy should also be checked.

B. If the gravida is HBsAg -ve, she is tested for HBsAb and HBcAb. Anti-HBcdetects evidence of current or past infection. However it may be false-positive andhence gravidas who are anti-HBc positive but anti-HBs negative are consideredas possibly non-immune. Non-immune gravidas are given HBV vaccine only ifthey are at high risk of acquiring the infection. Otherwise the vaccine is given afterthe pregnancy. If her anti-HBs level is >10 IU/ml, she is observed and the infant isgiven HBV vaccine at birth, and then at 2, 4 and 6 months of age. If her anti-HBslevel is <10 IU/ ml, she is given HBV vaccine and HBIG within 72 hours ofexposure at separate sites. Vaccine to also be given at 1 and 6 months after first

dose.

C. The gravida given HBV vaccine and HBIG is checked for HBsAg after 3months. If the test is –ve, no treatment is given to the infant. If it is +ve, HBIG andbirth dose of HBV vaccine are given to the infant within 12 hours of birth.

D. If the gravida is HBsAg +ve, she is tested for HBeAg/HBeAb, HBV DNA andliver function tests (LFTs). LFTs are useful to detect the existing state ofderangement of her hepatic function. If her HBV DNA level is >107 IU/ ml, she isgiven lamivudine, tenofovir or telbivudine from 30 weeks of gestation.Monotherapy with lamivudine and telbivudine has been tested for prevention ofperinatal transmission, but not with tenofovir. But tenofovir is a potent inhibitor ofHBV with a high barrier to resistance. Lamivudine is useful in gravidas withfulminant hepatic failure due to acute hepatitis B and an acute exacerbation ofchronic hepatitis B. HBIG and birth dose of HBV vaccine are given to the infantwithin 12 hours of birth. If maternal treatment is for preventing perinataltransmission alone, then it is stopped between 4 and 12 weeks postpartum. If it is inmaternal interest too, then the optimal time for stopping therapy postpartum has notbeen fixed. Factors like side effects, maternal liver fibrosis and risk of hepaticflares need to be considered while making the decision to stop therapy.

E. If her HBV DNA level is >107 IU/ml, the infant is given HBIG and birth dose ofHBV vaccine within 12 hours of birth.

A single dose of hepatitis B vaccine is 10 μg IM on anterolateral aspect of thethigh. The dose of HBIG is 0.5 ml IM on anterolateral aspect of the thigh.

269

Page 276: Differential Diagnosis and Management Options inuploads.worldlibrary.org/uploads/pdf/20180107071525differential... · 83 Peripartum cardiomyopathy 165 84 Hypertension 167 85 Convulsions

Check HBsAg(A)

-ve +ve

Check HBsAb and HBcAb (B) Check HBeAg/HBeAb, HBV DNA and LFTs (D)

HBsAb, HBcAb –ve anti-HBs >10 IU/ ml anti-HBs <10 IU/ ml HBV DNA >107 IU/ ml HBV DNA <107 IU/ ml (E)

Vaccinate gravida if athigh risk

Observe the gravidaHepatitis B vaccine tothe infant at birth, andthen at 2, 4 and 6months of age.

Give HBV vaccineand HBIG within 72hours of exposure atseparate sites. Thevaccine is given at 1and 6 months afterfirst dose too..

Treat gravida with lamivudine,tenofovir or telbivudine from30 weeks of gestation.Give HBIG and birth dose HBVvaccine to the infant within 12hours of birth.

Give HBIG and birth dose of HBVvaccine to the infant within 12hours of birth.

Check mother forHBsAg at 3 months (C)

-ve +ve

No treatment to infantGive HBIG and birth dose of HBV vaccine tothe infant within 12 hours of birth.

270

Page 277: Differential Diagnosis and Management Options inuploads.worldlibrary.org/uploads/pdf/20180107071525differential... · 83 Peripartum cardiomyopathy 165 84 Hypertension 167 85 Convulsions

136. Hepatitis C In PregnancyAbout 1-2% of women of childbearing age are carriers of hepatitis C virus (HCV).The incidence is up to 80% in women who are at high risk, including injecting-drug users, blood and blood product dependent patients, those with occupationalexposure, and sexual partners of those who are HCV +ve. Risk of co-infection isincreased two-three fold in HIV +ve patients. All such women and preferably allpregnant women should be screened for HCV. If a test for HIV has not been done,it is done at the earliest.

A. If HCV antibody test is –ve, no further action need be taken. If the test is +ve, aPCR test for HCV RNA and LFTs are done. The latter indicate the degree ofhepatic damage by HCV infection. If the gravida is HCV RNA -ve, it may be dueto a false positive antibody test, clearing of a past infection, or a level of viremiabelow the level of detection by the assay. Since a single negative test cannotexclude all risk of vertical transmission to the fetus, an HCV antibody test is doneon the infant at the age of 18 months. A gravida who has tested -ve for HCVantibody at first visit but remains at high risk of acquiring HCV infection is testedagain for HCV antibody in the third trimester. If she remains -ve, she is treated as

described above. If she tests +ve, she is treated as described under 'B'.

B. If the HCV RNA test is +ve, the risk of vertical transmission is 5%. LFTs arechecked in each trimester. Invasive procedures are minimized in the antenatalperiod (amniocentesis or chorionic villous sampling) and during labor (placementof fetal scalp electrode), to prevent vertical transmission. Early artificial rupture ofmembranes is avoided if possible, as rupture of membranes fo longer than 6 hoursincreases the risk of vertical transmission. There is no evidence that a cesareansection avoids vertical transmission. The baby is bathed to remove maternal bloodand body secretions prior to administering any intramuscular injections e.g.vitamin K. Breastfeeding does not increase the risk of vertical transmission. But ifthe nipples are cracked and bleeding, breastfeeding is stopped and manuallyexpressed breast milk is discarded until the cracks heal. The woman is treated afterbreastfeeding has been completed. Cure rates of 70-80% can now be achieved withtreatment. If not treated, patients with chronic HCV usually remain asymptomaticfor 20-30 years. About 20% develop cirrhosis of liver and 1-6% develophepatocellular carcinoma. A women on antiviral therapy for HCV should areadvised to avoid a pregnancy during therapy with Ribavirin and for 6 monthsthereafter because it is a teratogen. Use of two effective contraceptives together isrecommended.

271

Page 278: Differential Diagnosis and Management Options inuploads.worldlibrary.org/uploads/pdf/20180107071525differential... · 83 Peripartum cardiomyopathy 165 84 Hypertension 167 85 Convulsions

Check for HCV antibody (A)

Negative Positive

No further action needed. Hepatitis C RNA and LFTs (B)

-ve +ve

Check if the gravida remains at high risk of HCV infection

No Yes

Check for HCV antibody in the third trimester

-ve +ve

Avoid invasive procedures antenatally and during labor.Bathe the baby to remove maternal blood and body secretions.Permit breastfeeding unless nipples are cracked.

Treat the woman after breastfeeding has been completed.

Do HCV antibody test on the infant at the age of 18 months.

272

Page 279: Differential Diagnosis and Management Options inuploads.worldlibrary.org/uploads/pdf/20180107071525differential... · 83 Peripartum cardiomyopathy 165 84 Hypertension 167 85 Convulsions

137. Herpes Simplex Infection In PregnancyGenital infection with Herpes simplex virus (HSV-1) is more common with HSV-2. Primary HSV infection is one in which the patient has no prior exposure (i.e.there are no HSV-1 and HSV-2 antibodies). Recurrent HSV infection is one inwhich the patient had prior exposure (i.e. there are HSV-1 and/or HSV-2antibodies). In primary infection there are unilateral or bilateral vesicular lesionson the genital skin or adjacent areas. The base of the lesions is erythematous. Theychange into pustules, which break to form ulcers. Crust forms when the lesions areon keratinized skin. Atypical presentations include mild erythema and fissures. In arecurrent disease, there may be asymptomatic shedding of the virus or lesions as inprimary infection. HSV-2 can be found in the genital tract on 3% of days duringthe first year and 1% of days during the next 2 years. Neonatal HSV infection isacquired perinatally in 85% cases. Intrauterine infection occurs in 5% cases or less.

A. Distinguishing between primary and recurrent herpes is important because therisk of neonatal infection seems is greater with primary infection (30-50%) thanwith recurrent herpes (0-3%). In case of a recurrent infection, saline bathing locallyand paracetamol orally are given for active lesions, which heal in 7-10 days.Suppressive aciclovir therapy is given in a dose of 400 mg q8h PO from 36 weeksof gestation. Vaginal delivery is advised, as the risk of transmission to the baby islow. Use of fetal scalp electrode, forceps, and vacuum delivery are avoided, asthese procedures increase the risk of transmission to the baby. The risk oftransmission is low even if there are active lesions locally. In such cases thegravida is counseled and advised to have a vaginal delivery.

B. In case the gravida has primary herpes, The risk of transmission to the baby is

highest if delivery occurs within 6 weeks of the infection (41%). Overall, the risk ishigh if the infection occurs in the third trimester, while it is lower if the infectionoccurs during the first or the second trimesters. If the gravida is in labor when theinfection is diagnosed, LSCS is done to prevent transmission of the infection to thebaby. If she is not in labor, HSV type specific serology, HSV type specific PCRand HSV culture with genital swab are done. If her HSV antibodies (Ab) are +ve tothe HSV in genital swab, she is diagnosed to have recurrent herpes, and is managedas described under 'A'. If her HSV antibodies (Ab) are -ve to the HSV in genitalswab, she is diagnosed to have primary herpes. If she is in the first or secondtrimester at that time, she is given suppressive aciclovir therapy is given in a doseof 400 mg q8h PO from 36 weeks of gestation. Then she is managed during laboras in recurrent herpes.

C. If she has no antibodies to the HSV in genital swab, she is given aciclovir 400mg q8h from 36 weeks of gestation. If she is in the first or second trimester, she ismanaged subsequently as in recurrent herpes. If she is in the third trimester, she isdelivered by cesarean section. In case she has premature rupture of membranes, sheis delivered by LSCS immediately. If she opts for vaginal delivery despitecounseling, she is given aciclovir intravenously in a dose of 5 mg/kg q8hintrapartum. The neonate is given the drug in a dose of 20 mg/kg q8hintravenously.

In case of an HIV-positive women with HSV infection, the risk of transmission ofHIV to fetus is increased owing to genital ulceration. The gravida is given aciclovir400 mg PO q8h from 32 weeks of gestation to reduce such transmission, especiallyif a vaginal delivery is planned. Therapy is started so early in view of risk ofpreterm delivery due to HIV. Daily suppressive treatment of HSV is notrecommended if the woman is HIV Ab positive, HSV-1 or -2 seropositive, but hasno history of genital herpes.

273

Page 280: Differential Diagnosis and Management Options inuploads.worldlibrary.org/uploads/pdf/20180107071525differential... · 83 Peripartum cardiomyopathy 165 84 Hypertension 167 85 Convulsions

SUSPICION OF HERPES SIMPLEX INFECTION IN PREGNANCY

Check of past history of genital herpes infection(A)

Present Absent

Recurrent herpes Primary herpes

Suppressive antiviraltherapy from 36 weeks

Check if the gravida is in labor (B)

Speculum examination during labor No Yes

No active lesions Active lesions

Counseling

HSV type specific serologyHSV type specific PCR

HSV culture with genital swab (C)

Avoid using fetal scalp electrode,forceps, and vacuum delivery.Achieve vaginal delivery

HSV Ab +ve to the HSV in genital swab HSV Ab -ve to the HSV in genital swab

Check trimester of pregnancy

First or second Third

Suppressive antiviral therapy from 36 weeks

Deliver by LSCS

274

Page 281: Differential Diagnosis and Management Options inuploads.worldlibrary.org/uploads/pdf/20180107071525differential... · 83 Peripartum cardiomyopathy 165 84 Hypertension 167 85 Convulsions

138. HIV InfectionEarly diagnosis of HIV infection in pregnancy is important so as to reducematernal morbidity and prevent mother to child transmission. It is actuallypreferable that the test to detect HIV is done prior to conception, so that womenwith HIV can be counseled about their reproductive options and the care requiredduring pregnancy, childbirth and in management of their babies.

A. All gravidas should be offered HIV testing routinely, with appropriate pre- andpost-test counseling. The initial test is ELISA, and if it is positive, Western blot testis done to confirm the diagnosis. If the test result is negative, it should be repeatedafter 4 weeks in case of recent exposure or re-exposure. If it is negative, no furtheraction needs to be taken. This test should be repeated in each trimester if thewomen is at high and ongoing risk for HIV infection. If it becomes positive at anytime, action is taken as described under ‘B’.

B. If the gravida is HIV positive, her desire to continue the pregnancy is checked.If she does not wish to continue the pregnancy and her gestational age is less than20 weeks, an MTP is done. She is given contraceptive advice. If she desires tocontinue the pregnancy, or if her gestational age is 20 weeks or more (irrespectiveof her desire to have an MTP) management is done as in ‘C’.

C. The gravida is counseled that with the use of combination antiretroviral therapy(cART) and abstinence from breastfeeding, the risk of vertical transmission isless than 1%. Laboratory tests performed on her include CD4-cell count, HIV viralload, HIV resistance testing, liver and renal function tests, VDRL, Hepatitis B andC tests, and vaginal swab for group B streptococcus. Screening is done fortuberculosis.

D. The benefit of preventing vertical transmission of HIV is believed to outweighthe potential risks of cART. It is checked if the gravida is receiving ART. If she isalready receiving ART which is effective in suppressing HIV viral load and istolerated by the woman, it is continued. Nausea and vomiting of pregnancy maynecessitate adjustment of the therapy. If she is receiving efavirenz and presentsearly, she should be switched from efavirenz to an alternative antiretroviralagent which has greater safety and efficacy in pregnancy. If efavirenz has beenreceived during the first trimester, USG should be done to look for neural tubedefects. Any other drug for which safety and efficacy data are not available shouldalso be switched to another safer and better drug. Prophylaxis for opportunisticinfections is given as discussed under ‘E’.

E. Initiation of cART for all individuals regardless of CD4-cell counts may reducethe risk of HIV vertical transmission, but is associated with the risk of exposing

the fetus to cART. A gravida with CD4-cell count < 200 cells/mm3 is givencART immediately as she is at high risk of opportunistic infections. She is givenprophylaxis for opportunistic infections too. A gravida with CD4-cell countbetween 200 to 350 cells/mm3 is at risk of getting infections like herpes zosterand bacterial pneumonia. Hence cART is initiated after the first trimester iscompleted. If CD4-cell count is more than 350 cells/mm3, cART may be initiatedafter an anomaly scan has been performed. NACO recommends administration ofcART to all HIV positive gravidas irrespective of CD4-cell count and gestationalage. If CD4-cell count is below 200 cells/mm3 prophylaxis against pneumocystisjiroveci pneumonia is given. If the count is below 100 cells/mm3, additionalprophylaxis against Toxoplasmosis gondii is required. If it is below 50 cells/mm3,additional prophylaxis against mycobacterium avium complex has to be given.Continuous oral fluconazole and clarithromycin should be avoided, particularlyin the first trimester. Cotrimoxazole may be safely used throughout pregnancy,but the dose of folic acid should be increased to 5 mg per day in the first trimesterand the infant should be monitored for neonatal hyperbilirubinemia. NACOrecommends Tenofovir 300 mg + Lamuvidine 300 mg + Efavirenz 600 mg qd asthe first-line regimen. Alternative regimens to be used if the first line regimen isnot tolerated include Zidovudine + Lamuvidine + Efavirenz, Zidovudine +Lamuvidine + Nevirapine and Tenofovir Disoproxil Fumarate + Lamuvidine +Nevirapine.

F. Antenatal care is given to all gravidas as usual. Hemoglobin checked at 2, 4 8,12 weeks and 6 monthly thereafter. Liver function tests and CD4 count are doneafter 2 weeks and then 6 monthly. Associated infections like tuberculosis, hepatitisB, hepatitis C and opportunistic infections are treated appropriately.

G. If a gravida presents in labor without having undergone HIV test, a rapid test isdone. If it is positive, blood sample is collected for HIV testing by ELISA and CD4count (to be sent to the laboratory the next day). cART is initiated with Tenofovir300 mg + Lamuvidine 300 mg + Efavirenz 600 mg qd. If the test is positive, cARTis continued. If it is negative, no action need be taken.

H. Universal precautions are taken while conducting labor. Vertical transmissionrisk is increased by the prolonged rupture of membranes, repeated vaginalexaminations, fetal scalp blood monitoring and instrumental delivery. These areavoided as far as possible. A cesarean section is done only for obstetric indicationsand not for preventing vertical transmission.

I. Counseling is done about transmission through beastfeeding and top feeding isrecommended. The infant is given Nevirapine daily NVP for 6 weeks, irrespectiveof whether the infant is exclusively breastfed or receives exclusive replacementfeeding.

275

Page 282: Differential Diagnosis and Management Options inuploads.worldlibrary.org/uploads/pdf/20180107071525differential... · 83 Peripartum cardiomyopathy 165 84 Hypertension 167 85 Convulsions

ANTENATAL SCREENING FOR HIV ANTIBODY (A)

+ve -ve Patient presentswithout a test in labor

(G)

Confirm with Western blot Repeat test after 4 weeks in case of recent exposure or re-exposure Rapid test

+ve -ve

Antenatal care

+ve -ve +ve

No Check desire to continue pregnancy (B) Antenatal care cART

Check gestational age Yes Check of she is on ART

< 20 weeks > 20 weeks No (E) Yes (D)

Perform MTP Laboratory tests (C) cART Continue same treatmentunless includes teratogenor unproved drugs

Antenatal care (F)Laboratory tests

Management in labor (G)

Neonatal management

276

Page 283: Differential Diagnosis and Management Options inuploads.worldlibrary.org/uploads/pdf/20180107071525differential... · 83 Peripartum cardiomyopathy 165 84 Hypertension 167 85 Convulsions

139.Fever And Rash In Pregnancy

Skin rashes that develop during febrile illnesses are caused by various infectiousdiseases, which may be mild ones which resolve spontaneously or severe. Rashesare not important in themselves, but are useful in making the diagnosis of theunderlying diseases. Various types of skin rashes are macule (flat discoloration < 1cm in diameter), patch (flat discoloration > 1 cm in diameter), papule (solidelevated lesion < 1 cm in diameter), plaque (flat-topped, elevated lesion > 1.5 cmin diameter), nodule (rounded, elevated lesion > 1 cm in diameter), vesicle (fluid-filled, elevated lesion up to 1 cm in diameter), bulla (vesicle > 1 cm in diameter),pustule (pus-filled vesicle), wheal – well-demarcated, raised lesion lasting < 24hours), petechia (pinpoint hemorrhage, does not blanche with pressure), purpura(red macule/ papule that does not blanche with pressure), ecchymosis – large areaof bleeding into the skin, erythema (redness that blanches with pressure), crust(dried exudate of blood and/ or plasma) and eschar (hard crust or scab).

Fever and rash may be manifestations of a number of life threatening conditionslike Meningococcemia, Rocky Mountain spotted fever, ehrlichia, Capnocytophagainfection, toxic shock syndrome, typhoid fever, necrotizing fasciitis, viralhemorrhagic fever, Gram negative bacillary infections, and acute endocarditis.Henc an early diagnosis of the condition is important.

A detailed history is very important in differentiating the causes of fever and rash.It should include the following.1. Medications

2. Adverse effects related to any drugs3. Insect bites4. Occupational exposures5. Contact with ill individuals6. Contact with animals7. Sexual practices8. Ingestion of unusual foods9. Immunizations10. Intravenous drug use11. Alcoholism12. Travel13. Details of the rash: site of first appearance, spread to other sites, time over

which the spread occurred14. Associated symptoms including itching and/or pain, change in appearance of

the rash over time15. Diabetes mellitus16. Liver disease17. Malignancy18. Surgery: splenectomy19. Organ transplantation

The rash should be examined in good light (preferably natural) and should bepalpated as well. The distribution and arrangement of the lesions should be noted.

General and systemic examination should be done thoroughly, as the rash may notalways be just infective, but may be one manifestation among many others of thecausative disease.

277

Page 284: Differential Diagnosis and Management Options inuploads.worldlibrary.org/uploads/pdf/20180107071525differential... · 83 Peripartum cardiomyopathy 165 84 Hypertension 167 85 Convulsions

FEVER AND RASH IN PREGNANCY

Check the nature of the rash

Macules/papules Diffuse erythema Vesicles/bullae Nodules Petechiae/purpura Eschar Urticaria

Rash manifests in 7-10 days. The fever canprecede the rash. Both fever and rash resolve

after withholding the causative drug.

Thermaculopapular rash (begins on the face andspreads to most of the body, may clear with milddesquamation), fever, malaise, posterior cervicallymphadenopathy, petechiae on the soft palate.

Fever, rash (slapped-cheek appearance initially, then ageneralized maculopapular rash which may recur withexposure to sun, sometimes papular-purpuric glovesand socks syndrome manifest as painful and pruritic

swelling and erythema of the palms and soles), arthritis(symmetric, involving the peripheral joints) and

arthralgias, aplastic crisis, chronic anemia.Drug reaction Fever, coryza and a maculopapular rash (starts in the head and neck

and spreads centrifugally, may have mild desquamation), Early inthe course of the illness, Koplik's spots (small white spots like grains

of sand on the buccal mucosa opposite the lower molars)

Measles

Rubells

Fifth disease, or erythema infectiosum(Parvovirus B 19 infection)

Maculopapular rash of variable appearance,mononucleosis-type illness. pharyngitis,

adenopathy, and splenomegaly less commonthan with mononucleosis secondary to Epstein-

Barr virus), mild hepatitis.

Rash (maculopapular involving the face, trunk, and sometimesthe extremities, including palms and soles), fever, pharyngitis,

lymphadenopathy, sometimes mucosal ulcerations in the mouth,esophagus or genital region, a history of unprotected sexual

intercourse or sharing needles for injection of recreational drugs.

Rash (maculopapular, petechial, scarlatiniform,urticarial, or vesicular, seen in 5% cases),fever, pharyngitis, and lymphadenopathy,

thrombocytopenia, atypical lymphocytosis,heterophile antibodies measured by the

Monospot test

Cytomegalovirus infection Acute Retroviral Syndrome Epstein-Barr virus infection (infectiousmononucleosis)

Mononucleosis-like illness, rash(maculopapular, spares the face, appears after

resolution of fever), acute hepatitis.

Rash (maculopapular, sometimes petechial orvesicular), pharyngitis/diarrhea, aseptic

meningitis.

Fever, rash (diffuse maculopapular, involving the palms and soles),lymphadenopathy, mucous patches, condylomata lata are visible inthe mouth or on the labia or anus, hepatosplenomegaly, headache,aseptic meningitis, mild elevation of transaminases, spirochetes in

skin lesions.Roseola or exanthem subitum(Human Herpesvirus-6 infection)

Enterovirus infections Syphilis

Flu-like symptoms and rash (erythema migrans - develops 3 – 30days after the tick bite, rapidly enlarging erythematous macule > 5

cm in diameter, often with central clearing).

Fever, chills, headache, relative bradycardia, vomiting, diarrhoea,splenomegaly, abdominal pain, rash (centrally-distributed, erythematous

macules or papules).

Lyme disease Typhoid

Fever, headache,myalgias, and a

maculopapular rash(on the trunk and

extremities includingthe palms and soles).

Murine Typhus(Rickettsia typhi

infection)

278

Page 285: Differential Diagnosis and Management Options inuploads.worldlibrary.org/uploads/pdf/20180107071525differential... · 83 Peripartum cardiomyopathy 165 84 Hypertension 167 85 Convulsions

Macules/papules Diffuse erythema Vesicles/bullae Nodules Petechiae/purpura Eschar Urticaria

Pharyngitis or streptococcal infections atother sites, rash (erythroderma more

pronounced in the skin folds - Pastia'slines, followed by extensive

desquamation), circumoral pallor,strawberry tongue (white or beefy red with

prominent papillae).

Fever, rash (diffuse erythematous, followedby desquamation especially on the palms andsoles), hypotension, multisystem dysfunction.hyperemic mucous membranes negative bloodand skin cultures but staphylococci are grown

from colonized site.

Pain at the site of infection, shock, fever,rash (diffuse erythroderma, bullae at the site

of infection, desquamation duringconvelescence), multiorgan failure. nausea,vomiting, diarrhea, streptococci are grown

in blood and site of infection.

Fever, local pain, a tender,warm, erythematous and swollen

area with indistinct borders,history of trauma and chronic

edema of the extremity

Scarlet fever Stahylococcal Toxic shock syndrome Streptococcal Toxic shock syndrome Cellulitis

Fever, local pain, induration with distinct borders and a peau d'orangeappearance to the skin, bullae.

Pharyngitis, rash (diffuse erythrodermalocalized to the trunk and the proximal upper

extremities, desquamates later) in young adult.

Fever, rash at the site of minor trauma(scarlatiniform eruption followed by flaccid

bullae, denuded areas and then desquamation).

Erysipela Arcanobacterium haemolyticum infection Staphylococcal Scalded Skin Syndrome

Macules/papules Diffuse erythema Vesicles/bullae Nodules Petechiae/purpura Eschar Urticaria

Prodrome of nausea, myalgia, anorexia, and headache followed by rash(small, erythematous macules appear on the scalp, face, trunk, and

proximal limbs, vesicles even palms and soles, with rapid sequentialprogression over 12-14 hours to papules, clear vesicles, and pustules andsubsequent central umbilication and crust formation), intense pruritus at

vesicular stage, malaise, and a low-grade fever, painful, shallow,oropharyngeal or urogenital ulcers. Disseminated lesions are seen in the

immunocompromised cases.

Chicken pox

Vesicular lesions inoral or genital regions,

itching, fever,disseminated lesions,

hepatitis inimmunocompromised

patients.

Herpes simplex

Fever, rash (bullae atthe site of the cellulitis

or scattered withdissemination, healingwith eschar), contactwith salt water or raw

seafood

. Vibrio vulnificusinfection

Vesiculobullouseruption, fresh water

exposure to non-intact skin, medicinal

use of leeches.

Localized skininfection with

Aeromonas sps

Fever, rash(erythematous,

indurated lesion withbullous formation

followed by ulceration),blood culture positive

Pseudomonasaeruginosa septicemia

Fever, arthritis, tenosynovitis, rash(pustular, predominantly on the

extremities)

Fever, sore throat/mouth, vesicles on the buccal mucosa ortongue, rash (papular to vesicular lesions with a rim of

surrounding erythema, mainly on hands and feet),

Rash (central erythema surrounded by normal-appearing skinsurrounded by another ring of erythema, vesiculation of central area,

symmetric in distribution involving the palms and soles too),sometimes with mucosal involvement, conjunctivitis, stomatitis.

Disseminated Neisseria gonorrhoeae Enteroviral infection(Hand-Foot-and-Mouth disease)

Erythema multiforme (Stevens-Johnson syndrome when mucosa,conjunctiva are involved)

279

Page 286: Differential Diagnosis and Management Options inuploads.worldlibrary.org/uploads/pdf/20180107071525differential... · 83 Peripartum cardiomyopathy 165 84 Hypertension 167 85 Convulsions

Macules/papules Diffuse erythema Vesicles/bullae Nodules Petechiae/purpura Eschar Urticaria

Fever, skin nodules (well circumscribed,erythematous, firm and frequently with a pale

center), caused by Candida, Aspergillus,Sporothrix, Cryptococcus, Mucor, Fusarium

Coccidioides, and Histoplasma.

Fever, nodular lesions (disseminated,violaceous in color and can ulcerate).

Immunocompromised patients withatypical mycobacterial infection.

Fever, nodules (tender, erythematous, usuallyconfined to the lower limbs).

Erythema nodosum - non-infectious,inflammatory illnesses mediated by M.

tuberculosis, Chlamydia, and fungi.

Immunocompromised patients withhematological malignancies, bone marrow or

solid organ transplantation, and AIDS

Fever and nodular skin lesions (vascularappearance).

Bacillary angiomatosis

Fever, nodular skin lesions which ulcerate

Sweet's syndrome (febrile neutrophilicdermatosis)

Acute Hepatitis BEnteroviruses

Acuteschistosomiasis,strongyloidiasis,

Loa loa,trichinosis.

Macules/papules Diffuse erythema Vesicles/bullae Nodules Petechiae/purpura Urticaria Eschar

Fever, petechiae, DIC,multiorgan affection,

shock

Fever, petechiae, rapidprogression, systemictoxicity, meningitis.

High fever, severe frontal headache, rash(initially maculopapular, changes to petechial,

starts on the limbs including the palms andsoles, and spreads centrally).

Fever, petechiae, pre-existing valvular heartdisease, invasive procedure without antibiotic

cover, new or changing murmur.

Sepsis Neisseria meningitidisinfection

Rocky Mountain spotted fever Infective endocarditis

Fever, pharyngitis, malaise, adenopathy,petechiae, hepatitis and splenomegaly.

Epstein-Barr virus infections

Fever, life threatening infection.

Dengue, Ebola virus, Lassa fever virus,Machupo virus and Marburg virus infections

Rash (papule, which forms a vesicle that leadsto ulcer formation and eschar), fever, tender

regional lymph nodes, headache, and malaise.

Rash (painless pruritic papule progressing tovesicular center and finally eschar formation,

surrounded by non-pitting edema, satellitelesions), fever, headache and malaise, tender

lymphadenopathy, contact with infected cattle,goats, sheep, or horses.

Fever, headache,buboes, eschar,

history of flea bite.

Fever, headache, tender lymphadenopathy, rash(papule at the site of the bite of a mite

progressing to eschar, disseminated rash sparingpalms and soles).

Tularemia Anthrax Plague Rickettsial pox

280

Page 287: Differential Diagnosis and Management Options inuploads.worldlibrary.org/uploads/pdf/20180107071525differential... · 83 Peripartum cardiomyopathy 165 84 Hypertension 167 85 Convulsions

140. RubellaThe incubation period for rubella is 12-23 days. Rubella is asymptomatic in 25%-50% of cases. Others may have prodromal features like low-grade fever, malaise,headache, sore throat, coryza, conjunctivitis, and tender lymphadenopathy for 1-5days. Then there is development of a maculopapular scarletiniform rashwith/without mild pruritus, beginning on the face and spreading to the trunk andlimbs. It resolves in 3 days in the order of spread. There may be polyarthritis andpolyarthralgia. The patient is infectious from 7 days before to 5–7 days after theonset of the rash. Contact with a patient in this period is called an exposure. Thevirus crosses the placenta. Exposure to the virus in the first trimester may affectmultiple systems of the fetus, causing congenital rubella syndrome (CRS). Itsfeatures at birth include sensorineural deafness (60-75%), cardiac anomalies (10–20%) like pulmonary stenosis, patent ductus arteriosus and ventricular septaldefect, ophthalmic defects (10–25%) like pigmentary retinopathy, cataracts,microphthalmia and glaucoma, neurological defects (10-25%) like mentalretardation, microcephaly and meningoencephalitis, and miscellaneous conditionslike thrombocytopenia and purpura, hepatosplenomegaly and radiolucent bonedisease. Its late manifestations include diabetes mellitus, thyroiditis, behavioraldisorder and deficit of growth hormone.

A. Screening for rubella is done as a routine antenatally, by checking for rubellaIgG. If it is negative, the woman is susceptible, and is immunized after delivery. Ifit is positive and IgG level is more than 10 IU/ml, she is immunized and not likelyto be reinfected. She needs no further management. If it is positive and IgG is lessthan 10 IU/ml, she is at risk of getting rubella. She is advised to avoid contact withrubella and get immunized after delivery.

B. If the gravida has a contact with rubella or has developed rubella like illnesswith fever, joint pains and erythema, then rubella IgG and IgM are both tested. IfIgG is positive and IgM is negative, the woman has preconceptional immunity torubella, due to immunization or past infection. She needs no further management.If both IgG and IgM are positive, she may have a recent infection or a reinfection.Her IgG and IgM testing is repeated. If there is no rise in IgG and IgM is negative,she has preconceptional immunity to rubella, due to immunization or pastinfection. She needs no further management. But if her IgG level has risen and/orIgM has become positive, she has rubella infection (see under ‘C’). If her IgG andIgM are both negative and her contact with rubella was more than 3 weeks ago oronset of clinical disease more than 1 week ago, no further action is required. But if

the contact was less than 3 weeks ago or onset of clinical disease was less than 1week ago, her IgG and IgM levels are checked again. If her IgG and IgM are bothnegative, she is susceptible to rubella infection. She is advised to avoid contactwith rubella and get immunized after delivery. If either IgG or IgM or both becomepositive, she has rubella infection (see under ‘C’). If her IgG is negative and IgM ispositive, her IgG and IgM testing is done again. If her IgG and IgM are bothnegative, she is susceptible to rubella infection. She is advised to avoid contactwith rubella and get immunized after delivery. If either IgG or IgM or both becomepositive, she has rubella infection (see under ‘C’).

C. The risks of fetal infection and development of CRS depend on timing ofmaternal infection. In the first 12 weeks, the rate of fetal infection is about 80%and risk of CRS about 85%. Medical termination of pregnancy (MTP) is advised insuch cases. Between 12 and 16 completed weeks, the risk of fetal infection is about55% and of CRS is about 35%. Counseling is done, and if the patient desires, MTPis done. If not, fetal testing is done by rubella PCR, rubella culture and fetal IgMafter chorionic villus sampling (CVS) or amniocentesis. CVS is associated with therisk of contamination with maternal tissues and false positive rubella PCR testresult. Sensitivity of PCR has not been well validated. But a positive result isuseful in making a diagnosis. False negative fetal IgM is common until late inpregnancy. In case fetal infection is found, MTP may be done if so desired by thegravida. The risk of fetal infection and CRS between 17 and 22 weeks, 23 and 30weeks, 31 and 36 weeks, and beyond 36 weeks is about 36%, 30%, 60% and 100%respectively, while the risk of CRS is very low. MTP is not warranted in suchcases. There is no specific management for the gravida for her rubella infection.Rubella specific immunoglobulin is not effective in preventing maternal infectionafter contact with rubella. Normal human immunoglobulin is not indicated.

Rubella is best prevented by administration of MMR vaccine to a child 0.5 ml SCat the age of 12 months and between 4 and 6 years. A susceptible adult woman isgiven the vaccine 0.5 ml over triceps, repeated after 4 weeks. It is contraindicatedduring pregnancy, febrile illness, and with known allergy to MMR vaccine or itscomponents (e.g. gelatin or neomycin), immunodeficiency, thrombocytopenia, andrecent administration of high doses of immunoglobulins. However it can be givenalong with Rh-immune globulin if required. Adequate contraception should beused for 3 months after vaccination. There are no reports of CRS in the offspring ofwomen inadvertently given MMR vaccine during first trimester of pregnancy. SoMTP is not recommended for these patients. Vaccination of breastfeeding womenis safe.

281

Page 288: Differential Diagnosis and Management Options inuploads.worldlibrary.org/uploads/pdf/20180107071525differential... · 83 Peripartum cardiomyopathy 165 84 Hypertension 167 85 Convulsions

RUBELLA

Rubella IgG for screening (A) Rubella IgG,IgM (B)

-ve +ve, > 10 IU/ml +ve, < 10 IU/ml IgG +ve, IgM -ve IgG +ve, IgM +ve IgG -ve, IgM -ve IgG -ve, IgM +ve

Immunize afterdelivery

Minimal risk ofreinfection

Reimmunize afterdelivery

Past immunization orinfection

Immune to rubella, nofurther management.

Possibility of recentinfection orreinfection

Repeat IgG,IgM if < 3weeks from contact or< 1 weeks from onset

of illness

Repeat IgG,IgM

Repeat IgG, IgM IgG -ve, IgM -ve IgG or IgM +ve

No rise in IgG, IgM –ve Rise in IgG and/or IgM +ve Immunize afterdelivery

Maternal rubellainfection

Past immunization or infection Maternal rubella infection

Immune to rubella, no further management Check type of infection (C)

Primary infection Reinfection

Check trimester Asymptomatic, goodantibodies in past

Classical clinicalfeatures, doubtfulantibodies in past

Second trimesterinfection

First Second Third Observation Medical terminationof pregnancy

Fetal testing

Medical termination of pregnancy Fetal testing Conservativetreatment

Neonatal managementafter delivery

282

Page 289: Differential Diagnosis and Management Options inuploads.worldlibrary.org/uploads/pdf/20180107071525differential... · 83 Peripartum cardiomyopathy 165 84 Hypertension 167 85 Convulsions

141. ToxoplasmosisA. Screening for toxoplasmosis is not done as a routine in pregnancy. It may bedone in high risk groups, such as those who consume raw/undercooked meat and/orunwashed raw vegetables, have contact with young kittens and their litter, or do notwash hands after gardening, or are immunosuppressed or HIV-positive.Immunosuppressed or HIV-positive gravidas are at risk of reactivation andtoxoplasmosis encephalitis. Serological testing is also done when there aresymptoms suggestive of acute toxoplasmosis, including malaise, fever and cervicallymphadenopathy. It is also done when ultrasonography (USG) shows fetalintracranial calcification, microcephaly, hydrocephalus, hepatosplenomegaly,ascites, and/or severe intrauterine growth restriction. If both IgG and IgM arenegative, the gravida does not have toxoplasmosis. She is counseled to avoid eatingraw/undercooked meat, unwashed raw vegetables, eating without washing handsand contact with young kittens and their litter. If IgG is positive and IgM isnegative, she has had toxoplasmosis in the past. She requires no furthermanagement.

B. If IgG is positive or negative and IgM is positive, she probably hastoxoplasmosis. The diagnosis is confirmed by checking for symptoms oftoxoplasmosis, and repeating serological test with another kit, performing IgA testand IgG avidity. If she is asymptomatic for toxoplasmosis, IgM is low positive ornegative, IgA is negative and IgG avidity is high, she does not have toxoplasmosis.If she is symptomatic for toxoplasmosis, IgM is high, IgA is positive and IgGavidity is low, she has toxoplasmosis. IgM can remain positive for years. IgA,rising IgG level and low IgG avidity are more specific for infection within 3months. High IgG avidity after 16 weeks of gestation does not rule out infection inearly pregnancy.

C. The management of toxoplasma infection during pregnancy depends ongestational age. Fetal infection risk is low in the first trimester (4-15%), but if

infected, the risk of fetal damage is high (34-85%) and it is likely to be severe.Fetal infection risk in the second trimester is intermediate (25-44%). If infected,the risk of fetal damage is intermediate (18-33%) and it is likely to be less severe.Fetal infection risk is high (30–75%) in the third trimester. But when infected, therisk of fetal damage is low (4–17%), and the fetus is usually asymptomatic at birth.

D. Maternal treatment in the first trimester is with spiramycin. It is aimed atpreventing vertical transmission to the fetus. It is concentrated in the placentabut does not readily cross it. Hence it is not much useful for the treatment of fetalinfection. It is given in a dose of 1 g (3 million U) PO q8h. These patients need tobe evaluated by USG and amniotic fluid testing after 18 weeks (see under ‘E’).Pyrimethamine and sulphadiazine therapy is considered to be potentially toxic infirst trimester. Pyrimethamine is teratogenic too.

E. When the diagnosis is made in the second trimester, and also when those casesdiagnosed in the first trimester reach the second trimester, USG and amniocentesisfor Toxoplasma gondii PCR are done after 18 weeks. Amniocentesis is done atleast 4 weeks after suspected acute maternal infection. Amniocentesis should notbe done at less than 18 weeks of gestation and not less than 4 weeks after suspectedacute maternal infection so as to minimize the risk of false-negative results. Whenboth USG and PCR are negative, the fetus is not infected. Then maternal therapywith spiramycin is continued up to delivery. If PCR is positive, the fetus isinfected. Medical termination of pregnancy (MTP) is offered up to 20 completedweeks of gestation (the legal limit for performing it). If the gravida does not wantan MTP or is beyond 20 completed weeks of gestation, the treatment ispyrimethamine 100 mg loading dose PO followed by 25-50 mg PO qd plussulfadiazine 500-1000 g q6h. for 3 weeks, alternating with a 3-week course ofspiramycin in a dose of 1 g (3 million U) PO q8h.

A woman who has been diagnosed with an acute Toxoplasma gondii infection andis desirous of becoming pregnant should wait for 6 months before attempting tobecome pregnant.

283

Page 290: Differential Diagnosis and Management Options inuploads.worldlibrary.org/uploads/pdf/20180107071525differential... · 83 Peripartum cardiomyopathy 165 84 Hypertension 167 85 Convulsions

SUSPICION OF TOXOPLASMOSIS

Serological testing for toxoplasmosis (A)

IgG –ve, IgM –ve IgG +ve or –veIgM +ve (B)

IgG +ve, IgM -ve

No toxoplasmosis Suspicion of recent infection Past infection

Counseling Check for symptoms of ToxoplasmosisCheck IgG, IgM with a different kit

Check IgG avidity or IgA

Asymptomatic for toxoplasmosisIgM low +ve or –veHigh avidity for IgG

IgA –ve

Symptomatic for toxoplasmosisIgM high

Low avidity for IgGIgA +ve

Recent toxoplasmosis infection

Check gestational age (C)

First trimester (D) Second trimester (E) Third trimester

Maternal treatment Ultrasonography (USG), amniocentesis and Toxoplasma gondii PCR on amniotic fluid

USG –ve, PCR –ve PCR +ve, USG +ve/-ve

Maternal therapy MTP (up 20 weeks)Maternal therapy (after 20 weeks)

Maternal therapy

284

Page 291: Differential Diagnosis and Management Options inuploads.worldlibrary.org/uploads/pdf/20180107071525differential... · 83 Peripartum cardiomyopathy 165 84 Hypertension 167 85 Convulsions

142. Varicella ZosterVaricella zoster virus (VZV) infection is rare during pregnancy. But it is likely tocause significant complications for both mother and fetus, including maternaldeath, congenital varicella syndrome (CVS), and a severe or fatal form of varicellain the neonate. Its incubation period is usually 14-16 days ((range 10-21 days),which may be prolonged to 28 days if varicella zoster immunoglobulin (VZIG) hasbeen given. Vertical transmission to the fetus occurs during maternal viremia. Theincubation period for neonate is 11 days (range 9-15 days) from the onset ofmaternal disease. Infectivity begins 2 days before the appearance of rash and lastsuntil the lesions crust (4-5 days after the onset of the rash).

A. A past history of chickenpox or vaccination against VZV makes a womanimmune to it. In such cases no action need be taken if there is exposure to VZV(living in the same household as a patient with active chickenpox or herpes zoster,face-to-face contact with a case of chickenpox or zoster for at least 5 minutes orbeing in the same room as that person for one hour or more) during pregnancy. Ifthere is no such history or the status is unknown, then one has to check for VZIG inmaternal serum immediately. Its presence indicates maternal immunity, and nofurther action is required.

B. When VZIG is not found in maternal serum, and the time since exposure is 96hours or less, the gravida is given VZIG in a dose of 125 IU/10 kg of body weightIM, up to a maximum of 625 IU. Its protection lasts for 3 weeks, the half-life of theimmunoglobulin. If the duration is more than 96 hours, oral acyclovir is given in adose of 800 mg 5 times a day for 7 days. It is not very likely to be useful if started14 days after exposure.

C. If the gravida does not develop any rash after prophylactic treatment as in ‘B’,no further treatment is required. But if she develops rash despite the prophylactictreatment, she is observed for the development of complications like respiratorysymptoms, neurological symptoms, hemorrhagic rash or bleeding, persistent feverfor more than 6 days, and development of new lesions after more than 6 days, thenintravenous acyclovir in a dose of 10 mg/kg q8h and supportive treatment aregiven.

D. If there are no complications, then the interval from the onset of the rash ischecked. Oral acyclovir therapy is given if it is up to 24 hours, in a dose of 800 mg5 times a day for 7 days. It is not given if the interval is more than 24 hours.

E. All gravidas are allowed to deliver vaginally unless the fetal well being iscompromised or the gravida has respiratory failure in aggravated by pregnancy. Inthose two situations cesarean section is done.

F. Fetal varicella syndrome (FVS) develops in 0.55%, 1.4% and not at all withmaternal chickenpox at less than 12 weeks, 12 to 28 weeks and after 28 weeksrespectively. Most of the cases occur before 20 weeks. Its features include skinscars, eye abnormalities, limb abnormalities, low birth weight, cortical atrophy,mental retardation and poor sphincter control. If the gravida had chickenpox morethan 7 days before delivery, VZIG is not required. Intravenous acyclovir is givenonly if its maturity is less than 28 weeks or birth weight less than 1000 g. It neednot be isolated from the mother and breastfeeding is permitted. If the woman hadchickenpox up to 7 days before delivery or 2 days after delivery, the newborn isgiven VZIG. It need not be isolated from the mother and breastfeeding ispermitted. If the woman develops chickenpox 2 to 28 days after delivery, the babyis given VZIG only if its maturity is less than 28 weeks or birth weight less than1000 g. It need not be isolated from the mother and breastfeeding is permitted.

285

Page 292: Differential Diagnosis and Management Options inuploads.worldlibrary.org/uploads/pdf/20180107071525differential... · 83 Peripartum cardiomyopathy 165 84 Hypertension 167 85 Convulsions

EXPOSURE TO VARICELLA

Check history of past varicella infection or immunization (A)

Present Absent Unknown

No treatment required Check for VZV IgG antibody in maternal serum

Positive Negative

Check the time of exposure (B)

< 96 hours > 96 hours

Give Varicella zoster immune globulin Give oral acyclovirWatch for development of rash (C)

Rash does not develop Rash develops

No further treatment Check for the presence of complications

No complications (D) Complications

Check interval from onset of rash Give intravenous acyclovir and supportive therapy

< 24 hours

Oral acyclovir therapy

> 24 hours

ObservationObstetric management (E)Neonatal management (F)

286

Page 293: Differential Diagnosis and Management Options inuploads.worldlibrary.org/uploads/pdf/20180107071525differential... · 83 Peripartum cardiomyopathy 165 84 Hypertension 167 85 Convulsions

143. Zika Virus In PregnancyZika virus is a flavivirus transmitted by infected Aedes mosquitoes, which alsotransmit dengue and chikungunya viruses. Though it was first identified in humansin 1952, the first major outbreak was reported in 2015 in Africa. Human infectionis either asymptomatic or mild and self-limiting. It is important because it cancause Guillain-Barré syndrome in the adults and microcephaly and otherneurological complications in the fetus.

A. Exposure to Zika virus occurs when the gravida either resides in an area inwhich Zika virus transmission is taking place, or travels through that area. Thesegravidas are checked for clinical features of Zika virus infection, which includepruritic descending macular or maculopapular rash, fever, arthralgia or arthritis,conjunctivitis, headache, myalgia, retro-orbital pain, vomiting, edema, and isolatedor generalized lymphadenopathy.

B. If the gravida has no clinical features of the disease, an ultrasonographic (USG)anomaly scan is performed to detect fetal anomalies. If no anomalies are detected,normal antenatal care is given. If CNS anomalies are detected, laboratory tests aredone to detect Zika virus infection. RT-PCR is done on maternal serum within 7days of onset of symptoms to detect the virus in maternal serum. The test can bedone on urine of the gravida during the acute phase and up to three weeks after the

onset of illness too. Though RT-PCR can also be done on the woman's saliva andamniotic fluid, this is not done primarily. Serological test can be done to detectZika virus IgM antibodies in the serum of the gravida using ELISA orimmunofluorescence test from the seventh day after the onset of the illness. Crossreactions and false positive test results can occur with past contact with otherflaviviruses like dengue and yellow fever, and also with yellow fever vaccine. Ifthe test is negative or if the fetus has anomalies other than CNS anomalies,investigations are done to detect the cause of the anomalies, and they are managedappropriately. If the patient is confirmed to have Zika virus infection, no specifictreatment can be given as there is no antiviral agent available to treat Zika virusinfection. Only symptomatic and supportive treatment can be given. Rest is givenand measures are recommended to avoid spread of the illness to other people. Oralchlorpheniramine and topical calamine lotion or a menthol-based aqueous agentare used to treat generalized itching. Fever is managed with tepid sponging andparacetamol. Aspirin or other NSAIDs are avoided until dengue infection has beenruled out. Headache is treated with paracetamol. Medical termination of pregnancy(MTP) is offered if there are CNS malformations and the gestation is of less than20 weeks. MTP is not permitted legally beyond 20 weeks of gestation. If thepregnancy is continued, periodic assessment is done.

C. If a patient has clinical features of Zika virus infection and her laboratory test ispositive for that infection, she is managed as above. Ultrasonography is done todetect fetal CNS anomalies and they are managed as described under ‘B’ if present.

287

Page 294: Differential Diagnosis and Management Options inuploads.worldlibrary.org/uploads/pdf/20180107071525differential... · 83 Peripartum cardiomyopathy 165 84 Hypertension 167 85 Convulsions

ZIKA VIRUS EXPOSURE IN PREGNANCY

Check for features of Zika virus disease (A)

Absent Present

Fetal anomaly USG scan (B) Laboratory tests for Zika virus infection (C)

No anomalies Fetal CNS or other anomalies present Negative Positive

Antenatal care Laboratory tests for Zika virus infection Antenatal care Fetal anomaly USG scan

Negative Positive Fetal CNS or other anomalies present No anomalies

Investigate for causeof anomalies

Supportive treatmentAdvise MTP if appropriate and in legal time limit

Periodic assessment if pregnancy is continued

Supportive treatment

288

Page 295: Differential Diagnosis and Management Options inuploads.worldlibrary.org/uploads/pdf/20180107071525differential... · 83 Peripartum cardiomyopathy 165 84 Hypertension 167 85 Convulsions

144. Assessment of Fetal WellBeing

The fetal health may be compromised in utero due to a number ofconditions. Such fetuses need to be closely monitored for their well being, so thatobstetric intervention at appropriate time can save them and reduce their morbidity.Indications for antepartum fetal surveillance are as follows.

1. Pregnancy induced hypertension.2. Other hypertensive disorders.3. Intrauterine growth restriction.4. Postdatism.5. Previous intrauterine fetal death.6. Decrease in or loss of fetal movements.7. Diabetes mellitus.8. Maternal age above 40.9. Severe cardio-pulmonary disease.

10. Rh-isoimmunization.

A. Fetal movements counting is a simple method of fetal surveillance. In anuncomplicated case, maternal subjective assessment and assurance that the fetalmovements are as usual is adequate. In a high-risk case, some method of counting ispreferred. Every fetus has its own rhythm of movements, and no one system canaccurately define fetal compromise if based on achieving a given number in aspecified period. Cardiff’s count to 10 method is quite simple. The gravida startscounting at a fixed time in the morning every day e.g. 8.00 a.m. She notes the time atwhich 10 fetal movements are completed. This time should remain constant so longas the fetus remains well. In case the fetus is slow and does not complete 10movements by 10.00 p.m., the number of movements completed is noted. Thisnumber remains constant as long as the fetus remains well. Delay in completing 10movements or reduction in the number of movements up to 10.00 p.m. indicatespossible compromise of fetal well being.

B. A nonstress test is performed after 28 to 30 weeks of pregnancy. Fetal heart ratepattern is recorded for 20 minutes with an electronic monitor, the gravida being atphysical and mental rest. Presence of at least two fetal movements associated withacceleration of fetal heart rate by at least 15 bpm lasting for at least 15 secondsindicates a reactive and well fetus. It can be nonreactive with a compromise of fetalwell being, and even with a sleeping fetus. Hence it is repeated after 2 hours. If it isstill nonreactive, a stress test is required.

Fetal biophysical profile is a more recent method of fetal surveillance. It uses real-time ultrasonography for assessment of different fetal parameters and a nonstress test(table 144.1). Scores of 8 and 10 are normal, 4 and 6 equivocal, and 0 and 2abnormal. The interval of testing (1 week) is arbitrary, and abnormal scores havebeen found to develop within 24 to 48 hours. Arbitrary assignment of equal score toeach variable needs further assessment. Though recommended as a screening test, the

time and cost involved make it difficult to be so used. It has been said that if thescore is 8/8 for the four parameters other than the NST, it may not be performed.Since a nonstress test is the most important parameter from the 5 parametersassessed, it is perhaps preferable to use that alone for fetal surveillance, and use theprofile for generation of data to establish the value of individual parameters in thescore. Further studies are needed to resolve this issue.

C. A nonreactive fetus my be given stress in different ways. A noninvasive methodis vibroacoustic stimulation, in which a pure tone auditory stimulus of 2000 Hz isapplied over the fetal head as a 5 sec pulse, starting at 105 db and stepping up to 115and 120 db if required. If the baseline FHR increases by 15 bpm for > 3 minutes, thefetus is well. A contraction stress test is more invasive, but closer to labor conditionsand hence probably more accurate in predicting fetal well being. It is said to bepositive if there are late decelerations of FHR with < 3 uterine contractions lastingfor < 60 seconds in 10 minutes, induced by controlled, titrated infusion of oxytocin.Such a fetus is compromised and needs to be delivered urgently, if necessary bymaturation of its lungs by maternal betamethasone therapy.

D. Abnormal fetal umbilical artery Doppler waveforms precede abnormal nonstresscardiotocography results in case of compromise in fetal well being. When the resultsof both of the studies indicate a compromise, the false positive rate is very low. Areduction in fetal aortic diastolic flow is associated with fetal asphyxia. Absent enddiastolic flow indicates considerable risk to the fetus, though immediate delivery isnot warranted. A reversal of flow is associated with fetal death within 24 hours.Doppler study is a very sensitive method of fetal surveillance.

Table 144.1 Biophysical Profile (Manning)

Biophysical variable Normal (score 2) Abnormal (score 0)

Fetal breathingmovements (FBM)

At least 1 episode of FBMsof at least 30 s duration in30 min observation

Absent FBM or no episode of30 s in 30 min

Gross body movements At least 3 discrete body/limbmovements in 30 min

< 2 episodes of body/limbmovements in 30 min

Fetal tone At least 1 episode of activeextension with return toflexion of fetal limb(s) ortrunk. Opening and closingof hand is considerednormal tone.

Either slow extension withreturn to partial flexion ormovement of limb in fullextension or absent fetalmovement.

Reactive fetal heart rate(FHR)

At least 2 episodes of FHRacceleration of > 15 bpmand of > 15 s durationassociated with fetalmovements in 20 min.

Less than 2 episodes of FHRacceleration of > 15 bpm in40 min.

Qualitative amnioticfluid volume

At least 1 pocket ofamniotic fluid at least 1 cmin 2 perpendicular planes

Either no amniotic fluidpockets, or a pockets < 1 cmin diameter in 2perpendicular planes.

289

Page 296: Differential Diagnosis and Management Options inuploads.worldlibrary.org/uploads/pdf/20180107071525differential... · 83 Peripartum cardiomyopathy 165 84 Hypertension 167 85 Convulsions

ASSESSMENT OF FETAL WELL BEING

Assess risk

Low High

Fetal movement counting (A)

Normal Reduced

Continue daily movement counting Doppler study (D) Nonstress test, biophysical profile (B)

Fetus unwell Fetus well Nonreactive NST Reactive NST

Periodic assessment Repeat after 2 hours

Nonreactive Reactive NST

Vibroacoustic stimulation test / contraction stress test (C) Repeat every week

Negative VASTPositive OCT

Positive VASTNegative OCT

Assess fetal maturity Repeat every week

Mature Immature

Induction of labor Betamethasone to mature fetal lungs

290

Page 297: Differential Diagnosis and Management Options inuploads.worldlibrary.org/uploads/pdf/20180107071525differential... · 83 Peripartum cardiomyopathy 165 84 Hypertension 167 85 Convulsions

145. Fetal ArrhythmiasMost of the fetal arrhythmias are intermittent, caused by benign atrial,junctional or ventricular extrasystoles or transient variation in the heart rate.If sustained they require careful evaluation, as they can be fatal.A. Sustained tachycardia may result in hydrops fetalis due to inadequatecardiac output. Fetal echocardiography is done. Atrioventricular (AV)relationship is determined by simultaneously recording the motion of the atrialand ventricular walls. Doppler study with simultaneous recording of themitral inflow (representing atrial rate) and aortic flow (representingventricular rate) is also useful, but M-mode echocardiography is much moreaccurate. Atrial rate (A) more than ventricular rate (V) is diagnostic of atrialflutter, A:V being usually 2:1 or 3:1. Fixed 1:1 AV ratio is due to WPWsyndrome in 2/3 cases. Less often it is due to atrial tachycardia with 1:1conduction to ventricles or ventricular tachycardia with 1:1 conduction toatria. A larger 'V than 'A' is due to ventricular tachycardia. In fetaltachycardia, it may be difficult to evaluate ventricular systolic functionaccurately. Cardiac compromise is diagnosed by the presence of chamberenlargement, fluid collection in pericardial, pleural and peritoneal cavitiesand other features of hydrops fetalis. Color Doppler examination of AVvalves showing significant tricuspid and/or mitral regurgitation is a sign offetal compromise prior to development of hydrops fetalis. Control of fetaltachycardia becomes more difficult with maternally administered drugs oncefetal hydrops develops, due to impairment of transplacental transfer of thedrug.B. Maternally administered digoxin is the conventional treatment for fetalsupraventricular tachycardia. Its efficacy has not been proved. Direct fetaladministration may be more effective, but impractical for long-term control.Verapramil and propranolol can cause maternal hypotension and furthercompromise fetal cardiac function by their negative inotropic effect.Quinidine and procainamide may be effective but unsafe. Flecainide is quiteeffective. Sotalol is effective in converting atrial and supraventriculartachycardias. For rapid control of tachycardia in hydrops fetalis, intravenousamiodarone is used, followed by oral sotalol. If there is a failure of action ofany drug in 48 h, another agent is chosen. Adenosine given into umbilicalvein terminates supraventricular tachycardia, but is not used because therecurrence rate is high.C. Atrial tachycardia (flutter) is treated with a class I agent like quinidine orprocainamide, or a class III agent like sotalol or amiodarone. Digoxin orverapramil slows AV nodal conduction and increases AV block, giving timein which the tachycardia may terminate spontaneously. But it is notrecommended for a hydropic fetus, where the sinus rhythm has to beestablished urgently.

D. Ventricular tachycardia is rare and often requires no treatment due to itsintermittent nature. Sotalol appears to be the best drug to treat it.E. Once the tachyarrhythmia is controlled, a mature fetus is delivered. Apreterm fetus is delivered when mature, watching for recurrence. If thearrhythmia recurs, it is controlled and then the fetus is delivered.F. Sustained fetal bradycardia may be due to fetal hypoxia (fetal distress)or fetal AV block. A fetus with a complete AV block usually has normalmovements, and its ventricular rate is regular and unchanging, despitetreatment for acute fetal distress with maternal left lateral position andoxygenation.G. Fetal echocardiography during complete AV block shows a normalatrial rate and dissociated ventricular rate of 40-100 bpm. Above 30-50% ofthe cases have serious cardiac anomalies that have a perinatal mortality of upto 85%. Isolated AV block is almost always due to maternal connectivetissue disease which is clinically silent. The woman’s blood is checked forthe presence of SSA/Ro and SSB/La. If these are present, the cause is lupuserythematosus and Sjogren syndrome. These are treated appropriately. Afetus with AV block, no hydrops and ventricular rate above 55 bpm needscare as a normal fetus. Placement of an internal fetal lead during labor helpscheck the atrial rate for fetal monitoring. If it fails, fetal scalp pH monitoringis used. A fetus with hydrops or heart rate < 55 bpm is treated withisoprenaline or terbutaline administered to the mother. Digoxin is also usefulin some cases. Early delivery is achieved if drug therapy fails. Postnatally,permanent cardiac pacing is usually required for a neonate with a majorstructural cardiac disease, in about 50% of the cases of isolated AV block,having resting rates < 60 bpm, or a wide complex escape rhythm. If themother has antibodies to SSA (Ro) and/or SSB (La), prophylactic treatmentfor the connective tissue disorder may be offered to prevent the next fetusbeing affected, the risk being 33%.H. Simultaneous recording of atrial and ventricular wall motion and Dopplerexamination are useful in making the diagnosis of an irregular rhythm. Insecond degree AV block the atrial rate is normal, while ventricular rate isslower and usually irregular. It often progresses to complete AV block.Ventricular ectopy shows ventricular contractions preceding atrialcontractions. Second degree AV block is monitored like complete AV block.Intervention is required if it progresses to complete AV block. Atrial ectopyand ventricular ectopy are monitored by weekly fetal heart rate monitoring fordevelopment of sustained tachycardia. Frequent blocked atrial ectopic beatscan result in transient ventricular bradycardia, but that does not endanger thefetus. A fetus with ventricular ectopy should be assessed with ECG afterdelivery, and if the ectopy is persistent, with an echocardiography to see ifthere is myocardial damage. Structural heart disease is treated appropriatelyby a pediatric cardiologist.

291

Page 298: Differential Diagnosis and Management Options inuploads.worldlibrary.org/uploads/pdf/20180107071525differential... · 83 Peripartum cardiomyopathy 165 84 Hypertension 167 85 Convulsions

SUSPECTED FETAL CARDIAC ARRHYTHMIA

Perform fetal echocardiography, determine nature of fetal cardiac arrhythmia

Tachyarrhythmia (A) Bradycardia (F) Irregular rhythm (H)

More ‘A’s than ‘V’s Fixed 1:1 AVrelationship

More ‘V’s than ‘A’s Check for features of fetal distress

Present Absent

SeconddegreeAV block

Atrialectopy

Ventricularectopy

Structuralheartdisease

Atrial flutter Ventricular tachycardiaTreat fetal distress

Check for seriousWeekly FHRmonitoring

WPWsyndrome

Atrial tachycardia with 1:1AV conduction

Ventricular tachycardia with1:1 VA conduction

Fetal cardiac anomalies,SSA/Ro, SSB/La (G)

Check gestational ageCardiac anomalies SLE Sjogren syndrome Absent

Sustainedfetaltachycardia

No fetaltachycardia

> 37 weeks

Deliver fetus aftercontrollingarrhythmia

< 37 weeks

Check fetal decompensation and hydrops fetalisCounseling

Deliver at term

Appropriatetreatment

Check for hydrops fetalis/heart rate < 60 bpm Postnatal ECG

Echocardiography

Present Absent Present Absent

Fetal therapy Maternal drug administrationDrug therapy

Isolated AV block

Check for fetal arrhythmiaSucceeds Fails

Care as for a normalfetus

Supraventriculartachycardia (B)

Atrial tachycardiaflutter (C)

Ventriculartachycardia (D) Observation Early delivery

Check gestational age after control of arrhythmia

> 37 weeks < 37 weeks

Achieve delivery Observation, achieve delivery when mature

292

Page 299: Differential Diagnosis and Management Options inuploads.worldlibrary.org/uploads/pdf/20180107071525differential... · 83 Peripartum cardiomyopathy 165 84 Hypertension 167 85 Convulsions

146. Hydrops FetalisA. Hydrops fetalis is a condition characterized by accumulation of excessive fluidin at least two serous cavities of the fetus, the amniotic cavity being considered oneof the serous cavities. It is the final stage of a number of diseases of the mother orthe fetus. Usually the perinatal mortality with this condition is more than 50%, andmay be nearly 100% in some conditions. It is detected by ultrasonography as earlyas 11 weeks of gestation. Placental edema is most commonly associated withgeneralized skin edema. Ascites, pericardial effusion and pleural effusion can bedetected by ultrasonography. Once it is diagnosed, the plan of action is as follows.B. Immunological causes of hydrops fetalis account for 10 to 27% of the cases.Rh- isoimmunization (the most common), Kell antigen, Fy antigen (a few cases),ABO alloimmunization (extremely rare) cause it. The diagnosis is made bychecking the blood group of the gravida and her husband, and a positive indirectCoomb's test. See chapter 96 for the management.C. The woman's hemogram, serum protein levels and glucose tolerance tests areobtained. Severe anemia of the gravida, severe hypoproteinemia, and uncontrolleddiabetes mellitus may be responsible for hydrops fetalis.D. Serological tests are performed on the gravida for diagnosis of infections likesyphilis, toxoplasmosis, cytomegalovirus infection, rubella, Parvovirus infection,leptospirosis and Chagas' disease. With Parvovirus infection, hydrops developssuddenly due to development of severe anemia. Maternal serum alphafetoproteinmay be raised. Polymerase chain reaction may be used to diagnose this infection.Hydrops due to Parvovirus infection is known to resolve spontaneously, but thereis no way of predicting which cases will resolve.E. A large number of conditions associated with fetal anomalies may beresponsible for hydrops fetalis. An anomaly scan using both transabdominal andtransvaginal route reveals the anomalies.1. A chest mass may be echogenic, isoechoic or hypoechoic. The isoechoic

masses are difficult to diagnose. Their presence is suspected by shifting of theheart and mediastinum. An echogenic mass may be a cystic adenomatoidmalformation of the lung, lung sequestration, or obstruction of a major airway.In the absence of hydrops, fetal survival is 90%. If hydrops is present, openfetal surgery between 21 and 27 weeks of gestation may save such a fetus.Extralobar pulmonary sequestration is an abnormal, nonaerated mass ofextrapulmonary lung tissue supplied by a systemic artery. Palliative treatmentis possible. Congenital diaphragmatic hernia can be treated by fetal surgery.Other lesions responsible for hydrops fetalis include mediastinal teratoma andenteric cyst.

2. Urological anomalies include urethral stenosis or atresia, posterior bladderneck obstruction, spontaneous bladder perforation, neurogenic bladder withreflux, ureterocele and congenital nephrotic syndrome of Finnish type. All canbe diagnosed by ultrasonography. Maternal serum and amniotic fluid alphafetoprotein levels are raised in congenital nephrotic syndrome. It is autosomalrecessive and fatal in infancy.

3. Gastrointestinal anomalies include meconium peritonitis, obstruction,tracheoesophageal fistula, malrotation of intestine, volvulus of small bowel,

and duplication of bowel. Most of the cases of meconium ileus are due tocystic fibrosis. Fetal ascites may be treated successfully by placing a pigtailcatheter to drain the fluid into the amniotic cavity.

4. Hepatic disorders causing hydrops fetalis are hepatic calcification, hepaticfibrosis, cholestasis, biliary atresia, polycystic disease of liver, vascularmalformations, and familial cirrhosis. The fetal prognosis depends on thecause.

5. Fetal tumors associated with hydrops fetalis are teratoma, vascular tumors,neuroblastoma, rhabdomyoma, mesoblastic nephroma, thyroid adenoma andcerebellar medulloblastoma. These are managed as appropriate after delivery.

6. Conditions affecting the placenta and the umbilical cord that can causehydrops fetalis include chorioangioma of placenta, chorionic vein thrombosis,placental and umbilical vein thrombosis, torsion of the umbilical cord, trueknots in the cord, angiomyxoma of the cord, aneurysm of the umbilical artery,and fetomaternal hemorrhage. These conditions are managed as appropriateobstetrically. Often the diagnosis is made only after delivery of the fetus.

7. Miscellaneous conditions responsible for fetal hydrops include congenitallymphedema, congenital hydrothorax or chylothorax, polysplenia syndrome,torsion of ovarian cyst, tuberous sclerosis, and fetal trauma.

F. If the anomaly scan is negative, fetal echocardiography is done. Primary andsecondary intrauterine cardiac failure is the most common mechanism of fetalhydrops in the second and third trimesters. Up to 40% of the cases are due to fetalcardiac abnormalities, such as tachyarrhythmias, congenital heart block, oranatomical defects like atrial septal defect, ventricular septal defect, hypoplasticleft heart, pulmonary regurgitation, Ebstein's subaortic stenosis, dilatation of theheart, atrioventricular canal defect, single ventricle, premature closure of foramenovale, tetralogy of Fallot, subendocardial fibroelastosis, and dextrocardia combinedwith pulmonic stenosis. Other abnormalities which may be found are calcifiedaortic valve, myocarditis due to coxsackie virus, coronary artery embolus, atrialhemangioma, endocardial teratoma, and intracardiac rhabdomyoma. Structuraldefects in the heart are best treated after the delivery of the fetus. The degree ofpleural effusion in these patients may be predictive of survival. When pleuraleffusion is associated with a lung:thoracic ratio < 0.6, severe pulmonary hypoplasiacauses almost 100% perinatal mortality. Biventricular outer dimension of the heartis also a good predictor of the prognosis. Fetal supraventricular tachycardia can betreated medically with digoxin and verapramil administered to the mother.Atrioventricular block can be treated with digoxin and furosemide similarly. Fetalcardiac abnormalities may also be associated with genetic and chromosomalabnormalities too, as discussed later.G. Genetic causes of hydrops fetalis are found in about 15% of the cases between24 and 29 weeks of gestation. Trisomy 21 and monosomy X are the most commonones. Trisomy 13, trisomy 16, and trisomy 18 are also common. Rareabnormalities include mosaicism, translocation, deletion of the short arm ofchromosome 13 and pericentric inversions. Generalized skin edema onultrasonography has a high positive predictive value of aneuploidy and itspresence is an indication for fetal karyotyping before 20 completed weeks ofgestation. An increased nuchal translucency thickness is found in more than 80% oftrisomic fetuses between 10 and 14 weeks of gestation. High fetal loss rate (13%)

293

Page 300: Differential Diagnosis and Management Options inuploads.worldlibrary.org/uploads/pdf/20180107071525differential... · 83 Peripartum cardiomyopathy 165 84 Hypertension 167 85 Convulsions

with large nuchal translucency (> 5 mm) is due to the presence of associatedanomalies, most of which are detected only after 16 weeks of gestation.Association of increased nuchal translucency with generalized skin edema is a badprognostic sign. Cystic nuchal hygromas detected by 9 to 11 weeks of gestationare associated with a risk of aneuploidy of up to 60%. In the presence of a normalkaryotype, the outcome of a fetus with a cystic hygroma is normal in about 80% ofthe cases. Small hygromas may resolve later in the pregnancy and then theoutcome is good. Large hygromas in karyotypically normal fetuses may beassociated with Noonan or Robert's syndrome. In such cases the hygroma resolveslate in pregnancy or not at all. Multiple pterygium syndrome is characterized by acystic mass in the neck, multiple skin webs across joints, and multiple congenitalcontractures. Malformation syndromes associated with fetal hydrops arearthrogryposis multiplex congenital, thanatophoric dwarfism, asphyxiatingthoracic dystrophy, osteogenesis imperfecta, hypophosphatasia, achondrogenesis,Neu-Laxova syndrome, Saldino-Noonan syndrome, recessive cystic hygroma, andPena-Shokeir type I syndrome. Twin transfusion syndrome (TTS) is another majorcause of fetal hydrops. Though vascular connections are present in almost allmonochorionic twins, significant TTS develops in only 15% of the cases.Umbilical artery Doppler flow velocity waveforms are normal in a larger fetus,while a smaller fetus has episodes of low impedance with forward flow in diastoleimmediately followed by periods of absent and reverse flow. Hydropic changes

may be seen in the recipient, donor, or both, as early as 18 weeks of gestation.Presence of such changes in the second trimester indicates a poor prognosis. Thetreatment options are repeated amniocentesis, elective termination of pregnancy,selective feticide, and laser ablation of the vascular anastomoses. Cordocentesisgives fetal blood sample from which fetal hematological conditions can bediagnosed easily, though genetic studies can help make the diagnosis too.Homozygous alpha thalassemia (Hb Bart) is a major cause of fetal hydrops inSouth-east Asia. It is an autosomally recessive condition. Umbilical vein bloodflow and diameter are higher in these cases. Umbilical vein blood flow can becalculated using the following formula.

Blood flow = velocity X lumen area.Hydrops fetalis may be seen associated with other hematological conditions, suchas fetal anemia due to red cell aplasia, aplastic anemia, congenital leukemia,congenital dyserythropoietic anemia, G6PD deficiency, fetal anemia due topyruvate kinase deficiency, and red cell aplasia due to infection of fetal erythroidprecursors by human Parvovirus B 19. Hydrops due to fetal anemia can besuccessfully treated with intrauterine transfusion.Maternal complications of fetal hydrops are preeclampsia (it may develop earlyand necessitate delivery), anemia, antepartum hemorrhage, complications ofassociated hydramnios, higher need for cesarean section, postpartum hemorrhage,retention of placenta, and theca lutein cysts.

Confirm by ultrasonography (A)

SUSPICION OF FETAL HYDROPS

Assessment of immunological hydrops (B)

Immunological Nonimmunological Maternal hemogram, serum proteins, glucose tolerance test (C)

Maternal serological tests for fetal infection (D) Negative Positive

Positive Negative Anomaly scan (E)

Fetal echocardiography (F) Negative Positive

Positive Negative Amniocentesis, chorion villus biopsy or cordocentesis for karyotyping(G)

Evaluation after delivery Normal Abnormal Appropriate treatment

294

Page 301: Differential Diagnosis and Management Options inuploads.worldlibrary.org/uploads/pdf/20180107071525differential... · 83 Peripartum cardiomyopathy 165 84 Hypertension 167 85 Convulsions

147. Craniospinal and FacialAnomalies

A. Open neural tube defects (ONTDs) are multifactorial. Predisposing factorsinclude insulin-dependent diabetes mellitus, anticonvulsants, high doses of vitamin A,and folate deficiency. They are associated with aneuploidy and rare geneticsyndromes. High levels of maternal serum alpha fetoprotein suggest the diagnosis.Ultrasonography is diagnostic in most of the cases. MRI is quite useful too.

B. Anencephaly is diagnosed easily in the second and third trimesters, but less so in thefirst trimester. Ultrasonography shows a mass of thin-walled vessels (area vasculosa)protruding from the base of skull above the orbits, without any skull bones above theorbits. Such bones are seen in severe microcephaly. In early amnion rupture sequence,there is asymmetric malformation and presence of brain tissue. In acrania there isnormal brain without skull. Presence of a brain filled sac is due to an encephalocele.Medical termination of pregnancy is offered up to 20 weeks of gestation, and avaginal delivery thereafter. The baby cannot survive.

C. Presence of a defect in the bone under a sac-like protrusion from the head is due toa cephalocele. A cephalocele with brain in it is encephalocele, without brain iscranial meningocele. A skull defect near midline is due to failure of closure of neuraltube. A defect away from midline or in atypical location is due to early amnionrupture sequence. It is not seen in a cystic hygroma, teratoma, or hemangioma.Prognosis is better for frontal and isolated occipital cephaloceles. A vaginal deliveryis achieved in most cases, tapping a hydrocephalus if required. Cesarean section maybe required for a solid cephalocele causing dystocia.

D. Separation of the posterior ossification centers more widely than in the vertebraabove and below is due to spina bifida. In transverse plane, the centers appearsplayed, producing a U-shaped vertebra. In third trimester, laminae and spinousprocesses are not seen. Arnold-Chiari malformation is present in 90-95% cases.Lemon sign (scalloping of frontal bones from caudal displacement of cranialcontents producing a lemon-like appearance in axial plane in second trimester) andbanana sign (displacement of cerebellar hemispheres into cervical canal causing theirflattening in craniocaudal direction and obliteration of cisterna magna) may bepositive. More than 90% infants survive long term, but severely affected cases havegross permanent multisystem defects. Medical termination of pregnancy is offeredup to 20 weeks of gestation, and a vaginal delivery thereafter. Care is taken tominimize traction on the spine and trauma to the meningomyelocele.

E. Hydrocephalus is ventriculomegaly causing an enlargement of the head. It may bedue to an obstruction to outflow (non-communicating), impaired resorption of the

cerebrospinal fluid (communicating), overproduction of cerebrospinal fluid, orunderdevelopment or destruction of the cortical tissue increasing the ventricular sizerelatively (hydrocephalus ex vacuo). Hydrocephalus is diagnosed when the lateralventricular atrium measures > 11 mm (normal 7.6 mm). This is the earliest sign. It doesnot change with gestational age. Choroid plexus is seen in a dependent position inenlarged ventricle, while normally it fills posterior part of the lateral ventricle and isbilaterally symmetrical in all positions of the head. A primary hydrocephalus may beassociated with spina bifida, or congenital heart disease. Fetal karyotype is obtained as10% cases have chromosomal aberrations. A hydrocephalus may be secondary to DandyWalker syndrome, holoprosencephaly or thanatophoric dwarfism with clover-leaf skull.The treatment is to offer medical termination of pregnancy up to 20 weeks of gestation,and vaginal delivery after 20 weeks. Progressive cases need earlier delivery, afteradministration of betamethasone to mature the fetal lungs, if necessary. A very largehydrocephalus needs to be tapped during labor. It is done vaginally in cephalicpresentation and arrest of aftercoming head of a breech. The latter can be tappedsuprapubically too. The neonate is referred to a neonatologist and pediatric surgeon.See chapter 117 more details.

F. Central fluid collection in abnormal brain may be due to a number of conditions.Holoprosencephaly is due to an incomplete cleavage of the primitiveprosencephalon. It may be alobar, semilobar or lobar, though distinction betweensemilobar and lobar types may be impossible before childbirth. Most infants with lobartype die soon after birth, and the remaining have severe mental retardation. Other twotypes may have a normal life span, but many are severely mentally retarded. Themanagement is as of hydrocephalus. Recurrence in a future pregnancy is likely, the riskbeing 6% in the absence of a parental chromosomal abnormality, and 200 to 500 fold ifthere is insulin dependent diabetes mellitus.

Hydranencephaly is believed to be due to a severe destructive insult likebilateral occlusion of internal carotid arteries. Only the basal ganglia and remnants ofmesencephalon are seen within a normal skull above tentorium cerebelli, the subtentorialstructures being normal. Most babies die within the first year of life. Others have severemental retardation. Management is as of holoprosencephaly.

In porencephaly a portion of the cerebral cortex is replaced by a cystic cavitydue to a hemorrhagic, embolic, infectious, or traumatic insult. The cavity maycommunicate with a ventricle, subarachnoid space, or both. The outcome is poor. Avaginal delivery is achieved at term.

Aneurysm of vein of Galen is due to an arteriovenous malformation. It may beassociated with fetal hydrocephalus or other anomalies. Cardiomegaly and evenhydrops fetalis may occur due to increased intracranial blood flow. A vaginaldelivery is achieved in most cases.

295

Page 302: Differential Diagnosis and Management Options inuploads.worldlibrary.org/uploads/pdf/20180107071525differential... · 83 Peripartum cardiomyopathy 165 84 Hypertension 167 85 Convulsions

Neural tube defects (A)

CRANIOSPINAL AND FACIAL ANOMALIESUltrasonography

Central fluid collection in an abnormalbrain (F)

Absent vault of skull (B) Sac-like protrusions from headnot covered with bone (C)

Hydrocephalus (E)(See chapter 116)

Area cerebr-ovasculosa

Some part ofskull bones

Normal brain Check for bony defect in skull Hypotelorism,single common

ventricle, no

Cerebral cortexabsent

Solitary multipleecho-spared areas

within brainAnencephaly Severe

microcephalyAcrania Present Absent midline echo

Hydranencephaly PorencephalyAmniotic bands Large brain-filled sac Cephalocele Cystic

hygromaHemangioma Teratoma Holoprosencephaly

Early amnion rupturesequence

Encephalocele Check location of defect Nonpulsatile, tubular, fluid-filled structureoften behind third ventricle, Doppler study

Spina bifida (D)Near midline Away from midline

Atypical locationshowing blood flow within it

Check for lemon and banana signs Failure of neural tubeclosure

Amnion rupture sequence Aneurysm of vein of Galen

Absent Present Check for herniation of brain

Arnold-Chiarimalformation

Check presentationPresent Absent

Vertex

Vaginal delivery

Other

LSCSEncephalocele Cranial meningocele

Microcephaly Associatedanomalies

Large sac Associatedhydrocephalus

Dystocia due to alarge and solidcephalocele

Minimal braintissueherniation

Counsel parents about poorprognosis, vaginal delivery

Decompression and vaginaldelivery

Fetal indication for LSCS, parents accept risk of developmental defects

Yes No

LSCS Vaginal delivery

296

Page 303: Differential Diagnosis and Management Options inuploads.worldlibrary.org/uploads/pdf/20180107071525differential... · 83 Peripartum cardiomyopathy 165 84 Hypertension 167 85 Convulsions

148. Fetal AlloimmuneThrombocytopenia

It is a serious fetal disorder. Its incidence is 1:2000 pregnancies. About 50%of the cases occur in primigravidas, in whom the diagnosis is made only afterdelivery. The infant develops purpura, hematuria or gastrointestinal hemorrhage andin 20% of the cases intracranial hemorrhage. Chromosomal abnormalities andinfection with toxoplasma, rubella and cytomegalovirus have to be ruled out byappropriate investigations. The PLA1/PLA2 antigen system is responsible for 80% ofthe cases. Alloimmunization occurs when the gravida has PLA2/PLA2 platelets, andPLA1 platelets of the fetus enter her circulation. Then she forms antiplateletantibodies, which are passed into fetal circulation. They cause fetalthrombocytopenia.

The rate of recurrence of alloimmune thrombocytopenia depends on thealloantigen involved and the zygosity of the father. About 75% are PLA1/PLA1 and25% are PLA1/PLA2. Women at risk also include sisters of women whose babies diedof alloimmunization.

A. Monitoring of maternal anti-PLA1 antibodies is not useful in predicting fetalthrombocytopenia. High titers may be found in cases of mild thrombocytopenia andlow titers with severe thrombocytopenia. In some cases, maternal antibodies may notbe detectable. Hence cordocentesis is the best method of making a diagnosis. Thedisease is likely to be mild, moderate or severe if the previous baby's platelet countwas >100xl09/L, 50-100X109/L, and <50xl09/L respectively. In high risk cases,maternal treatment is given from twelfth week of gestation (see under ‘C’) andcordocentesis is done at 20 weeks. In moderate cases such treatment is not givenprior to cordocentesis. The following tests are done on the fetal blood sampleobtained.

1. Platelet count.2. Platelet group.3. Platelet bound antibodies.

If the platelet count is very low, serological typing of platelet alloantigensmay not be possible. In such cases the typing may be done with DNA amplificationtechniques.

If the fetal platelet count is <50X109/L, the risk of bleeding from the cordafter cordocentesis is increased. So fetal platelet count is done within 2 minutes ofcordocentesis, and if the count is low, platelets are transfused before the needle isremoved. Fetal transfusion of PLW platelets is given every week until delivery isachieved at 35 weeks. If the platelet count is 50-100X109/L, maternal treatment isgiven (see under ‘C’) and cordocentesis is repeated after 3 weeks. If the plateletcount is >100Xl09/L, no treatment is given to the mother, and cordocentesis isrepeated after 10 weeks. Subsequent management in both of these groups is based onthe new platelet count, as discussed before.

B. If the risk is low, cordocentesis may not be done at 20 weeks. The PLA genotype ofthe father is determined. If the father is heterozygous, there is a 50% chance of thefetus being affected. In that case amniocentesis or chorion villus biopsy is done todetermine fetal PLA genotype. If the fetus is PLA2 homozygous, further cordocentesisand maternal treatment are unnecessary. If it is heterozygous, cordocentesis is done.

C. All at-risk fetuses need to be treated to avoid intracranial hemorrhage (whichoccurs in utero in up to 50% of the cases) leading to fetal death, hydrocephalus, orporencephalic cyst. Maternal treatment includes the following.

1. Intravenous immunoglobulin.2. Glucocorticoids.

This treatment is given to all high risk cases from 2-16 weeks of gestation, andcordocentesis is done at 20 weeks.

297

Page 304: Differential Diagnosis and Management Options inuploads.worldlibrary.org/uploads/pdf/20180107071525differential... · 83 Peripartum cardiomyopathy 165 84 Hypertension 167 85 Convulsions

SUSPICION OF ALLOIMMUNE THROMBOCYTOPENIA

Assess risk

High (A) Medium Low (B)

Maternal treatment(C)

PLA genotyping of father

Cordocentesis Heterozygous Homozygous PLA2

Fetal platelet count Amniocentesis or chorion villus biopsy

< 50X109/L 50-100X109/L > 100X109/L Fetal PLA genotyping

Fetal transfusion ofplatelets every weekuntil delivery at 35

weeks

Maternal treatmentGlucocorticoids in 3

weeks

Repeat cordocentesisin 10 weeks

Heterozygous Homozygous PLA2 No further evaluationor treatment

298

Page 305: Differential Diagnosis and Management Options inuploads.worldlibrary.org/uploads/pdf/20180107071525differential... · 83 Peripartum cardiomyopathy 165 84 Hypertension 167 85 Convulsions

149. Intrapartum FetalMonitoring

A. Indications for intrapartum fetal heart rate (FHR) monitoring are as follows.

1. Pregnancy induced hypertension.2. Concurrent pregnancy and hypertension.3. Chronic renal disease.4. Diabetes mellitus.5. Cardiovascular disease.6. Intrauterine growth restriction.7. Postmaturity.8. Previous intrapartum fetal death.9. Rh-isoimmunization.

10. Maternal age above 40.11. Meconium staining of amniotic fluid.12. Use of oxytocin for induction or augmentation of labor.13. Prolonged labor.

B. An abdominal Doppler transducer is used prior to rupture of membranes.However its record may not be continuous owing to loss of signal during uterinecontractions and maternal movement due to pain. A scalp electrode is applied afterrupture of membranes. It picks up fetal ECG signal without interruption. Short term(beat-to-beat) variability is exaggerated with an external transducer. If the pattern isnormal, observation is continued.

C. An early deceleration is due to compression of the fetal head. There is a drop inthe fetal heart rate (FHR) with a uterine contraction, its nadir coinciding with thepeak of contraction, and it recovering as soon as the contraction wears off. It doesnot need any treatment.

D. A variable deceleration is due to compression of the umbilical cord, which maybe secondary to cord presentation, occult cord presentation, tight loop(s) of cordaround the neck, true knot in the cord, and oligohydramnios. There is a severe dropin FHR (usually below 100 bpm) with a shoulder of acceleration preceding it andanother following it. Such a gravida is given head low position which relieves cordcompression secondary to occult cord presentation. The pattern persists in cases ofoligohydramnios, true knot and loop of cord around the neck. If she hasoligohydramnios, saline amnioinfusion is done through an intraamniotic catheter (15ml/min). If it relieves compression of the cord by separating the uterine wall fromthe fetus, the amnioinfusion and observation are continued. If the pattern persists,fetal scalp blood pH analysis is required to diagnose fetal acidosis.

E. A late deceleration is due to fetal asphyxia during a uterine contraction, whenthe flow through uteroplacental vessels is cut off. It is seen with placentalinsufficiency. There is a drop in FHR which starts after the uterine contractionstarts, and persists beyond the contraction. Such a gravida is given a left lateralposition if she is already not in such a position. That relieves aortocaval compressionand hence improves perfusion of the uteroplacental vessels. Moist oxygen isadministered by a face mask to improve oxygenation of maternal blood and therebythat of the fetus. If the pattern is relieved by these measures, the same treatment andobservation are continued. If it is not relieved, fetal scalp blood pH monitoring isrequired.

F. Fetal scalp blood is sampled by making 1 or more (up to 3) stab incisions in thescalp with a 2 mm X 2 mm blade mounted on a handle. The blood is collected in aheparinized capillary tube. Normal values of fetal scalp blood analysis are shown intable 149.1.

Table 149.1 Fetal Acid-Base Values

Parameter Lower Upper Average

pH 7.15-7.30 7.30 - 7.47 7.25-7.35PC02 22-34 40-67 40-45PO2 7-17 23-36 20-25

Base excess -12.1 to -3.1 -1.5 to -5.1 > -10

If the pH is above 7.25, the abnormal FHR pattern is false positive, and observationis continued. A value between 7.2 and 7.25 needs to be repeated after 15 minutes. Ifit is > 7.25, the gravida is observed. If it remains between 7.2 and 7.25, it is repeatedevery 15 minutes. If it drops, an immediate delivery is achieved. A value below 7.2is repeated immediately, and if it is the same, an immediate delivery is achieved.Neonatal Apgar scores can be predicted accurately in 30% cases from a single pHvalue. A score of 7 or more is predicted by pH value of 7.2 or more. The score isbelow 7 if the value is below 7.2.

Continuous fetal tissue pH monitoring is now possible with the use of a pHsensitive glass tip electrode which fits in the fetal scalp at a depth of 3 mm. Acontinuous readout of fetal tissue pH every 1.5 seconds is displayed as a barographon uterine contraction channel of the fetal monitor record. Mean tissue pH is lowerthan capillary blood pH by 0.043 pH units.

Continuous fetal tissue oxygen level monitoring is also possible using atranscutaneous oxygen electrode. However its interpretation can be erroneousbecause it is affected by a large number of variables.

299

Page 306: Differential Diagnosis and Management Options inuploads.worldlibrary.org/uploads/pdf/20180107071525differential... · 83 Peripartum cardiomyopathy 165 84 Hypertension 167 85 Convulsions

Assess risk factor (A)

INTRAPARTUM FETAL MONITORING

Electronic FHR monitoring (B)

Early deceleration (C) Variable deceleration (D) Late deceleration (E) Normal pattern

Continue observation Head low position Left lateral positionOxygen by face mask

Continue observation

Pattern is relieved Pattern persists Pattern persists Pattern is relieved

Occult cord presentation Assess for oligohydramnios Continue observation

Continue observation Present Absent

Saline amnioinfusion

Pattern is relieved Pattern persists Scalp blood pH (F)

Continue amnioinfusion and observation > 7.25 7.2-7.25 < 7.2

Continue fetalmonitoring

Repeat pH after 15 minutes Repeat pH immediately

> 7.25 Stable Falling 7.2-7.25 < 7.2

Repeat after 15 minutes Immediate delivery

300

Page 307: Differential Diagnosis and Management Options inuploads.worldlibrary.org/uploads/pdf/20180107071525differential... · 83 Peripartum cardiomyopathy 165 84 Hypertension 167 85 Convulsions

150. Acute Fetal DistressA. An immediate delivery is sometimes required in fetal interest. Such a situation isseen when there is acute intrapartum fetal distress due to any cause from thefollowing.

1. Maternal conditions: supine hypotension, shock (hemorrhagic, cardiac, septic),hypoxia, hypercarbia, aspiration syndrome, bronchospasm, shock-lung syndrome,pregnancy induced hypertension, sickle cell crisis.

2. Placental conditions: abruption, placenta previa, premature aging, multipleinfarcts.

3. Uterine conditions: polysystole, hypertonia, overstimulation with oxytocin,obstructed labor in a multipara, uterine rupture.

4. Umbilical cord conditions: prolapse, true knot, tight loop(s) around the neck,short cord.

5. Fetal conditions: intrauterine growth restriction, postmaturity, Rh-isoimmunization, cardiac failure, hemorrhage.

B. A diagnosis of acute fetal distress is made by the following conditions.

1. Fetal heart rate (FHR) abnormalities: tachycardia (> 160 bpm), bradycardia (<110 bpm), irregularity of FHR.

2. Passage of meconium into the amniotic fluid.3. Electronic FHR monitoring: late or variable decelerations (see chapter 149).4. Fetal scalp blood pH analysis: pH below 7.2 (see chapter 149).

C. The following measures are taken to relieve acute intrapartum fetal distress.

1. Left lateral position.2. Hyperoxygenation: 6-7 L/min by a face mask.3. Correction of maternal hypotension, if any.4. Reduction of uterine activity: discontinuation of oxytocics, tocolytics (only if

immediate delivery is not possible).5. Correction of maternal hypoglycemia and acidosis, if any.

If the fetal distress is relieved by such measures, labor is allowed toprogress and close monitoring of fetal well being is continued. If there is no relief, animmediate delivery must be achieved or the fetus may die.

D. If the cervical dilatation is less than 10 cm, a cesarean section is done immediately.If it is 10 cm, the maternal pelvis is reassessed by internal pelvimetry. If it isinadequate, a cesarean section is required. If it is adequate, a choice of mode ofvaginal delivery is made based on findings of obstetric examination.

E. If the pelvis is adequate fetal presentation determines the mode of delivery.

1. Vertex presentation:a. Vacuum extraction: it is done when vertex is at station 0 to +2, or even

below, if the internal rotation is incomplete.b. Obstetric forceps delivery: it is done when the internal rotation is complete

and the station is below +2.c. Internal podalic version and breech extraction is done when the vertex of the

second of twins is floating.d. Cesarean section: for all cases with floating vertex other than second of

twins.2. Breech presentation: breech extraction is done if the breech is engaged. If it is

floating, a cesarean section is done.3. Transverse lie: internal podalic version (IPV) and breech extraction (BE) is

done for second of twins. All others are delivered by cesarean section.4. Face presentation: obstetric forceps delivery is achieved when the station is

below +2, and the internal rotation is complete. All other cases are delivered bycesarean section.

5. Brow presentation: all patients are delivered by cesarean section.

301

Page 308: Differential Diagnosis and Management Options inuploads.worldlibrary.org/uploads/pdf/20180107071525differential... · 83 Peripartum cardiomyopathy 165 84 Hypertension 167 85 Convulsions

Assess cause (A)

ACUTE FETAL DISTRESS

Symptomatic treatmentDiagnosis (B)

Relief No relief

Assess cervical dilatation (C)

< 10 cm 10 cm

Cesarean section Reassess pelvic dimensions (D)

Inadequate Adequate

Cesarean section Assess presentation (E)

Vertex Breech Transverse Face Brow

Assess station Floating Engaged Second oftwins

All others Assess station Cesarean section

< 0 0 to +2 +3 to +5 Cesareansection

BE IPV +BE

Cesareansection

< +2 +3 to +5

Second of twins All others Vacuumextraction

Assess internalrotation

Cesareansection

Lowforcepsdelivery

IPV + BE Cesareansection

Incomplete Complete

Vacuumextraction

Low forcepsdelivery

302

Page 309: Differential Diagnosis and Management Options inuploads.worldlibrary.org/uploads/pdf/20180107071525differential... · 83 Peripartum cardiomyopathy 165 84 Hypertension 167 85 Convulsions

151. Ambiguous GenitaliaAmbiguous external genitalia are now called as disorders of sexual development (DSD) isa complex condition that requires clinical acumen in making a diagnosis and tact indealing with anxieties of the parents until the diagnosis is made.

A. If an apparently female neonate has clitoromegaly and dysmorphic features,it may have Seckel's syndrome (microcephaly, dwarfism), lipodystrophy, Meldenhallsyndrome (acanthosis, insulin resistance), or skeletal anomalies. It may also bedue to some other rare syndromes which result in infant death.

B. If the features of the neonate are normal, local causes of clitoromegaly likehemangioma, lipoma and neurofibroma are looked for. These can be treated surgicallyat a later date.

C. A female fetus can get virilized by an exposure to androgens derived frommaternal ingestion of androgenic progestins, ovarian tumor (luteoma of pregnancy),or adrenal tumor. An exposure before 13 weeks of gestation results in formationof urogenital sinus and fusion of labial folds. Exposure after 13 weeks causesonly clitoral hypertrophy.

D. Neonatal hormone tests are done if there is no demonstrable maternal sourceof androgens, as judged from the history, clinical examination and investigationslike pelvic ultrasonography and abdominal CT scan or MRI. These tests includeserum 17 (OH) progesterone, and urinary excretion of 17 ketosteroids and pregnanetriol.The levels are abnormal in congenital adrenal hyperplasia (CAH) as shown in thefollowing table.

Deficiency of Serum17(OH)progesterone

Urinary 17ketosteroids

Urinarypregnanetriol

Serumcortisol

21hydroxylase High High High Low in saltlosing type

11 betahydroylase

High High High Normal

3 beta (OH)steroiddehydrogenase

Normal High Low

17 alphahydroxylase

Normal

E. The testes descend to the scrotum in a normal male and some cases of ambiguousgenitalia in a male. The ovaries are never found below their normal level in thepelvis. Thus if a gonad is palpable in a bifid scrotum, labium majus, or inguinalcanal, it is always a testis. But if it is not palpable, it does not necessarily mean

that the neonate has ovaries. The testes may be in the pelvis and may or maynot be found on a rectal examination. They may be found on an ultrasonic scanof the pelvis. In either case, a karyotype is obtained.

F. Most of the cases of mixed gonadal dysgenesis have a karyotype of 45X/46XY. The phenotype is male or female. There is a testis on one side and astreak gonad on the other side. Most phenotypic males show incomplete virilizationat birth, with some degree of phallic enlargement, urogenital sinus, and varyingdegrees of labioscrotal fusion. Mullerian structures are present on one side.Gonadoblastomas develop in 25% of the cases in childhood.

G. Ovotesticular DSD (formerly true hermaphroditism) is relatively rare. Testicularand ovarian tissues are found in the same or opposite gonads. The external genitaliareveal all gradations of ambiguity from the male to the female end. Internal genitaliacorrespond to the gonad on that side. A variable degree of virilization andfeminization develop at puberty. Gynecomastia develops in 60% of the cases, andmenstruation in 50% of the cases. Different karyotypes encountered are as shown inthe algorithm.

H. The karyotype is 46XY in 46,XY DSD (formerly male pseudohermaphroditism).The external genitalia are incompletely virilized. It may be due to abnormalsynthesis of androgens due to enzyme defects inherited as autosomal recessive traits.These enzymes are 20,22 desmolase, 3 beta (OH) steroid dehydrogenase, 17,20desmolase and 17 beta (OH) steroid dehydrogenase.

I. Incomplete androgen insensitivity syndrome is inherited as a sex-linked trait. Itsfeatures are a varying degree of breast development, virilization at puberty, andambiguity of the external genitalia varying from the completely female type (Lub'ssyndrome) to the completely male type (Reifenstein syndrome).

J. The karyotype in 5 alpha reductase deficiency is 46XY. There is pseudovaginalperineoscrotal hypospadias.

Treatment: the sex of rearing is determined as the first step. Until that decision ismade, it is preferable to tell the parents that the degree of differentiation is not yetcomplete, and it will be clear after the results of laboratory tests are available. Mostof the cases of 46,XX DSD have a normal potential for reproduction and should bereared as females. If the size of the phallus is adequate, the child is reared as a male.If not, it is reared as a female, performing a gonadectomy at puberty to preventmasculinization and development of testicular tumors. Ovotesticular DSD cases areusually reared as females, their testes being removed. Plastic surgery is required tocorrect genital ambiguity such as clitoral recession, vaginoplasty, labioscrotalreduction etc. Estrogen replacement therapy is required if the sex of rearing is femaleand the gonads are streak or surgically removed. CAH is treated medically withgluco- and mineralocorticoids.

303303

Page 310: Differential Diagnosis and Management Options inuploads.worldlibrary.org/uploads/pdf/20180107071525differential... · 83 Peripartum cardiomyopathy 165 84 Hypertension 167 85 Convulsions

AMBIGUOUS GENITALIA

Look for dysmorphic features (A)

Present Absent

Seckel’ssyndrome

Lipodystrophy Meldenhallsyndrome

Skeletalanomalies

Present Look for local causes of clitoromegaly in a female fetus (B)

Hemangioma Lipoma Neurofibroma Absent

Check for maternal ingestion of androgen, androgenic progestin,or production of androgen by an ovarian tumor (C)

Present Absent

Androgenic effect on a female fetus Hormone tests (D)

CAH Normal

Palpate for testes (E)

Palpable Not palpable

Karyotyping Karyotyping

45 X/46 XY 46 XX, 46 XY, 45 X/46 XY,46 XX/47 XXY, 46 XX/46 XY

46 XY

46,XY DSD

46 XX

46,XX DSD

Mixed gonadaldysgenesis

Ovotesticular DSD Abnormal androgensynthesis (G)

Incompleteandrogen

insensitivity (H)

Incomplete familialform of XY gonadaldysgenesis (I)

5 alpha reductasedeficiency (J)

304

Page 311: Differential Diagnosis and Management Options inuploads.worldlibrary.org/uploads/pdf/20180107071525differential... · 83 Peripartum cardiomyopathy 165 84 Hypertension 167 85 Convulsions

152. Postpartum HemorrhagePostpartum hemorrhage (PPH) is hemorrhage from or into the genital tract in excessof 500 ml at any time after the birth of the fetus up to the end of puerperium. It maybe third stage hemorrhage, primary PPH (within 24 hours after the third stage) orsecondary PPH (after the first 24 hours, during the puerperium). Resuscitation is doneby infusion of Ringer's lactate, plasma expanders, and whole blood as required.

A. If the placenta and membranes are still retained, placental separation is assessed.Two signs diagnostic of placental separation are apparent lengthening of theumbilical cord and placenta palpable in the vagina. A separated placenta is removedby Brandt-Andrews method. If it fails, or if the woman is actively bleeding while thesigns of placental separation are negative, the placenta is removed manually.

B. Usually it is easy to find a plane of separation between the placenta and theuterus. The procedure is difficult if some time has passed since the delivery of thefetus and the cervix has closed. In such a case, general anesthesia is required, usinghalothane, isoflurane, enflurane, desflurane or sevoflurane to relax the cervix. Theprocedure can fail also if the placenta lies above a constriction ring. Amyl nitrite,though a good drug for relaxing a constriction ring, is not available commercially.Such a case is treated under general anesthesia with any of the agents listed abovefor relaxing the ring. Another cause of failure is morbid adhesion of the placenta, asin placenta accreta, increta, or percreta. With partial separation of such a placenta,there is severe hemorrhage. If most of the placenta is removed, and only a small partis left attached to the uterus, sharp curettage is done. All other cases are best treatedby total abdominal hysterectomy early rather than late.

C. If the placenta and membranes have been expelled, the uterus is palpated todifferentiate between atonic and traumatic PPH. If it is soft and flabby, the PPH isatonic. The placenta and membranes are examined for completeness. If anycotyledon of the placenta or a portion of the membranes is missing, it is removedwith fingers passed into the uterine cavity. Even if they are complete, it is preferableto gently explore the uterine cavity for anything retained (e.g. succenturiate lobe ofthe placenta, fetus papyraceous). If the uterine cavity is empty, uterine contraction isachieved by the following measures.

1. Uterine massage.2. Bimanual compression and elevation.3. Uterotonic agents.

a. Methyl ergometrine 0.1 mg IV stat.b. Oxytocin infusion: 10 units in 500 ml Ringer lactate.c. PGF2 alpha: 250 μg IM.d. Misoprostol 600 μg PR.These may be used singly or in combination, after ruling out contraindications.

Usually these measures control atonic PPH. This woman needs close observation inthe fourth stage of labor for recurrence of PPH.

D. If the uterus is well contracted, the PPH is likely to be traumatic, such as due tocervical tear, vaginal tear, or uterine rupture. The former two can be diagnosed by aspeculum examination and easily repaired. Uterine rupture is diagnosed by presenceof features of internal hemorrhage, and passage of the examining hand into theperitoneal cavity when passed through the site of rupture through cervix. It ismanaged by exploratory laparotomy and repair, combined with a sterilizationoperation, and if that is not possible, by total abdominal hysterectomy.

E. If the uterus is well contracted and there is no injury to the birth passage, thewoman's coagulation profile is checked. See chapter 155 for the management of acoagulopathy.

F. Atonic PPH which cannot be controlled by measures described before, and PPHdespite normal coagulation profile in presence of a well contracted uterus is managedby uterine packing. If it succeeds, the pack is kept in place for 12 hours underantibiotic cover. If it fails, ligation of anterior division of the internal iliac artery isdone. If that also fails, total abdominal hysterectomy is done as a last resort.

G . If the uterus is well contracted and retracted to start with, the bleeding is likely tobe traumatic. It has been discussed under “D’.

H. If the uterus remains relaxed and continues to bleed after a cesarean section, theplacenta and membranes are checked. If they have been completely removed,bimanual compression and massage of the uterus with one hand behind the uterusand the other in front of the uterus is done. Uterotonic agents are used as describedunder ‘C’. If that does not control the bleeding, a uterine compression suture like B-Lynch suture is placed. If that does not control the bleeding, bilateral ligation of theanterior division of the internal iliac artery is done. If that fails to control hebleeding, a total abdominal hysterectomy is done. If the placenta is found to bemorbidly adherent and cannot be removed, a total abdominal hysterectomy is done.

305

Page 312: Differential Diagnosis and Management Options inuploads.worldlibrary.org/uploads/pdf/20180107071525differential... · 83 Peripartum cardiomyopathy 165 84 Hypertension 167 85 Convulsions

POSTPARTUM HEMORRHAGE

ResuscitateAfter vaginal delivery, uterus relaxed After vaginal delivery, uterus contracted (G) After a cesarean section (H)

Check placenta and membranes (A) Expelled Check placenta and membranes

Retained Palpate uterus Notpalpable

Completely removed Retained

Assess placental separation Relaxed

Examine placenta

Well contracted and retracted

Look for trauma to birth canalSee

chapter Manual removalSeparated Incompletely separated and membranes

Absent Present153

Brandt Andrews methodComplete Incomplete Coagulation

profileRepair trauma

Succeeds Fails

Removal complete Removal failsManual removal ofretained products

Abnormal NormalMorbidly adherent placenta

Manual removal of placenta (B)

Uterotonic drugs

See chapter155

Removal successful Removal fails Bimanualcompression-massage

Assess causePPH controlled PPH not controlled

Bimanual compression, massage,uterotonic agents

Constriction ring Placenta accreta Uterine packing (F)

Achieve relaxation ofring, manual removal

Check cleavage plane

PPH controlled PPH not controlled

Succeeds Fails

Present Absent Uterine compression suture

Partial placentaaccreta

Complete placentaaccrete, increta/percreta

Close observationRemove pack after 12

hours

Ligation of anteriordivision of internal

iliac artery

Succeeds Fails

Check degreePPH controlled PPH not controlled

Ligation of anterior division ofinternal iliac artery

Very small SignificantClose observation Total abdominal Succeeds Fails Total abdominal hysterectomy

Sharp curettage hysterectomy

Succeeds Fails Total abdominal hysterectomy

306

Page 313: Differential Diagnosis and Management Options inuploads.worldlibrary.org/uploads/pdf/20180107071525differential... · 83 Peripartum cardiomyopathy 165 84 Hypertension 167 85 Convulsions

153. Postpartum Hemorrhage:Uterus Not Palpable

See chapter 138 for preliminary discussion on postpartum hemorrhage(PPH). In traumatic PPH, and most of the cases of atonic PPH, the uterus is palpableabdominally. In the former, it is firm and well contracted. In the latter, it is soft andflabby. If the woman has severe bleeding vaginally and the uterus is not palpableabdominally, there are only two possibilities.

A. A vaginal and speculum examination is done for the diagnosis of acute inversion ofthe uterus. The inverted fundus is seen as a fleshy mass projecting out of the cervixor prolapsed outside the vulva. It causes shock by two mechanisms, hemorrhage(hypovolemic shock) and stretching of peritoneum (neurogenic shock). If the womanis not in shock, the inversion can be corrected immediately. If she is in shock, theinverted uterus is reposited in the vagina (if it is outside the vulva). The shock istreated. Then the cervix is assessed.

B. If the cervix is open, the inversion is corrected manually. A hand is passedinto the vagina and the inverted fundus of the uterus is held in the palm of thehand. The fornices are stretched in all directions by outstretched fingers. Thehand is elevated so that the uterine ligaments get stretched and they pull out theinverted portion, correcting the inversion.

C. Hydrostatic method of O'Sullivan is used if the manual method fails. The vaginais distended with sterile, warm saline passed into it through a catheter connected to adouche can at a height of 60 cm above it. A towel is placed over the vulva around thecatheter to prevent the fluid leaking out. Distension of the vagina stretches the

vaginal fornices and hence the cervix in all directions, and pushes on the invertedfundus upwards, correcting the inversion.

D. If the hydrostatic method fails, or if the cervix has closed, the inversion iscorrected manually under general anesthesia, with halothane or another uterusrelaxing agent to permit cervical dilatation. With any method of correction of theinversion, it is important to achieve good uterine contraction after the correction soas to prevent a recurrence of the inversion. Uterotonic agents are used for thispurpose, along with physical measures (see chapter 138).

E. If all the preceding methods fail to achieve a correction of the uterine inversion,any of the following methods may be used to correct it.

1. Huntington's method: a series of vulsellums are applied to the inverted uterinewall by an abdominal approach, and the inverted portion is pulled out.

2. Haultain's operation: the constricting ring of the cervix is cut posteriorly by theabdominal route. The inversion is then corrected and then the divided portion isrepaired.

3. Dobbin's operation: it is similar to Haultain's operation, except that the ring isdivided anteriorly.

4. Spinelli's operation: the constricting ring of the cervix is divided anteriorly bythe vaginal approach, the inversion is corrected, and then the incision isrepaired.

5. Kustner's operation: it is similar to Spinelli's operation except that the cervicalring is divided posteriorly.

F. If the uterus is not inverted in a case of PPH in which the uterine fundus is notpalpable abdominally, it lies under the diaphragm. This is due to atonic PPH, theuterus being filled with blood. See chapter 138 for discussion of atonic PPH.

307

Page 314: Differential Diagnosis and Management Options inuploads.worldlibrary.org/uploads/pdf/20180107071525differential... · 83 Peripartum cardiomyopathy 165 84 Hypertension 167 85 Convulsions

POSTPARTUM HEMORRHAGE: UTERUS NOT PALPABLE

Vaginal and speculum examinationCheck for uterine inversion (A)

Present Absent

Assess for shock Uterine distension by atopic PPH,fundus below the diaphragm

Present AbsentSee chapter 138

Reposit uterus into the vagina, treat shock. Judge condition of the cervix

Open Closed

Manual correction

Succeeds Fails

Manual removal under general anesthesia withuterus relaxing agent

Succeeds Fails

Treatment of atonic PPH

See chapter 138

308

Page 315: Differential Diagnosis and Management Options inuploads.worldlibrary.org/uploads/pdf/20180107071525differential... · 83 Peripartum cardiomyopathy 165 84 Hypertension 167 85 Convulsions

154. Late PostpartumHemorrhage

A woman is said to have late postpartum hemorrhage if she bleeds into orfrom the genital tract in excess of 500 ml at any time from the end of the first 24hours after delivery up to the end of the puerperium. Even if the bleeding does notexceed this specified volume, it warrants investigation, because the underlying causecould be serious.

A. A woman may need suppression of her lactation for various reasons. One of themethods used for this purpose is oral or parenteral administration of estrogen.Estrogen withdrawal bleeding follows. Reassurance is adequate for such cases,provided no other abnormalities are found on examination.

B. A cervical swab is obtained for microbiologic studies. A general, speculum andbimanual pelvic examination is done to assess features of puerperal sepsis (fever,foul vaginal discharge, vaginal warmth, pelvic tenderness, and subinvolution of theuterus). Subinvolution is found with retained products of conception and puerperalsepsis. It is diagnosed when the uterine size is more than expected. The fundal heightabove the top of the pubic symphysis is 15, 10, and 5 cm on days 0, 6, and 12 after

delivery. At the end of 2 weeks, it is at the level of the top of pubic symphysis. After2 weeks it is entirely in the pelvis. It is of normal size or less than normal size at theend of puerperium (the latter due to superinvolution owing to lactation). Uterotonicagents (see chapter 152) are administered to achieve a powerful contraction of theuterus and reduce the blood loss by constricting uteroplacental vessels passingbetween the oblique fibers of the myometrium. If there is puerperal sepsis, broadspectrum antibiotics are administered and a hemogram is obtained. If the bleedingdoes not get controlled, the antibiotics are changed based on antibiotic sensitivityreport. If the bleeding still continues after 48 hours, further action is taken asdescribed under ‘D’.

C. If there are no features of puerperal sepsis, abdominopelvic ultrasonography isdone. If the uterus is found to have retained products of conception, those areremoved by dilatation and curettage. The material removed is studied byhistopathology, so that a choriocarcinoma is not missed (see chapter 47).

D. If the bleeding cannot be controlled by the measures described, uterine arterialembolization is done by interventional radiology. If facilities are not available or itcannot be done due to any reason, anterior division of the internal iliac artery isligated bilaterally. It usually controls the bleeding. If it fails, a total abdominalhysterectomy is done.

309

Page 316: Differential Diagnosis and Management Options inuploads.worldlibrary.org/uploads/pdf/20180107071525differential... · 83 Peripartum cardiomyopathy 165 84 Hypertension 167 85 Convulsions

LATE POSTPARTUM HEMORRHAGE

Check history of estrogen administration (A)

Present Absent

Estrogen withdrawal bleeding Cervical swab for microbiologic studies (B)Speculum and bimanual pelvic examination

Puerperal sepsis No features of puerperal sepsis

Hemogram, abdominopelvic ultrasonographyBroad spectrum antibiotics, blood transfusion as required

Uterotonic agentsAbdominopelvic ultrasonography (C)

Bleeding is controlled Bleeding continues Retained products ofconception

Empty uterus

Change antibiotics based on microbiologic study report Dilatation and curettage Bleeding continues Bleeding is controlled

Bleeding is controlled Bleeding continues Choriocarcinoma

(See chapter 47)

Retained productsof conception

Bleeding continues Bleeding is controlled

Uterine artery embolization/ligation of anterior division of internal iliac artery (D)

Bleeding continues Bleeding is controlled

Total abdominal hysterectomy

310

Page 317: Differential Diagnosis and Management Options inuploads.worldlibrary.org/uploads/pdf/20180107071525differential... · 83 Peripartum cardiomyopathy 165 84 Hypertension 167 85 Convulsions

155. Suspicion of CoagulopathyA woman may be a known case of a coagulation disorder which is

genetically inherited. She may give a history of severe bleeding from cuts andwounds, and menorrhagia. If her condition has not been diagnosed, her bleeding time(BT) is checked.

A. If the bleeding time is abnormal, the platelet count is done. A low count meansthrombocytopenia. It may be autoimmune thrombocytopenic purpura (AITP) or anacquired condition like drug induced thrombocytopenia, thromboticthrombocytopenic purpura, aplastic anemia, folate deficiency, pregnancy inducedhypertension, and allergic disorder. An acquired disorder is treated appropriately. Incase of AITP, free and platelet-bound antiplatelet antibodies can be detected inmaternal circulation. If the maternal platelet count is 20X109/L or more, it ismonitored periodically and prenatal care is continued as usual. If it is below 20X109

or falls below that level, prednisone therapy is given. If the count remains below 50X109/L, monometric polyvalent human IgG is given as a transfusion, in a dose of 0.4mg/kg IV qd X 5d, starting 10-14 days before the due date. In all cases, fetal scalpblood platelet count is done in early labor. A vaginal delivery is allowed if the countis at or above 50 X 109/L, and a cesarean section is done if it is below that level toavoid fetal intracranial hemorrhage. Platelets are kept ready if a vaginal delivery isplanned.

B. If BT is abnormal and platelet count is normal, prothrombin time (PT) andpartial thromboplastin time (PTT) are tested. If both are normal, the diagnosisis platelet dysfunction. It requires platelet transfusions in labor.

C. If PT is normal and PTT is abnormal, it is von Willebrand's disease. Such awoman's factor VIII and platelet count are assayed periodically. She is given factorVIII prophylactically prior to labor so that a vaginal delivery can be allowed.

D. If PT and PTT are both abnormal, plasma fibrinogen is assayed. It is absent inafibrinogenemia. Such a woman needs fibrinogen before labor. It is low withexcessive fibrinolysis or disseminated intravascular coagulation (DIC). The former isvery rare in obstetrics. If fibrinogen is in the normal range, fibrinogen-fibrin

degradation product (FDP) level is checked. It is high in DIC, and normal withexcess of heparin therapy.

In obstetrics, disseminated intravascular coagulation (DIC) can develop due to anyof the following conditions.

1. Pregnancy induced hypertension.2. Abruptio placentae.3. Retained dead fetus syndrome.4. Septic abortion, chorioamnionitis, septicemia.5. Amniotic fluid embolism.6. Use of hypertonic saline for second trimester medical termination of pregnancy.7. Placenta previa.

The diagnosis is made by a coagulation profile, which shows prolongation ofbleeding time, clotting time, partial thromboplastin time, fibrinogen-fibrin degradationproducts, and lowering of platelet count and plasma fibrinogen level. If the patient isbleeding actively (bleeding from skin punctures, gums, incisions, hematemesis, andhematuria), the deficient factors are replaced with transfusion of fresh whole blood,fresh frozen plasma, cryoprecipitate, and platelets. Delivery is achieved when DIC iscontrolled. If she needs an operation, fresh frozen plasma, cryoprecipitate, andplatelets are given as required just before the operation.

E. If BT is normal, PT is normal, and PTT is abnormal, the woman has a deficiencyof factor VIII (hemophilia A carrier), factor IX, or factor XI. In case of the formertwo, the pregnancy is terminated if the fetus is male and affected, and a prophylactictransfusion of deficient factor is given before labor if the fetus is female or unaffectedmale. A vaginal delivery is then allowed.

F. If PT and PTT are normal, no treatment is required.

G. If PT is abnormal but PTT is normal, factor VII is assayed. If it is deficient,fresh frozen plasma or factor concentrate is given as required and before labor.If it is normal, the cause is malnutrition, liver disease, or warfarin effect. Theformer two are treated appropriately.

311

Page 318: Differential Diagnosis and Management Options inuploads.worldlibrary.org/uploads/pdf/20180107071525differential... · 83 Peripartum cardiomyopathy 165 84 Hypertension 167 85 Convulsions

SUSPICION OF COAGULOPATHY

Check bleeding time (A)

Abnormal Normal

Check platelet count Check PT,PTT

Low Normal PT normal,PTT abnormal (E)

PT, PTT normal (F) PT abnormal,PTT normal (G)

Thrombocytopenia Check PT, PTT Factor VIII, IX, XI assay Factor VII assay

Assess cause Normal (B) PT normal, PTTAbnormal (C)

PT, PTT abnormal(D)

Factor VIIIdeficiency

Factor IX deficiency Abnormal Normal

ATP Other Plateletdysfunction

von Willebrand’sdisease

Plasma fibrinogenassay

Hemophilia Acarrier

Factor VII deficiency

See chapter156

Appropriatetreatment Absent Low Normal Fetal sex determination

Assess medically

Afibrinoge-nemia

Excessivefibrinolysis

FDP assay Female Male Malnutrition

High NormalFetal plasma factor

VIII assayLiver disease

DIC Excess heparin Fetus unaffected Fetus affected Warfarin effect

Prophylactictransfusion of factor

VII.IXVaginal delivery

Medical terminationof pregnancy

312

Page 319: Differential Diagnosis and Management Options inuploads.worldlibrary.org/uploads/pdf/20180107071525differential... · 83 Peripartum cardiomyopathy 165 84 Hypertension 167 85 Convulsions

156. ThrombocytopeniaThrombocytopenia is the commonest platelet abnormality encountered in

obstetric practice. It is also the second most frequent hematological disorder, anemiabeing the commonest. It may be asymptomatic, being detected only onhematological testing. Or it may present as a coagulopathy (see chapter 155).

A. HELLP syndrome is a complication of pregnancy induced hypertension. Agravida in the third trimester presents with hypertension, proteinuria, with/withoutedema. She has features of hemolytic anemia, elevation of liver transaminases, andlow platelet count. Severe cases are best managed by immediate cesarean sectionafter correction of platelet count by transfusion of platelets. Milder cases can betreated more conservatively. Administration of dexamethasone results in someimprovement. Postpartum hemorrhage may occur. It is treated with platelettransfusion and replacement of blood as required. The condition improves afterchildbirth.

B. The gravida may present with anemia alone. On investigation she is found to havethrombocytopenia as well. This occurs in megaloblastic anemia (see chapter 76). Theplatelet count improves on treatment with folic acid in case of megaloblastic anemia.Aplastic anemia maybe chronic, or may develop during the pregnancy. The latter canbe recurrent in successive pregnancies. Pregnancy tends to aggravate both types.Thrombocytopenia can be profound. Treatment is as discussed in chapter 76. Platelettransfusions are required to prevent and or treat postpartum hemorrhage. If thegravida has anemia of any other type, the thrombocytopenia is not related to it, andmust be evaluated further.

C. Toxic depression of the bone marrow megakaryocytes may occur due to infection,drugs or alcoholism.

D Diagnosis of thrombotic thrombocytopenic purpura is made clinically in presence

of fever, hemolytic anemia, purpura, neurological symptoms, and renalabnormalities. A mature fetus is delivered immediately so that the gravida can betreated vigorously. But delivery usually does not affect the course of the disease.Treatment is administration of fresh frozen plasma and an antiplatelet agent likedipyridamole or low-dose aspirin to reduce platelet aggregation. If this treatmentfails, other treatment options are infusion of prostacyclin, plasmapheresis, steroids,immunosuppressants etc. The value of these measures is not yet proved.

E. Presence of IgG platelet bound or free maternal antiplatelet antibodies isdiagnostic of thrombocytopenia due to immune destruction of the platelets. This maybe the first manifestation of a more generalized autoimmune disease. DirectCoomb’s test, antiDNA antibody test, antinuclear factor test are performed. Ifpositive, the diagnosis is systemic lupus erythematosus (see chapter 93).Asymptomatic patients are treated with aspirin 75 mg a day orally, stopping oneweek before due date. If the gravida has established disease and is already onmedication it is continued such as paracetamol, aspirin, steroids, and azathioprine.Plasmapheresis may be used as required. Deep vein thrombosis can occur and ismanaged as discussed in chapter 91. If the tests for SLE are negative the patient hasautoimmune thrombocytopenic purpura. Maternal platelet count is checked everyweek. If it remains above 20X109/l, no further treatment is given. If it falls below20X109/l, prednisone therapy is given. It is a lowered to the lowest dose thatmaintains the count at or above 20X109/l. If at 37-38 weeks the count is below50X109/l , monomeric polyvalent IgG IV infusion is given in a dose of 0.4 mg/ kg/dfor five days. Platelet count begins to rise on day four and peaks five days later. Itlasts for three weeks, thereby covering the time of delivery.

In all cases of thrombocytopenia, fetal scalp blood platelets count is done inlabor. A vaginal delivery is allowed if it is 50X109/l or more. If it is less than that,cesarean section is done to avoid fetal intracranial hemorrhage. Platelets are keptready, should maternal hemorrhage take place.

313

Page 320: Differential Diagnosis and Management Options inuploads.worldlibrary.org/uploads/pdf/20180107071525differential... · 83 Peripartum cardiomyopathy 165 84 Hypertension 167 85 Convulsions

THROMBOCYTOPENIA

Check clinical features

Hypertension, edema, with/without proteinuria (A)

Anemia (B) Bone marrow depressing factor (C) Fever, anemia, purpura, neurologicalsymptoms, renal abnormalities (D)

AsymptomaticAccidental diagnosis

(E)

HELLP syndrome Determine cause Infection Drugs Thromboticthrombocytopenic

purpura

Check for the presence ofIgG platelet-bound or free

antiplatelet antibodies

Megaloblasticanemia

Aplasticanemia

Other Alcoholism None Assess fetal maturity Present Absent

Mature Immature Supportive treatment

Induction oflabor

Mature fetallungs with

betamethasone

Direct Coomb’s testAntiDNA antibodyAntinuclear factor

Check platelet count Negative Positive

Prednisone therapy <20X109/L >20X109/L Monitor count ATP SLE

Failure Remission Exacerbation Stable

Check count Fetal scalp blood platelet count in labor

< 50X109/L > 50X109/L < 50X109/L

Cesarean section

> 50X109/L

Keep platelets ready, vaginal deliveryIgG therapy

314

Page 321: Differential Diagnosis and Management Options inuploads.worldlibrary.org/uploads/pdf/20180107071525differential... · 83 Peripartum cardiomyopathy 165 84 Hypertension 167 85 Convulsions

157. Sudden Collapse inPregnancy

A. A gravida may suddenly collapse. The causes are different in the antepartum,intrapartum and postpartum periods. If the vital parameters are normal in the firsttwo trimesters and she has no pallor, the most likely diagnosis is a fainting spellcommonly encountered in the first trimester. It is relieved in lying down position dueto improvement in the blood supply of the brain. It is usually self-limiting and doesnot occur after the first trimester. If the woman is in shock (rapid thready pulse, cold-clammy extremities, and hypotension), and has severe pallor, vaginal bleeding islooked for. If it is present, she is likely to have an inevitable abortion (see chapter69). She may have a septic abortion and endotoxic shock too (see chapter 70). Severevaginal hemorrhage and shock in the second, and less commonly in the first trimesteris due to a vesicular mole. If she does not have any vaginal bleeding, butintraperitoneal hemorrhage (abdominal tenderness and distension, shifting dullness,and pallor) the diagnosis is either ruptured ectopic pregnancy (see chapter 71) orruptured corpus luteum hematoma in the first trimester, and rupture of a gravidrudimentary horn of a bicornuate uterus in the second trimester.

B. Excessive vomiting in the first trimester may cause hypotension, dehydration,electrolyte imbalance, and ketoacidosis.

C. In the third trimester, the gravida may have hypovolemic shock associated withvaginal bleeding (placenta previa) or concealed intrauterine bleeding (abruptionplacentae). If there is no bleeding and there is severe hypertension with proteinuria,the diagnosis is preeclampsia. There may be additional convulsions (eclampsia) orneurological signs (hypertensive encephalopathy or intracranial hemorrhage).

D. If the woman has been given an anesthesia for painless labor or an operation, anestheticcauses are looked for, such as pulmonary embolism (see later), acute tension pneumothorax(complicating positive pressure ventilation, leading to mediastinal compression andsubsequent cardiovascular compromise), residual neuromuscular blockade, patient-controlled analgesia (an overdose of pethidine causing apnea, sedation, and hypoxia),acute pulmonary edema (undiagnosed valvular heart disease), cerebrovascularaccident (a ruptured berry aneurysm due to the stress of the operation and anesthesia,particularly during intubation and extubation causing headache, vomiting, and apnea),severe hypotension with a regional block (in hypovolemic patients and supinehypotension syndrome), aspiration pneumonitis (see later), anaphylaxis oranaphylactoid reaction (see later).

E. Intrapartum and postpartum hemorrhage may be severe enough to cause

hypovolemic shock (see chapters 152and 153).

F. A woman in labor or soon postpartum may be hypoglycemic due to poor oralintake, vomiting, and associated illnesses like hepatitis, diabetes mellitus undertreatment. The patient has sweating, gooseflesh, tachycardia, confusion, fatigue,seizure, palpitations, tremor, and anxiety. The diagnosis is confirmed by a rapidrandom blood sugar level estimation. If that is not possible, a therapeutic trial byadministration of 50% dextrose IV is justified. Treatment is initial slow injection of50% dextrose (0.5 g/kg) followed by an infusion of 10% dextrose (0.10 g/kg perhour). The blood glucose level should be checked regularly thereafter.G. Sudden collapse may be due to use of different drugs administered to the gravida,such as anesthetic drugs (see under ‘D’), tocolytic therapy (cardiac dysrhythmias,myocardial infarction and congestive heart failure), accidental bolus doses ofmagnesium or vancomycin (hypotension and cardiac arrest), ergot derivatives(myocardial infarction and cardiac arrest), and anaphylactic reaction (see later).

H. Sudden collapse associated with respiratory distress may be due to a large numberof conditions like anaphylactic shock (history of drug administration followed byskin rash, bronchospasm, and cardiovascular collapse), amniotic fluid embolism(dyspnea, cyanosis, hypotension, seizure, DIC, cardiac arrest), pulmonarythromboembolism (chest and back pain, dyspnea, hemoptysis, syncope, wheeze – seechapter 92), air embolism (gasping, chest pain, cyanosis, high CVP, hypotension,cardiac arrest), or aspiration pneumonitis (respiratory distress, hypoxia, pulmonaryedema, and bronchospasm).

I. Acute inversion of the uterus causes sudden postpartum collapse. The patient goes inshock which is out of proportion to the amount of blood lost vaginally. If the placenta isstill attached to the uterus, there may not be any vaginal bleeding. See chapter 153 fordiagnosis and management of acute inversion of the uterus.

J. If a woman has received glucocorticoid therapy for prolonged periods, she is likely tohave chronic suppression of the adrenal cortex. Such patients stand the stress of labor,an operation or sepsis rather poorly, and may develop acute adrenocorticalinsufficiency. The symptoms are intense nausea, vomiting, and abdominal pain,lethargy, somnolence, and hypovolemic collapse. The condition is best prevented byadministration of hydrocortisone to all women in labor, undergoing an operation, orwith sepsis if they have received prolonged glucocorticoid therapy in the precedingsix months.

K. Women with conditions like septic abortion, chorioamnionitis, puerperal sepsis maydevelop septic shock. In hyperdynamic stage, the patient has tachycardia, reducedsystemic vascular resistance, and warm extremities. As sepsis progresses, the patientshows severe tachypnea, fever, diaphoresis, hypothermia, cool, mottled and oftencyanotic extremities, oliguria, renal failure.

315

Page 322: Differential Diagnosis and Management Options inuploads.worldlibrary.org/uploads/pdf/20180107071525differential... · 83 Peripartum cardiomyopathy 165 84 Hypertension 167 85 Convulsions

SUDDEN COLLAPSE

Check relation to parturition (A)

Antepartum Intrapartum Postpartum

Check trimester Third (C) Check for relation to anesthesia (D) Present

None

Check typeAppropriate

treatmentFirst and second Check for obstetric hemorrhage

Very high Check blood pressure Low

Check vital parameters Normal, no pallor Placentaprevia

Abruptioplacentae

Check for proteinuria

Present Absent

Normal Check for bleeding

Absent

Present (E)

Appropriatetreatment

Shock Fainting of normal pregnancy None

Check blood pressure

PIHEclampsia

Check for hypoglycemia (F)

Check for use of drugs (G) AbsentPresent

Check for excessive hemorrhage, vomiting Convulsions

Severe PIHCheck for

neurological signsPresent Absent

Appropriatetreatment

Vaginalbleeding

Intraabdominalbleeding

No bleeding,excessive

vomiting (B)Eclampsia

Present Absent

Check respiration and other related symptoms (H)

AbortionVesicular mole

Checktrimester Hyperemesis

gravidarumHypertensive

encephalopathy CVA

Dyspnea, cyanosis,hypotension, seizure,DIC, cardiac arrest.

Chest and back pain,dyspnea, hemoptysis,

syncope, wheeze.

Gasping, chest pain,cyanosis, high CVP,hypotension, arrest.

First SecondAmniotic fluid Pulmonary thrombo- Air embolism

Ectopic pregnancyRupture of rudimentaryhorn of bicornuate uterus

History of drug administration. Skin rash,bronchospasm, and cardiovascular collapse

embolism embolism

Respiratory distress, hypoxia, andRuptured corpus luteum hematoma

Anaphylactic shock Normal pulmonary edema, bronchospasm

Check miscellaneous conditions Aspiration pneumonitis

(I)Sudden collapse after childbirth,hypotension out of proportion tovaginal bleeding, placenta

(J)History of chronic steroid administration. Precipitated bysepsis or surgical stress. Intense nausea, vomiting, andabdominal pain, lethargy, somnolence, hypovolemic collapse.

(K)Temperature >380 C or <360 C, respiration >20/min, WBC 12000 or <4000/mm3 or with >10% immature (band) forms.

attached to inverted fundus ofuterus

Uterine inversion Acute adrenal insufficiency Septic shock

316

Page 323: Differential Diagnosis and Management Options inuploads.worldlibrary.org/uploads/pdf/20180107071525differential... · 83 Peripartum cardiomyopathy 165 84 Hypertension 167 85 Convulsions

158. Puerperal PyrexiaPuerperal pyrexia is an elevation of body temperature to 38° C or more on

any two of the first 10 days postpartum, exclusive of the first 24 hours.

A. A woman has a tendency for urinary retention in early puerperium due to loss ofdetrusor tone, lack of elasticity of the bladder, edematous swelling of the vulva andurethra, and reflex spasm of the urethral muscle due to pain in perineal sutures. Thewoman may need catheterization for such retention, or may have been catheterizedduring labor or before application of forceps, cesarean section etc. Bacteria are thusintroduced into the urinary tract and can cause acute cystitis and urethritis, which cancause puerperal pyrexia with chills and dysuria. Urinalysis confirms the diagnosis.Microbiological studies are done to determine antibiotic sensitivity and thenappropriate antibiotic therapy is given.

B. The renal pelvis and ureter are dilated during pregnancy, due to the smoothmuscle relaxant action of progesterone, and pressure of the gravid uterus. Thechanges are more marked on the right side. These changes take up to 8 weeks toregress completely. Cystitis may progress to acute pyelonephritis due to ascendinginfection. The woman has fever with chills and renal angle tenderness. Appropriateantibiotic therapy is given, based on microbiologic studies on urine.

C. The breasts can be the seat of inflammation due to problems arising fromlactation. Such a woman complains of pain in the breast(s). Examination shows anyof the following.

1. Engorgement: both the breasts are enlarged, warm, firm, tender, nodular, andhave visible veins on the surface. Breast engorgement occurs due to faultytechnique of lactation or an inability of the neonate to suckle. Such a womanneeds lactation advice and correction of any faults in her technique of feedingthe baby. Any milk left in the breasts after feeding the baby should beexpressed out manually. If she is unable to do so, a breast pump can be usedconveniently. Administration of oxytocin (10 units IM) 10 minutes beforeemptying the breasts with hands or a pump is useful.

2. Mastitis: the concerned breast or part of a breast is inflamed. It is red, hot, andtender. There is no fluctuation. A cracked and infected nipple is oftenresponsible for it. The treatment is an antibiotic, an NSAID, and local hotfomentation. Lactation is continued on the other side.

3. Breast abscess: there is acute pain in one breast. Examination reveals a tender,hot lump in one breast. Fluctuation test is positive. The treatment is aspirationof the pus, preferably under ultrasonographic control. If that is not possible,incision and drainage is done, followed by administration of a broad spectrum

antibiotic, and an NSAID. Microbiological studies are done on the pus drainedto determine antibiotic sensitivity and then antibiotic therapy is changed basedon the report. Lactation is continued on the other breast.

D. If the woman has foul smelling lochia, lower abdominal and pelvic pain, puerperalsepsis is suspected. Examination reveals vaginal warmth, uterine tenderness, pelvictenderness, and in some cases, a tender adnexal mass on one or both sides (tubo-ovarian mass). An ultrasonography helps in making a diagnosis of complications, ifany.

1. Infected episiotomy: the suture line is tender, red, and pus may be seenescaping from the punctures of the sutures. The sutures are removed to permitfree drainage of the pus, which is sent for microbiologic studies. The wound iscleaned with hydrogen peroxide and saline twice a day and a broad spectrumantiseptic cream is applied to it locally. Perineal wash is given twice a day andevery time the woman passes urine or stools. Appropriate antibiotic is given.When the wound has healthy granulation tissue, secondary suturing is done,putting monolayer vertical mattress sutures of No. 1-0 monofilament nylon.

2. Endometritis-salpingooophoritis-parametritis: the clinical picture is as describedbefore. A cervical swab is sent for microbiologic studies. Appropriate antibioticsand NSAID are used. An exploratory laparotomy is required for the followingindications. Total hysterectomy is usually done.a. Septicemia with or without septic shock not responding to medical treatment.b. Infected leiomyoma.c. Abscess or gangrene of the uterine wall.d. Tubo-ovarian mass expanding despite treatment or rupture of the same.

3. Pelvic peritonitis: besides treatment outlined before, head high position is givento help localize pus to the pelvis, and nasogastric tube drainage is done todecompress the gastrointestinal tract.

4. Pelvic abscess: it causes rectal tenesmus in addition to other features ofpuerperal sepsis. It is drained by posterior colpotomy if close to posteriorvaginal fornix, and extraperitoneally abdominally through a grid-iron incision ifhigh up in the pelvis. Drainage under ultrasonographic control is alsoacceptable. If the abscess fill up again despite antibiotic therapy, it is drained bythe open technique.

5. Pelvic thrombophlebitis: the diagnosis is confirmed by color Doppler studiesand CT or MRI of the pelvis. The treatment is antibiotics and heparin.

E. A respiratory tract infection causes cough, expectoration, and is associated withchest signs of consolidation, cavitation, and/or effusion. That is treated appropriately.Any other medical cause of fever is also treated appropriately, such as malaria,tuberculosis, enteric fever, etc.

317

Page 324: Differential Diagnosis and Management Options inuploads.worldlibrary.org/uploads/pdf/20180107071525differential... · 83 Peripartum cardiomyopathy 165 84 Hypertension 167 85 Convulsions

PUERPERAL PYREXIA

History, clinical examination

Dysuria, bladdertenderness (A)

Renal angletenderness (B)

Pain in breasts (C) Foul smelling lochia,pelvic tenderness, pelvic

mass(es), vulvitis (D)

Other infections(E)

Cystitis Acutepyelonephritis

Breastengorgement

Mastitis Breast abscess Puerperalsepsis

Urinalysis, urine culture, appropriateantibiotic therapy

Lactation advice, manualexpression of breast milk

AntibioticsNSAIDs

Incision and drainageCulture of pus, Antibiotics

Determine type

Infected episiotomy Endometritis, salpingo-oophoritis, parametritis

Pelvicperitonitis

Pelvic abscess Pelvic thrombophlebitis

Remove sutures, perineal wash,local antiseptics, antibiotics,secondary suturing.

Cervical swab cultureAntibiotics, NSAIDs

Head high position, gastricdecompression, antibiotics,exploratory laparotomy ifrequired.

Incision and drainage orultrasound-guided aspiration,culture of pus, antibiotics.

Heparinization, antibiotics

Respiratory infection Malaria Tuberculosis Enteric fever Other infections

Appropriate treatment

318

Page 325: Differential Diagnosis and Management Options inuploads.worldlibrary.org/uploads/pdf/20180107071525differential... · 83 Peripartum cardiomyopathy 165 84 Hypertension 167 85 Convulsions

159. Postpartum MastalgiaA. A lactating woman may complain of pain in one or both the breasts. Theconcerned breast is examined for the cause of the pain. Bilateral pain is due toengorgement of the breasts (See chapter 158). Sometimes there can be unilateralengorgement of the breast due to unilateral problem in the nipple (e.g. retractednipple). Faulty emptying of the breast by a feed causes engorgement. Use of aWoolwich shield during feeding and manual expression of the milk cause resolutionof the problem.

B. Localized tenderness in a breast can be due to a disease in the nipple, or in thebreast. The nipple may be sore. Soreness is due to faulty position duringbreastfeeding. Correction of the position is more important than local application ofa soothing or nipple-hardening cream.

C. A galactocele is a localized collection of milk in the breast. It is due to a block inthe duct of the lobule. It is usually mildly tender. It should be observed for up to 48

hours while breast feeding is continued. If it resolves spontaneously, no treatment isrequired. If it persists, it is aspirated under local anesthesia with a needle. There is arisk that it may get infected and form a breast abscess. If it does develop, it is treatedas discussed in chapter 158.

D. Mastitis and breast abscess cause unilateral breast pain and tenderness. Theseconditions have been discussed in chapter 158.

E. Acute lactating carcinoma of the breast (mastitis carcinomatosa) accounts for 2%of breast cancers. The affected breast is painful. In 50% cases there is a retraction ofthe nipple. The skin over the breast is red and warm. There is cutaneous edema dueto blockage of subdermal lymphatics with the cancer cells. Axillary lymph nodes areenlarged. This condition may be mistaken for a breast abscess and incised fordrainage. It differs from an abscess in that the edema is more extensive, there is anabsence of fever and leukocytosis, and there is a failure to respond to antibiotics.Fine needle aspiration cytology confirms the diagnosis. A woman with this conditionneeds a surgical referral for management with radical mastectomy and radiotherapy.

319

Page 326: Differential Diagnosis and Management Options inuploads.worldlibrary.org/uploads/pdf/20180107071525differential... · 83 Peripartum cardiomyopathy 165 84 Hypertension 167 85 Convulsions

POSTPARTUM MASTALGIA

Check side

Unilateral Bilateral

Local examination Local examination

Generalized tenderness (A) Localized tenderness (B) Breast engorgement

Breast engorgement Nipple Galactocele (C) Infection (D) Acute lactatingcarcinoma (E)

(See chapter 158)

See chapter 144 Sore Cracked Observation Check type

HygieneLactation support

ExpressionLocal treatmentNipple shield

Mastitis Abscess Appropriatetreatment

Resolves Persists See chapter 158

Needle aspiration

320

Page 327: Differential Diagnosis and Management Options inuploads.worldlibrary.org/uploads/pdf/20180107071525differential... · 83 Peripartum cardiomyopathy 165 84 Hypertension 167 85 Convulsions

160. Cervical IncompetenceCervical incompetence is a syndrome characterized by a failure of the

cervix to remain closed beyond the first trimester, such that the pregnancy ends as asecond trimester abortion, or an early preterm delivery.

A. It may be caused by any of the following causes.

1. Gynecologic cervical trauma: rapid dilatation (beyond 9 mm), amputation,conization, and Fothergill's operation.

2. Obstetric cervical trauma: traumatic forceps or vacuum delivery, precipitatelabor, and bucket-handle tear of the cervix (with the use of prostaglandins forsecond trimester MTP or failure to remove cervical cerclage in presence ofuncontrollable uterine contractions).

3. Associated with congenital malformations of the uterus: bicornuate or septateuterus, T-shaped uterine cavity (with intrauterine exposure to DES).

4. Congenital: preponderance of smooth muscle over fibrous tissue in the cervix.

B. Cervical incompetence is characterized by painless dilatation and effacement ofthe cervix, followed by profuse watery discharge vaginally. The membranes ruptureearly. Then there is a short period of painful uterine contractions ending withexpulsion of the fetus. In successive pregnancies, fetal wastage occurs either at thesame gestational age, or earlier, but never at a later gestational age. Earlier loss isdue to further damage to the cervix during that pregnancy loss. If the history isclassical, it is preferable to perform a cerclage electively in the next pregnancy,before the cervix starts dilating and effacing. This is because the results of electivecerclage are better than those of emergency cerclage.

C. If the cervix is normal or minimally torn, either Shirodkar's cerclage orMacDonald's cerclage can be performed. MacDonald's operation is technicallyeasier, faster, and associated with less morbidity than Shirodkar's operation. Shirodkar'soperation achieves tightening at the level of the internal os, while MacDonald'soperation achieves tightening at the junction of vaginal and supravaginal cervix.Thus the former should achieve better results than the latter. However the results ofboth the operations are similar.

D. If the cervical dilatation and effacement are minimal, either Shirodkar's orMacDonald's cerclage can be performed. If the cervix is 1 to 3 cm dilated, MacDonald's

operation is performed when the cervical effacement is less than 40%, and Wurmprocedure is performed when the effacement is 40% or more. With advancedeffacement, it is not possible to perform any operation other than Wurm procedure,in which the external os is almost closed with two horizontal mattress suturesperpendicular to each other. If the vaginal cervix is absent (as with amputation),badly scarred, or torn upwards beyond the vaginal fornix, any form of vaginaloperation is not successful. Benson and Durfee's cerclage places a ligature aroundthe upper end of the cervix by the abdominal route, and is the operation of choice insuch cases.

Contraindications for cervical cerclage are as follows.

1. Threatened abortion or preterm labor.2. Placenta in the lower segment.3. Ruptured membranes.4. Intrauterine fetal death.5. Fetal malformations incompatible with life.6. Intrauterine sepsis.7. Cervical or vaginal sepsis.8. Cervical dilatation > 4 cm, effacement > 50%.9. Gestational age < 14 weeks, > 32 weeks.

Prerequisites for performing cervical cerclage are as follows.

1. Confirmation of the diagnosis of cervical incompetence.2. Ultrasonography of the lower abdomen and pelvis: for localization of the

placenta, demonstration of fetal life, and ruling out congenital malformations.3. Hanging drop and culture of cervical and/or vaginal discharge, if any infection

is suspected.E. The patient is examined every week. If the cerclage is intact, antenatal care iscontinued. If the cerclage has become loose, the need for and feasibility ofrepeating the cerclage is assessed. If it is not feasible, antenatal care is continuedunder close observation. If the cerclage is cutting through the cervix, it is removed.The pregnancy is continued giving conservative treatment as long as possible.Preterm labor is managed as discussed in chapter 101. If it fails to get controlled,the cerclage is removed. Preterm premature rupture of membranes is managed byremoval of the cerclage and treatment as described in chapter 102. The cerclage isremoved at term.

321

Page 328: Differential Diagnosis and Management Options inuploads.worldlibrary.org/uploads/pdf/20180107071525differential... · 83 Peripartum cardiomyopathy 165 84 Hypertension 167 85 Convulsions

Etiological factors (A)

CERVICAL INCOMPETENCE

Assess nature of cerclage requiredHistory (B)

Elective Emergency

Assess cervix for old tears and scarring (C) Assess cervical dilatation (D)

Absent Minimal Forniceal tear Minimal 1-3 cm Any, with fornicealtear

Shirodkar’s or MacDonald’s cerclage Benson and Durfee’scerclage

Shirodkar’s or MacDonald’scerclage

Assess cervicaleffacement

Benson and Durfee’scerclage

< 40% > 40%

MacDonald’s cerclage Wurm procedure

Weekly reassessment (E)

Cerclage intact Cerclage loose Cerclage cuttingthrough cervix

Preterm labor Premature rupture ofmembranes

Term

Continuereassessment

Check feasibility ofrepeat cerclage

Remove cerclage,conservative

treatment

Tocolysis,betamethasone

Remove cerclage

See chapter 102

Remove cerclage

Follow-up Successful control Failure

Remove cerclage

322

Page 329: Differential Diagnosis and Management Options inuploads.worldlibrary.org/uploads/pdf/20180107071525differential... · 83 Peripartum cardiomyopathy 165 84 Hypertension 167 85 Convulsions

161. Difficult Cesarean SectionDifficulties may be encountered during a cesarean section at different

stages.

A. Access is usually easier and faster with a vertical midline incision. If theaccess is inadequate, the incision can easily be extended upwards, if necessaryby going around the umbilicus. Such a situation is encountered when there isfibrosis in the abdominal wall due to previous exploratory laparotomy, especiallywith a Pfannenstiel incision. If the access is found to be limited with a Pfannenstielincision, there is no scope for extension of the incision. In that case the rectihave to be divided. This nullifies the advantage of making a Pfannenstiel incision,even if the recti are sutured back after the operation.

B. If the woman has had a cesarean section(s) in past, the urinary bladder islikely to be adherent to the front of the lower segment, especially over the scar.In such cases, the bladder is separated from the lower segment by sharp dissection.Blunt dissection is likely to traumatize the bladder. If there has been no suchoperation in the past, a combination of sharp and blunt dissections can be usedto separate the bladder from the lower segment.

C. If the lower segment is well formed, a transverse incision is made in it.Usually it provides adequate access for delivery of the baby. If the access is inadequate,the lateral ends of the incision are extended further laterally, curving the endsupwards along the lateral wall of the uterus. An inverted-T shaped incision isnot recommended, because it extends into the upper segment, and has all thedisadvantages of an upper segment operation. If the lower segment is poorlyformed, as in cases of prematurity, transverse lie, and placenta previa, a transverseincision long enough to permit delivery of the fetus would almost separate thepart of the uterus above the incision from that below it. In such cases, a verticalincision is made in the lower segment. If it is found to be too small, it is extended

upwards.

D. Various difficulties may be encountered in delivery of the presenting part.

1. Deep engagement of the head: an assistant may be asked to push up the head bypassing a hand into the vagina. This may cause extension of the uterineincision. An alternative technique is modified Patwardhan's maneuver, in whichthe upper limbs are delivered first (keeping the back anterior), followed by thethorax, abdomen, lower limbs, and the head in that order.

2. High floating head: fundal pressure through maternal abdominal wall usually isadequate. Alternatively obstetric forceps (Hale) may be used to deliver thehead.

3. Transverse lie: Patwardhan's maneuver is the preferred method. A lower limb isdelivered first followed by the other lower limb, keeping the back anterior. Thenthe fetus is delivered like a breech presenting fetus. An alternative is modifiedPatwardhan's maneuver (discussed before).

4. Breech presentation: it is delivered as in breech extraction.5. Face presentation: the chin is brought forward, and then the head is delivered by

flexion.6. Brow presentation: the head is rotated into the anteroposterior diameter, and is

then delivered by extension.7. Placenta previa: the placenta may be encountered in the lower segment incision,

if it is anterior. It is reflected downwards to expose membranes above it. Thenthey are ruptured and the fetus is delivered. If it is not possible to do so, theplacenta is cut across and the fetus is delivered through it. If that is not possibletoo, the placenta is delivered first, and then the fetus. In the second and thirdoptions, the delivery of the fetus has to be quick and the cord has to be clampedfast so as to reduce fetal blood loss in the former and start neonatal resuscitationearly in the latter option.

8. Multiple pregnancy: one fetus is delivered at a time, using the method appropriatefor its presentation as described before.

323

Page 330: Differential Diagnosis and Management Options inuploads.worldlibrary.org/uploads/pdf/20180107071525differential... · 83 Peripartum cardiomyopathy 165 84 Hypertension 167 85 Convulsions

DIFFICULT CESAREAN SECTION

Make abdominal incision, assess degree of access (A)

Inadequate Adequate

Vertical incision Transverse incision Divide uterovesical fold of peritoneum and dissect urinary bladderaway from lower segment (B)

Extend incision upwards Split recti transversely and repair afterwards No previous cesarean section Previous cesarean section

Sharp/blunt dissection Sharp dissection

Assess width of the lower segment (C)

Adequate Inadequate (poorly formed)

Lower segment transverse incision Lower segment vertical incision

Adequate access Inadequate access Inadequate access Adequate access

Extend incision obliquely upwards in broad ligament area Extend incision upwards

Delivery of the presenting part (D)

Deeply engaged head High floating head Breech Face Brow Placenta previa Multiple pregnancy

Push up vaginally ormodified Patwardhan

maneuver

Fundal pressure,obstetric forceps

Breech extraction Deliver head byflexion

Deliver head byextension

Reflect placenta downward, cutthrough placenta or deliver

placenta first

One fetus at atime

324

Page 331: Differential Diagnosis and Management Options inuploads.worldlibrary.org/uploads/pdf/20180107071525differential... · 83 Peripartum cardiomyopathy 165 84 Hypertension 167 85 Convulsions

162. Desire for ContraceptionA couple may desire contraception for various reasons, such as to delay the

first pregnancy after marriage, space successive pregnancies, avoid any pregnancyin future, avoid pre- or extramarital pregnancy, and prevent transmission of agenetic or other disease to the offspring.A. A large number of contraceptive methods are available. A cafeteria approach isadopted in selection of the method of contraception. Merits and demerits of all themethods are explained to the couple, including their failure rates, and then the choiceis made by the couple. If the couple desires temporary contraception, prior to anycounseling the following information is obtained - menstrual history, obstetrichistory, gynecologic history, contraceptive history, medical history and findings ofgynecologic examination. Any contraindications and risk factors for the differentmethods of contraception are looked for. If any such factors are present, counselingis done against the use of those methods of contraception, and safer alternatives aresuggested. If there are no such contraindications or risk factors, and if the couple isnot willing to use any contraceptive, the rhythm method of contraception isrecommended. If the male partner wishes to use a contraceptive, a condom isadvised. If the female partner wishes to use a contraceptive, the frequency of coitusis checked.B. If they have frequent coitus and the woman is willing to handle her genitalia, afemale condom, diaphragm (with an intravaginal spermicide), or vaginal ring isrecommended. If she is unwilling to handle her genitalia and is nulliparous,combination contraceptive pills, centchroman, injectables (combination orprogesterone-only), or implants can be used. An intrauterine contraceptive device(IUCD) is not suitable for a nulliparous woman. If she is parous, one has to check ifshe is lactating or not.C. A lactating woman is advised to use minipill, IUCD, centchroman, injectableprogestins, or implants. Contraceptives containing estrogen are not used becauseestrogen suppresses lactation. If she is not lactating, she can use combination pills,IUCD, centchroman, injectables, or implants. She can use a minipill too, but it is notas good because it has to be taken continuously and can cause menstrualirregularities. The injectables and implants can cause menstrual irregularities andamenorrhea, but have to be used if other forms of contraceptives are not suitable oracceptable to the woman. If she has menorrhagia with an IUCD or even beforeinsertion of an IUCD, she can use a progestin releasing IUCD like progestasert orlevonorgestrel releasing IUCD, which controls the blood loss and givescontraception too.D. If the woman is exposed to coitus without adequate contraception, as with casual,unplanned sex or rape, or after rupture of a condom during sex, she is at risk ofdeveloping an unwanted pregnancy, especially if the coitus takes place at midcycle.In such cases postcoital contraception is required. Hormonal methods are useful inall women. Though estrogen alone or progestin alone has been used for this purpose,the simplest, most economical, and still quite effective method is to administer orallytwo tablets of a combination contraceptive pill (each containing 0.05 mg of ethinylestradiol) as soon as possible after the coitus, and two more 12 hours later. To beeffective, the drug must be given not later than 72 hours of the coitus. An alternative

is to use levonorgestrel 750 μg PO, to be repeated after 12 hours. A recent advancein emergency contraception is to give ulipristal acetate 30 mg PO within 120 hoursafter unprotected sex. If this contraception fails and the woman gets pregnant, sheshould undergo a medical termination of pregnancy. An alternative to hormones isinsertion of a Cu-T within 5 days of the coitus. It has an additional advantage in thatit gives continued protection from a pregnancy for 3 more years.E. If the couple desires a permanent method of contraception, the choice is betweenvasectomy for the male and tubal ligation for the female partner. Male sterilizationby vasectomy is far superior to female sterilization. It is a minor procedure easilyperformed under local anesthesia. It does not involve entry into the peritoneal cavitylike the female sterilization by tubal ligation does, and is thus free of all the relatedcomplications. It has few, if any, significant complications. Its reversal also does notinvolve a laparotomy like reversal of a female sterilization does, and is thus easierand associated with fewer complications. If the woman opts to undergo sterilizationand is menstruating, the postmenstrual phase is the time at which a tubal ligation isdone, so that there is no risk of a pregnancy being present at the time of theprocedure. The uterus is of normal size and is a pelvic organ. Laparoscopicsterilization is the procedure of choice for such a woman. If the fallopian tubes arenormal, silastic bands are applied to them. But if they are edematous, thick, and/orfixed by adhesions, silastic bands cannot be applied without the risk of partialocclusion of the tube or transection. A spring-loaded clip is applied to such a tube. Ifit is not available, electrocauterization of the tube (bipolar, unipolar, or if facilitiespermit Semm's endocoagulation) or a minilaparotomy and tubal ligation by anyopen technique is performed. In early puerperium, the uterus is palpableabdominally. The tubes are easily accessible through a small abdominal incision 2cm below the level of the uterine fundus, made under local anesthesia. It must beremembered that when tubal ligation is done in puerperium or with a cesareansection, the failure rates are higher as compared to those with an interval procedure.Hence a method must be used which is known not to fail e.g. modified Uchida'smethod, Parulekar's method, Irving's method, and Oxford method. The latter twomethods need greater exposure than the first two methods, and can be done with acesarean section but not through a minilaparotomy incision. In late puerperium, ifthe uterus has fully involuted, a laparoscopy may be performed if notcontraindicated. Otherwise a minilaparotomy is done, through a suprapubic, small,transverse incision. A woman after a second trimester abortion (spontaneous orinduced) has a large uterus and is sterilized as during puerperium, through a smallincision placed 2 cm below the level of the uterine fundus. After a first trimesterabortion (spontaneous or induced) the uterus is smaller, and a laparoscopy orminilaparotomy can be performed conveniently. It must not be done before amedical termination of pregnancy (MTP) is complete, because the uterus is largeand soft, so that it can be injured easily during manipulations with an intrauterineinstrument. The tubes are also less readily accessible owing to the large size of theuterus. A woman may desire a sterilization procedure along with another operation.If the abdomen is opened (e.g. for myomectomy, oophorectomy, removal of anectopic pregnancy, sling procedure for uterine prolapse etc.) an open techniqueis used as discussed before. In case of Fothergill's or modified Fothergill's operationsterilization is done through posterior colpotomy.

325

Page 332: Differential Diagnosis and Management Options inuploads.worldlibrary.org/uploads/pdf/20180107071525differential... · 83 Peripartum cardiomyopathy 165 84 Hypertension 167 85 Convulsions

DESIRE FOR CONTRACEPTION

Check if temporary or permanent (A)

Temporary Permanent (E)

Check willingness to use a contraceptive Check if make or female

No Yes Female Male

Rhythm method Check which partner will use contraceptive Gynecologic assessment Vasectomy

Male Female Postmenstrual Postabortal(first trimester)

Postabortal(second

trimester)

Postpartum

Condom Check frequency of coitus

Laparoscopic sterilization Minilap TL Puerperal TLFrequent (B) Unprotected (D) Postcoital

contraceptionInfrequent

Assess fallopian tubesCheck willingness to handle genitals Female condom,

diaphragm,spermicidals

Yes No Check parity (C)Normal Thick, edematous Fibrotic, rigid, fixed

Female condom, diaphragm,vaginal ring

0 > 1

Silastic bands Spring-loaded clipsCombination pill, centchroman,

injectables, implants.Check if lactating

No Yes

Combination pill, IUCD, centchroman,injectables, implants

Minipill, IUCD, centchroman, injectableprogestins, implants

326

Page 333: Differential Diagnosis and Management Options inuploads.worldlibrary.org/uploads/pdf/20180107071525differential... · 83 Peripartum cardiomyopathy 165 84 Hypertension 167 85 Convulsions

163. Missing Threads of anIUCD

A. Majority of the intrauterine contraceptive devices (IUCD) used in modern timeshave threads, which are cut 1 cm below the external os after insertion. The purposeof the threads is to let the woman feel them herself periodically to confirm that theIUCD is within the uterus, and to let her gynecologist see them on speculumexamination for the same reason. Another purpose is to help removal of the IUCD.All one has to do is to grasp the threads with a hemostat and make traction.

B. If the threads are not felt and seen, the first possibility to be considered is apregnancy, resulting either due to absence of the IUCD in the uterine cavity, ordespite its presence (failure of IUCD). See chapter 62 for methods of diagnosis of apregnancy. See chapter 165 for the management of a pregnancy due to a failure of anIUCD.

C. If the woman is not pregnant, the uterine cavity is sounded. An IUCD may be feltwhen the sound makes a contact with it. In such a case, the threads are drawn up theuterocervical canal. They can be drawn out again by passing in a cytobrush androtating it a few times to entangle the threads before drawing it out. A threadretriever may also be used quite conveniently. Application of suction with a Karmansyringe and cannula is useful in some cases to draw the threads out. The woman canthen follow up as advised before. If that is not possible, the IUCD is removed withan IUCD removing hook, Novak's endometrial biopsy curette, or Shirodkar's hook.A new IUCD is then inserted. If the IUCD does not get removed by these means, anyof the following methods may be used.

1. Hysteroscopy to draw the threads out.

2. Dilatation and curettage.3. Exploration of the uterine cavity with a curved long hemostat.

D. If the IUCD is not felt with a sound, pelvic ultrasonography is done to locate it. Ifit is intrauterine, it is managed as discussed above. If it is seen to be in the peritonealcavity, it is removed laparoscopically. Usually two puncture technique is required, thesecond puncture being used for passage of grasping forceps. Since most if not all thedevices used currently are medicated (copper or progestin releasing IUCDs), whichcause intense intraperitoneal adhesions, all devices lying in the peritoneal cavitymust be removed. If laparoscopic removal is not possible due to adhesions, theIUCD is removed by a laparotomy.

E. If the IUCD is not seen anywhere on ultrasonography, it is possible that it hasbeen expelled from the uterus. However it is also possible that it lies within loops ofbowel, and is not seen because gas within the bowel stops ultrasonic waves. In sucha case, a sound is passed into the uterine cavity and is tied to the vulsellum applied tothe anterior lip of the cervix, so that it does not get displaced during the woman'smovements. Anteroposterior and lateral radiographs of the pelvis are obtained. Iffacilities for ultrasonography are not available, this test can be used instead quiteeffectively. If the IUCD is found to be away from the sound in either or both of theradiographs, it is diagnosed to have passed out of the uterus through a perforation. Ifit is in the peritoneal cavity, it is removed as discussed before. If it lies within thebroad ligament, an exploration is required.

F. Rarely the IUCD passes into the urinary bladder by perforating the uterus andbladder both. It can be diagnosed by ultrasonography and removed by cystoscopy. If ithas been within the bladder for a long time, a urinary calculus may form around it.The woman is then referred to a urologist for removal of the stone by litholapaxythrough a cystoscope, or by suprapubic cystotomy.

327

Page 334: Differential Diagnosis and Management Options inuploads.worldlibrary.org/uploads/pdf/20180107071525differential... · 83 Peripartum cardiomyopathy 165 84 Hypertension 167 85 Convulsions

MISSING THREADS OF AN IUCD

Speculum examination (A)

Threads are not seen Threads are seen

Assess for the presence of a pregnancy (B) Reassurance

Present

See chapter 165

Absent

Sound uterine cavity (C)

IUCD is felt IUCD is not felt

Retrieve threads Pelvic ultrasonography

Possible Not possible Intraperitoneal IUCD (D) IUCD is not seen (E) Intravesical IUCD (F)

Remove IUCD and insert another Radiographic assessment Cystoscopy orsuprapubiccystostomy

Possible Not possible Laparoscopic removal Intraperitoneal IUCD IUCD absent

Removal by hysteroscopy or dilatation and curettage

Possible Not possible Spontaneous expulsion

Removal by laparotomy Insert another IUCD

328

Page 335: Differential Diagnosis and Management Options inuploads.worldlibrary.org/uploads/pdf/20180107071525differential... · 83 Peripartum cardiomyopathy 165 84 Hypertension 167 85 Convulsions

164. Failure of ContraceptionIf a couple is using some from of contraception and the woman misses a

period, the possibility of a pregnancy due to a failure of the contraceptive is the firstconsideration. The diagnosis of the pregnancy is made as discussed in chapter 62.

A. A couple uses contraception because a pregnancy is not wanted. Thus it would belogical to believe that a pregnancy resulting from a failure of contraception is alwaysunwanted. However some couples decide to have the baby. Medical termination ofpregnancy (MTP) is permitted by the Indian MTP Act for a pregnancy resulting froma failure of contraception. If the couple opts for MTP, it is performed, and a moreeffective mode of contraception is offered. Even if the option chosen is to continuethe pregnancy, it may not always be advisable to do so. The options depend upon themethod of contraception that has failed.

B. If the woman is using hormonal contraception when she gets pregnant, the fetusgets exposed to the hormones. Sex steroids are believed to cause adverse effects onthe fetus, e.g. genital lesions with diethyl stilbestrol (cervical adenosis, cervicalcockscomb, vaginal adenosis, T-shaped uterine cavity, stumpy fallopian tubes,cervical or vaginal adenocarcinoma), VACTERL syndrome with progestins(vertebral, anal, cardiac, tracheal, esophageal, renal, and limb malformations),masculinization of a female fetus with exposure to androgenic progestins etc. Thecouple is counseled on this possibility, and that ultrasonography and MRI may not beable to detect these malformations or late effects (like carcinoma). If the coupleaccepts the risk, the pregnancy is continued and antenatal care is given. If MTP isopted for, it is done using an appropriate technique (see chapter 165).

C. Mechanical barrier contraceptives like condom and diaphragm have no adverseeffect on the fetus and the pregnancy is continued safely if so desired.

D. If an intravaginal spermicidal agent has been used during a pregnancy, therisk of exposure of the fetus to the agent after vaginal absorption and passageacross the placenta is explained to the couple. A decision can then be allowedto be made regarding continuation of the pregnancy or MTP.

E. If a pregnancy occurs despite insertion of an IUCD, the first step is to determine ifit is intrauterine or extrauterine (see chapter 71). If the pregnancy is extrauterine isremoved appropriately. The IUCD is removed too and the couple is advised to useanother form of contraception. If the pregnancy is intrauterine, and the threads of theIUCD are seen at the external os, the IUCD is removed by making traction on thethreads and an MTP is done because the copper or progesterone in the IUCD has aharmful effect on the fetus.. If the threads are not seen, an ultrasonography of theabdomen and pelvis is done. If the IUCD is intrauterine, an MTP and removal of theIUCD are done. If the IUCD is not seen on ultrasonography, it might have beenexpelled or might have perforated and be hidden by loops of bowel (the gas within

which stops ultrasonic waves). Hence an abdominopelvic MRI is done. If the IUCDis absent, the pregnancy may be continued if so desired. If the IUCD is extrauterine,it has to be removed by appropriate means (see chapter 163). In such cases it may bebetter to perform an MTP at the same time, because there can be harmful effects ofthe anesthetic agent on the fetus in the first trimester.

F. Effect of centchroman on a fetus has not been studied adequately. It is better tohave an MTP if the woman gets pregnant while using centchroman.

G. Rhythm method of contraception is not associated with any problem for the fetus,and the pregnancy can be continued if the couple so desires. If not, an MTP is done.

H. A pregnancy resulting from a failure of a sterilization procedure can be quitedisturbing to the couple. The man or the woman has already undergone an operationso as to avoid having another baby. Thus the morbidity and the risk of complicationsassociated with that procedure have already been borne by the person who hasundergone the operation, and still the woman has got pregnant. Sometimes thepregnancy is detected too late, because the couple believes that a pregnancy cannotoccur after the sterilization procedure and does not see a doctor about missing amenstrual period. In those cases the pregnancy has to be continued. Rarely thecouple decides to have this baby, either because one of the previous babies has died,or sometimes even when all the previous babies are alive and well. If the pregnancyis less than 20 weeks, an MTP is done, followed by either vasectomy or tubalsterilization by a minilaparotomy. It is preferable to use an open method with zerofailure rate. If the pregnancy has progressed beyond 20 weeks, antenatal care isgiven. A puerperal tubal ligation or vasectomy is carried out after the delivery of thebaby.

Table 164.1 Failure Rates of Sterilization Methods

Method Failure rate (%)

Silastic band 0Spring loaded clip 0.2 - 0.6Tantalum hemoclip 5.0 - 18.0Pomeroy 0 - 0.4Modified Madlener 0.3 - 2.0Uchida 0Parulekar 0Wood 0Irving 0Fimbriectomy 0Salpingectomy 0 - 1.9Cornual resection 2.8 - 3.2Simple ligation 20.0Electrocoagulation 0.1 - 2.0Chemical sclerosis of tube 20 - 30

329

Page 336: Differential Diagnosis and Management Options inuploads.worldlibrary.org/uploads/pdf/20180107071525differential... · 83 Peripartum cardiomyopathy 165 84 Hypertension 167 85 Convulsions

Confirm presence of a pregnancy

FAILURE OF CONTRACEPTION

Assess desire to continue the pregnancy (A)

Yes No

Check which contraceptive was used MTP plus more effective contraception

Hormonal (B) Mechanical barrier (C) Spermicide (D) IUCD (E) Centchroman (F) Rhythm method (G) Sterilization (H)

MTP after counseling Antenatal care MTP Check location ofpregnancy

MTP Antenatal care Antenatal care,sterilize male or

female partner afterdelivery

Intrauterine Extrauterine

Check for presence of threads of IUCD at the external os See chapter 71

Present Absent

Pelvic ultrasonography

Intrauterine IUCD No IUCD Extrauterine IUCD

MTP plus removal of IUCD Pelvic MRI

Antenatal care No IUCD Extrauterine IUCD MTP plus removal of IUCD by appropriatemethod

330

Page 337: Differential Diagnosis and Management Options inuploads.worldlibrary.org/uploads/pdf/20180107071525differential... · 83 Peripartum cardiomyopathy 165 84 Hypertension 167 85 Convulsions

165. Desire for MedicalTermination of Pregnancy

A. Medical termination of pregnancy (MTP) is permitted by the Indian MTP Act upto 20 completed weeks of pregnancy for a large number of indications. Failure ofcontraception and rape are two important ones out of these. Before a womanundergoes an MTP, she has to understand various complications associated with it. Ifthe woman desires an MTP even after adequate counseling, it may be done.

B. The methods used for MTP vary according to the uterine size (gestational age).Up to 6 completed weeks of pregnancy, the products of conception are quite smalland can be removed without undue morbidity using menstrual regulation (MR), inwhich manual vacuum aspiration is done using Karman syringe (50 ml) and 6 mmcannula, which does not need cervical dilatation. MR does not need electricity as asuction machine would. It does not need anesthesia because the cervix does not haveto be dilated. Thus complications of MR are fewer than those of dilatation andcurettage (D&C). Even confirmation of pregnancy is not necessary, and it can bedone if the woman has missed her menstrual period by up to 2 weeks. But it has afailure rate of up to 3.7%, and if the woman does not get her next period as expected,D&C is required. As an alternative to MR, a woman may opt for a medical methodfor MTP or D&C.

C. Up to 9 weeks of pregnancy, the woman has an option of undergoing a medicalMTP and D & C. Medical MTP can be performed up to 9 weeks of gestation. In thismethod 200 mg of mifepristone is given orally, followed by misoprostol 600 μgorally or vaginally after 24 hours. The patient aborts, and if a complete abortion doesnot occur, a blunt curettage is done. D&C is performed under a paracervical blockcombined with parenteral analgesia, or general anesthesia. The cervix is dilatedrapidly to a size in millimeters equal to the number of weeks of gestation, theproducts of conception are removed with a suction cannula with the same externaldiameter in millimeters as the number of weeks of gestation, and the completeness ofthe procedure is confirmed by a blunt curettage of the endometrium.

Difficulties may be encountered in D&C. It may not be possible to pass even auterine sound through the cervical canal due to cervical stenosis. This problem isovercome by passing a fine probe into the canal, followed by half-graduated cervicaldilators. The uterus is quite soft during pregnancy and may get perforatedaccidentally. The damage can be extensive if it is perforated by a suction cannula,which can suck in bowel and injure it. Such a complication necessitates an immediate

exploratory laparotomy and treatment of the bowel injury (if any) by resection andend-to-end anastomosis. The MTP is completed by the vaginal route while anotherperson watches the site of perforation through the laparotomy incision. If theperforation has been caused by a sound, dilator, or a blunt curette, a laparotomy neednot be done as the first measure. A laparoscopy is done to observe the site of uterineperforation. If it is not bleeding actively, the MTP is completed by the vaginal routewhile another person observes the site of perforation through the laparoscope toconfirm that the instrument used for MTP does not pass into the peritoneal cavitythrough the perforation. On the other hand, if the perforated site is bleeding actively,a laparotomy is done, the MTP is completed under direct visual control, andhemostatic sutures of No. 1-0 polyglactin are placed over the perforation.Experienced endoscopic surgeons can trace bowel by laparoscopy and a laparotomyneed not be done in such a situation.

D. Between 9 and 12 weeks MTP is done by D&C as described before.

E. Between 12 and 14 weeks MTP can be postponed up to 16 completed weeks, sothat a second trimester method can be applied more effectively. However if thepatient is very keen on an immediate MTP, D&C can be done 2 to 3 hours afterintramuscular administration of 250 μg of PGF2α which achieves powerful uterinecontractions, cervical dilatation, and minimizes blood loss.F. Between 14 and 16 weeks of gestation, the patient is advised to wait up to 16completed weeks of gestation, because the second trimester methods are not veryeffective in this period. Between 16 and 20 weeks of gestation one has a choicebetween extraovular instillation of ethacridine lactate (EOE), intramuscularprostaglandins (IM-PG), and vaginal misoprostol. In EOE 150 ml of 0.1%ethacridine lactate is instilled extraovularly through a Foley's catheter. If the womandoes not abort in 72 hours and does not even get uterine contractions, the procedureis considered to have failed and either the procedure is repeated or one of the othertwo methods is used. The process if dilating the cervix slowly with laminaria orisapgol tents followed by the use of an oxytocin infusion is an alternative.Unfortunately tents are not available commercially. In IM-PG, 250 μg PGF2α isadministered 3 hourly up to 24 hours, monitoring uterine contractility and cervicaldilatation before administration of each dose. The dose is withheld if there are verypowerful uterine contractions while the cervix remains tightly closed, so as to avoiduterine rupture. Misoprostol is used in a dose of 400 μg PV every 4 hours, up to atotal of 5 g. Aspirotomy is another option which we do not favor, because it involvesremoval of a very large fetus piece-meal after slow dilatation of the cervix. It cancause serious blood loss or uterine injury or both. Hysterotomy is an obsoleteprocedure.

331

Page 338: Differential Diagnosis and Management Options inuploads.worldlibrary.org/uploads/pdf/20180107071525differential... · 83 Peripartum cardiomyopathy 165 84 Hypertension 167 85 Convulsions

Assess indicationCounseling

DESIRE FOR MTP

Valid indication, persistent desire No indication/loss of desire

Assess uterine size (A) Antenatal care

< 6 weeks (B) < 9 weeks (C) 9-12 weeks (D) 12-14 weeks (E) 14-16 weeks (F) 16-20 weeks (G)

Menstrual Check choiceregulation

Assess willingness to wait Wait up to 16weeks

Check forcontraindications

for PG

Medical Surgical Unwilling Willing to wait upto 16 weeks

Present AbsentMifepristone +

misoprostolRapid dilatation, suction evacuation, blunt curettage PG + Dilatation

and suctionevacuation

Succeeds Failstotally/partially

Extraovularethacridine lactateor normal saline

Misoprostol orPGF2α

Rapid dilatation, suction evacuation, blunt curettage

Succeeds Difficulty in passing through cervix Uterine perforation

Appropriate management

332

Page 339: Differential Diagnosis and Management Options inuploads.worldlibrary.org/uploads/pdf/20180107071525differential... · 83 Peripartum cardiomyopathy 165 84 Hypertension 167 85 Convulsions